首页

部编版九年级语文上册全册教学课件PPT

资源预览文档简介为自动调取,内容显示的完整度及准确度或有误差,请您下载后查看完整的文档内容。

1/1330

2/1330

3/1330

4/1330

剩余1326页未读,查看更多内容需下载

部编版九年级语文上册第一单元第一课沁园春雪 名师导学1沁园春·雪一、课前自学知识储备1.文章结构的作用:①总领全文;②起承上启下的过渡作用;③为下文做铺垫,使前后文有所照应;④总结全文,点明中心,深化主旨。2.常见的照应方法有三种:首尾照应、前后照应、文题照应。照应的作用是使文章内容更为完整,结构更为紧密。 一课一法体会课文结构的严谨性:(1)“总—分”结构:“望”字统领下文,展现了长城、黄河、山脉、高原这些最能反映北国风貌的雄伟景观;“惜”字统领下文,从而展开作者对各代英雄的品论。(2)过渡自然:“江山如此多娇”承上,总括上阕的写景;“引无数英雄竞折腰”启下,展开下阕对历代英雄的评论。这一过渡使全诗浑然一体,结构严密。(3)前后照应:“长城内外”“大河上下”,各照应“万里”“千里”;“惟余莽莽”“顿失滔滔”,分别照应“雪飘”“冰封”。 二、内容梳理(根据课文填空,理清文脉)《沁园春·雪》分上下两阕。上阕描写北国雪景,展现祖国山河的壮丽;下阕由祖国山河的壮丽引出英雄人物,纵论历代英雄,抒发作者的抱负。沁园春·雪上阕(描写)总写①__________的开阔宏大北国雪景分写望”:长城、黄河、山脉、高原——表现作者②__________的胸襟“须”:想象雪后晴日——抒发作者对北国风光的③__________之情博大赞美下阕(议论、抒情)过渡句:“④__________________________________________________________”纵论英雄:“惜”字寄寓作者⑤_______________________主旨句:“⑥___________________________________________________________”江山如此多娇,引无数英雄竞折腰伟大的抱负俱往矣,数风流人物,还看今朝 名师导练基础训练1.根据拼音写出相应的词语。(1)大河上下,/顿失tāotāo(   )。(2)须晴日,/看红装素裹,/分外yāoráo(   )。(3)江山如此多娇,/引无数英雄竞zhéyāo(   )。(4)唐宗宋祖,/稍逊fēngsāo(   )。/一代tiānjiāo(   ),/成吉思汗,/只识弯弓射大雕。/俱往矣,/数fēngliú(   )人物,/还看今朝。滔滔妖娆折腰风骚天骄风流 2.下列句子中加点的词语使用不恰当的一项是()A.碧绿的海水将厦门和鼓浪屿分成两部分,隔海望去,坐落在大海中的鼓浪屿犹如一座海上公园,山清水秀,独领风骚。B.大江东去,浪淘尽,千古风流人物。C.春节期间,当越来越多的人忙于抢红包并乐此不疲时,世界上最遥远的距离就变成了“我看着你,你却在抢红包”。D.他在班会上的即兴讲话逻辑严密,语无伦次,博得同学们的阵阵掌声。D 3.下列对病句的修改不正确的一项是()A.由于六环路全线贯通,标志着北京高速公路网基本建成。(删去“由于”)B.未来五年,国家将通过改造棚户区,建造公租房等方式,增加中低收入居民的住房供给,解决当前存在的房源不足。(在“房源不足”后加“的问题”)C.近日,东方出版社出版引进了诺贝尔化学奖得主姜·范恩撰写的科普书《热的简史》。(把“引进”调到“出版”的前面)D.2009年10月9日,美国半人马座火箭、月球坑观测和传感卫星先后两次相继撞击月球的南极地区。(删去“两次”)D 4.阅读下面材料,按要求作答。追求健康长寿是现代人的普遍愿望。近年,美国农业部推出了“食物指南金字塔”图形,这成为美国人摄取食物的最佳方案。“金字塔”底层为面包、米饭等谷类食物,是饮食的基础;第二层为蔬菜水果,是维生素的主要来源;第三层为肉、蛋、奶等,是蛋白质的来源;顶层为脂肪、油类、甜食,人们只需少量摄取即可。许多食品学家都认为这种饮食结构设计合理,有利于人们科学饮食,追求健康长寿。中国20世纪90年代也曾颁布过《食物结构改革与发展纲要》,强调“食物要多样,粗细要搭配,三餐要合理,营养要保证,甜食不宜多,油脂要适量”,这与“金字塔”图形十分相似。 (1)请用一句话概括以上内容,不超过30字。____________________________________________________________________________________________________________________(2)根据材料内容,以“健康”为描述对象,用上一种修辞方法,写一句话。____________________________________________________________________________________________________________________科学的饮食结构有利于人的健康长寿。(或:合理的饮食搭配是人健康长寿的条件)合理饮食是打开健康之门的金钥匙,平衡营养是走向长寿之路的通行证。 课内精读5.赏析课文中精妙的句子。(6分)(1)“千里冰封,万里雪飘”一句运用了什么修辞方法?有什么作用?________________________________________________________________________________________________________________________________________________________________________________________________________________________________________(2)“山舞银蛇,原驰蜡象,欲与天公试比高”一句运用了什么修辞方法?有什么作用?______________________________________________________________________________________________________________________________________________________________________________运用了互文的修辞方法。写出了北国雪景的广袤无垠,流露了作者的豪迈,抒发了作者对北国雪景的热爱之情,表现出作者的博大胸怀。比喻、拟人。化静为动,生动形象地写出了“山”“原”腾飞的气势,写出了“山”“原”无畏的气概和雄心勃勃的精神面貌。 6.如何理解课文的主旨?(3分)____________________________________________________________________________________________________________________通过对北国雪景的描写,抒发了作者对祖国山河的热爱,表现了作者豪迈的胸怀。散文阅读技法专练——文章结构的安排春天的风阿 土①我记着这样一个故事,有位自认禅学很深的居士,一天在寺外与庙里的高僧相遇,便故意向高僧提出“佛何为”的话来。高僧并未接话,而是平静地望着春田里劳作的农人。居士再问,高僧却出口说道:“春天的风好吗?”居士大惊,警戒之心陡起,随即谨慎言道:“什么风?”高僧听了,淡淡一笑自转身走了,居士则沉思良久终不得解。故事到此止住,不得解的不仅是那个居士,我也一样如坠五里云雾。 ②春天的风好吗?③其实,风本身没有什么不同,不同的只是你的心境!这本不是秘密,我却荒废了无数个春天,才不经意发现,而使我有此发现的还是一个名不见经传的普通老农。④我喜欢在春天来到乡下,这种喜爱常常让我不能自已。其实,人的喜恶在很多时候常常取决于自己对待事物的心态,也因此导致他没有平等对待其他事物的机会。⑤我喜欢春天的乡下,喜欢那里散发出的泥土气息,喜欢出现在田野里的所有事物。小的时候,我还喜欢打着赤脚与春天的泥土一次次亲近,尽管那时候的泥土还很凉,顺着脚心往上冲的冷气有些刺骨的感觉,但对于一个习惯于乡村生活的孩子来说,依然有说不尽的快乐,我们会为此做出不同的运动,直到让冷气变成暖气。如今,我再也不能像小的时候光着脚丫子在泥土上欢蹦乱跳了。人到了一定的时期,就得真实地遵从那个时期的规矩呀! ⑥田埂向远方延伸着,在不远处的地头坐着一位老人。他安静地坐着,嘴里含着一根长长的烟袋,双手环抱膝上,眼睛直直地望着田里,锄头在他的屁股下横着。我走近了他,发现他的头发已几近灰白,脸上的皱纹很深,让人觉得就是放上一支笔也不会掉下来。老人神往地眯着眼睛,很专注的样子,似乎连我的到来也没有注意。他的样子有些像我十多年前就已去世的伯父,不由得给我一种亲近感。⑦“看什么呢?”我在他的身边坐下,怕惊了他,而轻轻地问着。“春天的风。”老人不假思索,甚至没有来得及取出嘴里早已熄了的旱烟袋。“春天的风?”我不由一愣。“是呀。”老人回过神,转过脸,松开抱在膝上的手,朝我微微地点了一下头笑着说:“春天的风好呀!” ⑧“春天的风好呀!”我突然觉得这句话竟是那样的熟悉,便不由得随口问道:“春天的风有什么不同吗?”⑨“其实,风本身没有什么不同,不同的是你在春天的心境而已!”老人缓缓地说着,“在充满爱意的心境里,春天的风不轻狂、不暴躁,它温和、美丽,能催出生命的新绿,能温暖冰凉的心。”顿一顿,老人又接着道:“在满是怨愤的心境下,春天的风是料峭的、寒冷的,让你猝不及防。但那才是真正春天的风呀,就像人生没有什么可以永恒一样,变幻无常才是真正的世象。”老人边说边指着田里的庄稼,“就像它们在春风里不停地扭着身子,是感激还是挣扎与反抗呢?” ⑩是呀!老人的话再次让我想起那个有关高僧与居士的故事,心里竟慢慢地有了些感悟。其实高僧的表达很简单,他只是在告诉我们保持一颗平常心而已,因为我们常常由于执着而不知所以!我不得不对这个看上去并不起眼的老农另眼相待,他不仅有着如此缜密的心思,还有着独具的智慧与心性。是的,一切事物都有它不同寻常的契机,没有什么是可以忽略的!我望着老农,想着他说的话。人生并不是所有的事物都要彰显出来,有些时候隐忍着的更具有不可估量的深度。风从田野上吹过,轻缓而温柔,它吹过路面,路面的小草钻了出来;它吹过小河,小河上的冰慢慢地薄了。(选自《读者》,有删改) 7.文章开头为什么从一个故事开始,这样写有什么好处?(4分)______________________________________________________________________________________________________________________________________________________________________________8.“是的,一切事物都有它不同寻常的契机,没有什么是可以忽略的”一句中“契机”在文中具体指什么?(3分)______________________________________________________________________________________________________________________________________________________________________________①巧妙地引出“春天的风”这个话题,照应题目;②设置悬念,引起读者的阅读兴趣,并引起读者对“春天的风”的思考;③为下文做铺垫。“契机”本意指事物转化的关键,在文中指春天,“我”在乡下遇到一位老人,老人的一番话使“我”不经意地发现了一个道理。 9.联系全文,谈谈你对“其实,风本身没有什么不同,不同的是你在春天的心境而已”的理解。(5分)________________________________________________________________________________________________________________________________________________________________________________________________________________________________________10.文章最后一段在全文中有何作用?(5分)____________________________________________________________________________________________________________________作为一种自然现象,风本来无所谓好坏,而人的不同心境却影响着人对风的看法,有什么样的心态就有什么样的春风。因此,人应该用平和、乐观、积极的心态来看待客观事物,对客观事物要保持一颗平常心。照应题目和开头,前后呼应,首尾圆合;表现出作者有所感悟后的乐观、积极的心态和欣悦之情。 中考考点实训名著阅读模拟训练太  阳从远古的墓茔/从黑暗的年代/从人类死亡之流的那边/震惊沉睡的山脉/若火轮飞旋于沙丘之上/太阳向我滚来……它以难遮掩的光芒/使生命呼吸/使高树繁枝向它舞蹈/使河流带着狂歌奔向它去当它来时,我听见/冬蛰的虫蛹转动于地下/群众在旷场上高声说话/城市从远方/用电力与钢铁召唤它于是我的心胸/被火焰之手撕开/陈腐的灵魂/搁弃在河畔/我乃有对于人类再生之确信(节选自《艾青诗选》) 1.请概括“太阳”这一意象的主要特征及其象征意义。(3分)______________________________________________________________________________________________________________________________________________________________________________2.简要分析诗的前二节中运用的修辞方法及其作用。(4分)________________________________________________________________________________________________________________________________________________________________________________________________________________________________________主要特征:①诞生于黑暗和死亡;②炽焰熊熊、恢宏伟大;③势不可挡;④创造万物。象征意义:太阳象征着光明、希望和新生。第一节中,诗人运用比喻的修辞方法,把太阳比作火轮,形象生动地展现了太阳炽焰熊熊、滚滚而来的奔涌之势。第二节中,诗人运用拟人的修辞方法,描写“高树的舞蹈”“河流的狂歌”等,勾画了一幅众生热烈欢迎太阳、万物复苏的情景。 3.诗歌采用“墓茔”“黑暗”“死亡”“舞蹈”“河流”“虫蛹”等意象有什么作用?(3分)__________________________________________________________________________________________________________________________________________________________________________________________________________________________________________________________________________________________________太阳的象征意义是光明、希望和新生,所以诗歌通过“墓茔”“黑暗”“死亡”等意象来反衬太阳,并通过“舞蹈”“河流”“虫蛹”等一系列万物复苏的动态意象来正面烘托太阳,使其象征意义格外鲜明。 2019秋部编版九年级语文上册第一单元第二课我爱这土地 名师导学2我爱这土地一、课前自学知识储备象征手法:象征是诗歌、散文中常见的写作手法,是以具体事物去表现某种抽象概念或不便表达的概念的一种文学手法。象征手法的作用:一般用来赞颂美好的事物,体现作者对理想境界的追求,但有时也可用来讽刺丑恶的事物,抨击不合理的社会现象。●上册● 一课一法写实和象征手法的交织:诗人运用写实和象征的手法,描绘了一组鲜明的诗歌意象,分别赋予“土地”“河流”“风”“黎明”等意象不同的象征和暗示意味,同时流露出作者对祖国的“黎明”抱有乐观的信念。二、内容梳理(根据课文填空,理清文脉)《我爱这土地》是一首在现代诗歌史上广泛传诵的抒情名篇,以一只鸟生死眷恋土地作比,形象地抒发了诗人深沉而真挚的爱国情感。诗人艾青用“嘶哑”来形容鸟儿鸣唱的歌喉,更能表达出诗人为祖国前途、命运的担忧,心力交瘁的情状。全诗共两节: 我爱这土地第一节(铺陈叙述)声音嘶哑的鸟——象征①________________悲愤的爱国者“歌唱”(总领下文)暴风雨打击的土地——象征②__________________________汹涌着悲愤的河流——象征③__________________________无止息的激怒的风——象征不屈不挠的斗争精神来自林间的温柔的黎明——象征④__________________________遭受侵略蹂躏的祖国顽强的反抗精神独立自由的曙光第二节(直接抒情):“为什么”——因为爱 名师导练基础训练1.根据拼音写出相应的词语。(1)假如我是一只鸟,/我也应该用sīyǎ()的喉咙歌唱。(2)这被暴风雨所打击着的土地,/这永远xiōngyǒng()着我们的悲愤的河流,/这无止息地吹刮着的激怒的风,/和那来自林间的无比温柔的límíng(   )……(3)——然后我死了,/连羽毛也fǔlàn()在土地里面。嘶哑汹涌黎明腐烂 2.下列句子中加点的词语使用不恰当的一项是()A.一个打扮并不华贵却端庄严肃而有美德的人,是令人肃然起敬的。B.贺州客家围屋以其独特的建筑风格吸引众多游客,我们不得不叹服建造者的别具匠心。C.朗诵文天祥的《过零丁洋》会让我们的爱国情怀油然而生。D.在“红五月”读书活动期间,语文老师就如何“读好书”的话题说长道短,让我们受益匪浅。D 3.下列对病句的修改不正确的一项是()A.中学生之所以喜欢网络文学作品的原因,是因为这些作品不仅表现手法自由活泼,而且思想感情丰富细腻。(删去“的原因”)B.广东省政府深化改革,加快广东自贸区。(把“加快”改为“加强”)C.近200年来,地球的森林已有三分之一左右被采伐和毁掉,尤其在近50年,部分地区森林减少了一倍。(把“一倍”改为“一半”)D.我们要养成爱读书,特别是读经典,读名著,让书香飘满我们的生活。(在“爱读书”后加“的习惯”)B 4.阅读下面的材料,按要求作答。毛笔是一种源于中国的传统书写工具,有玉管、翠管等雅称,被列为中国的文房四宝之一。毛笔是汉族在生产实践中发明的。毛笔初用兔毛,后亦用羊、鼬、狼、鸡、鼠等动物毛,笔管以竹或其他材料制成。它头圆而尖,用于传统的书写和图画。一支好的毛笔应具有“尖、齐、圆、健”的特点。“尖”就是笔锋尖锐;“齐”就是笔锋修削整齐;“圆”就是笔头圆润;“健”就是毛笔弹性强,写出的字锐利矫健。我国最有名的毛笔是出自浙江湖州的湖笔、河南的太仓毛笔、河北的侯店毛笔、湖南湘阴的长康毛笔以及江西的文港毛笔。 (1)请用一句话概括以上内容,不超过30字。__________________________________________________________(2)根据材料内容,以“毛笔”为描述对象,用上一种修辞方法,写一句话。____________________________________________________________________________________________________________________毛笔是以兽毛为笔头而制成的中国传统书写和绘画工具。几千年来,毛笔如一位德高望重的老者,慢慢书写着汉族光辉灿烂的文化历史,描摹着劳动人民智慧挺拔的丰姿。 课内精读5.赏析课文精妙的句子。(6分)(1)“——然后我死了,连羽毛也腐烂在土地里面”一句有何深意?______________________________________________________________________________________________________________________________________________________________________________(2)“为什么我的眼里常含泪水?因为我对这土地爱得深沉”一句运用了什么修辞方法?有什么作用?______________________________________________________________________________________________________________________________________________________________________________生动地刻画了这只鸟饱受磨难却拼尽全力地用整个生命去歌唱的形象,表达出诗人对土地的眷恋;将自身融进大地,隐含了一种敢于牺牲自我的精神。设问。一问一答,直抒胸臆,以“我的眼里常含泪水”的情状,寄托诗人那颗真挚、炽热的爱国之心,形象地表达了诗人对土地、对祖国的眷恋和热爱。 6.这首诗的第一、二节之间有什么内在联系?(4分)______________________________________________________________________________________________________________________________________________________________________________散文阅读技法专练——文章表现手法之象征最温暖的一片阳光李云迪①母亲真的老了,有点像孩子般缠人。她每天都会打来电话。她耳朵有点儿背,电话里声音总是很大。其实她也没什么事,只是问我是否安好。在得到肯定的答复后,她就高兴地把电话挂掉。有时我忙,就会打断她的话,我能听得出那边轻轻的叹息。但母亲还是在每天上午9时左右打过来,正是太阳高升的时候。这两节诗都饱含着诗人对祖国深深的眷恋和无尽的热爱之情。第一节是对“爱土地(祖国)”主题的抒情性的铺陈描述,第二节可看作是感情的迸发和升华,让读者更强烈地感受到这种真挚的感情。 ②这一天,母亲又来电话,除了问好外,还告诉我她去采了很多婆婆丁。母亲过去常用婆婆丁做蒸菜,婆婆丁一定要野生的,最好在打春时没有开花前去采,这时的最新鲜。小时候我很爱吃母亲做的这个菜。我有些担心地告诉母亲,您都80多岁了,千万不要去野外采野菜了,我会不放心的。母亲答应得很痛快。③第二天正午时分,有人叩门,打开门见是母亲。她脸上汗津津的,花白的头发也有些凌乱,我急忙把母亲扶进屋内,埋怨她为什么不打电话告诉我。可母亲不理我的不满,喜滋滋地取出她做的蒸菜,看不出一丝倦意。我见她的手青筋暴露,手上有结痂的血口子,这一定是她采野菜时留下的。她说,快吃吧,还温热着呢。我怎么也想不到,母亲为了让我吃到她做的菜,竟然沐着风雨去野外,顶着晨星点灶火,用一股爱焰把菜做熟,搭乘公共汽车,颠簸几个小时,给我送来。在世上,凡是有母亲的地方,就会有奇迹。 ④母亲真的老了,眼前的事情她往往记不住,可过去的事情却记得很清晰。母亲也有青春,只是离今天很远。她曾把箱子里保存的一条长裙拿给我看,告诉我她就是穿了这条长裙和父亲在向日葵下相识的。那长裙依旧如新,可裙子的主人却被时光洗老,腰弯了,头发白了,白得像天上的一片云。⑤母亲生在青川江边,长在稻花乡里。母亲居住的老屋前有一条唱着歌的小溪,每当燕子呢喃着飞来的时候,桃花就开了,开着开着,桃花就落了,花瓣落在溪水里打着旋儿流远了,就像流远的岁月。老家的院子里种着两棵杏树,每当圆润的青杏结满枝头的时候,不等熟透,母亲就和小姐妹们采下来吃。那味道酸酸的,就像那时辛酸的日子。母亲在艰苦的岁月中学会了很多家务活,有一手好针线,成家后很是勤俭。在那艰苦的日子里,她用那根银针缝补着旧衣裳一样的破旧岁月。父母感情很深,生活得很和睦。父亲在知天命之年患了重病,离开了人世。母亲开始独自生活。我担心她孤寂,执意要把她接到身边来,可她不肯,她说,她能自理,也不想给我们增添麻烦。我给她请了保姆,可没几天就被她打发走了。 ⑥忽然想起,有两天没有接到母亲的电话了。我急忙打电话过去,多次无人接听,我有了不祥之感,乘夜车就往家里赶。赶到家里时,见母亲躺在床上,额头敷着毛巾。邻居大婶告诉我,母亲发了两天烧,烧得重时不断念叨着我的名字。我守候在母亲的床边,两手紧攥着她那像树枝一样干瘪的手。母亲问我,你知道我最希望的事是什么吗?我摇摇头,她告诉我,最希望的事是让我永远不会老,永远不得病,如果有可能,让她代替我老,代替我生病,她也乐意。就是走了,她也不想扰动我,想一个人悄悄去找我的父亲。我听着,心被刺痛,泪如泉涌。我把脸紧紧贴在母亲的手上,整个人似乎已经融化了。⑦往后的日子里,每天上午9时左右,也就是太阳高升的时候,我都先打电话给母亲,问候她是否安好。我还会幸福地眯着眼睛,虔诚地半张着嘴,侧着耳朵凝神听母亲讲着过去的故事。这个时候,我真的很温暖。母亲就是给我温暖的那一缕阳光……(有删改) 7.请简要概述文章的主要内容,并说说文章题目“最温暖的一片阳光”运用了什么修辞方法?有什么作用?(5分)____________________________________________________________________________________________________________________________________________________________________________________________________________________________________________________________________________________________________________________________________________________________本文主要讲述了母亲给“我”打电话以及送蒸菜的事情,表达了母亲对“我”无微不至的关爱以及“我”对母亲的理解与感激。象征。题目不仅用“最温暖的一片阳光”象征母亲、母爱,而且与文章结尾相呼应,突出了母爱的伟大。 8.品味语言,回答问题。(4分)(1)从人物描写方法的角度,赏析“她脸上汗津津的,花白的头发也有些凌乱”这个句子。____________________________________________________________________________________________________________________(2)结合语境,赏析“那长裙依旧如新,可裙子的主人却被时光洗老”一句中加点词语的表达效果。______________________________________________________________________________________________________________________________________________________________________________这句话运用了外貌描写的方法,写出了母亲着急赶来给“我”送蒸菜时的样子,表现了母亲对“我”无微不至的关爱。“洗”是“冲洗”的意思,在这里是指母亲随着时光的流逝慢慢变老。作者巧用动词,生动传神地写出了岁月的无情以及“我”的伤感。 9.请结合第③段,谈谈你对该段中画线句子的理解。(4分)__________________________________________________________________________________________________________________________________________________________________________________________________________________________________________________________________________________________________只要有母亲在,就会有想象不到的不平凡的事情发生。文中是指母亲不顾年老体衰,克服重重困难,采摘野菜并做好美味的蒸菜给“我”送来,令“我”震撼的事情。这句话体现了母爱的伟大,表达了“我”对母亲的感激之情。 10.文章在构思和写作方法方面富有特色,请结合文章内容,就其中一点写出你的见解。(4分)______________________________________________________________________________________________________________________________________________________________________________________________________________________________________________________________________________________________________________________________________________________________________________________________________________________示例一:运用对比的修辞方法。如文章开篇写母亲主动给“我”打电话和结尾“我”主动给母亲打电话形成对比,更好地突出了“我”对母爱的理解与赞美,以及对母亲的感激之情。示例二:运用插叙的写法。文章插叙了母亲过去的生活经历,使文章内容充实,使人物形象丰满,突出母亲一生的艰辛,更为真切感人。 中考考点实训名著阅读模拟训练礁  石一个浪,一个浪/无休止地扑过来/每一个浪都在它脚下/被打成碎沫,散开……它的脸上和身上/像刀砍过的一样/但它依然站在那里/含着微笑,看着海洋……(节选自《艾青诗选》) 1.试分析“礁石”这一形象的特点及其象征意义。(3分)______________________________________________________________________________________________________________________________________________________________________________2.赏析“但它依然站在那里/含着微笑,看着海洋”一句中加点词语的表达效果。(3分)______________________________________________________________________________________________________________________________________________________________________________3.诗歌描写了向礁石“无休止地扑过来”的海浪,有什么作用?(4分)______________________________________________________________________________________________________________________________________________________________________________长年累月在狂风巨浪的拍打下坚强不屈、坚定不移的礁石形象。它不仅是不向命运低头的诗人自身的象征,而且是我们中华民族不畏强暴、不屈不挠的精神象征。借助“站在那里”“含着微笑”等词语,诗人赋予礁石以生命,使之人格化,并将其长期受海浪迫害却依然坚强不屈、乐观自信的精神形象生动地表现了出来。①向礁石“无休止地扑过来”的海浪,象征着迫害他人的人;②以海浪的汹涌嚣张与礁石的乐观坚挺做对比,更加突出礁石的形象特点。 2019秋部编版九年级语文上册第一单元第三课乡愁 名师导学3乡愁一、课前自学知识储备1.文学作品中常见的写作手法:联想、想象、象征、对比、衬托、反衬、以小见大、借景抒情、直接(间接)描写、欲扬先抑或欲抑先扬。2.反衬手法:采用相反的、相对立的事物来衬托主体,从而使主体更形象、更突出的一种写作技巧,如以美衬丑、以乐衬悲,以恶衬善等。●上册● 3.对比和反衬的区别:对比和反衬是文章常用的表现手法,两者易于混淆,同学们要注意进行区别。对比的两个事物间的关系是并列的,结果是突出对比的双方;反衬却可以明显地突出被衬托的一方。如“朱门酒肉臭,路有冻死骨”运用的是对比,而“蝉噪林愈静,鸟鸣山更幽”则用了反衬手法来突出环境的幽静。一课一法诗歌中的反衬手法:诗歌中“小小的”“窄窄的”“矮矮的”“浅浅的”四个形容词都以一种看似轻描淡写的方式,把乡愁浓缩于四个面积小、程度轻的对象之上,反衬出诗人内心深处浓烈的思乡情感。 二、内容梳理(根据课文填空,理清文脉)《乡愁》体现了余光中思念家乡与亲人的真挚情感。在诗中,随“乡愁”在诗人人生每个阶段对应物的改变,“乡愁”的情绪越来越浓,最终由个人的故乡之思上升到带有普遍意义的家国之思。时间顺序意象感情第一节小时候(少年)小小的邮票①__________第二节长大后(青年)②_______________③__________第三节后来(中年)④_______________生死悲痛第四节现在(老年)⑤_______________家国之思窄窄的船票矮矮的坟墓浅浅的海峡母子深情夫妻恩爱 名师导练基础训练1.根据拼音写出相应的词语。(1)小时候/乡愁是一枚小小的yóupiào(    )。(2)后来啊/乡愁是一方矮矮的fénmù(    )。(3)而现在/乡愁是一湾浅浅的hǎixiá(    )。邮票坟墓海峡 2.下列句子中加点的词语使用不恰当的一项是()A.海水剧烈地翻腾起雪白的浪花,活像一朵硕大的莲花。B.脱口秀主持人喜剧演员艾伦·德杰尼勒斯以她温厚亲切的幽默而大受观众喜爱。C.蓝的天,白的云,清的水,绿的山,还有房前屋后篱笆上盛开的蔷薇花,好一派田园风光,真是目不暇接。D.忽然间,走近这爱丽丝仙境般的地方,富丽堂皇的装饰让我一时间眼花缭乱。C 3.下列句子中没有语病的一项是()A.经调查,“8·12”天津港爆炸事故原因是瑞海公司违规经营、违规储存危险货物以及安全管理极其混乱所造成的。B.面对叙利亚小难民艾兰浮尸海滩的照片,使欧洲一些国家终于松口,允许更多难民入境。C.磁州瓷器工艺精湛,具有高雅、时尚、个性的艺术享受,是一种观赏价值极高的艺术品。D.屠呦呦用青蒿素治疗疟疾的研究,有效降低了疟疾患者的死亡率,为医学发展做出了卓越的贡献。D 4.阅读下面的材料,按要求作答。杭州的建筑工地每年有1000多个,如果把这些工地的围墙适当“美容”一下,它将成为街头一道独特的风景。为进一步打造“国内最清洁城市”,今后,杭州新开工的建筑工地的围墙都要进行美化。现在不少城市的墙绘主要是广告,而杭州的墙绘则比较有江南特色,有些还融入了地域文化。有关部门要求杭州建筑工地的墙绘今后要消除营利性广告,逐步走向公益化,赋予其丰富的文化内涵,体现出杭州的文化特质。 (1)请用一句话概括以上内容,不超过30字。__________________________________________________________(2)根据材料内容,以“文化墙绘”为描述对象,用上一种修辞方法,写一句话。______________________________________________________________________________________________________________________________________________________________________________杭州建筑工地围墙要美化得有文化味。文化墙绘是传统文化与现代文明的美丽邂逅,在带来视觉享受的同时,也形象生动地传递着传统文化,展现着现代文明的风采,滋润着人们的心田,丰富着群众的精神世界。 课内精读5.请联系全文,说说诗歌中的感情是如何层层递进的。(3分)____________________________________________________________________________________________________________________6.全诗共四节,在结构形式上有何特点?(5分)__________________________________________________________________________________________________________________________________________________________________________________________________________________________________________________________________________________________________诗中感情是层层深入的,先是母子别、夫妻别,再是生死别、故国别,诗人由个人情感上升到民族情感。①“小时候”“长大后”“后来”“现在”这种表示时间的时序语句,表现出时间的变化;②“乡愁是……”这一句式的反复出现,营造出一种回环往复、一唱三叹的旋律;③“这头”“那头”“里头”“外头”自然地显示出空间的隔离,反复使用营造出一种低回掩抑、如泣如诉的气氛。 散文阅读技法专练——文章表现手法之欲扬先抑仙人掌花曹乾石见别人家阳台上花草缤纷,煞是艳羡,激起了我养花的兴致。我从花草市场陆续买了一些回家种养,海棠、月季、杜鹃、米兰、文竹……我开始还能记着浇水、松土、施肥,但终究是很难坚持,结果一盆盆花草都枝枯叶落。我这才意识到,养花并非只是闲情逸致,那完全是精明勤快人的事儿,与我这个愚汉、懒汉无缘。例外的是,只有一盆仙人掌活了下来。不论阴晴雨雪,也不论那一小盆泥土如何枯涸龟裂,它总是绿色挺拔,每一根毛刺都伸出渴求的喙,贪婪地啄食着阳光和空气。 可这并未给我带来欢欣。它既不美,也不雅。我只是不无遗憾地想,为什么美的总那么娇弱,而不美的却总是那么倔强呢?要是海棠、月季、杜鹃、米兰也这样易于生存该有多好啊。对于仙人掌的存在,我也不再关心。许久后的一个黄昏,我倚着阳台观望,无意中又瞥见这盆仙人掌。它竟横竖添了许多新节,刀戟般凛凛叉开,待细看时,我却忍不住大笑起来。不是吗,且不说它的大小掌节是怎样歪歪斜斜,怪模怪样,只看那绿色的掌面吧,竟皱皱巴巴,细纹密布,活像一张老人的脸。我真不知道,它还会怎样疯长下去。也许叫它死去比叫它活着都困难。这可怕的东西! 我的养花历史就此告一段落,自然是没养出一朵花来。谁想夏天一到,奇迹竟发生了。一个雨后的清晨,我推开阳台门,蛋青色的光亮和泉水般的空气扑面而来。我发现那盆仙人掌上有一大团很耀眼的东西。是霞光吗?是雨水折映的太阳吗?我定睛一看,什么?竟然是花。是的,是一朵花,并且是一朵异常漂亮的花。十二片金黄色绸缎般富有光泽的花瓣,敏感地轻颤着流苏般柔软的花穗,细密地遍布着雪乳般滋润的花粉。它凝然静立,却闪烁着宫殿般辉煌灿烂的光芒。这是我看过的最美的花!此刻,仙人掌正骄傲地高擎着它。仙人掌上的皱纹是愈加深刻了。是啊,它赖以立身的不过是一抔泥土,要开出这样震撼人心的金色花朵,能不倾注全部心血吗? 可以想象,这仙人掌,它曾经有过柔软的叶,窈窕的枝。但为了抗拒沙漠的压榨,它才变得冷峻而坚强。正是有了这冷峻和坚强,在沙漠吞噬掉无数娇美的花之后,我们还能欣赏到仙人掌神奇的金色花。它绝非一朵仅仅使人赏心悦目的花,它的不甘泯灭的美,令弱者肃然起敬。美,一旦与顽强结合,就是不可摧毁的。(选自《中华活页文选》,有删改) 7.请根据文章内容,概括补充作者对这盆仙人掌态度的转变。(3分)只有一盆仙人掌活了下来意外仙人掌既不美,也不雅(1)___________________横竖添了许多新节,怪模怪样(2)___________________仙人掌开出了金色的花朵(3)___________________遗憾、漠视嘲笑、不屑赞美、敬佩 8.文章开头写“我”养海棠、月季、杜鹃、米兰、文竹等,有什么作用?(4分)____________________________________________________________________________________________________________________9.赏析下列句子的表达效果。(4分)(1)只看那绿色的掌面吧,竟皱皱巴巴,细纹密布,活像一张老人的脸。____________________________________________________________________________________________________________________(2)此刻,仙人掌正骄傲地高擎着它。仙人掌上的皱纹是愈加深刻了。____________________________________________________________________________________________________________________开头写海棠、月季、杜鹃、米兰、文竹等植物的枯落反衬出仙人掌的顽强,也为下文“我”对仙人掌感情态度的变化做铺垫。运用比喻的修辞方法,把仙人掌的掌面比作老人的脸,突出仙人掌的丑陋,表现了“我”对仙人掌的不屑。运用拟人的修辞方法,将仙人掌美的花和深的皱纹结合起来,突出仙人掌的刚毅与柔美,表现了“我”对仙人掌的敬畏与喜爱之情。 10.文章主要采用了什么写作手法?有什么作用?(6分)__________________________________________________________________________________________________________________________________________________________________________________________________________________________________________________________________________________________________文章主要采用了先抑后扬的写作手法。作者先写仙人掌的丑陋是“抑”,后写它也能开花并且花朵非常娇艳是“扬”,前后形成鲜明对比,突出了“美,一旦与顽强结合,就是不可摧毁的”的主题,更能深入人心,给读者留下深刻的印象。 中考考点实训名著阅读模拟训练煤的对话你住在哪里?我住在万年的深山里/我住在万年的岩石里你的年纪——我的年纪比山的更大/比岩石的更大你从什么时候沉默的?从恐龙统治了森林的年代/从地壳第一次震动的年代你已死在过深的怨愤里了么?死?不,不,我还活着——/请给我以火,给我以火!(节选自《艾青诗选》) 1.诗歌采用了怎样的表现形式?这样写有什么好处?(3分)________________________________________________________________________________________________________________________________________________________________________________________________________________________________________2.诗歌借煤抒情,说说诗歌的主旨是什么。(2分)____________________________________________________________________________________________________________________对话的形式。作用:①使煤的内心世界得以表白,呈现出灵活、有趣、吸引人的形象;②诗人运用平静的问话与煤炽热如火的回答之间一冷一热的反差,用“我”的冷静反衬煤的热烈,使煤的自白给人以强烈的感染力。诗人通过树木被埋到地层深处变成煤后,渴望在烈火中再生的诉说,歌颂了被压迫民族不甘屈辱、自强不息的精神。 3.这首诗作于1937年春天,那时正是中华民族处于危急存亡的关头。你能结合时代背景和煤的特点简单分析一下这煤和我们的民族有什么共同之处吗?(5分)________________________________________________________________________________________________________________________________________________________________________________________________________________________________________煤的特点是深藏在地下,热能巨大,一旦燃烧便烈火熊熊。煤和中华民族的共同之处是当时我们遭受着和煤一样的灾难,又像煤一样长久地沉默,更像煤一样身上蕴含着无穷无尽的热量,并和煤一样憧憬着光明灿烂的前景。 2019秋部编版九年级语文上册第一单元第四课你是人间四月天 名师导学4你是人间的四月天——一句爱的赞颂一、课前自学知识储备1.首尾呼应:指文章的开头和结尾说的是同一个意思,二者互相照应。其构思方法及作用如下:(1)先叙后议:文章开头对事物进行客观叙述,而在结尾,针对所写的事物生发议论,这样,先叙后议,既首尾呼应,又深化了文章的中心。●上册● (2)回环往复:开头以为起点,结尾再重新回到这个起点上,形成循环往复之感,突出主题中心。(3)前因后果:文章开头交代某种原因,结尾写出原因所产生的结果。因果呼应,顺理成章,结构严谨。(4)情景相生:文章的开头通过对景物的描写,渲染一种气氛,结尾又重新回到相应的景物描写上来。反复渲染,增强感染力。2.暗喻、联想:暗喻是诗歌中常用的修辞方法,通过类比联想或接近联想,用一种事物代替另一种事物,可以为读者提供更广阔的想象空间,让人联想翩翩。 一课一法体会诗歌首尾呼应的作用:诗歌中开头“我说你是人间的四月天”和结尾“你是人间的四月天”首尾呼应,使得整首诗结构更严谨,韵律更完美和谐,主题更加突出。二、内容梳理(根据课文填空,理清文脉)《你是人间的四月天》是一首新格律诗,该诗充分体现了新诗创作的“三美”原则——音乐美、绘画美与建筑美。这首诗语言简单,色彩鲜明,韵律优美,意境清新,充满欢快喜悦之情。全诗共五节: 你是人间的四月天第一节:春光风舞图笑响点亮风(声音)风在春光中(光影)爱如四月天光艳轻灵第二节:风烟星雨图①__________(静态美)风吹星闪雨洒(动态美)云烟爱如四月天柔和恬静第三节:②__________图月夜花开百花(娉婷、鲜妍)圆月(天真、庄严)爱如四月天鲜妍庄严第四节:雪后新绿图(鹅黄、绿、白)——爱如四月天新鲜柔嫩第五节:③__________图花开燕语花盛开(看)燕呢喃(听)爱如四月天融暖缠绵 名师导练基础训练1.根据拼音写出相应的词语。(1)我说你是人间的四月天;/笑响点亮了四面风;qīnglíng()/在春的光艳中交舞着变。(2)那轻,那pīngtíng(),你是,xiānyán()/百花的guānmiǎn()你戴着,你是/天真,庄严,你是夜夜的月圆。(3)你是一树一树的花开,是燕/在梁间nínán(   ),——你是爱,是暖,/是希望,你是人间的四月天!轻灵娉婷鲜妍冠冕呢喃 2.下列句子中加点的词语使用不恰当的一项是()A.每年夏天,我们看到很多小太平洋蛙栖息在花园里的花朵上,享受着阳光。B.阅读课上,同学们像一只只勤奋的小蜜蜂,扑在书上,汲取着知识的营养。C.清末文豪王闿运是齐白石的恩师,素有冰鉴之称,他看人没看走眼,看诗却看走眼了,他对齐白石的旧体诗莫衷一是。D.生存还是毁灭?哈姆雷特陷入进退维谷的境地,他既担负着复仇的使命,又承受着亲情、爱情、友情等复杂情感的折磨。C 3.下列对病句的修改不正确的一项是()A.春节回到家乡,我又看到了母亲那亲切的笑容和久违的乡音。(在“久违”前加“听到”)B.每年全国青少年科技创新大赛有超过1000万名左右的青少年参加。(删去“超过”或“左右”)C.我们欣赏古代诗词,应该了解作者的全面生平以及他所处的时代环境。(把“环境”改为“背景”)D.回忆自己的成长,每一步都离不开“园丁”的培育。(在“成长”后加“历程”)C 4.阅读下面的材料,按要求作答。被称为“龙卷风之乡”的美国,东濒大西洋,西靠太平洋,南面又有墨西哥湾,大量的水汽不断从东、西、南面流向美国大陆。水汽多,容易形成雷雨云,当雷雨云积聚到一定强度后,就会产生龙卷风。美国所在的北美大陆中部是南北贯穿类似“U”形的地貌,西边有科迪勒拉山系,东部由阿巴拉契亚山脉和拉布拉多高原构成,而中部大多是平原,“方便”了来自北方的高压气流和南方的低压气流从中部“走廊”长驱直入并直接汇合,从而产生强大的气旋。龙卷风经过之处,常会发生拔起大树、掀翻车辆、摧毁建筑物等现象,它往往使成片庄稼、成万株果木瞬间被毁,令交通中断,房屋倒塌,人畜生命和经济遭受损失等。 (1)概括说明美国龙卷风的形成与哪些因素有关,不超过30字。____________________________________________________________________________________________________________________(2)根据材料内容,以“龙卷风”为描述对象,用上一种修辞方法,写一句话。____________________________________________________________________________________________________________________地理位置、气候条件、地形特点、大气环流特征(冷暖气流汇合)。龙卷风是大气中最强烈的旋涡现象,当猛龙一般的龙卷风席卷着大地时,它可扭折树枝,吹掀屋瓦,撞断电线…… 课内精读5.赏析课文精妙的句子。(6分)(1)“笑响点亮了四面风;轻灵/在春的光艳中交舞着变”一句运用了什么修辞方法?有什么表达效果?______________________________________________________________________________________________________________________________________________________________________________(2)“你是一树一树的花开,是燕/在梁间呢喃”一句从哪些感官描述了“你”怎样的特点?______________________________________________________________________________________________________________________________________________________________________________通感。“笑响”是听觉,“点亮”描述的却是视觉,既生动形象地写出了风的轻柔,营造出光亮明艳的氛围,又借助声音引向春光,写出了人间四月天的生机明媚。诗人从视觉和听觉描述了春天,燕子的呢喃声与满树繁华构成了一幅有声有色的春天画卷,生动地展现了人间四月天万物勃发、温暖美好的特点。 6.诗歌以“我说你是人间的四月天”开头,以“你是人间的四月天”结束,这样写在结构和内容上分别有什么作用?(4分)__________________________________________________________________________________________________________________________________________________________________________________________________________________________________________________________________________________________________结构上:首尾呼应,使行文结构更加严谨,主题更加突出。内容上:开头直接提出主题,但“我说”二字表明诗人的态度是含蓄而矜持的,而最后诗人直接肯定地说“你是人间的四月天”,感情真挚而炽烈,反复的修辞突出强化了诗人对爱的赞颂。_____ 散文阅读技法专练——文章精美语句的赏析四月醉芳菲①正应了那句“忽如一夜春风来,千树万树梨花开”的古诗,仿佛在一日之间,那被凛冽的寒风吹得只剩下枯枝和荒芜的小城大地,眨眼之间便在四月的一天桃红柳绿、春色阑珊了。②四月天,人间芳菲开遍。所有的日子,都被一种肆无忌惮的明媚所笼罩,仿佛一切都像琥珀一般,闪着别样的光鲜,润出别样的色泽。路边的迎春花是一树树开的,黄灿灿的花儿一下压住了树枝的全部,稀疏的地方,也是满枝条的绿芽。轻风吹来,黄绿相间着一种陶醉,映亮了心灵。园里的垂柳挥舞着柔软的纤手,似乎在微微地向林荫道上的人们问好致意,草坪也在一夜之间布满了嫩绿色,用恬淡清新诉说着季节的生动。我站在柏油路上,仰头感受暖暖的春日,一瞬间,如同被什么所击中,或被攫住了似的,只感到流光中的迷离与幻动,令人无可抗拒地几乎融入了和煦的暖风、游离的暗香以及不息的声色中。 ③四月的芳菲使人醉呵!④一时间,缤纷的思绪像杏花雨般落下……⑤想去踏青。踩着油菜金黄、绿草青青的田垄,寻找一些儿时的记忆,让自己从纷杂和困惑里释然,深深地呼吸,大声地呼喊,自在地和自然做个亲密的接触,做最放纵的自我。然后采一束野花带回家,放在窗台那个已经空了很久的花瓶里,把春天收藏在心里,把芬芳留在属于自己空间里的四月。⑥想去远行。想去“梨花一枝春带雨”的江南,感受烟雨霏霏;着一身丝绸的长裙,寻一些天上人间的浪漫;和着踏歌的节奏,赏大街小巷细致的风情;于亭台楼阁里觅一些让心悸动的情感,使喧嚣中有些浮躁的心静下来,思索,再思索,坦然面对生活中的酸甜苦辣。 ⑦想去耕种。踏着四月泥土的松软,架一柄木犁,以蹒跚的姿势,把种子播在春梦的端头;担一把锄头,擦掉额头的汗水,在古诗平平仄仄中,深谙着“粒粒皆辛苦”的哲理;握一把牛鞭,赶一架老车,轧着车轱辘菜的新绿,吱吱扭扭地奏出岁月的向往;或者穿透四月薄薄的雨雾,听蛙鸣声声,如一首温馨的歌谣,唱响内心深处清脆的乡音。⑧四月,融融的春光,无处躲藏的明媚饮醉了山水,也酣畅了爱情。人间四月天,在林徽因笔下是一曲爱的礼赞:“你是一树一树的花开,是燕/在梁间呢喃,——你是爱,是暖,/是希望,你是人间的四月天!”以几近阳光的心情去听这首美妙的抒情诗吧,你会感觉那一树花开就是一片林的爱,那一声燕呢就是一首爱的乐章。你会不经意间想起某个人,曾经擦肩而过,却成为永远的风景,永远的记忆。 ⑨四月,多少缱绻的情怀,摩挲着春光流溢的感触。将思绪放飞,随影而行,随风遁远。四月,一半是醉人的芳菲,一半是疯长的憧憬。(选自黑龙江大庆中考语文试卷,有删改)7.文章综合运用了哪些表达方式?有什么表达效果?(4分)____________________________________________________________________________________________________________________文章综合运用了描写、抒情等表达方式。描写了四月明媚的春光,抒发了作者对春天的喜爱和赞美,对美好人生的向往和憧憬。 8.根据要求赏析下列句子。(4分)(1)“草坪也在一夜之间布满了嫩绿色,用恬淡清新诉说着季节的生动”一句运用了什么修辞方法?有什么作用?______________________________________________________________________________________________________________________________________________________________________________(2)“四月,一半是醉人的芳菲,一半是疯长的憧憬。”结合加点词语理解句子的内涵。______________________________________________________________________________________________________________________________________________________________________________运用拟人(夸张)的修辞方法。“一夜之间”“嫩绿”“诉说”等生动形象地写出了草坪的春意盎然和恬淡清新,表现了春天的美好和生动,表达了作者的陶醉之情。美丽的春天既使人陶醉,又使人对未来充满希望。 9.第③④两段都是一句单独成段,在结构上起什么作用?(4分)__________________________________________________________________________________________________________________________________________________________________________________________________________________________________________________________________________________________________10.阅读文章第⑤~⑦段,简述作者有哪些思绪?(5分)______________________________________________________________________________________________________________________________________________________________________________第③段“四月的芳菲使人醉呵”一句总结上文,强调了春天百花盛开使人陶醉的美好景象;第④段“一时间,缤纷的思绪像杏花雨般落下”一句自然地引出下文,单独成段,使得文章结构清晰,层次分明。回忆过去,让自己从纷杂和困惑里释然;向往远方,让浮躁的心情宁静;播种梦想,畅想奋斗的乐章。 中考考点实训名著阅读模拟训练旷野(节选)薄雾在迷蒙着旷野啊……看不见远方——/看不见往日在晴空下的/天边的松林,/和在松林后面的/迎着阳光发闪的白垩岩了;/前面只隐现着/一条渐渐模糊的/灰黄而曲折的道路,/和道路两旁的/乌暗而枯干的田亩……田亩已荒芜了——/狼藉着犁翻了的土块,/与枯死的野草,/与杂在野草里的/腐烂了的禾根;/在广大的灰白里呈露出的/到处是一片土黄,暗赭,/与焦茶的颜色的混合啊……/——只有几畦萝卜,莱蔬/以披着白霜的/稀疏的绿色,点缀着/这平凡,单调,简陋/与卑微的田野。 那些池沼毗连着,/为了久早/积水快要枯涸了;/不透明的白光里/弯曲着几条淡褐色的/不整齐的堤岸;/往日翠茂的/水草和荷叶/早已沉淀在水底了;/留下的一些/枯萎而弯曲的枝杆,/呆然站立在/从池面徐缓地升起的水蒸汽里……山坡横陈在前面,/路转上了山坡,/并且随着它的起伏/而向下面的疏林隐没……/山坡上,/灰黄的道路的两旁,感到阴暗而忧虑的/只是一些散乱的墓堆,/和快要被湮埋了的/黑色的石碑啊。一切都这样地/静止,寒冷,而显得寂寞……(节选自《艾青诗选》) 1.诗歌的开篇以“薄雾在迷蒙着旷野啊”一句作为第一节,这样写有什么作用?(3分)______________________________________________________________________________________________________________________________________________________________________________2.诗歌第二节对旷野从远到近地眺望,远近之景在意境上有何不同?(3分)______________________________________________________________________________________________________________________________________________________________________________3.诗歌的第六节在内容和结构上各有什么作用?(4分)______________________________________________________________________________________________________________________________________________________________________________“薄雾在迷蒙着旷野啊”一句以写实隐喻,奠定了这首诗的整体性意境;而在句尾加上“啊”字变为感叹句,奠定了这首长诗的抒情基调。远眺旷野的怡人景色,是一个新鲜而明朗的场景,而近景却是灰黄而衰败的景象,一明一暗,形成强烈的对比,同时这也是欲扬先抑的写法。内容上,是叙述者对前面写生式的旷野做客观展示的概括;结构上,以叙述者对旷野“静止,寒冷,而显得寂寞”的主观评价作为第一部分的结束。 2019秋部编版九年级语文上册第一单元第五课我看 名师导学5我看一、课前自学知识储备1.文章的表达方式:记叙、描写、抒情、议论、说明。2.抒情:抒发和表现作者的感情,它是抒情文体的主要表达方式;在一般的文学作品和记叙文中,也常常作为重要的辅助表达方式。抒情的方式有两种,一种是直接抒情,即直接抒发作者内心的情感;一种是间接抒情,即通过记叙、描写等方式来抒情。●上册● 一课一法体会课文抒情的表达效果:(1)课文采用了大量的感叹句直接抒情。①“哦,逝去的多少欢乐和忧戚,/我枉然在你的心胸里描画!”②“哦!多少年来你丰润的生命/永在寂静的谐奏里勃发。”③“哦,让我的呼吸与自然合流!”(2)课文通过描写“生命的飞奔”“鸟的歌唱”“云的流盼”“树的摇曳”等充满热烈情感的意象,间接抒发了诗人对生命的热爱,对自由的憧憬。 二、内容梳理(根据课文填空,理清文脉)《我看》是一首优美流畅的生命之歌,既客观描述了具有蓬勃生命力的春意,又抒发了诗人主观的感受。全诗将热烈的意象、主观的激情和理性的思维交织融合为一体,使人们感受到强烈的美,也激发了人们对生命更深刻的体验。全诗共五节: 我看第1~2节:描写客观的春意选取意象:画面特点:第3~5节:抒写主观的感受①_____________________________________________________________生机蓬勃春风、春草、远水、绿潮、飞鸟、晴空、流云、大地②___________、宁静悠远赞美大自然永恒的美抒发热爱生命、向往自由之情感悟到生命的流变是自然规律 名师导练基础训练1.根据拼音写出相应的词语。(1)哦!多少年来你fēngrùn( )的生命/永在寂静的谐奏里bófā( )。(2)我看流云慢慢地红晕/无意chénzuì( )了凝望它的大地。(3)哦,逝去的多少欢乐和yōuqī( ),/我枉然在你的心胸里描画!(4)如今却只见他生命的静流/随着季节的起伏而piāoyì( )。丰润勃发沉醉忧戚飘逸 (5)去吧,去吧,哦生命的飞奔,/叫天风挽你坦荡地mànyóu( )。(6)像鸟的歌唱,云的liúpàn( ),树的yáoyè( )。漫游流盼摇曳 2.下列句子中加点的词语使用不恰当的一项是()A.我爱自然,因为春天的生机盎然,因为夏天的绿意勃发,因为秋天的安宁沉静,因为冬天的万物萌生。B.当暴风雨来袭时,林海枝舞叶涌,俯仰起伏,万千树干就是万千根摇曳的琴弦,弹奏出惊心动魄的乐曲。C.为了顺利通过验收,大家领取任务,个个斗志昂扬,分道扬镳,各干各的事情去了。D.很多教师和学生都有这样的经验和体会,在考试前一定要保持轻松的心态,采用疲劳战术和题海技术只能事倍功半。C 3.下列对病句的修改不正确的一项是()A.节能环保型社会里,人们的低碳意识正在逐步增大。(把“逐步”改为“逐渐”)B.学习成绩提高,主要取决于学生自身是否努力。(在“提高”前加“能否”)C.防止不再发生类似的伤害事故,政教处采取了很多安全措施。(删去“不”)D.这种新发现的流行病,促使我们改正并认识了自己不良的卫生习惯。(把“改正”与“认识”调换位置)A 4.阅读下面的材料,根据要求作答。诺贝尔文学奖获得者秘鲁作家略萨的作品从社会层面讲,笔触相当广泛,拉丁美洲的土著居民、市镇市民、知识分子、艺术人物、官僚机构、军队士兵等,都是他的描述对象。他的小说敏感地揭露和批判社会现实问题:如独裁统治、官僚腐败现象、贫富差距悬殊、阶级压迫、种族歧视、军警特务横行、党派竞争等。略萨坚信,“小说需要介入政治”,这是让小说变得尖锐而有力的重要武器之一。略萨被誉为“拉美结构主义文学大师”,与墨西哥作家卡洛斯·富恩特斯、阿根廷作家利奥·科塔萨尔、哥伦比亚作家加西亚·马尔克斯并称为“拉丁美洲文学爆炸的四大王将”。与马尔克斯的《百年孤独》绮丽魔幻的叙述风格不同,略萨的小说更注重叙事结构的布局,经常会同时并行几条线索,然后逆转、反转,叙事技巧无与伦比。 (1)阅读材料,请概括出略萨获得诺贝尔文学奖的理由。____________________________________________________________________________________________________________________(2)根据材料内容,以“小说”为描述对象,用上一种修辞方法,写一句话。____________________________________________________________________________________________________________________他的小说揭露和批判现实,反对独裁,注重叙事结构的布局,叙事技巧无与伦比。小说是生活的收纳盒,容纳人间悲苦,收藏生命喜乐;小说是社会的反射镜,照射人性丑陋,反射灵魂良善。 课内精读5.赏析课文中精妙的句子。(9分)(1)“我看它们低首又低首,/也许远水荡起了一片绿潮”一句运用了哪些修辞方法?有什么作用?__________________________________________________________(2)“我看飞鸟平展着翅翼/静静吸入深远的晴空里”,分析加点词语的表达效果。____________________________________________________________________________________________________________________运用拟人、反复的修辞方法,生动形象地写出了春草的蓬勃生机。“吸”字生动准确地展示了飞鸟跃动高飞的情景,既反衬了晴空的高远辽阔,又能表现出春天生机蓬勃的力量。 (3)“哦,让我的呼吸与自然合流!/让欢笑和哀愁洒向我心里,/像季节燃起花朵又把它吹熄。”如何理解句子的深刻含义?__________________________________________________________________________________________________________________________________________________________________________________________________________________________________________________________________________________________________6.研读第五节诗歌,说说诗人在这里因自然而激发出了怎样的愿望。(3分)__________________________________________________________诗人运用隐喻的修辞方法,含蓄地表达了对生命真谛的感悟,人应融入自然,肯定自然,返回自然,才能获得自身内心的安宁,所以洒向“我”心里的不仅有追求自由潇洒的“欢笑”,也有“哀愁”,正如花开花落一般自然。这一句不仅增加了诗的韵律和美感,更使我们对生命有了深刻、真切的体验。融入自然,追求自由快乐;在自然中感悟、寻找生命的真谛。 散文阅读技法专练——文章的表达方式(甲)上学的路上,有一条必经的小河。那年初春的一个早晨,当我匆匆忙忙赶往学校时,河上的冰还没融化,我三步并作两步冲上冰面。刚刚走到河中央,“扑哧”一声,一处冰面塌落了,我的两条腿踏进了冷彻骨髓的河水里。我穿着湿透的棉裤坐在教室里,全身都麻木了,我沉默地挺着,脸和唇都变成了青紫色。中午回到家,母亲很惊讶,却什么也没问,只是让我围着被褥坐在烧得热热的炕头上取暖,她蹲在火炉前为我烧火烤棉裤。炉火灼灼,不时爆出些响声来,而母亲却始终一言不发。当母亲把烘干的棉裤交给我时,我的眼圈有些红肿。日子一天天过去,高考迫在眉睫。有一夜下了暴雨,当我清早上学来到那条河边时,我发觉有什么人早在暴涨的河水里砌了一些大石头。那石头一块块稳稳地立在水里,石面高出了水面数厘米,远远看去就像一座简陋的石桥。我踏石而过的时候,心里淌过一股莫名的感激。那天晚上回到家我无意中看到母亲手上缠着白纱布,血 透出来,湿了一片,惊问她为什么受伤,母亲淡淡地答道:“不小心碰的。”站在一边的妹妹抢着告诉我:“妈妈的手是搬石头砸伤的。”“妈妈搬石头干什么?”我不解地追问。“搬石头搭桥呀!”那瞬间,我的心轰响着,久久说不出话来。那年9月,我踏过“母亲桥”去远方上大学。独在异乡,仍是母亲用无言的关注帮我度过每一次的疲惫和寂寞。渐渐地,我知道,是母亲用青春搭起我的生命之桥,用希望和奉献搭起我通向成功的幸福之桥。如今,河上的“母亲桥”已经为风雨所侵蚀残损,而在我的心灵中,“母亲桥”却永远踏不断。(节选自《母亲桥》,有删改) (乙)父母的爱是天地间最伟大的爱。一个孩子,自从呱呱坠地,父母就开始爱他,鞠之育之,不辞劬劳(生养他,教育他,不辞劳苦)。稍长,令之就学,督之课之,唯恐不逮(督促他,唯恐达不到目的)。及其成人,男有室,女有归,虽云大事已毕,父母之爱固未尝稍杀(不曾有丝毫的松懈或减少)。父母的爱没有终期,而且无时或弛(没有一时松弛过)。父母的爱也没有差别,看着自己的孩子牙牙学语,无论是伶牙俐齿或笨嘴糊腮,都觉得可爱。眉清目秀的可爱,浓眉大眼的也可爱;天真活泼的可爱,调皮捣蛋的也可爱;聪颖的可爱,笨拙的也可爱;像阶前的芝兰玉树固然可爱,癞痢头儿子也未尝不可爱,只要是自己生的……父母的爱是天生的,是自然的……是无条件的施与而不望报。父母子女之间的这一笔账是无从算起的。父母的鞠育之恩,子女想报也报不完。(节选自梁秋实的《父母的爱》,有删改) 7.甲文所记叙的中心事件是_________________________________________________________________________________________;这正是乙文中心句“___________________________”的具体表现。(4分)8.从表达方式看,甲文和乙文分别以哪种表达方式为主?(2分)_________________________________________________________9.甲文先后写了三次母亲对“我”的无言的关注,第一次是_____面描写,第二次是_____面描写,第三次是概括描写。乙文从三个方面来赞美“父母的爱”,这三个方面是(用原文语句填空):(1)____________________;(2)______________________;(3)_____________________。(5分)母亲在暴涨的河水中用石头为“我”搭了一座“母亲桥”父母的爱是天地间最伟大的爱甲文以记叙为主,乙文以抒情为主。正侧父母的爱没有终期父母的爱也没有差别父母的爱是天生的 10.理解甲文中画线句子的深刻含义。(4分)河上的“母亲桥”已经为风雨所侵蚀残损,而在我的心灵中,“母亲桥”却永远踏不断。____________________________________________________________________________________________________________________________________________________________________________________________________________________________________小河上的“母亲桥”是用石头搭的,所以容易被风雨侵蚀残破;而在“我”心灵上的“母亲桥”是用亲情搭的,所以牢不可摧,表现了母亲对“我”的深远影响,令“我”终生难忘,也表达了“我”对母亲的感激之情。 中考考点实训名著阅读模拟训练北方一天/那个科尔沁草原上的诗人/对我说:/“北方是悲哀的。”不错/北方是悲哀的。/从塞外吹来的/沙漠风,/已卷去北方的生命的绿色/与时日的光辉/——一片暗淡的灰黄/蒙上一层揭不开的沙雾;/那天边疾奔而至的呼啸/带来了恐怖/疯狂地/扫荡过大地;/荒漠的原野/冻结在十二月的寒风里,/村庄呀,山坡呀,河岸呀,/颓垣与荒冢呀/都披上了土色的忧郁……/孤单的行人,/上身俯前/用手遮住了脸颊,/在风沙里/困苦地呼吸/一步一步地/挣扎着前进……/几只驴子/——那有悲哀的眼和疲乏的耳朵的畜生,/载负了土地的/痛苦的重压,/它们厌倦的脚步/徐缓地踏过/北国的/修长而又寂寞的道路…… 那些小河早已枯干了/河底已画满了车撤,/北方的土地和人民/在渴求着/那滋润生命的流泉啊!/枯死的林木/与低矮的住房/稀疏地,阴郁地/散布在灰暗的天幕下;/天上,/看不见太阳,/只有那结成大队的雁群/惶乱的雁群/击着黑色的翅膀/叫出它们的不安与悲苦,/从这荒凉的地域逃亡/逃亡到/绿荫蔽天的南方去了……北方是悲哀的/而万里的黄河/汹涌着浑浊的波涛/给广大的北方/倾泻着灾难与不幸;/而年代的风霜/刻画着/广大的北方的/贫穷与饥饿啊。而我/——这来自南方的旅客,/却爱这悲哀的北国啊。/扑面的风沙/与入骨的冷气/决不曾使我咒诅;/我爱这悲哀的国土,/一片无垠的荒漠/也引起了我的崇敬/——我看见/我们的祖先/带领了羊群/吹着笳笛/沉浸在这大漠的黄昏里;/我们踏着的/古老的松软的黄土层里/埋有我们祖先的骸骨啊,/——这土地是他们所开垦/ 几千年了/他们曾在这里/和带给他们以打击的自然相搏斗,/他们为保卫土地/从不曾屈辱过一次,/他们死了/把土地遗留给我们——/我爱这悲哀的国土,/它的广大而瘦瘠的土地/带给我们以淳朴的言语/与宽阔的姿态,/我相信这言语与姿态/坚强地生活在大地上/永远不会灭亡;/我爱这悲哀的国土,古老的国土/——这国土/养育了为我所爱的/世界上最艰苦/与最古老的种族。(节选自《艾青诗选》) 1.这首诗主要描写了哪些意象?这些意象有什么特点和作用?(4分)____________________________________________________________________________________________________________________________________________________________________________________________________________________________________________________________________________________________________________________________________________________________2.诗人反复三次深情地抒发了“我爱这悲哀的国土”,这样写有什么作用?(3分)________________________________________________________________________________________________________________________________________________________________________________________________________________________________________意象:荒漠的原野、颓垣与荒冢、孤单的行人、悲哀的眼和疲乏的耳朵的畜生、干枯的小河、枯死的林木、低矮的住房、惶乱的雁群、浑浊的波涛、扑面的风沙等。特点及作用:诗人选取一系列冷色调的意象,营造了灰暗阴郁、死寂苍凉的氛围,塑造了痛苦疲乏的流民群体形象,刻画出北方的典型特征,传达出自己的悲哀之情。诗人反复强调和渲染,既突出了“悲哀”的现实,含蓄地抒发了诗人看到伤痕累累的祖国时深沉的悲哀和痛苦之情;而反复抒写的“爱”又让读者感到一种发自内心的、富有震撼力的声音,给人以力量。 3.简析诗歌在表达方式上的特点。(3分)____________________________________________________________________________________________________________________诗歌前三节极写北方的“悲哀”,最后一节转而直接抒发深爱之情。诗人在前后对比中传达出深沉的爱国之情。 2019秋部编版九年级语文上册第二单元第六课敬业与乐业 名师导学6敬业与乐业一、课前自学知识储备1.议论文的整体结构:引论(提出问题)——本论(分析问题)——结论(解决问题)。2.本论部分常见的论证结构:(1)“总—分—总”式结构:这是议论文运用最多的一种结构方式。在论证思路中,有的是先总说后分说,有的先分说后总说,或者先总说后分说再总说。这就是“总—分—总”式结构方式。(2)并列式结构:在论证思路中,将文章的中心论点分解成几个 平行的、并列的分论点,或是把论据并列起来,论证的几个层次或段落之间的关系是平行的,这就是并列式。(3)对照式结构:在论证思路中,把两种事物(或意见)加以对比,或者是用另一种事物(或意见)来烘托某一种事物(或意见),这就是对照式。(4)递进式结构:在论证思路中,几个观点之间由浅入深,层层深入,步步推进,这就是递进式结构。它的特点是各层次的前后顺序有严格要求。 一课一法理清课文的论证结构和思路:这篇演讲词从整体结构来看,采用了“总—分—总”的结构方式,按照“有业——敬业——乐业”的顺序展开论述,思路清晰,标志性词语的运用更使该议论文显得条理分明。(1)“先要说说”“第一”“第二”就分别领起“有业”“敬业”和“乐业”。(2)在论述“乐业”时,作者在提出“凡职业都是有趣味的,只要你肯继续做下去,趣味自然会发生”的观点后,理由也分“第一”“第二”“第三”“第四”分条陈述,有条有理,思路清晰。二、内容梳理(根据课文填空,理清文脉)《敬业与乐业》是一篇演讲词。演讲的对象是一群职业学校的学生,为的是向他们说明对职业应有“敬业”和“乐业”的态度,并说明如何培养“敬业乐业”的精神。文章的总体结构是“总—分—总”式,条理清晰,纲举目张。全文可分为三个部分: 敬业与乐业第一部分(第1段):提出论点——提出“①___________________________________________”的中心论点有业第三部分(第9段):总结全文‘敬业乐业’四个字,是人类生活的不二法门第二部分(第2~8段):分析问题孔子言论——②__________(论证方法)道理论证百丈禅师——③__________(论证方法)举例论证敬业:④_________——⑤______________——⑥_________(论证思路)含义为什么“敬”怎样“敬”乐业主观条件:⑦____________________________________客观条件:⑧____________________________________苦乐全在主观的心凡职业都是有趣味的⑨________即是责任心⑩________即是趣味敬业乐业强调中心 名师导练基础训练1.根据拼音写出相应的词语。(1)我这题目,是把《礼记》里头“敬业乐群”和《老子》里头“安其居乐其业”那两句话,duànzhāngqǔyì( )造出来的……但我确信“敬业乐业”四个字,是人类生活的bùèrfǎmén( )。(2)群居终日,yánbùjíyì( ),好行小慧,难矣哉!(3)这位言行相顾的老chánshī( ),老实不客气,那一天便绝对地不肯吃饭。断章取义不二法门言不及义禅师 (4)敬字为古圣贤教人做人最简易、直捷的法门,可惜被后来有些人说得太jīngwēi( ),倒变得不适实用了。(5)凡做一件事,便忠于一件事,将全副精力集中到这事上头,一点不pánɡwù( ),便是敬。(6)唯一的mìjué( )就是忠实,忠实从心理上发出来的便是敬。(7)大家同是替社会做事,你不必xiànmù( )我,我不必xiànmù( )你。(8)一个人对于自己的职业不敬,从学理方面说,便是xièdú( )职业之神圣。(9)我想天下第一等苦人,莫过于无业游民,终日闲游làngdàng( ),不知把自己的身子和心摆在哪里才好。精微旁骛秘诀羡慕羡慕亵渎浪荡 (10)专心做一职业时,把许多游思、wàngxiǎng( )dùjué( )了,省却无限闲烦恼。(11)我自己常常力求这两句话之实现与tiáohé( )。妄想杜绝调和 2.下列句子中加点的词语运用不恰当的一项是()A.大学不是幻想,不是梦想,更不是妄想,而是我们伟大的理想。只要我们为之奋斗,为之拼搏,总有一天我们会满载而归的!B.初冬的校园,无论向蓝白调和而成的天空凝望,还是朝衰弱无力的树木远眺,这被凄冷的风轻抚过的画面,总是抹上一层厚厚的凉意。C.老师只读了散文的一部分便说出了文章的主旨,这种断章取义的能力赢得了同学们的热烈掌声。D.固定时间的睡眠有助于规律生活习惯的养成,让孩子调节自我生理时钟是营造优质睡眠的不二法门。C 3.下列对病句的修改不正确的一项是()A.时至今天,王阳明的思想还在继续支配着中国读书人的一些头脑。(把“一些”放在“中国读书人”前面)B.为了更好地加强未成年人的思想教育,我们一定要做好引导学生学习先进人物。(在句末加“的工作”)C.它使你从来没有如此鲜明地感受到生命的活跃、强盛和存在。(把“活跃”与“强盛”调换位置)D.能否营造人人敬业奉献的浓厚氛围,关键是提升公民的道德修养。(删去“能否”)C 4.仿照画线句子,在后面补写一句话,使整段话语意连贯。梦想潜伏在我们每个人的心底,它给了我们划破天空的翅膀,让我们在广袤的长空中翱翔;_______________________________,___________________________________。它给了我们驰骋草原的骏马让我们在宽广的牧场上执鞭狂奔 课内精读5.从论证方法的角度赏析课文中精妙的句子。(9分)(1)没有职业的懒人,简直是社会上的蛀米虫,简直是“掠夺别人勤劳结果”的盗贼。______________________________________________________________________________________________________________________________________________________________________________(2)《庄子》记佝偻丈人承蜩的故事,说道:“虽天地之大,万物之多,而唯吾蜩翼之知。”____________________________________________________________________________________________________________________比喻论证。用“蛀米虫”和“盗贼”的比喻,形象地表现了作者对“没有职业的懒人”彻底讨伐的态度,也从侧面说明了“有业”之必要。举例论证。举出佝偻丈人承蜩的故事,具体典型地论述了怎样敬业,强调了做事要专心的观点,通俗易懂,使听众易于接受。 (3)曾文正说:“坐这山,望那山,一事无成。”……一个人对于自己的职业不敬,从学理方面说,便是亵渎职业之神圣。__________________________________________________________________________________________________________________6.阅读课文第6~7段,简析作者是如何层层深入地论证“敬业的重要性”的。(6分)_____________________________________________________________________________________________________________________________________________________________________________________________________________________________________________________________________________________________道理论证。引用曾文正的话论述了敬业的重要性,增强了文章的说服力。①首先解释什么是“敬”。引用朱熹的“主一无适便是敬”,解释“敬业”的含义就是专心致志、心无旁骛;②接着,用设问句提出“业有什么可敬呢”“为什么该敬呢”;③最后用设问句“怎样才能把一种劳作做到圆满呢”过渡到阐述敬业的做法——“唯一的秘诀就是忠实”“是敬”。 议论文阅读技法专练——理清论证思路古今中外,凡是获得重大成就的人,都具有坚持精神。无产阶级革命导师马克思为了撰写《资本论》,不顾疾病折磨、生活窘迫和反动势力的迫害,坚持写作40年,终于完成了这部辉煌巨著。数学家陈景润在攻克“哥德巴赫猜想”这个数学堡垒的过程中,不怕讽刺挖苦,忍受着疾病的折磨,在工作条件很差的情况下,夜以继日地学习、钻研,仅他运算过的稿纸就有几麻袋,有个英国数学家称赞他在数学上“移动了群山”。全国劳动模范、北京市百货大楼营业员张秉贵,热情地接待顾客,26年如一日,没有红过一次脸,吵过一次嘴。由此可见,坚持是取得一切成就的必由之路。(节选自荆幸连的《要有坚持精神》,有删改) 7.请简析文段的论证思路。(6分)(1)首先,作者表明观点:_________________________________________________________________________________________。(2)然后,列举______________________________________________________________________________________的事例来证明观点。(3)最后得出结论:_____________________________________。8.选段主要采用了什么论证方法?有什么作用?(3分)______________________________________________________________________________________________________________________________________________________________________________古今中外,凡是获得重大成就的人,都具有坚持精神马克思著作《资本论》,陈景润攻克“哥德巴赫猜想”和张秉贵26年如一日热情地接待顾客坚持是取得一切成就的必由之路选段主要采用了举例论证的方法。具体典型地论证了“古今中外,凡是获得重大成就的人,都具有坚持精神”这一观点,增强了文章的说服力。 个人应该懂得畏惧。孔子说:“君子有三畏:畏天命,畏大人,畏圣人之言。”天命就是老天爷赐予的命运;大人是指有地位、有号召力的人;圣人之言,是指古往今来那些道德高尚的人所发表的言论。孔子所说的“君子三畏”是否适用于今天的社会生活,我们姑且不去讨论,但对于他“人要知畏惧”的主张,我双手赞成。在现代社会,我觉得人应该“怕”的至少有两种:一是应该“怕”法律,就是要遵守基本的法律与规章制度,用古人的话说,叫“畏法度”;二是应该“怕”道德,也就是那些虽不违法,但明显违背社会道德、会被人戳脊梁骨的事绝对不做。那些因为图一己之欲、逞一时之快而无视法律和道德的人,当他们为自己的“无所畏惧”痛悔不已的时候,不也正是用自己的亲身经历为我们敲响警钟吗?(节选自游字明的《君子知“怕”》,有删改) 9.请简析文段的论证思路。(6分)(1)首先,作者提出“_______________________”的观点。(2)然后,____________________________,从正面论证“人要知畏惧”,接着阐述了“怕”的内涵。(3)最后,____________________________________,从反面论证“个人应该懂得畏惧”的道理。10.选文中的画线句子采用了什么论证方法?有什么作用?(3分)__________________________________________________________________________________________________________________个人应该懂得畏惧引用孔子的名言作为道理论据举出一些人因无所畏惧而痛悔不已的例子道理论证。引用孔子的名言作为道理论据,具体有力地论证了“个人应该懂得畏惧”的道理,同时,自然地引出下文的论述。 中考考点实训议论文阅读模拟训练敬业·职业·精业唐宋①一位朋友出差归来,对当地人的敬业精神和职业水准印象深刻,留下了美好的记忆。②我国古代思想家朱熹说:“敬业者,专心致志以事其业也。”认认真真、尽职尽责的敬业精神,是职业精神的首要内涵,是职业道德和优秀品格的集中体现。作为一种文化精神,敬业精神不仅是通向职场的“绿卡”,更是民族素质的重要内涵。③马马虎虎不能强国富民。早在20世纪早期,鲁迅先生就一针见血地指出,中国四亿人生着一种病,那名称就是马马虎虎,不医 好这个病,是不能救中国的。胡适先生曾写过《差不多先生传》,批评当时的国人缺少认真的精神。有没有认真的工作态度,有没有敬业的精神,不仅关系到一个国家精神风貌的好坏,更关系到国家的强弱、民族的兴衰。④没有敬业精神建不成现代化。懒懒散散,工作就没有效率;粗枝大叶,容易造成失误、留下隐患。在这方面,我们有成功的经验,也有惨痛的教训。重大安全生产事故、重大食品、药品安全事故的背后,麻痹大意、失职渎职是重要的原因。⑤敬业精神是竞争力的重要基础,却不是竞争力的全部。在科技日新月异、一日千里的今天,我们不仅要敬业,还要专业、职业、精业,才能从尽职尽责跨越到尽善尽美,才能从优秀跨越到卓越。⑥在我们身边,大体有四类人。有的人既敬业又职业,是企业的核心人才、核心竞争力。有的人敬业不职业,这样的人吃苦耐劳、精神可嘉,遇到紧急情况招之能来,可是,来了却不一定能战。有 的人职业不敬业,虽然业务素质高、解决问题的能力很强,却三心二意、毛手毛脚,容易“大意失荆州”,一失足成千古恨。有的人既不敬业又不职业,“当一天和尚撞一天钟”,每天浑浑噩噩混日子,误己误人。⑦敬业不易,精业更难。梅兰芳在舞台上顾盼生辉、流光溢彩,可是很少有人知道,为了让眼神活起来,眼睛近视的他每天早晨放飞鸽子,极目苍穹,苦练眼功。邓亚萍打球快速凶狠,可是,很少有人知道,为了增强手腕的力量,身材娇小的她曾用铁拍子练球。成功没有捷径,辉煌的背后,是鲜为人知的努力和付出。⑧在职场上,既敬业又精业的人永远是供不应求的“抢手货”,既不敬业又不精业的人,常常成为被淘汰的“处理品”。同样,在国际舞台上,如果一个国家的人民既勤勤恳恳又有创新能力,就能引领科技、创造品牌,走在时代的前沿;如果一个国家的人民对工作敷衍了事,就很难拿出有品质、有创意、有科技 含量的产品,落伍是早晚的事。⑨不断强大的国家、民族,无不有着不断进取、精益求精的精神。在这个以实力说话的时代,在应对国际金融危机的今天,只有秉承敬业、职业、专业、精业的精神,才能迎接挑战,后来居上,中华民族伟大复兴的梦想才能变成现实。 1.下列与原文内容不相符的一项是()(3分)A.敬业精神的内涵具体指的是人们应该怀有恭敬之心,认认真真、尽职尽责地对待自己从事的职业。B.职业、精业可以帮助人们从尽职尽责跨越到尽善尽美,从优秀跨越到卓越。C.敬业精神是职业精神的首要内涵,是竞争力的重要基础;职业强调业务素质高、解决问题的能力强;精业既强调职业精神又强调业务素质。D.既敬业又职业的人才是企业的核心人才和核心竞争力。A 2.下列对文章的分析理解,不正确的一项是()(3分)A.文章的中心论点是“只有秉承敬业、职业、专业、精业的精神,才能迎接挑战,后来居上,中华民族伟大复兴的梦想才能变成现实”。B.选文第⑦段画线的句子是个过渡句,“敬业不易”是承接上文,而“精业更难”是引出下文。C.选文第②段引用朱熹的话,自然地引出文章论述的中心话题,起到统领全文的作用。D.选文第⑧段画线的句子运用正反对比论证,通过“抢手货”与“处理品”的对比,突出了“只有秉承敬业、职业、专业、精业的精神,才能迎接挑战”的观点。C 3.分析选文第⑦段的论证思路。(4分)____________________________________________________________________________________________________________________________________________________________________________________________________________________________________本段文字首先提出“敬业不易,精业更难”这一观点,接着举梅兰芳苦练眼功和邓亚萍铁拍练球的例子具体论证观点,最后强调成功、辉煌的背后是鲜为人知的努力和付出。 2019秋部编版九年级语文上册第二单元第七课就英法联军远征中国致巴特勒上尉的信 名师导学7就英法联军远征中国致巴特勒上尉的信一、课前自学知识储备1.书信的格式:一般包括称呼、问候语、正文、祝愿语、署名、日期六部分。2.反语的修辞方法:(1)反语:运用跟本意相反的词语来表达此意,却含有否定、讽刺以及嘲弄的意思,是一种带有强烈感情色彩的修辞方法。(2)反语的作用:多用在揭露、批判、讽刺等方面,产生极其尖 锐的讽刺意味。如课文中作者运用了一系列的反语如“漂亮”“丰功伟绩”“收获巨大”“文明”“全部赞誉”等词语来谴责英法联军的强盗行为,具有极其辛辣和尖锐的嘲讽意味。3.理清“概括——具体——概括”的写作方法:先概括介绍提出主题,然后围绕主题进行具体叙述,最后对上文进行总结的写作方法。采用这种方法安排材料,可以使文章层次清晰,主题鲜明。 一课一法体会课文中雨果描述圆明园的“概括——具体——概括”的写作方法:(1)概括:总体评价圆明园是“世界奇迹”。(2)具体:①首先与巴特农神庙比较,指出圆明园的艺术性质和成就;②其次具体讲述园中之物,依次介绍建筑材料、建筑、陈设、装饰、园林景观,以耗费劳动之巨作结。(3)概括:最后总述圆明园是属于全人类的亚洲文明杰作。二、内容梳理(根据课文填空,理清文脉)法国著名作家雨果就英法联军远征中国一事,愤怒谴责英法联军的强盗行为,愤怒谴责英法联军毁灭世界奇迹——圆明园的罪行,他深切同情中国所遭受的空前劫难,表现出对东方艺术、对亚洲文明、 就英法联军远征中国致巴特勒上尉的信第一部分(第1~3段):赞美圆明园总体评价圆明园是①__________对中华民族的充分尊重,这封信的内容包含两个部分:世界奇迹用②_______的修辞方法突出圆明园的艺术成就对比文明剪影第二部分(第4~10段):讽刺侵略者过渡句:“③__________________________”用④________的修辞方法谴责英法联军的强盗行径野蛮强盗这个奇迹已经消失了反语 名师导练基础训练1.根据拼音写出相应的词语。(1)而你很想知道,我对英法的这次胜利会给予多少zànyù( )。(2)请你想象有一座言语无法形容的建筑,某种huǎngruò( )月宫的建筑,这就是圆明园。(3)请你用大理石,用玉石,用青铜,用瓷器建造一个梦,用雪松做它的屋架,给它上上下下缀满宝石,披上chóuduàn( )……饰以liúli( ),饰以珐琅,饰以黄金,施以脂粉。(4)这是某种令人jīnghài( )而不知名的杰作,在不可名状的晨曦中依稀可见,宛如在欧洲文明的地平线上piējiàn赞誉恍若绸缎琉璃惊骇 ( )的亚洲文明的jiǎnyǐnɡ( )。(5)他们对圆明园进行了大规模的jiélüè( ),zānɡwù( )由两个胜利者均分。(6)从前他们对巴特农神庙怎么干,现在对圆明园也怎么干,不同的只是干得更彻底,更漂亮,以至于dàngránwúcún()。我们把欧洲所有大教堂的财宝加在一起,也许还抵不上东方这座了不起的fùlìtánghuáng()的博物馆。那儿不仅仅有艺术珍品,还有大堆的金银制品。fēnggōngwěijì()!收获巨大!(7)将受到历史zhìcái( )的这两个强盗,一个叫法兰西,另一个叫英吉利。瞥见剪影劫掠赃物荡然无存富丽堂皇丰功伟绩制裁 2.下列句子中加点的词语运用不恰当的一项是()A.时光的剪影里,暖了多少相遇,又惆怅了多少离别。B.院中有两棵梨树,等到月升中天,清光从树间筛洒而下,地上阴影斑斓,恍若波光点点,尤为幽绝。C.世界上一些知名的大厦由于采用了玻璃幕墙或不锈钢幕墙,金光闪烁,富丽堂皇,显示了建筑技术高度发达的时代特征,也赋予了建筑材料以崭新的含义。D.老天津卫古建筑荡然无存,志愿者呼吁将老城古建筑尽早纳入天津历史文化名城保护规划。D 3.下列句子中没有语病、句意明确的一项是()A.提供网络搜索服务的企业必须整改,是因为竞价排名机制影响了搜索结果的公正性是重要原因。B.历史和现实都告诉我们,青年一代有理想、有才能,国家才有前途,民族才有希望。C.随着近年来草原自驾的兴起,严重危害了草原的生态环境,正在成为草原不可承受之重。D.我原打算介绍王宾向张凯借阅《平凡的世界》,没想到他已经离开学校,跟父母回老家探亲去了。B 4.仿照画线句子,在后面仿写两个句子,使之构成排比。冰雪融化,浅草萌发,昭示了早春的来临;骄阳似火,绿树成荫,书写着盛夏的热烈;_________,__________,________________;__________,__________,____________________。大自然的一幅幅画卷,让我们心旷神怡,流连忘返。天高云淡黄叶满地点染出秋天的色彩雾凇沆砀寒梅怒放凝成了严冬的风骨 课内精读5.赏析课文中精妙的句子。(6分)(1)“他们手挽手,笑嘻嘻地回到欧洲”一句有什么特殊的表达效果?____________________________________________________________________________________________________________________(2)“这就是文明对野蛮所干的事情”一句运用了什么修辞方法?有什么作用?______________________________________________________________________________________________________________________________________________________________________________以嘲讽的形式形象地再现了强盗相互勾结的丑恶嘴脸和肆意掠夺的卑鄙行径,讽刺力强。反语。句中的“文明”“野蛮”等反语,正是强盗恬不知耻的狡辩口吻,有辛辣的讽刺意味,强烈谴责侵略者颠倒黑白、厚颜无耻的丑恶嘴脸。 6.就英法联军远征中国这件事,雨果是什么立场和态度?表达了作者什么样的思想感情和品格?(4分)_____________________________________________________________________________________________________________________________________________________________________________________________________________________________________________________________________________________________雨果站在人类的立场上,立场明确,态度坚定,对英法联军的罪恶行径进行了强烈谴责和辛辣讽刺。对被侵略者和被掠夺者表示巨大同情,饱含着深厚的人道主义精神,表现了雨果正直、公正、人道、是非分明和胸怀博大的品格。 议论文阅读技法专练——掌握“总—分—总”的论证结构①古诗文的底子要从小打下,更要常读常新。②古人云:“读书百遍,其义自见。”它的意义也正在于此。③就拿“但愿人长久,千里共婵娟”一句来说,小时候我们按照表面意思理解,或许只能明白这是苏轼借月亮表达对弟弟的思念之情。④但若干年后,当我们一个人身居异乡,又恰逢中秋之夜,我们抬头仰望圆月,心中再次涌现出这句词时,自然别有一番滋味,对词的意境和感情的理解也更深了一层。⑤所以,从小打下深厚的古诗文功底会终身受益。(节选自《练就古诗文的“童子功”》,有删改) 7.请用“/”为选段划分层次。(2分)①②③④⑤8.简析选段的论证结构。(4分)________________________________________________________________________________________________________________________________________________________________________________________________________________________________________选段采用了“总—分—总”的论证结构。首先指出“古诗文的底子要从小打下,更要常读常新”,接着举出解读“但愿人长久,千里共婵娟”的例子论证观点,最后总结上文,强调要从小打下古诗文的功底,呼应开头。// ①历史上的圣哲先贤,他们之所以能建功立业,名垂千古,原因就在于他们都具备博采众长的精神,敢于宣称“吾不知”。②孔子问学生子贡,你和颜回比,谁的能耐大?子贡回答说,颜回闻一而知十,我只是闻一知三。子贡慨叹“弗如也”,寥寥数字,掷地作金石声。③《史记·高祖本纪》载,刘邦在总结楚汉之争的得失时说:“夫运筹帷幄之中,决胜于千里之外,吾不如子房(张良);镇国家,扶百姓,给馈饷,不绝粮道,吾不如萧何;连百万之众,战必胜,攻必取,吾不如韩信。此三子者,皆人杰也,吾能用之,此吾所以取天下者也。”刘邦如此知人善任,得天下当然是势所必然了。④读《广师》,我们可以领教明末清初思想家顾炎武的类似高论:“艰苦力学,无师而成,吾不如李学中;险阻俱尝,与时 屈伸,吾不如路安卿;好学不倦,笃于朋友,吾不如王石山。”顾老夫子谦逊若此,被誉为“一代通儒”,令人诚服。⑤瞿秋白在29岁时被选为中共中央政治局负责人,在一次会议上他恳切地表白:“搞农运,我不如彭湃、毛泽东;搞工运,我不如苏兆征、邓中夏;搞军事,我不如叶挺、贺龙。”虚心谨慎,溢于言表,瞿秋白之所以为瞿秋白,基于此。(节选自《人贵敢说“吾不知”》,有删改) 9.请简析选文的结构特点。(4分)_____________________________________________________________________________________________________________________________________________________________________________________________________________________________________________________________________________________________10.选文第②~⑤段的顺序能否调换?为什么?(3分)__________________________________________________________________________________________________________________选文采用了“总—分”的论证结构。第①段总写,提出“历史上的圣哲先贤,他们之所以能建功立业,名垂千古,原因就在于他们都具备博采众长的精神,敢于宣称‘吾不知’”的观点;接着在第②~⑤段具体列举了古今圣哲先贤的具体事例来论证观点,增强说服力。不能调换。因为这四段的事例是按照从古到今的时间顺序排列的。 中考考点实训议论文阅读模拟训练人当有所畏惧郭庆晨①在对待“畏惧”的问题上,一直有两种说法,一种是“无所畏惧”,一种是“有所畏惧”。年轻的时候,听到的多是对无所畏惧的推崇,加之年轻气盛,便总有一种大无畏的劲头。待过了知天命之年,身上的锐气消减,有些事就不免畏首畏尾、怕这怕那。经历了这两种说法的打架,心里常常会困惑:是无所畏惧对,还是有所畏惧对?琢磨的结果是:人当有所畏惧。②为什么人当有所畏惧呢?因为人生在世不能没有理想和信念,为了追求和坚守自己的理想和信念,人就必须有所畏惧。孔 子就曾说过:“君子有三畏:畏天命,畏大人,畏圣人之言。”这里孔子指出君子畏惧的对象,其实也是他们崇信的对象。人们常说的“敬畏”,其实就是由敬而畏。如果没有信仰,没有崇敬,则很难生畏。③人有所畏惧才能有操守和原则,才能严于律己、堂堂做人,才能安身立命。东汉杨震升任东莱太守,上任途中经过昌邑。昌邑县令王密是杨震荐举的官员。闻知恩公到来,王密带十斤黄金于夜晚前往驿馆拜访杨震。杨震不受。王密以为他故作客气,说:“夜幕无知者。”杨震来气了,反驳道:“天知、地知、你知、我知,怎说无知?”从此,“四知”便传为佳话,流传至今。④无所畏惧者往往过高地估计自己的能力,因有恃无恐而栽跟头,而且会栽得很惨。《三国演义》中几个人物的命运就很典型。比如何进。东汉末年,汉灵帝驾崩,大将军何进贵为国舅,又是辅政大臣,可谓权倾天下。这时,有人提醒他十常侍要谋反。可何进 并不以为然,说:“吾掌天下之权,十常侍敢待如何?”结果怎么样?不久,何进就身首异处了,杀他的人正是十常侍。这是恃权而无恐。⑤吕布自恃勇武过人,一般人他都不放在眼里。他动不动就会说:“吾有画戟、赤兔马,有何惧哉!”可是,在白门楼,他的赤兔马和方天画戟相继被不满于他的部下偷走,他本人也因为失去了坐骑和武器成了曹操的阶下囚,最后被缢死。这是恃器而无恐。⑥还有那个“死读兵书”的马谡,把兵书上的“凭高视下,势如破竹”奉为教条,盲目地以为只要将兵马“置之死地”,就自然可以“而后生”了。结果,他虽将兵马“直至绝地”,却没能“后生”,并且落得个几乎全军覆没的结果。这是恃书而无恐。⑦经验证明,与无所畏惧、有恃无恐相伴的,往往是失败;相反,倒是有所敬畏、自知自律常常与成功为伍。这是因为,无 所敬畏的人不懂得人并不能主宰一切,需要敬重圣贤、敬重自然、敬重百姓的道理。既然违背了客观规律,这所谓的“无所畏惧”也就只是盲目的“无畏”,顶多也就是蛮勇罢了,这跟真正意义上的“大无畏精神”实在是相去甚远,不能同日而语。(选自《刊授党校》,有删改) 1.下列能准确概括文章中心论点的一项是( )(3分)A.人有所畏惧才能有操守和原则,才能严于律己、堂堂做人,才能安身立命。B.人当有所畏惧。C.有所敬畏、自知自律常常与成功为伍。D.人们常说的“敬畏”,其实就是由敬而畏。B 2.下列对文章的理解分析,不正确的一项是()(3分)A.文章第④~⑥段列举了“恃权而无恐”“恃器而无恐”“恃书而无恐”三种“有恃无恐”。B.文章第②段的画线句子是设问句,既能激发读者的思考,又自然地引出下文。C.文章主要采用举例论证,第③段举出杨震的例子是正面例子,第④段举出何进的例子是反面例子,从正反两方面论证了“人当有所畏惧”的观点,使得论证更全面有力。D.文章采用“总—分—总”的论证结构,第①段提出论点,第②~⑥段论证观点,最后一段总结全文,强调“人当有所畏惧”的中心论点。D 3.你认为“四知”的故事被“传为佳话,流传至今”的原因可能是什么?(4分)____________________________________________________________________________________________________________________________________________________________________________________________________________________________________示例:杨震在深夜无人知的情况下,能拒绝重金的诱惑,说出“四知”的道理,体现了他难能可贵的优秀品质,值得人们敬仰和学习。(或:从这件事中能看出,杨震有所畏惧,有操守、有原则,能严于律己,堂堂做人,这值得人们钦佩和学习) 2019秋部编版九年级语文上册第二单元第八课论教养 名师导学8*论教养一、课前自学知识储备叙议结合的写作手法:一篇议论文,只叙不议,则是论据的堆砌;只议不叙,又会流于空洞的说教。好的议论文,往往要做到叙议结合,即把叙事与议论有机地结合起来。(1)叙述的作用:叙写生活现象,观察细致,针对性强,为下文的议论提供基础,也使得文章行文方式活泼灵动,有理有据。(2)议论的作用:对于某个人或某件事做出一定的评价,主要是 为了突出表现议论对象的特点和作者的观点。(3)叙议结合的作用:作者在叙写生活中的例子后发表自己的看法,从而突出文章的观点,增强文章的说服力。一课一法体会课文论证方法的作用:(1)对比论证:先列举生活中无教养的例子,再谈有教养的表现,对比鲜明,具体有力地强调了真正的教养是无论何时何地都尊重别人这一观点。(2)举例论证:列举生活中大量生动的例子,针对性强,富有浓厚的生活气息,增强文章的说服力。 论教养第一部分(第1段):开门见山,引出论题——教养②_____论证、二、内容梳理(根据课文填空,理清文脉)第二部分(第2~12段):论述真正的教养第三部分(第13~18段):论述优雅风度列举有教养的表现——必定①________别人尊重举例列举无教养的表现——在外和在家的不同表现③______论证对比批驳人们对风度的错误认识反面论述养成风度的基础——不应该④_______他人妨碍正面总结养成风度的准则——尊重和智慧⑤_______论证、举例⑥_______论证对比 名师导练基础训练1.根据拼音写出相应的词语。(1)我不敢màorán( )提供有关教养的“处方”,因为我不认为自己是教养完美的典范。(2)假如一个男人跟朋友和熟人见面时bīnbīnyǒulǐ( ),可是在家里对妻子儿女动不动就dàfāléitíng( )——那就可以肯定他不是一个有教养的人。(3)他都不管不顾地打开收音机或者电视,并且把音量放得很大,或者suíxīnsuǒyù( )地高声说话——那么,可以断定这个人缺乏教养,而且他永远也不会把自己的子女培养成有hányǎnɡ( )的人。贸然彬彬有礼大发雷霆随心所欲涵养 (4)假如一个人喜欢跟妻子或者孩子们开玩笑,却不顾及他们的自尊心,尤其是当有外人在场的时候,还要yīyìgūxíng( ),恕我直言,这样的人简直愚蠢到了极点!(5)有教养的人待人处世绝不会zìchuīzìléi( )。(6)有句谚语说得好:国王的礼貌是kèshǒu( )时间。(7)在社会交往中,比如出门做客或者在家接待客人,在剧场,在工作场合,人究竟该如何zìchí( )?(8)遗憾的是,人们却很少从这些书中jíqǔ( )有益的见解。(9)有些人有一种错觉,似乎优雅风度就是jiǎoróuzàozuò( ),是出于无聊,是fùyōngfēngyǎ( ),是毫无意义的niǔniēzuòtài( )。一意孤行自吹自擂恪守自持汲取矫揉造作附庸风雅扭捏作态 (10)问题的关键究竟何在呢?要养成优雅风度应该zūnxún( )哪些准则?(11)搜集那些难以逐一熟记的关于行为举止的“道德zhēnyán( )”,是不是一件轻而易举的事情呢?(12)如果你懂得了这一点,再加上几分suíjīyìngbiàn( )的智慧,那么风度就会自动来到你的身边。遵循箴言随机应变 2.下列句子中加点的词语使用不恰当的一项是()A.优秀的语言艺术家,总是像海绵吸水似的,汲取大量的富有生命力的词语。B.坐落于庭院清阁的梅,总能穿过尘世喧嚣与繁华,自持一份冰清玉洁。C.行走在斗转星移的人生之旅,切勿一意孤行,也勿相信一切人,相信自己,也能听取别人的意见,这才是风雨人生路。D.东房房客是个饱学之士,为人谦和,气度不凡,真是个附庸风雅的谦谦君子。D 3.下列对病句的修改不正确的一项是()A.随着镰刀头羊的那声吼叫,使整个斑羚群迅速分成两拨,老年斑羚一拨,年轻斑羚一拨。(删去“随着”或“使”)B.大型文化传承节目《中国诗词大会》受到媒体的广泛关注,掀起了一股追捧传统文化。(把“关注”改为“报道”)C.今年四川企业“走出去”呈爆发式增长态势,其中投向“一带一路”沿线的项目就约占40%左右。(删去“约”或“左右”)D.是否认识到屏在建筑美中应起的作用,是做到造型轻巧、色彩协调、绘画富有诗意的前提。(删去“是否”)B 4.仿照画线句子,在后面补写一句话,使整段话语意连贯。如果说友谊是一棵常青树,那么,浇灌它的必定是出自心田的清泉;_______________________________,那么,______________________________________________。如果说友谊是一朵开不败的鲜花照耀它的必定是从心中升起的太阳 课内精读5.赏析下列句子中加点词语的表达效果。(6分)(1)恕我直言,这样的人简直愚蠢到了极点!____________________________________________________________________________________________________________________(2)一个有教养的人,必定从心里愿意尊重别人,也善于尊重别人。______________________________________________________________________________________________________________________________________________________________________________“简直”一词加重语气,从反面强调了在家里不尊重家人的行为是无教养的行为,表达了作者对此类行为的批判。“必定”一词表示肯定,强调了“有教养”和“尊重”之间的必然联系,突出“尊重”的重要性,突出作者的观点,加深读者的印象,增强了文章的说服力。 6.课文第3~12段主要运用了什么论证方法?有什么作用?(4分)______________________________________________________________________________________________________________________________________________________________________________________________________________________________________________________________________________________________________________________________________________________①举例论证,通过列举生活中大量生动细致的例子,具体阐述了“一个人是不是真正有教养,首先要看他在自己家里、在自己亲属之间的表现”这一观点,增强了文章的说服力;②对比论证,通过把无教养的表现和有教养的表现进行对比,突出了“教养的本质特点是无论何时何地,他都必定从心里愿意尊重别人,也善于尊重别人”这一观点。 议论文阅读技法专练——分析对比论证的作用①专门研究“穷人的哲学”的古先生说:“如果一个人一生能坚守一个行业或一个地方,相信他会有所成就,在某个领域成为德高望重之人。但是如果他不停地迁徙,或许会有更多的机会,人生会有更多的风景。很难说哪种方式更好,两种人会各有所获。但现代社会的开放性,使每个人都能面临更多的机会,如果你没有去尝试,你就不知道自己的潜能,不知道是否会有更好的前途。”②常言道:树挪死,人挪活。对候鸟式生存的人来说,尽管付出了动荡不安的代价,但他们却因此获得了许多发展机遇,有了更多成功的希望。而我们之中的大多数人,往往就是因为对动荡不安的生活方式怀有深深的恐惧,才错失了很多发展的机会。我们常常忘记自己也有一双“翅膀”,因而便老是在生活的“悬崖”面前惊恐万状,踌躇不前。(节选自《做一只生命的“候鸟”》,有删改) 7.选文第①段引用了名人名言做论据,这属于_________论证。(2分)8.选文第②段先从_______面论证,后从_______面论证,其作用是_______________________________________________________________________________,从而增强了论证的力量。(4分)道理正反从正反两个方面进行对比论证,突出论述了勇于候鸟式生存的人的勇敢和可贵 中考考点实训议论文阅读模拟训练人生因阅读而气象万千陈凌①每到世界读书日,这样一张照片所定格下的历史瞬间总会被人提起:1940年10月22日,英国遭受空袭,位于伦敦的荷兰屋图书馆也难以幸免,几乎被炸成废墟,墙壁倾颓,砖石满地,但有3名男子竟不顾敌机刚刚离去,在尚未倒塌的书架前翻捡书籍。战火的残酷与读书所展现的不屈意志,两相对比,不仅给人以强烈的视觉冲击,更给人以持久的感动。②或许,也会有人不解:战火纷飞,还不忘阅读,为的是什么?“很多人被生活的艰难折磨得心灵枯萎,但有书香滋润的灵 魂不会。”——不久前,在一家书店里看到的一则读者留言,或许可以作为解答。对于那3名男子而言,阅读绝非是暂时忘记战乱的“镇定剂”,而是勇敢面对现实,让精神再次振奋、让意志战胜泪水的方法和途径。③阅读何以有这样的伟力?一位作家曾指出,阅读实际上会给人以两种收获,一种是通过读书,知道自己原来不知道的东西;另一种是通过读书触发反思,知道自己本来就有的东西,并激活它。前者是知识,后者是智慧。越是走在人生的“三岔口”,越是面临困难和选择之时,阅读的力量就越能显示出来。一个人的努力往往会让自己有更多的选择,知识让我们理解每一项选择的意义,而智慧还能进一步让人知道,自己的选择是符合内心的,从而坚定前行的意志。④人生如果远离了阅读,就等于一间房子没有窗户。曾国藩一生饱览群书,文章笔力雄厚,就连梁启超都称赞他“可以入文苑 传”。然而,即便有如此深厚的功底,他依然认为自己有“三耻”,居第一位的,就是对天文和算学“毫无所知”。一物不知,便深以为耻,曾国藩之所以会有如此的认识,就因为他深知,“书籍并非是装点门面的饰品,而是精神的营养品”。⑤读书或许并不必然导向外在的成功,但它必然指向内在的丰沛。这些年,两季《中国诗词大会》的冠军,惊艳了无数人,一位是第二季的武亦姝,另一位是第三季的雷海为。令人动容的,并不仅仅是武亦姝2000多首诗词记忆量的才情,也不全是雷海为以外卖小哥身份成功逆袭的不凡,更是因为他们身上散发的那种“闲看花开花落,漫随云卷云舒”的从容淡定。这样一份独特的气质,正源于阅读的滋养。有句话说得好:“读书多了,容颜自然改变。许多时候,自己可能以为看过的许多书籍都成了过眼烟云,不复记忆,其实它们仍是潜在的,在谈吐上、在胸襟中、在气质里,当然也可能显露在生活和文字中。” ⑥有人以为,读书太多,容易脱离生活。其实不然,读书并不是要逃离这个世界,而是要让人拥有平视世界的眼睛,阅读本身就是为了更好地生活。正如哲人所言:“一个人和书籍接触得愈紧密,他便愈加深刻地感到生活的统一,因为他的人格复化了,他不仅用他自己的眼睛观察,而且用着无数心灵的眼睛观察。”⑦阅读既塑造丰盈的内心,又给人以丰富的生活,让人生气象万千。这便是阅读之于人生的独特意义。(选自《人民日报》2018年4月23日,有删改) 1.下列准确概括本文中心论点的一项是()(3分)A.阅读既塑造丰盈的内心,又给人以丰富的生活,让人生气象万千。B.阅读能使得人生具有独特意义。C.阅读是勇敢面对现实,让精神再次振奋、让意志战胜泪水的方法和途径。D.阅读能培养人们独特的气质,让人生气象万千。A 2.下列对本文信息理解不正确的一项是()(3分)A.选文第④⑤段对应的是第⑦段中的“阅读塑造丰盈的内心”这个分论点;而第⑥段对应的是“阅读给人以丰富的生活”这一分论点。它们属于一一对应的关系。B.选文第①段中的画线句子运用了对比论证的方法。C.第⑤段中武亦姝和雷海为身上那种从容淡定的独特气质就是“内在丰沛”的体现。D.选文从多角度论述了阅读的作用,尤其是第⑥段针对一般人可能有的认识进行论述,使论证更严密。B 3.下面一则材料能否证明本文的中心论点?请说明理由。(4分)二战期间,美国士兵往往因为战争间歇期间生活乏味而士气涣散、训练消极,长官为此头痛不已。在政府向军队提供了1.2亿本“军供版”图书后,士兵很快就恢复了士气,积极投入训练中。___________________________________________________________________________________________________________________________________________________________________________能。事例中士兵通过阅读丰富了精神生活,振奋了士气,具体论证了本文“阅读既塑造丰盈的内心,又给人以丰富的生活”的中心论点。 2019秋部编版九年级语文上册第二单元第九课精神的三间小屋 名师导学9*精神的三间小屋一、课前自学知识储备1.议论性散文的特点:用“散文”的笔法“发议论”(或:以阐述某个观点为中心)。从议论角度看,它是“议论文”,是要阐明一个论点;从笔法角度看,它是“散文”,它不像一般议论文注重理性和逻辑,它侧重的是形象的描绘和情感的抒发。2.议论性散文中表达方式的作用:(1)议论:作者对某个议论对象发表见解,以表明自己的观点和 态度。它的作用在于使文章鲜明、深刻,具有较强的哲理性和理论深度。(2)抒情:抒发和表现作者的感情。它的作用主要是反映社会生活的精神方面,便于读者了解作者的感情倾向。 一课一法体会“精神的三间小屋”的内涵,把握比喻论证的作用:本文主要采用了比喻论证的方式,增强了材料的形象性,使要阐述的道理浅显易懂。“精神的三间小屋”的实际含义分别是:(1)给爱留下足够的容量,使希望永恒在前。(2)使事业和人生,呈现缤纷和谐、相得益彰的局面。(3)完善自我,弘扬个性。二、内容梳理(根据课文填空,理清文脉)《精神的三间小屋》是一篇集议论、描写、抒情于一体的议论性散文。文章以三间小屋为载体,阐述了精神追求的内涵及其意义,激励人们关注自我心灵,提升精神境界,使人格得到升华,表现了关 精神的三间小屋第一部分(第1~6段):引出话题——对两句名言的感慨和思考注个性、关注自我、关注人的精神生活的思想,文章条理清晰,结构严谨,可分为三个部分:第二部分(第7~18段):分析如何建立精神小屋第一间:盛放着①__________的小屋爱和恨第二间:盛放着②__________的小屋事业第三间:安放着③__________的小屋自身第三部分(第19~20段):总结发表议论:美好人性,健康人格提出建议:④________________________________________________增修新舍,矗立精神大厦 名师导练基础训练1.根据拼音写出相应的词语。(1)面对那句“人的心灵应该比大地、海洋和天空都更为博大”的名言,人们往往会zìcánxínghuì( )……不知累积至那种guǎngmào( ),需如何jīzǎn( )每一粒泥土、每一朵浪花、每一朵云霓?(2)也许因为我们不过是小小的草民,即便怀有效仿的渴望,也总是kěwàng’érbùkějí( ),便以位卑kuānyòu( )了自己。(3)这些复杂对立的情感,林林总总,会将这间小屋挤得满满的,jiānbùróngfà( )。自惭形秽广袤积攒可望而不可即宽宥间不容发 (4)在某一个jīngētiěmǎ( )之夜,它们会无师自通,与天地呼应,铮铮作响。(5)假若爱比恨多,小屋就光明温暖,像一座金色池塘,有红色的鲤鱼yóuyì( ),那是你的大福气。(6)假如恨比爱多,小屋就凄风苦雨,愁云惨雾,你会精神悲戚压抑,xíngxiāogǔlì( )。(7)无论一生遭受多少kùn’è( )欺诈,请依然相信人类的光明大于暗影。(8)不要轻觑了事业对精神的rúyǎng( )或反之的腐蚀作用,它以深远的力度和广度,挟持着我们的精神,以成为它huīxià( )持久的人质。金戈铁马游弋形销骨立困厄濡养麾下 (9)这不但因为相宜的事业,并非像雨后的菌子一样俯拾即是,而且因为我们对自身的认识,也如chōusībōjiǎn( ),需要水落石出的流程。(10)否则,jiūzhànquècháo( ),lǐdàitáojiāng( ),那屋内必是鸡飞狗跳,不得安宁。(11)规划自己的职业生涯,使事业和人生呈现缤纷和谐、xiāngdéyìzhāng( )的局面,是第二间精神小屋坚固优雅的要决。(12)我们说出的话,无论声音多么嘹亮,都是别的喉咙dūnɑng( )过的。(13)它纸糊的墙壁化为huījìn( ),白雪的顶棚变作泥泞,露水的地面成了沼泽,江米纸的窗棂破裂,露出惨淡而真实的世界。抽丝剥茧鸠占鹊巢李代桃僵相得益彰嘟囔灰烬 2.下列句子中加点的词语运用不恰当的一项是()A.有一种号称“海中之虎”的虎鲸胃口大得令人惊骇。B.突然,那深邃辽远的宝石蓝的天空中,绽出了一团炽烈耀眼的火光,划出一条弧形的漂亮的轨道,拖曳着一条极灿烂的光束,恰似一条美丽的长翎,向着无穷的广袤天空悠然而逝。C.经过警方抽丝剥茧,历时五个月的辛苦工作,这场以招工招生为由进行诈骗的案件终于真相大白了。D.近年来,随着流域经济的快速发展,松花江污染问题日渐严重,因此恢复松花江的生态功能间不容发。D 3.下列对病句的修改不正确的一项是()A.中国科学院最近研究发现,喜马拉雅山冰川退缩,湖泊的面积扩张,冰湖溃决危险性增大,引起了研究者的广泛关注。(删去最后一句)B.成千上万的亚运志愿者都在忙碌着,他们在共同努力,完成举办一次令亚洲乃至全世界都瞩目的文明亚运的理想。(把“完成”改为“实现”)C.在这次考察活动中,我们了解了有关许多湖光岩形成的科学知识。(把“了解”改为“学习”)D.省政府要求各地要立足防大汛、抢大险、抗大灾,做到排查在前、排险在前、预警在前,确保群众的生命财产安全。(把“排查在前、排险在前、预警在前”改为“预警在前、排查在前、排险在前”)C 4.仿照画线句子,在后面补写一句话,使整段话语意连贯。我们把自己的头脑,变成他人思想汽车驰骋的高速公路,却不给自己的思维,留下一条细细的羊肠小道;我们把自己的头脑,变成一个搜罗娱乐新闻的集装箱,却不给自己的思想,留下一个小小的储物盒;______________________,______________________________________,__________________,_______________________。我们把自己的头脑变成一本收录中外艺术大成的作品集却不给自己的幻想留下一张薄薄的纸 课内精读5.赏析下列句子所蕴含的思想情感。(6分)(1)假若爱比恨多,小屋就光明温暖,像一座金色池塘,有红色的鲤鱼游弋,那是你的大福气。____________________________________________________________________________________________________________________(2)我们把自己的头脑变成他人思想汽车驰骋的高速公路,却不给自己的思维留下一条细细的羊肠小道。______________________________________________________________________________________________________________________________________________________________________________运用比喻的修辞方法,把有爱的小屋比作金色的池塘,生动形象地写出了小屋的美好和幸福,强调爱对人类生活的重要作用。运用比喻的修辞方法,从反面论述人应该保有自己的个性和思维,不能随波逐流,丢失自己,说理生动形象,通俗易懂,使得文章文采飞扬。 6.课文最后两段采用了什么表达方式?有什么作用?(4分)_____________________________________________________________________________________________________________________________________________________________________________________________________________________________________________________________________________________________课文最后两段主要采用议论的表达方式。既总结全文,指出建立精神的栖息地是我们的义务和权利,又深化主题;采用排比句表现作者对美好人性和健康人格的期盼,最后提出建议,希望大家建立辽阔的精神世界。 议论文阅读技法专练——体会说理方式的表达效果①一篇文章怎样才好看呢?先抛开内容不说,手法必须有变化。最常用的手法有描写、叙事、抒情、说理等。如就单项技巧而言,描写而不单调,叙事而不拖沓,抒情而不做作,说理而不枯燥,文章就算做好了。但更多时候是这些手法的综合使用。变化再多,基本的东西只有几样,概括说来就是:形、事、情、理、典五个要素,我们可以称之为“文章五诀”。②正如一根单弦也可以弹出一首乐曲,只跑只跳也可以组织一场体育比赛。但毕竟内容丰富、好听、好看的还是多种乐器的交响和各种项目都有的运动会。所以无论哪种文体,单靠一种手法就想动人,实在很难。一般只有五诀并用才能做成斑斓锦绣的五彩文章。试用这个公式来检验一下名家名文,无不灵验。范仲淹的《岳阳楼记》是一 篇“记”,但除用一两句小叙滕子京谪守修楼之事外,其余,“淫雨霏霏”和“春和景明”都是写形,“感极而悲”“其喜洋洋”是写情,而最后推出一句震彻千年的大理“先天下之忧而忧,后天下之乐而乐”。形、事、情、理,四诀都已用到,文章生动而有深意,早已超出记叙的范围。毛泽东的《为人民服务》从追悼会现场说起,是形;讲张思德烧炭,是事;沉痛哀悼,是情;为人民服务,是理;引司马迁的话,“或重于泰山,或轻于鸿毛”,是典。五诀俱全,如山立岸,沉稳雄健,生机勃勃。有人说马克思的文章难读,但是你看他在剖析劳动力被作为商品买卖的本质时,何等的生动透彻:“原来的货币占有者作为资本家,昂首前行;劳动力占有者作为他的工具尾随于后。一个笑容满面,雄心勃勃;一个战战兢兢,畏缩不前,像在市场上出卖了自己的皮一样,只有一个前途——让人家来鞣。”在这里,“形”字诀的运用,已不是一个单形,而是组合形了。可知,好文章是很少单用一诀一法,唱独角戏,奏独弦琴的。(节选自梁衡的《文章五诀》,有删改) 7.选文第②段在论证时,采用了什么写作结构?请做简要分析。(4分)______________________________________________________________________________________________________________________________________________________________________________________________________________________________________8.选文中的画线句子采用了哪种说理方式?有什么作用?(3分)____________________________________________________________________________________________________________________________________________________________________________________________________________________________________选文第②段用了“总—分—总”的论证结构。开始用几句话概括说明“只有五诀并用才能做成斑斓锦绣的五彩文章”,然后用三个事例进行论证,最后用“可知,好文章是很少单用一诀一法,唱独角戏,奏独弦琴的”进行总结。采用了比喻说理的方式。把运用五诀中的一诀比喻成用一根单弦弹一首乐曲,只跑只跳组织一场体育比赛,说明其单调。这种说理方式的使用,生动形象,易于让读者明白只用五诀中的一诀的弊端。 中考考点实训议论文阅读模拟训练收获精神生命的成长欧阳锡龙①小的时候,每年寒暑假,我和哥哥都会跟着父母去田里种庄稼。老两口一辈子都在土地上耕作,什么季节种什么作物,哪个季节收获哪样庄稼,样样门清。这也让我慢慢悟出一个道理:读书就如种庄稼,只有精耕细作,才能收获精神生命的成长。②播下良种,才能有好收成。种庄稼,首先就要选育良种。读书,同样需要有一种挑剔的眼光。优秀的书籍就像粒粒饱满健硕的种子,能在人的心灵原野植下美好,生根发芽;相反,将时间花在粗制滥造、内容廉价的书籍上,不仅于知识、德行的增长 无益,还容易让心灵受到污染,得不偿失。古人讲,读书欲精不欲博,用心欲专不欲杂。这里面的道理,不仅仅在于书需要“读透”,更在于书需要“选好”。③好的书籍,能将历史定格,变柔弱为坚强。“火把虽然下垂,火舌却一直向上燃烧”,这是我每次重读路遥《平凡的世界》的一个感受。它让我懂得,痛苦与幸福实际上是相对的。要想在精神上保持富足,就需要不断超越简单的物质追求,并多加审视在不完美状态下的生命意义与价值。曾有一段时间,我经历了一些挫折和失败,一度心灰意冷,但每每在空闲时读起这本书,浮躁的内心就能平静下来,而孙少平永不服输的那种拼搏精神和永不气馁的那股韧劲,也总能深深地感染我。合上书本,再反观现实,就能明白:生命中的每一次挫折,不也是铺通前行之路的基石吗?更何况,只是一时的挫折,与其埋怨,毋宁改变。一本好书,力量如斯。④“要想庄稼种得好,还得力气下得巧。”一分耕耘,一分收 获。只是站在田头观望别人播种,自己却不付出任何努力的人,在丰收时节也只能远远地观望他人的成果。想要从书中汲取智慧,就不能坐而空谈,更不能叶公好龙,而应该在书籍的“稻田”里,与文字和思想为伍,或读,或抄,或默,或思,或悟。就好比播种、除草、施肥、浇水等诸多农活,自始至终都付出汗水,才能在丰收时收获喜悦。⑤清代文学家袁枚在《随园诗话》中的一句话说得好:“善破其卷而取其神,非囫囵用其糟粕也。蚕食桑而所吐者丝,非桑也;蜂采花而所酿者蜜,非花也。”对于一本好书,如果只是生吞活剥、生搬硬套,而做不到理性反思、融会贯通,就很难有机会吸收书籍当中的营养,反倒可能让自己患上“厌食症”,损害独立思考的能力。⑥正如庄稼的成长会碰到风霜雨雪一样,阅读的过程,也往往会遭遇困难,尤其是在阅读经典的时候。当时的语言习惯、思 考方式,往往与现在有着极大差别,这样的“硬骨头”,要啃下来,绝非易事。比如,大家都知道《资本论》是马克思的重要著作,放在当下,也有很强的解释力,依然没有过时,但真正从头到尾细细读过的人,恐怕并不多。其实,越是难读,越见勇气和毅力。读好书、啃经典,就需要“不破楼兰终不还”的决心。要知道,经典是时代和民族文化的结晶,是精神源泉,读经典的“大书”虽难,却能让人终身受益。⑦“没有种不好的庄稼,只有不会种庄稼的农民。”细细品味父母种庄稼的朴素道理,我坚信,在书海耕耘,能让人遇见不同的风景,成就人生的品质与高度。(选自《人民日报》,有删改) 1.下列说法与文章内容不相符的一项是()(3分)A.种庄稼时,选种、辛勤耕耘、经历风霜雨雪等过程和读书的过程非常相似。B.好的书籍,会给人精神上的力量,如读《平凡的世界》,心灰意冷的“我”变得平静,明白了挫折是前行之路的基石。C.种庄稼需力气下得巧,正如读书也要巧下工夫,讲究方法,才能收获喜悦。D.阅读的过程一定会遭遇困难,这就需要勇气和毅力,读书虽难,却能让人受益终身。D 2.下列对文章的分析理解,不正确的一项是()(3分)A.文章开篇从生活经验写起,自然地引出文章的中心论点:“读书就如种庄稼,只有精耕细作,才能收获精神生命的成长。”B.文章在分析论证观点时采用了并列式的写作结构,从选书写到读书,条理清晰,结构严谨。C.文章第⑤段主要采用了道理论证,引用袁枚的话论证了读书需要思考和融会贯通,要巧下力气的观点,增强了文章的说服力。D.文章结尾与开头相呼应,使得文章内容更严谨,结构更完整。B 3.文章主要采用什么说理方式?请概述出种庄稼与读书的共同之处。(4分)____________________________________________________________________________________________________________________________________________________________________________________________________________________________________比喻说理。①种庄稼,首先就要选育良种,正如读书首先要选好书;②种庄稼,力气要下得巧才能有收获,正如读书要讲究方法,才能收获生命的成长;③庄稼的成长会碰到风霜雨雪,正如读书也往往会遭遇困难。 2019秋部编版九年级语文上册第三单元第十课岳阳楼记 名师导学10岳阳楼记一、课前自学知识储备“记”:古代的一种文体,主要是记载事物,并通过记事、记物、写景、记人来抒发作者的感情或见解,即借景抒情,托物言志。“记”在写法上可以分为两类:(1)一类是作者不直接说,或基本不表露自己的观点或感情状态,而是寓情理于事、景、物的描述之中,例如《桃花源记》。(2)另一类是由事、景、物的描述中,自然生发出情理,而常以“卒章显志”的方式表现出来。本文属于第二类。 文章技法1.品味本文将叙事、写景、抒情和议论融为一体的写作方式:本文由叙事入手交代写作缘由,再借景物描写抒发“迁客骚人”的悲喜情怀,巧妙地与“古仁人之心”做对比,自然地引出议论,抒发“先天下之忧而忧,后天下之乐而乐”的政治理想,揭示主旨。2.体会文章对比手法的作用:(1)写阴景,抒悲情;写晴景,抒喜情。一明一暗,一悲一喜,对比鲜明。(2)“迁客骚人”悲喜的个人情怀和“古仁人”“不以物喜,不以己悲”的对比,突出“古仁人”的博大胸襟,也为下文揭示主旨做铺垫。3.文章骈散结合,语言凝练,文质兼美,具有很强的艺术感染力。 二、内容梳理(用原文语句填空,理清文脉)《岳阳楼记》生动地描写了洞庭湖的雄伟景色及“迁客骚人”登楼观景所触发的“悲”“喜”之情;通过褒扬“古仁人之心”,贬抑了一般“迁客骚人”的“悲”“喜”观,从而抒发了作者“不以物喜,不以己悲”的博大胸襟和“先天下之忧而忧,后天下之乐而乐”的远大政治抱负。全文可分为三个部分: 岳阳楼记叙述(第1段):交代了重修岳阳楼的背景、盛况以及作记的缘由议论、抒情(第5段)描写、抒情(第2~4段)描写洞庭湖全景空间:“①_________________________________________________”衔远山,吞长江,浩浩汤汤,横无际涯时间:“②____________________”朝晖夕阴,气象万千写暗景抒悲情写景:抒情:“登斯楼也,则有③_________________________,满目萧然,感极而悲者矣”“淫雨”“阴风”“浊浪”“樯倾”“楫摧”“虎啸”“猿啼”去国怀乡,忧谗畏讥写明景抒喜情写景:“春和”“景明”“沙鸥”“锦鳞”“岸芷”“汀兰”“皓月”“浮光”“静影”“渔歌”抒情:“登斯楼也,则有④____________________,把酒临风,其喜洋洋者矣”心旷神怡,宠辱偕忘“古仁人”的胸怀:“⑤_______________________”不以物喜,不以己悲点明主旨:“⑥_______________________________”先天下之忧而忧,后天下之乐而乐 名师导练一、字词解释1.解释下列句子中加点词语的意思。(1)滕子京谪守巴陵郡__________(2)百废具兴__________________________(3)属予作文以记之_________________________(4)横无际涯_________(5)前人之述备矣__________(6)连月不开_______________(7)日星隐曜_______________贬官。同“俱”,全、皆。同“嘱”,嘱托。边际。详尽。指天气放晴。光芒。 (8)樯倾楫摧____________________(9)去国怀乡__________(10)春和景明__________(11)宠辱偕忘____________________(12)古仁人之心_______________________2.解释下列句子中加点的多义词。(1)和政通人和____________至若春和景明__________(2)记属予作文以记之__________岳阳楼记____________________倒下。折断。指国都。日光。荣耀。一起。古代品德高尚的人。和乐。和煦。记载。古代的一种文体。 (3)观予观夫巴陵胜状__________此则岳阳楼之大观也__________(4)空浊浪排空__________而或长烟一空_______________(5)极南极潇湘____________感极而悲者矣__________(6)薄薄暮冥冥__________如履薄冰_________________________(7)或而或长烟一空__________或异二者之为_________________________________(8)一而或长烟一空__________一碧万顷__________在洞庭一湖__________看。景象。天空。消散。至、到达。极点。迫近。厚的反义词,厚度小。有时。或许、也许,表示委婉语气。全。一片。整个。 3.文言词汇选择。(1)下列各组句子中,加点词语意思相同的一项是()A.登斯楼也/微斯人B.此乐何极/南极潇湘C.予观夫巴陵胜状/此则岳阳楼之大观也D.而或长烟一空/或异二者之为(2)下列各组句子中,加点词语意思相同的一项是()A.此则岳阳楼之大观也/叹为观止B.前人之述备矣/筹备会议C.长烟一空/焕然一新D.把酒临风/身临其境AC 二、句子翻译1.越明年,政通人和,百废具兴。____________________________________________________________________________________________________________________2.衔远山,吞长江,浩浩汤汤,横无际涯,朝晖夕阴,气象万千。____________________________________________________________________________________________________________________3.登斯楼也,则有去国怀乡,忧谗畏讥,满目萧然,感极而悲者矣。______________________________________________________________________________________________________________________________________________________________________________到了第二年,政事顺利,百姓和乐,各种荒废了的事业都兴办起来了。它连接着远方的山脉,吞吐着长江的水流,浩浩荡荡,宽阔无边,早晚阴晴明暗多变,景色的变化无穷无尽。(这时)登上这座楼啊,就会产生离开国都,怀念家乡,担心被说坏话,惧怕被批评指责的心情,(放眼望去)满眼萧条景象,一定会感伤到极点而悲痛万分。 4.而或长烟一空,皓月千里,浮光跃金,静影沉璧。____________________________________________________________________________________________________________________5.不以物喜,不以己悲。_________________________________________________________6.居庙堂之高则忧其民,处江湖之远则忧其君。____________________________________________________________________________________________________________________7.其必曰“先天下之忧而忧,后天下之乐而乐”乎!__________________________________________________________8.噫!微斯人,吾谁与归?__________________________________________________________有时大片烟雾完全消散,皎洁的月光一泻千里,浮动的光像跳动的金子,静静的月影像沉入水中的玉璧。不因外物和自己处境的变化而喜悲。在朝廷做官则为平民百姓忧虑,被贬谪到边远地区做地方官则替君主担忧。大概一定会说“在天下人忧之前先忧,在天下人乐之后才乐”吧!啊!如果没有这种人,我同谁一道呢? 三、内容理解1.内容理解默写。(1)范仲淹赞扬滕子京政绩的句子是:_____________,_____________。(2)概括洞庭湖全景的句子是:__________,__________,____________,___________,___________,____________。(3)分别写出与“以物喜”“以己悲”相照应的句子。“以物喜”:_____________,________________,______________,____________,________________;“以己悲”:_____________,________________,_____________,____________,______________________。政通人和百废具兴衔远山吞长江浩浩汤汤横无际涯朝晖夕阴气象万千登斯楼也则有心旷神怡宠辱偕忘把酒临风其喜洋洋者矣登斯楼也则有去国怀乡忧谗畏讥满目萧然感极而悲者矣 (4)描述“古仁人”博大胸襟的句子是:________________,_______________。(5)由“古仁人”的博大胸襟和高尚情操而得出的论断是:_______________________,________________________。(6)抒写作者有远大的政治抱负的句子是:___________________,_______________________。2.内容理解简答。(1)课文标题为《岳阳楼记》,内容上却大量描写洞庭湖的景色,试分析作者的用意。___________________________________________________________________________________________________________________________________________________________________________不以物喜不以己悲居庙堂之高则忧其民处江湖之远则忧其君先天下之忧而忧后天下之乐而乐因为前人对岳阳楼的描绘已经很详尽了,于是作者另辟蹊径以湖的景色来烘托岳阳楼;用湖的“浩浩汤汤”“气象万千”的景物特点引出后面的登楼之情,为抒发自己的政治抱负做铺垫。 (2)登楼者面对两种不同的景色产生了“悲”和“喜”的感情,作者写这一内容的目的是什么?______________________________________________________________________________________________________________________________________________________________________________(3)“微斯人,吾谁与归”是一个反问句,它表达了作者怎样的思想感情?____________________________________________________________________________________________________________________写登楼者“以物喜”“以己悲”的感情,意在与“古仁人之心”做对比,突出“古仁人”“不以物喜,不以己悲”的豁达胸怀和高尚情操。表达了作者希望与“古仁人”同道的思想感情,强烈而又含蓄地表达了作者的自勉之意及对朋友的期望之情。 (4)课文写出了作者怎样的精神境界?有何深远意义?__________________________________________________________________________________________________________________________________________________________________________________________________________________________________________________________________________________________________精神境界:表达出作者“不以物喜,不以己悲”的旷达胸襟和“先天下之忧而忧,后天下之乐而乐”的伟大抱负。深远意义:强调为人要有宽广的胸襟和崇高的人格,以天下为己任,吃苦在前,享乐在后,激励古往今来无数仁人志士忧国忧民,无私奉献。 3.综合分析选择。(1)下列对课文的分析理解,不正确的一项是()A.全文既有对湖光山色的描写,又有精辟深刻的议论,还有惆怅悲沉的抒情,将抒情、议论、写景自然地结合起来。B.“薄暮冥冥,虎啸猿啼”是实写之景,渲染出一种悲凉凄惨的氛围,烘托了登楼者的感伤之情。C.全文体现了范仲淹“以民为本”的思想,表现了他博大的胸襟。D.课文第2段开头从空间和时间两个角度来描写洞庭湖:空间上突出洞庭湖的广大,时间上突出洞庭湖早晚景色多变。B (2)下列对课文内容与写法的分析理解,不正确的一项是()A.作者先写岳阳楼的雄伟景观,再借景抒发不同的览物之情,卒章显志,点明主旨。B.“微斯人,吾谁与归?”此句既是慰勉滕子京,也体现了作者欲与“古仁人”同道的旷达胸襟。C.全文的中心句是:“不以物喜,不以己悲。”D.课文写景、抒情、议论相结合,由景抒情,由情入议。句式骈散结合,行文富于变化。C 中考考点实训课外文言文阅读模拟训练司马光爱书费衮至于启卷,必先几案洁净,藉以茵褥①,然后端坐看之,或欲行看,即承以方版,未尝敢空手捧之,非惟手汗渍及,亦虑触动其脑②。每至看竟一版即侧右手大指面衬其沿而覆以次指面捻而夹过,故得不至揉熟③其纸。每见汝辈多以指爪撮起,甚非吾意。今浮图、老氏④尤知尊敬其书,岂以吾儒反不如乎?汝当谨志之。【注释】①茵褥:被褥。②脑:书脑,即书脊。③揉熟:揉坏。④浮图、老氏:指佛教徒、道教徒。(节选自《凉溪漫志》,有删改) 1.下列各组句子中,加点词语意义和用法相同的一项是()(3分)A.至于启卷/不愤不启B.藉以茵褥/属予作文以记之C.未尝敢空手捧之/浊浪排空D.汝当谨志之/志大才疏2.请用三条“/”给文中画线的句子断句。(3分)每至看竟一版即侧右手大指面衬其沿而覆以次指面捻而夹过。3.选文是司马光训诫儿子的一段话,其中能体现出司马光对书有着怎样的态度呢?(3分)__________________________________________________________________________________________________________________B///①他担心书籍受到污损,体现出司马光对书的爱惜;②司马光的语言和动作体现出他对书的尊敬。 2019秋部编版九年级语文上册第三单元第十一课醉翁亭记 名师导学11醉翁亭记一、课前自学知识储备文言文特殊句式——判断句的辨别和理解:判断句是对事物的性质、情况、事物之间的关系做出肯定或否定判断的句子。(1)用“也”表判断:“望之蔚然而深秀者,琅琊也。”(《醉翁亭记》)(2)用“者”表判断:“隶而从者,崔氏二小生。”(《小石潭记》) (3)句末用“者也”表判断:“莲,花之君子者也。”(《爱莲说》)(4)用动词“是”表判断:“问今是何世,乃不知有汉。”(《桃花源记》)(5)用“即”“乃”“则”“皆”“本”“诚”“亦”“素”“必”“谓”等副词表示肯定判断:①“此则岳阳楼之大观也。”(《岳阳楼记》)②“环滁皆山也。”(《醉翁亭记》)③“是谓大同。”(《大道之行也》)(6)无标志判断句:文言文中的判断句有的是没有任何标志,直接由名词对名词做出判断。例如:“刘备天下枭雄。”(司马光《赤壁之战》)注意:判断句中出现在谓语前的“是”一般都不是判断词,而是指 示代词,做判断句的主语;而有些判断句中的“是”也并非都不表示判断,“是”在先秦古汉语中很少做判断词,汉朝以后做判断词的情况则开始增多。一课一法1.体会文章融抒情、写景和叙事多种表达方式于一体的写法:文章始终把山水风光的描写,游人活动的叙述以及个人感情的抒发紧密地交融在一起,而以抒情贯穿始终,多种表达方式相融合,创设了优美的意境。2.文章结构严谨,前有伏笔,后必照应:前有伏笔,后必照应是《醉翁亭记》结构安排的特点,如开篇“名之者谁?太守自谓也”呼应文尾“太守谓谁?庐陵欧阳修也”,首尾呼应,结构完整;而“夕阳在山,人影散乱”与前文的“暮而归”又恰成照应。文章处处注意呼应和照应,使得全文内在结构经纬分明,严谨周密。 二、内容梳理(用原文语句填空,理清文脉)《醉翁亭记》是一篇优美的抒情散文。以“乐”字作为贯穿全文的主线,生动地描写了醉翁亭的秀丽环境和自然风光,层层展开记叙了山水乐——四时乐——宴酣乐——禽鸟乐——游人乐——太守乐,勾勒出一幅太守与民同乐的画面,表现了作者随遇而安、与民同乐的旷达情怀。 醉翁亭记第1段:点出“乐”字亭的环境:环滁皆山——西南诸峰——琅琊——酿泉——醉翁亭(整体——局部)名字由来:(太守)“①________________________________________________”饮少辄醉,而年又最高,故自号曰醉翁也暗示主旨:“山水之乐,得之心而寓之酒也”(“醉”与“乐”)第2段:山水之乐山间一日:“②__________________________________________”日出而林霏开,云归而岩穴暝,晦明变化山间四时春景:“③____________________”野芳发而幽香夏景:“④____________________”佳木秀而繁阴秋景:“⑤____________________”风霜高洁冬景:“⑥____________________”水落而石出第3段:宴酣之乐滁人游:“歌”“休”“呼”“应”“往”“来”(动作欢快)太守宴:“溪鱼”“泉酒”“山肴”“野蔌”(食物富足)众宾欢:“射”“弈”“觥筹”“起坐”(活动热闹)太守醉:“苍颜白发,颓然乎其间者,太守醉也”(神态欢乐)第4段:太守之乐禽鸟之乐——游人之乐——太守之乐揭示主旨:“⑦_______________________________________”(“醉”与“乐”)醉能同其乐,醒能述以文者,太守也 名师导练一、字词解释1.解释下列句子中加点词语的意思。(1)蔚然而深秀者____________________(2)峰回路转_________________(3)有亭翼然临于泉上者________________(4)醉翁之意不在酒_________________(5)云归而岩穴暝______________(6)负者歌于途_______________(7)泉香而酒洌_______________茂盛的样子。曲折、回环。居高面下。意趣,情趣。昏暗。背着东西。清。 (8)杂然而前陈者__________________(9)游人去而禽鸟乐也____________(10)太守谓谁_____________2.解释下列句子中加点的多义词。(1)环环滁皆山也__________如鸣珮环____________(2)谓太守谓谁____________太守自谓也__________(3)秀望之蔚然而深秀者__________佳木秀而繁阴__________陈列,摆开。离开。为,是。环绕。玉环。为,是。命名。秀丽。茂盛。 (4)意醉翁之意不在酒____________________鸣之而不能通其意_______________(5)临有亭翼然临于泉上者_________________临溪而渔_____________3.文言词汇选择。(1)下列各组句子中,加点词语解释有误的一项是()A.环滁皆山也(环绕)/山肴野蔌(菜蔬)B.醉翁之意不在酒(意趣,情趣)/山水之乐,得之心而寓之酒也(寄托)C.野芳发而幽香(花)/酿泉为酒,泉香而酒洌(凉,冷)D.宴酣之乐,非丝非竹(尽兴地喝酒)/树林阴翳,鸣声上下(遮盖)意趣,情趣。意思。居高面下。靠近。C (2)下列各组句子中,加点词语意思相同的一项是()A.若夫日出而林霏开/云开雾散B.四时之景不同/春和景明C.醉翁之意不在酒/出其不意D.环滁皆山也/险象环生A 二、句子翻译1.峰回路转,有亭翼然临于泉上者,醉翁亭也。____________________________________________________________________________________________________________________2.醉翁之意不在酒,在乎山水之间也。__________________________________________________________3.山水之乐,得之心而寓之酒也。__________________________________________________________4.若夫日出而林霏开,云归而岩穴暝。__________________________________________________________5.野芳发而幽香,佳木秀而繁阴。__________________________________________________________山势回环,路也跟着转弯,有一座亭子,(亭角翘起)像鸟张开翅膀一样,高踞于泉水之上,这就是醉翁亭。醉翁的情趣不在于(喝)酒,而在于欣赏山水美景。欣赏山水的乐趣,领会于心间,寄托在酒上。像那太阳出来,树林里的雾气散开,云雾聚拢,山谷就显得昏暗了。野花开放,有一股清幽的香味,好的树木枝叶繁茂,形成浓密的绿荫。 三、内容理解1.内容理解默写。(1)为后文写景抒情定下了基调的句子是:___________________,_____________________。(2)描写山间朝暮之景的句子是:___________________,______________,____________,___________________。(3)描写山间四时之景的句子是:__________________,_______________,_____________,___________________,___________________。(4)文中与“饮少辄醉,而年又最高”相照应的句子是:___________,___________________,_________________。(5)文中点明全文主旨的句子是:_________________,__________________,_______________。醉翁之意不在酒在乎山水之间也若夫日出而林霏开云归而岩穴暝晦明变化者山间之朝暮也野芳发而幽香佳木秀而繁阴风霜高洁水落而石出者山间之四时也苍颜白发颓然乎其间者太守醉也醉能同其乐醒能述以文者太守也 2.内容理解简答。(1)文中极力渲染一个“乐”字,有什么作用?表达了作者怎样的思想感情?___________________________________________________________________________________________________________________________________________________________________________(2)请说说“太守之乐”的内涵。_____________________________________________________________________________________________________________________________________________________________________________________________________________________________________________________________________________________________用“乐”作为主线贯穿全篇,从山水、禽鸟、游人、宾客、太守五个方面来叙其乐,最后点出醉翁之乐,意在表现政治清明的社会现实,表达了作者随遇而安、与民同乐的旷达情怀。“太守之乐”归纳起来有三个方面:一是“山水之乐”,并因“四时之景不同,而乐亦无穷也”;二是“宴酣之乐”,近取“山肴野蔌”,佐酒“非丝非竹”,众人“起坐而喧哗”,太守颓然其间;三是“乐人之乐”,故作文以叙其事。三乐归一,都是“与民同乐”。 (3)文中不仅写了太守与众宾客的游乐,还写了滁人的游乐。这样写有什么作用?____________________________________________________________________________________________________________________(4)作者很注重动静的搭配,无论景与景、人与景、人与人,作者都注意以一动辅以一静,静极生动,动极生静,相辅相成,相映成趣。请简要分析下列句子的表达效果。①“峰回路转,有亭翼然临于泉上者,醉翁亭也。”结合“回”“转”“翼”三个词语简析句子的表达效果。______________________________________________________________________________________________________________________________________________________________________________为下文写“太守之乐”做铺垫,反映欧阳修治理有方,其管治的地区政通人和,人民安居乐业。“峰”和“路”都是静物,但作者用“回”和“转”化静为动,形象地表现了泉流傍着山径的曲折蜿蜒之状;“翼”本是个名词,作者却使它动化,把醉翁亭居高临下的飞动之势表现得栩栩如生。 ②“觥筹交错,起坐而喧哗者”与“苍颜白发,颓然乎其间者”分别叙写了哪两个场景?有什么表达效果?________________________________________________________________________________________________________________________________________________________________________________________________________________________________________“众宾欢”和“太守醉”。这两个场景,一动一静,这种动静相生相成之法,使画面既充满生机又不显得纷扰和杂乱,既极力渲染了“乐”,又暗示了文章的主题。 3.综合分析选择。(1)下列对文章的分析理解,不正确的一项是()A.《醉翁亭记》是一篇散文,文章写了两部分内容:第一部分,重点是写亭;第二部分,重点是写游。而贯穿全篇的却是一个“醉”字。B.文章融情于景,既讴歌了滁州优美山水风景,又抒发了作者对建设和平安定、与民同乐的理想社会的努力和向往。C.文章开篇从山落笔,由山写到泉,由泉写到亭,既介绍了醉翁亭周围的环境,又为写亭做了自然的铺垫。D.作者从多个角度写景,从山水相映之美、朝暮变化之美、四季变换之美等角度巧妙地创设了一个优美的意境,使散文的意境绘形绘声,山光、水色、人情、醉态并于一画,各具情致。A (2)下列对文章的分析理解,有误的一项是()A.作者以富有灵性的笔触,生动形象地描绘了琅琊山变幻无穷的美丽景色,赏山乐水的欢畅与惬意跃然纸上。B.文章写“滁人游”“太守宴”“众宾欢”“太守醉”“众宾归”均围绕“乐”字展开,既写出太守与民同乐,又表现了当时滁州的政治清明。C.游记着力写太守与滁人沉醉于山水之间的情状,意在表现作者因仕途不得意而沉迷山水的消极人生态度。D.文章骈散结合,句式整齐又富有变化,尤其是“也”和“而”字的灵活运用,使得语言具有回环往复的音韵美。C 中考考点实训课外文言文阅读模拟训练喜雨亭记(节选)苏轼亭以雨名,志喜也。古者有喜则以名物,示不忘也。周公得禾,以名其书;汉武得鼎,以名其年;叔孙胜狄,以名其子:其喜之大小不齐,其示不忘一也。余至扶风之明年,始治官舍,为亭于堂之北而凿池其南引流种木以为休息之所。是岁之春,雨麦于岐山之阳,其占为有年。既而弥月不雨,民方以为忧。越三月乙卯乃雨,甲子又雨,民以为未足;丁卯大雨,三日乃止。官吏相与庆于庭,商贾相与歌于市,农夫相与忭于野,忧者以乐,病者以愈,而吾亭适成。 于是举酒于亭上,以属客而告之曰:“五日不雨可乎?”曰:“五日不雨则无麦。”“十日不雨可乎?”曰:“十日不雨则无禾。”“无麦无禾,岁且荐饥,狱讼繁兴而盗贼滋炽。则吾与二三子,虽欲优游以乐于此亭,其可得耶?今天不遗斯民,始旱而赐之以雨,使吾与二三子,得相与优游而乐于此亭者,皆雨之赐也。其又可忘耶?”1.下列各组句子中,加点词语意思和用法不同的一项是()(3分)A.志喜也/处处志之B.古者有喜则以名物/名之者谁C.民方以为忧/先天下之忧而忧D.商贾相与歌于市/负者歌于途A 2.请用三条“/”给文中画线的句子断句。(3分)为亭于堂之北而凿池其南引流种木以为休息之所。3.文章写了一件什么喜事?表达了作者怎样的思想感情?(3分)__________________________________________________________________________________________________________________/久旱逢甘霖(天降喜雨)。表达了作者关心百姓疾苦,与民同乐的思想感情。// 2019秋部编版九年级语文上册第三单元第十二课湖心亭看雪 名师导学12*湖心亭看雪一、课前自学知识储备白描:中国画技法名,源于古代的“白画”。在文学创作上,白描作为一种表现手法,是指用最简练的笔墨,不加烘托,描绘出鲜明生动的形象。白描手法的作用:(1)白描用于写人,只要三言两语即可勾画出人物的外貌和神态,使读者如见其人。(2)白描用于写事,只要几笔就可使事态毕现,生动形象。 (3)白描用于写景,只要几笔就可勾勒出一幅鲜明的图画,使人如临其境。文章技法1.赏析课文的白描手法:文章采用白描手法写雪景,抓住景物的突出特征,“一痕”“一点”“一芥”“两三粒”简洁概括,表现出悠远脱俗的情味。2.体会课文中对比手法的作用:“天与云与山与水,上下一白”——极力渲染了空旷辽远的氛围;“长堤一痕、湖心亭一点、与余舟一芥、舟中人两三粒”——极力突出景物的渺小。从整体到局部的写景顺序,宇宙的空阔与人的渺小构成了强烈的对比,含蓄地表达了作者对于人生渺茫的深沉感慨。3.品味课文中叙事、描写、抒情的表达效果:(1)叙事:叙事是行文的线索,作者开头交代了看雪的时间、目 的地和天气状况,中间交代了游踪——“到亭上”“及下船”,并且记叙了湖心亭巧遇一事。(2)描写:描写雪后西湖的奇景,湖心亭巧遇中人物的神态、动作和语言描写。(3)抒情:湖心亭巧遇虽是叙事,但重在抒情。作者与金陵客因相遇而惊喜,因相处而畅快,因相别而伤感,更因天涯遇知音而喜悦。二、内容梳理(用原文语句填空,理清文脉)《湖心亭看雪》以精练的笔墨,记叙了作者在湖心亭看雪的经过,描绘了一幅幽静深远、洁白广阔的雪景图,记叙了湖心亭的奇遇,表达了幽远脱俗的闲情雅致和孤芳自赏的情怀,同时也寄托了人生渺茫的慨叹。文章可分为两个部分: 湖心亭看雪湖中雪景叙事:交代看雪的时间、地点和天气写景全景:“①_________________________________________________________”近景:“②___________________________________________________________________________________________”雾凇沆砀,天与云与山与水,上下一白湖上影子,惟长堤一痕、湖心亭一点、与余舟一芥、舟中人两三粒而已亭上的人叙事:相遇——相处——相别(志趣相投)抒情:大喜——畅快——伤感(释然) 名师导练一、字词解释1.解释下列句子中加点词语的意思。(1)大雪三日__________(2)湖中人鸟声俱绝_______________(3)余拏一小舟______________(4)雾凇沆砀____________________(5)焉得更有此人____________(6)余强饮三大白而别__________________________(7)客此__________(8)及下船____________下雪。消失。撑(船)。白汽弥漫的样子。哪能。古人罚酒时用的酒杯。客居。等到。 2.解释下列句子中加点的多义词。(1)一上下一白__________长堤一痕__________(2)更是日更定________________________更有痴似相公者__________(3)是是日更定__________是金陵人__________________(4)绝湖中人鸟声俱绝_______________与妻子邑人来此绝境_______________完全。数词。古代夜间的计时单位。还。这。判断动词。消失。与世隔绝。 3.文言词汇选择。(1)下列各组句子中,加点词语意思不相同的一项是()A.湖中人鸟声俱绝/哀转久绝B.是日更定矣/问今是何世C.上下一白/长烟一空D.独往湖心亭看雪/往来而不绝者(2)下列各组句子中,加点词语意思相同的一项是()A.余强饮三大白而别/上下一白B.及下船/及郡下C.湖中焉得更有此人/是日更定矣D.客此/迁客骚人BB 二、句子翻译1.是日更定矣,余拏一小舟,拥毳衣炉火,独往湖心亭看雪。____________________________________________________________________________________________________________________2.雾凇沆砀,天与云与山与水,上下一白。____________________________________________________________________________________________________________________3.惟长堤一痕、湖心亭一点、与余舟一芥、舟中人两三粒而已。______________________________________________________________________________________________________________________________________________________________________________这天晚上八时左右,我撑着一条小船,裹着裘皮衣服,围着火炉,独自到湖心亭看雪。冰花周围弥漫着白汽,天与云、与山、与水,浑然一体,白茫茫一片。只有西湖长堤在雪中隐隐露出一道痕迹,湖心亭也成了一个点,和我像草芥那样渺小的一叶小舟,舟中两三个像米粒般大小的人影罢了。 4.舟子喃喃曰:“莫说相公痴,更有痴似相公者。”__________________________________________________________三、内容理解1.内容理解默写。(1)文中描写雪景的句子是:__________,__________________,_____________,_____________、_________________、_____________、______________、______________________。(2)文中点明作者偶遇知音的惊喜之情的句子是:_______________________________。(3)文中以舟子的喃喃之语收束全文的句子是:_______________,_____________________。船夫自言自语道:“别说相公痴迷,还有和相公一样痴迷的人。”雾凇沆砀天与云与山与水上下一白湖上影子惟长堤一痕湖心亭一点与余舟一芥舟中人两三粒而已余强饮三大白而别莫说相公痴更有痴似相公者 2.内容理解简答。(1)“独往湖心亭看雪”一句中“独”字有什么表达效果?______________________________________________________________________________________________________________________________________________________________________________(2)“天与云与山与水”一句连用三个“与”字写出了什么景象?____________________________________________________________________________________________________________________(3)作者为何要以舟子的喃喃之语来收束全文?______________________________________________________________________________________________________________________________________________________________________________“独”字映衬出环境的幽静和空旷,表现了作者遗世独立的高洁情怀和不随流俗的生活方式,也暗示作者的“痴”(不同凡响),为下文做铺垫。连用三个“与”字反复渲染和突出了天空、云层、山峦和湖水浑然一体、举目皆白的景象。①作者引用舟子的话包含了对“痴”情的赞赏,从侧面烘托出作者痴迷于天人合一的山水之乐,痴迷于世俗之外的雅情雅致;②暗含了作者天涯遇知音的愉悦。 3.综合分析选择。(1)下列对文章的分析理解,不正确的一项是()A.“雾凇沆砀,天与云与山与水,上下一白”一句中,作者采用了宽视野、远镜头,营造了天空、云层、山峦、湖水连成一片,白茫茫浑然难辨,天长水远的阔大境界。B.“一痕”“一点”“一芥”“两三粒”写出了宇宙的空阔与人的渺小,表现了悠远脱俗的精神。C.两个赏雪人见到志趣相投的作者,不停劝酒,作者被迫喝下三大杯酒后赶紧离去。D.作者以舟子的喃喃之语来收束全文,深化意境,反衬作者痴迷于天人合一的山水之乐,痴迷于世俗之外的闲情雅致。C (2)下列对文章的分析理解,不正确的一项是()A.“大雪三日,湖中人鸟声俱绝。”作者从听觉起笔,让人感受到了万籁无声的寂静气氛。B.文章可谓是痴人赏奇景,奇景“奇”在万籁俱寂,天地一色,万物渺小;痴人“痴”在不顾天寒地冻,独往赏雪,醉心于雪景,闲情雅趣异于常人。C.文章采用白描手法,勾勒写意,使西湖奇景与游者雅趣相互映照。D.文章描写雪后奇景和湖心亭巧遇,主要表达了作者孤芳自赏的厌世之情。D 中考考点实训课外文言文阅读模拟训练卫南之战泽①兵进至卫南,度将孤兵寡,不深入不能成功。先驱云②前有敌营,泽挥众直前与战,败之。转战而东,敌益生兵③至。王孝忠战死,前后皆敌垒。泽下令曰:“今日进退等死,不可不从死中求生。”士卒知必死无不一当百斩首数千级金人大败,退却数十余里。泽计敌众十余倍于我,今一战而出其不意,势必复来。使悉其铁骑夜袭吾军,则危矣。乃暮徙其军。金入夜至,得空营,大惊,自是惮泽,不敢复出兵。(节选自《宋史》,有删改)【注释】①泽:宗泽,南宋将领。②云:报告。③生兵:新兵。 1.下列各组句子中,加点词语意义和用法不相同的一项是()(3分)A.度将孤兵寡/寡不敌众B.今日进退等死/且欲与常马等不可得C.泽计敌众十余倍于我/不可计数D.大惊,自是惮泽/见余大喜2.请用三条“/”给文中画线句子断句。(3分)士卒知必死无不一当百斩首数千级金人大败。3.请用简洁的语言概括宗泽在卫南之战中取胜的原因。(3分)_________________________________________________________C///他善于鼓舞士气,又能审时度势,灵活用兵。 2019秋部编版九年级语文上册第三单元第十三课唐诗三首 名师导学13诗词三首诗歌背诵名句赏析主旨理解行路难(其一)李白金樽清酒斗十千,玉盘珍羞直万钱。停杯投箸不能食,拔剑四顾心茫然。欲渡黄河冰塞川,将登太行雪满山。闲来垂钓碧溪上,忽复乘舟梦日边。行路难,行路难,多歧路,今安在?长风破浪会有时,直挂云帆济沧海。(1)金樽清酒斗十千,玉盘珍羞直万钱。赏析:此句以宴席之精美来侧面烘托诗人内心的茫然,并与下句“停”“投”“拔”“顾”形成对比。(2)欲渡黄河冰塞川,将登太行雪满山。赏析:“欲渡黄河”“将登太行”比喻诗人即将迈向自己的理想;“冰塞川”“雪满山”比喻理想受阻。(3)行路难,行路难,多歧路,今安在?赏析:反复、疑问的修辞和紧张急促的短句,表现了诗人内心极度的迷惘与失落。(4)长风破浪会有时,直挂云帆济沧海。赏析:此句为本诗的主旨句,化用典故,体现诗人自信乐观、对理想的执着追求和博大的胸襟。诗人面对仕途中巨大的打击,以真实的笔触记录了自己从怀才不遇和仕途不顺的茫然——对仕途前程的期盼——坚信理想能实现的过程,表现了诗人积极乐观的情怀。 诗歌背诵名句赏析主旨理解酬乐天扬州初逢席上见赠刘禹锡巴山楚水凄凉地,二十三年弃置身。怀旧空吟闻笛赋,到乡翻似烂柯人。沉舟侧畔千帆过,病树前头万木春。今日听君歌一曲,暂凭杯酒长精神。(1)巴山楚水凄凉地,二十三年弃置身。赏析:荒僻的环境侧面烘托出诗人无限的辛酸,也流露出诗人长期被贬的愤懑心情。(2)沉舟侧畔千帆过,病树前头万木春。赏析:诗人以“沉舟”“病树”自喻,饱含感慨身世、惆怅忧伤之情;但他对未来并不失望,坚信新事物必然会代替旧事物,表现出诗人对人事变迁和仕宦沉浮的豁达豪迈的胸襟。诗人在诗中不仅表达了政治失意的愤懑,而且表现出对生活、对未来积极乐观和豁达的态度。 诗歌背诵名句赏析主旨理解水调歌头苏轼丙辰中秋,欢饮达旦,大醉,作此篇,兼怀子由。明月几时有?把酒问青天。不知天上宫阙,今夕是何年。我欲乘风归去,又恐琼楼玉宇,高处不胜寒。起舞弄清影,何似在人间。转朱阁,低绮户,照无眠。不应有恨,何事长向别时圆?人有悲欢离合,月有阴晴圆缺,此事古难全。但愿人长久,千里共婵娟。(1)我欲乘风归去,又恐琼楼玉宇,高处不胜寒。赏析:词人以超越自然的奇想表达他对现实人间的不满,但急转直下,故意找出天上的美中不足来坚定自己留在人间的决心。一正一反,更表露出词人对人间生活的热爱。(2)转朱阁,低绮户,照无眠。赏析:此句运笔流畅,表面写对月圆人不“圆”的怨恨,其实蕴含了词人对亲人的思念之情。(3)人有悲欢离合,月有阴晴圆缺。赏析:词人对月怀人,由感伤离别转为探求人生哲理。(4)但愿人长久,千里共婵娟。赏析:突破时间的局限与空间的阻隔,共同欣赏这一轮明月,表达了词人对一切经受离难的人们的美好祝福,表现了他旷达的胸襟和积极乐观的人生态度。《水调歌头》这首词的上阕把酒颂月,抒写了由幻想超尘到复归现实的思想过程;下阕对月怀人,阐发“月有阴晴圆缺,人有悲欢离合”的人生哲理。全词意境深远,笔调委婉,表达了词人对胞弟苏辙的无限怀念,表现出他旷达乐观的胸襟。 名师导练一、直接默写1.请把李白的《行路难(其一)》默写完整:金樽清酒斗十千,______________。停杯投箸不能食,________________。欲渡黄河冰塞川,将登太行雪满山。__________________,________________。行路难,行路难,__________,今安在?___________________,__________________。2.请把刘禹锡的《酬乐天扬州初逢席上见赠》默写完整:巴山楚水凄凉地,二十三年弃置身。_________________,________________。______________,_________________。今日听君歌一曲,暂凭杯酒长精神。玉盘珍羞直万钱拔剑四顾心茫然闲来垂钓碧溪上忽复乘舟梦日边多歧路长风破浪会有时直挂云帆济沧海怀旧空吟闻笛赋到乡翻似烂柯人沉舟侧畔千帆过病树前头万木春 3.请把苏轼的《水调歌头》默写完整:明月几时有?把酒问青天。________________,______________。我欲乘风归去,又恐琼楼玉宇,高处不胜寒。______________,_______________。__________,__________,照无眠。不应有恨,何事长向别时圆?人有悲欢离合,月有阴晴圆缺,此事古难全。_______________,_______________。不知天上宫阙今夕是何年起舞弄清影何似在人间转朱阁低绮户但愿人长久千里共婵娟 二、理解性默写1.《行路难(其一)》中比喻人生路上艰难险阻、壮志难酬的诗句是:_____________________,_____________________。2.《行路难(其一)》中用典故表达了诗人渴望得到明君重用,成就功名的诗句是:________________,___________________。3.《行路难(其一)》中写出诗人内心的苦闷、抑郁及感情激荡变化的诗句是:__________________,___________________。4.《行路难(其一)》中写出尽管前路障碍重重,但仍能到达理想的彼岸,表现了诗人积极乐观的精神的诗句是:_______________,___________________。欲渡黄河冰塞川将登太行雪满山闲来垂钓碧溪上忽复乘舟梦日边停杯投箸不能食拔剑四顾心茫然长风破浪会有时直挂云帆济沧海 5.《行路难(其一)》中以“__________,________,_________,__________”一句正面写“行路难”来比喻人生道路的艰难险阻、失意与无奈。6.《酬乐天扬州初逢席上见赠》中叙述自己被贬时间之长、被贬之地边远,饱含诗人无限的辛酸和愤懑的诗句是:_______________,__________________。7.《酬乐天扬州初逢席上见赠》中通过引用典故来表现岁月流逝,人事全非,自己恍若隔世之人的诗句是:____________________,____________________。8.《酬乐天扬州初逢席上见赠》中蕴含哲理,表明新事物必将取代旧事物的诗句是:_________________,____________________。行路难行路难多歧路今安在巴山楚水凄凉地二十三年弃置身怀旧空吟闻笛赋到乡翻似烂柯人沉舟侧畔千帆过病树前头万木春 9.《酬乐天扬州初逢席上见赠》中用比喻指出久遭贬谪的诗人想摆脱现实处境,展望未来风光,借景物变化暗示社会发展的诗句是:__________________,___________________。10.《酬乐天扬州初逢席上见赠》中既是对友人关怀的感谢,也是和友人共勉,且表现了诗人坚定的意志和乐观精神的诗句是:__________________,___________________。11.《水调歌头》中通过写明月来表达对天下人美好的祝愿的词句是:_________________,________________。12.《水调歌头》中“此事古难全”里“此事”是指:_________________,__________________。13.《水调歌头》中抒写词人矛盾心情的句子是:_____________,_________________,________________。沉舟侧畔千帆过病树前头万木春今日听君歌一曲暂凭杯酒长精神但愿人长久千里共婵娟人有悲欢离合月有阴晴圆缺我欲乘风归去又恐琼楼玉宇高处不胜寒 14.《水调歌头》中从“隔千里兮共明月”演化而来的句子是:_________________,________________。15.宋代著名词人苏轼在《水调歌头》的下阕开头描写月光流转的名句是:__________,__________,__________。课内精读1.“金樽清酒斗十千,玉盘珍羞直万钱”两句描写了怎样的场面?有什么作用?__________________________________________________________2.诗句“行路难,行路难,多歧路,今安在”显然不同于其他的诗句,分析其独特之处及其表达效果。____________________________________________________________________________________________________________________但愿人长久千里共婵娟转朱阁低绮户照无眠描绘了隆重而丰盛的宴会场面。反衬作者悲愤、失望的情绪。运用反复的修辞方法,尾句是疑问句,写出了诗人面临的巨大困难:遭遇坎坷,突显了他苦闷、彷徨、茫然的心理。 3.透过《行路难(其一)》全诗,你看到了一个什么样的诗人形象?____________________________________________________________________________________________________________________4.下列对《行路难(其一)》的理解有误的一项是()A.诗的开头两句以夸张的笔法,营造了欢乐的宴饮气氛,体现了诗人愉悦的心情。B.诗中以“欲渡黄河冰塞川,将登太行雪满山”来比喻人生道路中的艰难险阻。C.诗中运用典故,含蓄地表达了诗人渴望得到朝廷重用的心理。D.诗的最后两句写出了诗人坚信远大的抱负必能实现的豪迈气概。一个虽饱含苦闷、愤懑迷茫之情,却又充满信心,执着地追求理想,积极乐观的诗人形象。A 5.“怀旧空吟闻笛赋,到乡翻似烂柯人”中的两个典故,分别用来表达诗人怎样的思想感情?______________________________________________________________________________________________________________________________________________________________________________6.“沉舟侧畔千帆过,病树前头万木春”一句历来受到人们的赞赏,你怎样理解这两句诗?____________________________________________________________________________________________________________________“怀旧空吟闻笛赋”——对亲旧凋零的伤痛悼念、人事全非的怅惘;“到乡翻似烂柯人”——暗示贬谪时间长久,表现了诗人对世态变迁的怅惘心情。这两句诗生动形象地揭示了新陈代谢的自然规律,表现出诗人积极进取的人生态度。 7.下列对《酬乐天扬州初逢席上见赠》这首诗的内容的理解,错误的一项是()A.首联中的“凄凉地”“弃置身”这两个词语,暗含诗人长期谪居的痛苦和抑制已久的愤激心情。B.颔联写诗人回到了久别的家乡,虽人事全非,心里却有欣慰之情。C.颈联“沉舟”和“病树”是比喻久遭贬谪的诗人自己。D.尾联点明酬赠题意,既是对友人关怀的感谢,也是和友人共勉。8.《水调歌头》一词前小序的作用是什么?_________________________________________________________9.指出下列词语或句子所表达的情感。(1)“何事长向别时圆”中的“何事”表达的情感是:__________________________________________________________________B主要是向读者陈述写这篇词时的心情、地点,交代写作的原因。不能和亲人团聚的惆怅。(或:希望和亲人团聚的愿望) (2)“但愿人长久,千里共婵娟”表达的情感是:_________________________________________________________10.下列对《水调歌头》赏析不正确的一项是()A.这首词上阕写词人对天上宫阙的向往和对人间生活的热爰,下阕写词人在月下对亲人的怀念和祝福,自始至终洋溢着乐观向上、积极奋进的情绪。B.“不应有恨,何事长向别时圆?”这一句抒发了思念亲人的感情,表面看来,好像是对月有怨恨之意,其实是表达亲人不能团聚的惆怅。C.“人有悲欢离合,月有阴晴圆缺,此事古难全。”这句话从人到月,从古到今,对自然和社会做了高度概括,很有哲理意味。D.全词情景交融,意境豪放阔大,描写虚实结合,充满了丰富的想象和浪漫的情致,具有强烈的艺术感染力。对亲人的美好祝愿;乐观旷达的情怀。A 中考考点实训古诗文默写模拟训练根据课文默写古诗文。(10分)1.但愿人长久,_____________。(苏轼《水调歌头》)(1分)2._________________,玉盘珍羞直万钱。[李白《行路难(其一)》](1分)3.然则何时而乐耶?其必曰“__________________,_________________”乎!(范仲淹《岳阳楼记》)(2分)4.李白在《行路难(其一)》中引用典故,委婉含蓄地表达自己对被重新任用有所期待,对未来仍抱有希望的句子是:_________________,__________________。(2分)千里共婵娟金樽清酒斗十千先天下之忧而忧后天下之乐而乐闲来垂钓碧溪上忽复乘舟梦日边 5.请把刘禹锡的《酬乐天扬州初逢席上见赠》默写完整。(4分)巴山楚水凄凉地,二十三年弃置身。___________________,___________________。___________________,__________________。今日听君歌一曲,暂凭杯酒长精神。怀旧空吟闻笛赋到乡翻似烂柯人沉舟侧畔千帆过病树前头万木春 2019秋部编版九年级语文上册第四单元第十四课故乡 名师导学14故乡一、课前自学知识储备小说刻画人物的手法:人物形象是小说的中心,常见的描写人物的方法有心理描写、动作描写、语言描写、外貌描写、神态描写。 一课一法对比手法:是文学作品中一种常用的表现手法,它通常将不同事物或同一事物的两个方面,安置在一定条件下,使之集中在一个完整的艺术统一体中,加以对照,有利于充分显示事物的矛盾,突出被表现事物的本质特征,使形象更加鲜明,同时也加强了文章的艺术效果和感染力。分析课文中对比手法的作用:对比手法作用环境的对比揭示了在帝国主义和封建主义的残酷蹂躏下日益破产的社会现实。闰土的前后对比揭示了在半殖民地半封建的黑暗制度下,农民日益悲惨的命运。杨二嫂的前后对比她的变化说明了城镇小市民的贫困化,也从侧面反映了农村经济的破败,反映了当时社会的弊病。“我”与闰土关系的对比反映了封建等级观念及封建礼教对人的毒害。闰土和杨二嫂的对比文中用杨二嫂的自私刻薄衬托了闰土的纯朴善良。 二、内容梳理(根据课文填空,理清文脉)《故乡》以“我”回到故乡的见闻和感受为线索,通过闰土20多年前后的变化,描绘了辛亥革命后十年间中国农村衰败、萧条、日趋破产的悲惨景象,揭示了广大农民生活痛苦的社会根源,表达了作者改造社会、创造新生活的强烈愿望。小说按照时间顺序,可分为三个部分:故乡第一部分(第1~5段):回故乡所见萧索的景象所感卖屋的悲凉作用是:①________________________________________渲染悲凉气氛,为下文做铺垫第二部分(第6~77段):在故乡忆少年闰土:②____________________________的小英雄勇敢聪明、活泼可爱、热情开朗见中年闰土:③_____________________的木偶人贫穷困苦、迟钝麻木睹杨二嫂:④______________________________的小市民尖酸刻薄、庸俗势利、自私贪婪第三部分(第78~88段):离故乡——“我”的感受是⑤_______________________________________痛苦、失望、希望、憧憬 名师导练基础训练1.根据拼音写出相应的词语。(1)时候既然是深冬;渐近故乡时,天气又yīnhuì()了,冷风吹进船舱中,呜呜的响,从篷隙向外一望,苍黄的天底下,远近横着几个xiāosuǒ()的荒村,没有一些活气。(2)这jìsì(),说是三十多年才能轮到一回,所以很郑重。(3)这畜生很línglì(),倒向你奔来,反从胯下窜了。(4)我èrán()了。(5)然而圆规很不平,显出bǐyí()的神色,仿佛chīxiào()法国人不知道拿破仑,美国人不知道华盛顿似的。阴晦萧索祭祀伶俐愕然鄙夷嗤笑 (6)我一面yìngchou(),偷空便收拾些行李,这样的过了三四天。(7)这是第五个孩子,没有见过shìmiàn(),躲躲闪闪……(8)我和母亲也都有些wǎngrán(),于是又提起闰土来。(9)我希望他们不再像我,又大家gémó()起来……(10)然而我又不愿意他们因为要一气,都如我的辛苦展转而生活,也不愿意他们都如闰土的辛苦麻木而生活,也不愿意都如别人的辛苦zìsuī()而生活。应酬世面惘然隔膜恣睢 2.下列句子中加点的词语使用不恰当的一项是()A.我愕然地抬起头,正好对上他那双波澜不兴的紫眸,那种紫色,仿佛水晶一般透明。B.信念是在远方闪烁的灯,它在你奔走在幽暗阴晦的山野小径上时给你指点方向;信念是在耳边鸣响的号,它在你懈怠于疲惫困顿的旅途中时给你以力量。C.成乐高铁一旦开通,那么在乐山,水运的顺畅与高铁的快捷相得益彰,这将大大提升乐山经济发展的速度。D.面对小区保安的盘问,作案后的小偷义愤填膺,反应相当激烈,然而,在铁证面前不得不束手就擒。D 3.下列对病句的修改不正确的一项是()A.在保留现有水源的基础上,杭州将逐步形成以千岛湖为主,钱塘江、东苕溪为辅的多水源供水,从而提高居民的用水品质。(在“多水源供水”后加“的格局”)B.如何在加快发展经济的同时,减少人口增长速度,保持人类与生态环境的平衡,这是摆在一些国家面前的重大课题。(把“减少”改为“减慢”)C.数学老师运用交通事故统计数据进行教学,使学生不仅增强了安全意识,还学会了计算方法。(把“增强”改为“提高”)D.当灵感迸发、文思泉涌时,美妙的文辞会源源不断地流泻到笔下,这都源于写作者平时注重知识和生活的积累为基础。(删去“为基础”)C 4.压缩下列语段,不超过45字。一到秋天,森林里又换了一番景色。落叶松和各种阔叶树的叶子变成深浅不同的黄色。有些树的叶子都变得火一样红。不落叶的树显得更苍翠了。秋风摆动树林,哗哗的声音好像海边的浪涛。各种山果子都熟了:榛子、野枣、山里红、山葡萄……说也说不完。松鼠忙起来了,它不停地把山果子搬进树洞里去。东北的森林,十月就飞雪了,松鼠不得不早做准备,好度过漫长寒冷的冬天。____________________________________________________________________________________________________________________秋天来到森林,树木的叶子颜色美丽,各种山果子都熟了,松鼠忙着储备果子。 课内精读5.赏析课文中精妙的句子。(6分)(1)“那西瓜地上的银项圈的小英雄的影像,我本来十分清楚,现在却忽地模糊了,又使我非常的悲哀。”这句话的含义是什么?____________________________________________________________________________________________________________________(2)“现在我所谓希望,不也是我自己手制的偶像么?只是他的愿望切近,我的愿望茫远罢了。”这句话中“手制的偶像”指的是什么?为什么闰土的愿望“切近”,而“我”的愿望“茫远”?__________________________________________________________________________________________________________________________________________________________________________________________________________________________________________________________________________________________________本来“我”对故乡的记忆是美好的,但又觉得美好的未来是渺茫的,以至于眼前的影像模糊起来。“手制的偶像”指对美好生活的憧憬和希望。闰土崇拜神鬼的偶像,态度虔诚,他希望自己的生活能够好起来,这种愿望是“切近”的;而“我”对未来充满希望,“我”的希望是让后辈过上幸福的生活,实现这一希望是艰巨而且漫长的,因而“我”的愿望是“茫远”的。 6.课文最后再次描写海边奇异的画面,有什么作用?(4分)__________________________________________________________________________________________________________________7.课文结尾“我想:希望是本无所谓有,无所谓无的。这正如地上的路;其实地上本没有路,走的人多了,也便成了路”一句采用了什么表达方式?如何理解?(5分)_____________________________________________________________________________________________________________________________________________________________________________________________________________________________________________________________________________________________结构上:呼应上文;内容上:表达了“我”对美好希望的想象和憧憬。作者采用了议论的表达方式。在这个充满哲理的结尾中,作者把希望比作地上的路,意思是:只空有希望而不去奋斗、追求,希望便“无所谓有”;有了希望并始终不渝地斗争、实践,希望便“无所谓无”。人们都满怀希望奋斗,就会迎来新生活,就能创造新生活。这个句子表达了作者对踏出希望之路的信心。 小说阅读技法专练——体会对比手法的作用我的心忽然感到无比欣慰。我们常常行进在布满漩涡、处处危险的浊流中,孩子的童真与善良,却让我仿佛在浊流中看到了一股清泉,这股清泉让我感受到了人性中的甘甜,让我看到了人间最美好的那一面!这份童真与善良我曾经是那样的熟悉,而现在又是如此的陌生。(节选自陈敏的《童真》,有删改) 8.从修辞方法的角度赏析选段中的画线句子。(4分)________________________________________________________________________________________________________________________________________________________________________________________________________________________________________运用了比喻、对比的修辞方法,把危险和世俗比作浊流,把孩子的童真与善良比作清泉,把浊流与清泉做对比,表达了“我”对孩子的童真与善良的赞美之情;“我”从这股“清泉”中感受到了甘甜,突出了童真的难能可贵。 “先生,请问您要办理什么业务?”门口站着的美女微微躬身发问。“问我吗?”被称为先生,刚子有点心虚。美女笑得很含蓄,说:“是的,先生。”简直受宠若惊了,刚子赶紧挤出一丝笑,说:“我给我媳妇汇点钱。我在工地干活,她在农村老家……”刚子还想多说几句,美女已经将一张薄薄的纸片儿交到刚子手中。“先生,这是您的排号,请等叫号后,到7号窗口办理。”美女再次躬身,并做出一个优雅的手势。银行大厅里很暖和,与屋外的阴冷截然不同。刚子环视四周,也没发现空调。也许是快下班了,大厅里的顾客并不多,各忙各的,安静有序。(节选自赵春亮的《不明白》,有删改) 9.选文最后一段的画线句子写到“大厅的暖和”和“屋外的阴冷”,这两处环境描写的作用是什么?(4分)___________________________________________________________________________________________________________________________________________________________________________“大厅里的暖和”与“屋外的阴冷”形成鲜明对比,既突出了银行环境的舒适,又写出了刚子受到优待和尊重时内心的温暖和幸福。 中考考点实训文学类文本阅读模拟训练奶奶的第一次合影陈敏①奶奶一生有两个忌讳:照相和过生日。她认为照相会吸走人的精气,过一次生日就会折寿半年。她将这一理念坚持了一辈子,于是,我们做晚辈的,都尽量顺着她,不为她庆生,也不为她照相。尽管我曾偷拍过她几次,但终因怕她知道了不悦,也一直没让她看。她低调对待生命,不想在这个世界上留下她的任何印记。②奶奶很上相。她坐在屋檐下剥玉米,银白色的头发贴在脑后,身材瘦削,穿着一件灰蓝色的长对襟衣,那模样本身就是一 幅水墨画。我一直想给她拍张彩照,等她百年之后使用,毕竟,她已年过九十了。但她从不配合,我的想法一直未能实行。③年过九十的人几乎是风里的一盏灯。④入秋的那场雨下过,天气“呼啦”一下就凉了。奶奶的老毛病气管炎犯得很准时,年年秋后如期而至。奶奶尽量压低喘息声,但咳嗽声还是传了过来,一阵一阵的,将她的身子压得越来越低,让她的脚步更加蹒跚。⑤门外的那棵油桐树叶黄了,秋雨过后,“啪啪”地往地上落。“叶落归根了!”奶奶嘴里嘀咕一声,缩了缩身子,弯着腰回屋里了。奶奶怕风吹,就待在黑洞洞的屋子里不出来,有时,看半天没动静,喊她几声,她才应一声。⑥转眼冬天来了。父亲打来电话,声音大得可怕,简直是吼,说:“老房门前被画了白线,有高速公路要从门口过,这叫人咋活呀?”⑦这是很多农民当下面临的一个难题,谁也没办法。我还一直 为我们家地处偏僻地带,不会被现代化设施骚扰而得意呢,没想到还是没躲过拆迁这一劫,想着想着,头就大了,跟老板请了假,回去看看。⑧院子里依然静悄悄的,只是房阶下三米以外的地方醒目地画了一条白线。那棵高高挺立的油桐树首当其冲,白线就画在它的根部。⑨那条白线就像一道符,在太阳的光里显得阴森而恐怖。耳后,一阵急促的喘息声传来,回头看,是奶奶。⑩奶奶今天显得很精神,她将头发梳得光溜溜的,走出她“宅”了许久的屋子,微笑地看着我说:“二丫,你知道不,咱家门前的这棵油桐树是你爷爷盖房时栽的。你爹才7岁时,你爷爷蹬脚走了,他走时树才有碗口那么粗。”⑪我只知道吃从树上掉下来的油桐籽,却从不知道这棵树竟然有这么长的历史。它现在粗得我一抱都搂不严。 ⑫奶奶说后,用手捋了捋头发,又说:“过不了几天,这树就要被砍了,咱们的老屋和院墙肯定也保不住了。二丫,你给我照张相吧,我要和树连同这老屋在一起,这么多年了,也该到头了。”我的眼泪竟然“哗”地落了下来,拿相机的手颤抖得厉害。⑬奶奶端端地站在油桐树下,望着我,脸上透出少有的光。透过被泪水模糊的眼,我看了看镜头里的奶奶,使劲地摁了快门。奶奶看了看相机里的照片说:“我不敢看了,收起来吧,也不知道你爷爷敢不敢看。”她颤颤巍巍地走回了屋里。⑭一年后,奶奶和那棵老油桐以及老屋永远地待在了一起。(有删改) 1.根据文章第①段的内容,用两个词语概括奶奶的性格特点。(3分)______________________________________________________2.文章中多处提到油桐树,请说说油桐树在文中有哪些作用。(5分)____________________________________________________________________________________________________________________________________________________________________________________________________________________________________________________________________________________________迷信和固执。①油桐树贯穿文章的主要情节;②推动情节的发展,油桐树是爷爷栽下的树,对奶奶来说有特殊意义,它将被砍倒,奶奶才主动提出要照相,才有了第一次合影;③以老油桐树的形象来衬托奶奶的形象,突出奶奶对爷爷的怀念和对家的热爱。 3.请从修辞方法的角度赏析第⑨段中的画线句子。(4分)______________________________________________________________________________________________________________________________________________________________________________4.小说以《奶奶的第一次合影》为题有什么作用?(5分)__________________________________________________________________________________________________________________________________________________________________________________________________________________________________________________________________________________________________运用比喻的修辞方法。把那条白线比作一道“阴森恐怖的符”,表现“我”面对拆迁惊慌、无奈的心态;也说明拆迁给“我”的冲击之大,因为爷爷栽的油桐树即将消失,奶奶也即将逝去。①设置悬念,“第一次”合影是什么情况,和谁“合影”,这些悬念吸引读者继续阅读文章;②“合影”二字点出贯穿全文的线索“照相”;③“第一次合影”是全文的核心内容,揭示作品的主旨,表现了奶奶对爷爷和过往生活的深切怀念以及对家的热爱。 2019秋部编版九年级语文上册第四单元第十五课我的叔叔于勒 名师导学15我的叔叔于勒一、课前自学知识储备1.小说的故事情节包括开端、发展、高潮、结局四个部分。2.理清小说情节各个部分的作用:(1)开端:交代背景,铺垫下文;(2)发展:刻画人物,反映性格;(3)高潮:表现冲突,揭示主题;(4)结局:深化主题,留下思考。 3.记叙的顺序一般分为顺叙、倒叙、插叙和补叙。4.插叙:在叙述中心事件的过程中,为了帮助展开情节或刻画人物,暂时中断叙述的线索,插入一段与主要情节相关的回忆或故事的叙述方法。其作用是:有助于情节的展开、刻画人物性格、补充背景材料,使人物形象生动完整,突出其某个性格特点,或交代细节,深化主题。一课一法课文中两处插叙的作用:(1)第一处插叙:补充交代于勒去南美洲前后的情况,揭示了“我们”全家急切盼望于勒的原因,展示了全家从怕于勒到盼于勒的变化,对比鲜明,突出人物形象。(2)第二处插叙:交代二姐的婚事,既生动地揭示了资本主义社 会的金钱关系,深化主题,又能自然地引出下文。二、内容梳理(根据课文填空,理清文脉)《我的叔叔于勒》通过对比手法,运用语言、动作、神态描写刻画人物的心理世界,描述了菲利普夫妇对待亲兄弟于勒的前后截然不同的态度,刻画了资本主义社会里因贫穷而兄不认弟的触目惊心的惨象,揭露了资本主义社会人与人之间的赤裸裸的金钱关系。全文可分为四个部分: 我的叔叔于勒开端:盼于勒家里生活①________,母亲内心②________家人衣着③________,父亲的话④__________拮据痛苦整齐永不变更发展:赞于勒插叙往事,于勒挥霍家产时父母的态度是⑤__________引用书信,于勒经商发财时父母的态度是⑥__________惧怕憎恨奉为福星高潮:遇于勒⑦_______描写揭开于勒事实上穷困潦倒的真相⑧______________________描写表明父母的深恶痛绝外貌语言、动作、神态结局:躲于勒景物描写衬托家人⑨_______________的心情回航改船躲避于勒失望、沮丧 名师导练基础训练1.根据拼音写出相应的词语。(1)我母亲对我们的jiéjū()生活感到非常痛苦。(2)可是每星期日,我们都要衣冠整齐地到海边zhànqiáo()上去散步。(3)据说他当初行为不正,zāotà()钱。(4)果然,十年之久,于勒叔叔没再来信。可是父亲的希望却yǔrìjùzēng()。(5)对于叔叔回国这桩shínájiǔwěn()的事,大家还拟定了上千种计划,甚至计划到要用这位叔叔的钱置一所biéshù()。拮据栈桥糟蹋与日俱增十拿九稳别墅 (6)我一直盯着父亲,看他zhèngzhòngqíshì()地带着两个女儿和nǚxu()向那个衣服lánlǚ()的年老水手走去。(7)我也duānxiɑng()了一下那个人。他又老又脏,满脸皱纹,眼光始终不离开他手里的活儿。(8)父亲客客气气地和船长搭上话,一面gōngwéi(),一面打听有关他职业上的事情。(9)听说他在那边kuòchuò()过一个时期,可是您看他今天已经落到什么田地!(10)我父亲脸色早已shàbái(),两眼呆直。郑重其事女婿褴褛端详恭维阔绰煞白 2.下列句子中加点的词语运用不恰当的一项是()A.不再去违心讨好,不再有刻意恭维,不求人,不必屈就妥协,无须逢迎献媚。岁月老了,心却逐渐丰厚,是该做回自己的时候了。B.也颇有那种虚假的傲气,总愿意掩饰自己的贫穷,假装阔绰。C.《财富》杂志说,近30年来新成立的科技公司的总资产已经接近一万亿美元,而这个资产总量还在一天天地与日俱增。D.人群中挤攘了一下,一个梳着童花头的男孩子跑过来,郑重其事地把一块钱放进了捐款箱。C 3.下列对病句的修改不正确的一项是()A.“中国汉字听写大会”的举办,既可以让汉字文化得以传承,让汉字书写美德得以传递,又可以检验我国民众整体的汉字书写水准。(后三个分句改为“既可以检验我国民众整体的汉字书写水准,让汉字书写美德得以传递,又可以让汉字文化得以传承”)B.与《舌尖上的中国3》第一集的原画面相对比,该片涉嫌抄袭的镜头不少于十多处。(删去“多”)C.3月30日,书记、副书记和其他县领导出席了党的群众路线教育动员大会。(把“其他县领导”改为“县其他领导”)D.南平市政府深入贯彻党中央精神,加快海峡西岸经济区。(把“加快”改为“加强”)D 4.阅读下面材料,按要求作答。海底是一个神奇的世界,海洋生物是这个神奇世界的缔造者,人们只有不断地探索才能走进这个神秘莫测的世界。比如一只海星模样的动物挥舞着五条长长的手臂游过来,它看上去没有头也没有眼睛,但事实上每条手臂都遍布着“眼睛”,全身就是一只巨大的复眼。美国贝尔实验室的科学家说,他们在研究一种被称为“海蛇尾”的动物时,偶然发现它身体表面由碳酸钙构成的骨板是由许多极为细小的凸透镜构成的。这些微小透镜就组成了一只可以看到四面八方的复眼,与昆虫的复眼相似,对它躲避天敌非常有用。所以说,美丽的海洋生物是神秘的,是令人无比迷恋的,我们在认识它们的同时,同样应该留心它们身上蕴藏的秘密。 (1)请用简明的文字概述“海蛇尾”的特点和功能,不超过35字。__________________________________________________________________________________________________________________(2)根据材料内容,以“海洋生物”为描述对象,用上一种修辞方法,写一句话。____________________________________________________________________________________________________________________体表遍布的细小凸透镜,组成了一只巨大的复眼,有利于其躲避天敌。大自然造就了神奇美丽的海洋生物,它们就像藏在深海中的宝藏,蕴藏着无穷的财富,等待着人们去探索和追寻。 课内精读5.课文中菲利普夫妇对于勒的态度产生变化的原因是什么?反映了怎样的社会现实?(4分)______________________________________________________________________________________________________________________________________________________________________________6.赏析课文中精妙的句子。(9分)(1)“父亲总是重复他那句永不变更的话:‘唉!如果于勒竟在这只船上,那会叫人多么惊喜呀!’”这句话运用了什么描写方法?有什么作用?______________________________________________________________________________________________________________________________________________________________________________菲利普夫妇态度的变化是因为于勒经济状况的变化:于勒发财时奉为福星,急切盼望;于勒贫穷时深恶痛疾。揭露了资本主义社会人与人之间赤裸裸的金钱关系,反映了资本主义社会的黑暗与腐败。运用语言描写的方法。揭示了菲利普夫妇急切盼望于勒回来,幻想发财的心理,与下文躲于勒形成对比,刻画了人物势利、奉金钱至上的特点。 (2)“最后我父亲终于说:‘您船上有一个卖牡蛎的,那个人倒很有趣。您知道这个家伙的底细吗?’”结合加点词语赏析句子。___________________________________________________________________________________________________________________________________________________________________________(3)“我心里默念道:‘这是我的叔叔,父亲的弟弟,我的亲叔叔。’”这个句子运用了什么修辞方法?有什么作用?____________________________________________________________________________________________________________________________________________________________________________________________________________________________________菲利普为了不被船长发现他的用意,更害怕船长知道他和于勒的关系,只得东拉两扯,但又急于想转入正题,“最终”一词含有一种按捺不住的急迫,从中表现出他的虚伪与无情。运用了反复的修辞方法。突出人物的关系,强调了于勒与“我们”的血缘之情,反映了“我”对处于贫困的于勒叔叔深切的同情和对父母亲六亲不认的困惑、苦闷及不满,这是作者对菲利普夫妇辛辣的讽刺。 小说阅读技法专练——品味插叙的作用谢谢,你拉得真好青水螅①和朋友一起外出,在天桥上遇见了一个少年在卖艺。②少年一身破旧的衣服,肩上的小提琴也很破旧,他的脸上一副麻木的表情,身前的盆子里散落着几枚硬币。我不知道他努力演奏的是什么音乐,但是我能清楚地分辨出,他的技艺并不高超。③我掏出钱包,想同样丢下几枚硬币。朋友阻止了我的举动,拉着我站在那里倾听起来。少年的眸子显露出一丝生气,琴声也不再那么枯涩。一曲奏罢,朋友对少年鞠了一躬,掏出几张人民币放在盆子里,对少年说了句:“谢谢,你拉得真好。”我清楚地看到,少年的眸子里有亮晶晶的东西滑落。 ④朋友是个小有名气的小提琴家,他怎么会赞扬如此蹩脚的技艺呢?和朋友离开天桥后,我问起朋友刚才的举动。朋友叹了口气说:“这个孩子像极了当年的我。”于是,朋友对我讲了这样的一个故事。⑤在他在读高中的时候,决定报考音乐专业,但贫寒的家境在为他买了一把便宜的小提琴后,根本无法把他送到辅导学校去学习小提琴。为了筹集自己的学费,他带着小提琴,用他那蹩脚的技艺在天桥上卖艺。天桥上人来人往,偶尔有好心人丢下几枚硬币,伴随着的是怜悯的目光。本就有些自卑的朋友感觉自尊心受到了严重的打击,他感觉到他的技艺逐步下降,他的行为是对艺术的一种亵渎。有很多次他都想跑回家,发誓再也不去卖艺了。⑥有一天,在他卖艺的时候,有位老人站在他的旁边静静倾听。在临走的时候,老人对他鞠了一躬,对他说:“谢谢你,你拉得真 好。”朋友第一次感觉到了尊重,感觉到了对艺术的尊重,对人的尊重。从那以后,朋友在外出卖艺的时候,再也不是以自卑的心态去演奏,而是倾注了所有的热情,用心去演奏,技艺也不断提高。⑦“后来,我读了大学,却再也没有见到过那位老人。”朋友对我说,“时至今日,我一直都在感激这位老人,是他告诉我,尊重远远比廉价的怜悯更为重要。”⑧我深深地被打动了。是啊,对于一个落魄的人来说,尊重远远比廉价的怜悯更为重要。(选自《传承》,有删改) 7.文章的第⑤⑥段运用了哪种记叙顺序?有什么作用?(3分)___________________________________________________________________________________________________________________________________________________________________________插叙。补充交代了“我”朋友对天桥上的少年说“谢谢,你拉得真好”的原因,突出了文章的中心:尊重比廉价的怜悯更重要;使行文有起伏,避免了平铺直叙。 ①母亲病了,躺在床上,我就坐在母亲的床边玩电脑。无意中转头看看母亲,她正睁着眼看着天花板出神。我心里突然一动。我名义上是在这里陪伴母亲,可除了能为她做点饭倒杯水外,还做了什么?②母亲是生我时落下了病根,多年来一直备受疾病的折磨,今年又添上了许多老年病。也许因为母亲长年生病,她自己不在乎了,我们也习以为常了。有时周末回家,也很少想着要主动帮她干点儿什么,总是她一个人里里外外地干这干那。③母亲这次犯病,就是累病的。她病了,不但得不到我们的同情,反而谁见了她都会抱怨,都觉得她把自己累病了,害得我们也跟着受累。每当我们抱怨的时候,母亲总是一声不吭,仿佛她真的做错了什么一样。 ④我关掉电脑,给母亲倒上一杯水。还没等我说什么,母亲却说:“看电脑累眼睛吧?快躺下闭着眼睛歇一歇。”我答应着,拉了一床被子就躺在母亲身边。我说:“妈,你给我说说你以前的事,我看看能不能写篇文章。”⑤母亲惊喜得一时不知该说什么。我静静地躺着,耐心地等着她说。突然觉得,原来躺在母亲身边,就是这样的感觉——暖暖的。我却已经遗忘多年了。(节选自王晓明的《躺在母亲身边》,有删改) 8.文章第②③段运用了哪种记叙顺序?分析这两段在文中的作用。(3分)____________________________________________________________________________________________________________________________________________________________________________________________________________________________________插叙。这两段补充交代了母亲生病的原因,以及母亲生病时仍常年一个人忙里忙外、默默承受疾病折磨,最后积劳成疾却遭受家人抱怨的事实。这样写能使母亲的形象更加鲜明,文章内容更加充实,中心更加突出。 中考考点实训文学类文本阅读模拟训练最暖心的事[美国]鲍勃·布劳顿①十年前,我从得克萨斯州的乡村来到纽约开出租车谋生。开的士会碰到形形色色的人,有的人幽默诙谐,有的人失意忧郁,还有的人自命不凡。但让我印象最深的莫过于一个老太太。②那是今年5月份第2个星期日的深夜,我接到城郊的一个约车的电话。我想,也许是一些参加完晚会的人,或是某个刚赶到这个城市过母亲节的人。③我到达目的地时是3时30分。一个破败的公寓楼黑黢黢地立在我的眼前,只有一楼有一个房间透出一点灯光。这种情况下, 大多数司机顶多只会按一两声喇叭,稍等片刻,然后开车走人。因为这个时间和地点时常会出现治安问题。然而,我也知道这个时间在这样的地方打车不易,再说也许这个客人有点困难需要我帮一把手呢。于是,我走到亮灯的那户人家敲了敲门。④“等一会儿。”回答我的是一个苍老虚弱的声音。我听到屋内有什么东西在地上拖动。隔了好久,门开了,一个80多岁的瘦小老太太吃力地拖着一个萨斯包走了出来。她身穿一件印第安大花布上衣,头戴一顶圆桶形帽子,帽子上还罩了一条面纱,活脱脱一个20世纪40年代好莱坞电影里走出的人物。⑤“你能帮我拎一下包吗?”她说。我先将她的包拎上车子,然后又回头搀扶她。她走得很慢,边走边对我表示感谢。⑥“这没什么。”我说,“我这是为我的客人服务。再说,我希望我的妈妈在外面也能得到同样的服务。”⑦“你真是一个好人。”她感激地说道。进了车子,她给了我 一个地址,问:“能不能从城里走?我很想再看看这座城市……”⑧“能,不过这就不是最近的路了。”我答道。⑨“这不要紧。”她说,“我不着急。我是菲奥娜小姐,不过人们都叫我菲奥娜太太,麻烦去圣洛安敬老院。”⑩我从后视镜中看了她一眼。菲奥娜太太的眼窝里有一滴亮晶晶的东西。“我孤寡一人。”她继续说道,“医生说我剩下的时间不多了。不然我不会去的。”⑪我悄悄地伸手关掉了计程表。经过城里的路程一刻钟就能走完,然而我们却花了足足有两个多小时,因为她一会儿让我慢行,一会儿让我停车,还不时地讲着话。菲奥娜太太指着一座大楼,告诉我她曾在这儿干过电梯操作员的工作。经过一个居民区时,她说她和丈夫结婚的新房就是在这里。她要我将车子在一个商场前停了一会儿。她说这里曾是个舞厅,年轻时她在舞厅当过舞蹈指导老师。有时,她会让我在某一个地方放慢速度,然后默 默凝视前方,一句话也不说。⑫当第一缕阳光露出地平线的时候,菲奥娜太太这才说:“我累了。走吧。”⑬车子来到了她要去的圣洛安敬老院前。敬老院的两个工作人员正在等着我们,工作人员说:“这位老太太一直不肯来敬老院,现在她患了肺癌,才同意来敬老院,而且必须在今年的母亲节来敬老院。”工作人员说着给她推来了轮椅。⑭“我应该付给你多少钱?”菲奥娜太太取出钱包问我。⑮“不要钱。”我答道。⑯“你也要养家。”菲奥娜说。⑰“还有其他客人呢。”我说,接着几乎是不假思索地弯下腰拥抱了她。她紧紧地抱住我。“你给了一个老太太一小会儿快乐的时光。”她说,“谢谢你。” ⑱我最后握了握她的手,然后走向暗淡的晨曦。我的身后响起了关门的声音。这是一个即将结束的生命发出的声音。一路上,我在想,如果今天带菲奥娜太太的是一个脾气急躁没有耐心的司机,如果我在公寓楼前按一两声喇叭后就把车开走,又会是怎样一种情形呢?⑲我做的这件事情似乎微不足道,但是现在想起来,却是我一生中最暖心的一件事情。 1.本文记叙了一件什么事?请抓住关键要素,用一两句话概括故事内容。(3分)______________________________________________________________________________________________________________________________________________________________________________________________________________________________________________________________________________________________________________________________________________________2.阅读第段,仔细揣摩并分析菲奥娜太太经过城里这两个多小时的心理感受。(3分)_________________________________________________________“我”在母亲节深夜开车送一个即将告别生命的老人到敬老院时,满足她的每一个愿望(或:送她到她想去的任何地方),最后到达目的地,不收她的出租车费,还给了她一个拥抱,她感到很快乐,“我”也很温暖。(或:“我”开车送一个即将告别人生的孤寡老人到敬老院时,满足她的每一个愿望并且不收费,使她感受到了生命最后时刻的温暖)对过去美好生活的眷念和对生命的不舍。 3.本文运用了细节描写的方法,请从文中找出关于老太太的两处细节描写,简要分析这些细节的丰富内涵。(4分)__________________________________________________________________________________________________________________________________________________________________________________________________________________________________________________________________________________________________4.文章题目为“最暖心的事”,结合全文谈谈你对“暖心”的理解。(3分)________________________________________________________________________________________________________________________________________________________________________________________________________________________________________作者对老太太的外貌(肖像)、神态作了细致描写,写老太太穿戴过时、花哨,以及沉浸在过去的美好回忆中的神态(或:“默默凝视”的神态),生动形象地表现了老太太热爱生活(或:爱美),对年轻时代美好生活的眷恋(或:怀念),表达了作者对老太太的怜悯、欣赏和爱心。示例一:菲奥娜太太在生命结束前得到了一个出租车司机的关怀和帮助,她感到快乐和温暖。示例二:“我”帮助了一个生命即将结束的老太太,“我”感到温暖。 5.读罢此文,你在为人处世或者对人生的认识方面产生了哪些感悟?请用凝练的语言写出来。(4分)________________________________________________________________________________________________________________________________________________________________________________________________________________________________________示例一:对待弱势群体要友善、有耐心。示例二:在生活中多做一些“微不足道的能帮助人的小事”,也许会给别人带去很大的温暖。 2019秋部编版九年级语文上册第四单元第十六课孤独之旅 名师导学16*孤独之旅一、课前自学知识储备小说环境描写的作用:①渲染气氛,烘托人物形象;②为下文做铺垫;③推动情节发展;④深化作品主题。 一课一法体会课文中环境描写的作用:环境描写作用鸭群“鸭群在船前形成一个倒置的扇面形,奋力向前推进,同时,造成了一个扇面形水流。”突出划船赶鸭子的速度很快,烘托了父子俩对前途感到茫然无助的心情。“鸭们十分乖巧。也正是在夜幕下的大水上,它们才忽然觉得自己已成了无家的漂游者了。”以鸭子“无家”“漂流”及对唯一的小船的依靠烘托出父子俩此时孤独、恐惧、无依的心理。“鸭们也长大了,长成了真正的鸭。它们的羽毛开始变得鲜亮,并且变得稠密,一滴水也不能泼进了。公鸭们变得更加漂亮,深浅不一样的蓝羽、紫羽,在阳光下犹如软缎一样闪闪发光。”以鸭子的成长象征着杜小康的成熟,深化了小说主题。芦苇荡“当杜小康一眼望去,看到芦苇如绿色的浪潮直涌到天边时,他害怕了……芦荡如万重大山围住了小船。杜小康有一种永远逃不走了的感觉。”渲染了空旷气氛,衬托出主人公内心的恐惧和害怕。暴风雨“天黑,河水也黑,芦苇荡成了一片黑海。杜小康甚至觉得风也是黑的……四下里,一片呼呼的风声和千万枝芦苇被风折断的咔嚓声。”推动故事情节的发展。 二、内容梳理(根据课文填空,理清文脉)《孤独之旅》展示了主人公杜小康在放鸭生活中,战胜恐惧和孤独而成熟、坚强的成长历程。小说命名为“孤独之旅”,蕴含深刻意味。课文中杜小康的人物形象,是在成长过程中丰满起来的。而小说中出色的自然环境描写又为人物成长提供了广阔的背景。文章明线与暗线交相辉映,互相衬托。孤独之旅背景:家道中落,失学放鸭开端:①______________——茫然、恐惧、恐慌(逃避)离家放鸭发展:芦荡放鸭——②__________________(面对)孤独,不再恐慌高潮:③___________——④______________(挑战)风雨寻鸭勇敢、坚强结局:⑤____________——⑥_________________(成熟)鸭子下蛋长大了,坚强了 名师导练基础训练1.根据拼音写出相应的词语。(1)油麻地jiādǐ()最厚实的一户人家,就是杜小康家。(2)他对杜小康带着哭腔的请求,zhìzhībùlǐ(),只是不停地撑着船,将鸭子一个劲儿赶向前方。(3)杜雍和现在只是要求它们向前游去,不停顿地游去,不肯给它们一点儿mìshí()或xīnào()的可能。(4)没有其他声音,天地又如此kōngkuàng(),因此,这叫声既让人觉得寂寞,又使人感到振奋。(5)未知的东西,似乎更能liáodòu()一个少年的心思。(6)此时,热气与chuīyān(),就再也无法分得清楚了。家底置之不理觅食嬉闹空旷撩逗炊烟 (7)鸭们十分guāiqiǎo()。(8)芦苇丛中还有一种不知名的香草,一缕一缕地chānzá()在芦叶的清香里,使杜小康不时地去用劲儿嗅着。(9)但,这一切无论如何也不能完全qūchú()杜小康的恐慌。(10)第二天,父子俩登上芦苇滩,找了一个合适的地方,用liándāo()割倒一大片芦苇,然后将它们扎成把。(11)不一会儿,暴风雨就xiēsīdǐlǐ()地开始了,顿时,天昏地暗,仿佛世界已到了末日。(12)然而这群平时很wēnshùn()的小东西,今天却都疯了一样,只顾没头没脑地乱窜。(13)一样的芦苇,一样chóngchóngdiédié()无边无际。乖巧掺杂驱除镰刀歇斯底里温顺重重叠叠 2.下列句子中加点的词语使用不恰当的一项是()A.群山都落在脚下,显得空旷高远,高得可以同月牙儿拉手,同太阳亲脸。B.勇敢的海鸥,乖巧的驯鹿,还有善良的蜘蛛夏洛,它们让那些原本苍白单调的梦境美好得如同一个个精致的蝴蝶标本。C.我想告诉你:要笑就笑个痛痛快快,要哭就哭个歇斯底里。想笑就笑,想哭就哭,不必在意别人的看法,活出真正的自己,做你自己。D.一个人越是成功,他所遭受的委屈就越多,要使自己的生命获得极值和炫彩,就不能太在乎委屈,要学会置之不理,淡然待之。D 3.下列句子中没有语病的一项是()A.自从来到这个世界上,人类就一直面临着两种挑战,一是来自人类本身,一是来自大自然。B.首赴亚丁湾、索马里海域执行护航任务的中国海军的三艘军舰和几十名特种兵,将进行登舰检查、与海盗火力对抗、保护相关海域通行船只的通行安全等活动。C.良好的心态,是我们参加中考能否取得成功的关键。D.作家莫言采用魔幻现实主义的手法,将民间故事和历史事实融入小说创作并取得杰出成就,被评为2012年“诺贝尔文学奖”。A 4.根据下面的材料,给“播客”下个定义。播客的英文名称为Podcast,我们将播客简单地视为个人的网络广播。播客作者借助数字广播技术,将其录制的声音和视频节目上传到互联网上,通过专门的软件与便携播放器实现播放,从而与广大网友分享。网友不必实时接收,也不必端坐在电脑前,可将节目下载到自己的播放器中,随时随地收听收看,自由地享受。____________________________________________________________________________________________________________________播客是借助数字广播技术将声音和视频节目上传到互联网以供分享的个人网络广播。 课内精读5.从描写手法等角度赏析下列句子在课文中的表达效果。(6分)(1)天黑,河水也黑,芦苇荡成了一片黑海。杜小康甚至觉得风也是黑的。临近中午时,雷声已如万辆战车从天边滚过来,不一会儿,暴风雨就歇斯底里地开始了。______________________________________________________________________________________________________________________________________________________________________________(2)雨后天晴,天空比任何一个夜晚都要明亮。杜小康长这么大,还从未见过蓝成这样的天空,而月亮又是那么的明亮。____________________________________________________________________________________________________________________通过环境描写和运用反复、比喻的修辞方法,极力渲染了环境的恶劣,既推动故事情节的发展,为杜小康找鸭群、与父亲走失、经历磨难做铺垫,又烘托出杜小康坚强勇敢的人物形象。通过对雨过天晴、月亮明亮的环境描写,烘托了杜小康经历暴风雨找到鸭群后的兴奋和感到自己忽然长大后的欣慰。 6.课文中杜小康经历暴风雨,找到走散的鸭子后,反而哭了。他为什么哭?谈谈你的理解。(4分)___________________________________________________________________________________________________________________________________________________________________________①暴风雨中追赶鸭子这样艰辛的生活不是他这个年龄的人应该承受的,所以委屈而哭;②他经历了这样的磨难,觉得自己忽然长大了,坚强了,所以激动得哭。 小说阅读技法专练——分析环境描写的作用好人的成本马德①去年盛夏的一天,一丝风也没有,天闷热得厉害。街道上,行人寥落,车辆仿佛也耐不住这火烧火燎的热,箭一般穿梭着。②午后刚过,先是有一阵劲风刮来,接着,豆大的雨点便噼里啪啦砸了下来。这雨多少有点猝不及防。街两边的摊档立刻忙乱了起来,大家都急着收摊,谁也顾不上谁。正准备打车往单位赶的他,见路边有一个衣服摊还没有拾掇完,便赶紧跑过去帮忙。这是一个女人的摊位,女人正忙乱而惊慌失措地收拾着。“大姐,我来帮你。”他喊了一嗓子。女人几乎连看都没顾上看他一眼就点了点头。他利落地把挂在衣架上的衣服摞在一起,又利落地用塑料布包裹起 来。还有一个小女孩,也跑前跑后地忙着。倒是女人颇为吃力,她把衣物整理在一起打包的时候,甚至用上了膝盖和下巴。着急也不至于这样吧,他觉得有些蹊跷。待女人转过身的时候,他发现,女人只有一条胳膊,而另一条被雨淋湿的空袖管,此刻正紧贴在上衣的前襟上。③他的心中立刻升腾起一种悲怆的责任感。他加快了速度,当更加密集的雨到来的时候,所有衣物都已经装好并放置在一旁的农用三轮车里,就连布篷、铁架等零散的东西,也都归置到了背雨的地方。④大家都湿透了。雨水顺着发梢流下来,模糊了他的眼睛,他抹了一把,四下环顾,见一切都收拾停当了,就和女人打招呼,说:“大姐,我走啦。”女人拉着女孩的手,眼中满浸着泪水,嘴唇哆 嗦着,似乎要说什么,但说不出来。他看着女人紧贴在衣襟上的空袖管,也苦笑着,不知道说什么好。⑤与她们告别后,他的心里很不是滋味,毕竟,今天这个在生活的艰辛中挣扎的女人给了他很大的触动。刚走了没几步,那个女孩气喘吁吁地追上了他,女孩说:“叔叔,妈妈说,你是一个好人!妈妈下岗半年以来,你是第一个肯帮助妈妈的陌生人。谢谢你,叔叔!”说完后,女孩向他深深地鞠了一躬。⑥你是一个好人!坐在出租车中的他反复地默念着这句话。他回想着刚才所做的一切,他发现,在这个世界上,做一个好人,需要付出的,其实,并不是很多。车窗外,雨停了,迎面吹来了一股清凉的风,远处一条彩虹横架空中,是那么的美丽。(选自《高中生之友》,有删改) 7.文中第①段的环境描写运用了哪些修辞方法?突出了景物的什么特点?这处自然环境描写在文中起到什么作用?(5分)__________________________________________________________________________________________________________________8.文中多处写到雨,这样描写有什么作用?(4分)__________________________________________________________________________________________________________________9.文中第⑥段画线的句子有什么作用?(4分)__________________________________________________________________________________________________________________夸张、比喻、拟人。突出了天气的闷热。作用:交代了故事发生的时间、背景,为情节的发展做铺垫。推动了故事情节的发展,渲染出女人悲苦挣扎的形象特点,衬托出他善良高大的形象。通过环境描写,渲染了明亮轻快的气氛,烘托他帮助人后愉悦的心情,突出他善良的品质和乐于助人的好人形象。 中考考点实训文学类文本阅读模拟训练路灯下的身影①父亲是一名工人,一名普普通通的工人。但父亲用他的双手为我撑起了一片天,一片阳光明媚的天。②一次,鹅毛大雪纷纷扬扬地飘舞着,寒风放肆地呼啸着。“当——”已经12点了,早已疲倦的我伸着懒腰,这么晚了,爸爸还没有回来,我焦躁地拉开了书桌前的窗帘。大雪被寒风吹得乱舞,迷蒙了我的双眼。小街上的路灯还亮着,我忽然看见路灯下有个人影在徘徊,这身影是如此熟悉,真是太熟悉了,像爸爸,太像了,可是爸爸不是去朋友家做客了吗?那身影突然停了下来, 仰头朝我的小阁楼上望着。③是爸爸,是他。他似乎在不停地咳嗽着。我的心里不禁一颤,明白了许多。爸爸的老毛病又犯了,为了不影响我的学习,为了给我一个安静的学习环境,他每天晚上总说去拜访亲戚,而那只是他的托词罢了,也许他每天都在这昏黄的路灯下徘徊。腊月里的天冷得让温室里的我都缩手缩脚的,更何况是街道上的爸爸呢?而且他的身体又那么差……④我的眼睛湿润了,泪在眼眶里打着转儿,我强忍着不让它流下来。外面的雪似乎又大了起来,爸爸还在灯下徘徊,他不停地咳嗽着,咳得很厉害,我只看见他的肩膀在不停地颤抖着。他搓着手,跺着脚,终于,他难过地蹲下了身子,很艰难地。我的泪顿时像断了线的珠子不停地往下滴,我拉开了窗户,朝着街道上喊:“爸爸, 爸爸……”我哽咽了,一句话都说不出来,只觉得心里好乱好乱。爸爸艰难地站起来,冲我尴尬地说着:“爸爸酒喝多了,蹲在这歇一会儿,不早了,你也该休息了。”⑤我抓起爸爸的外套便往楼下奔,朝街道上跑去,我哭,不停地哭,冷风不禁让我连打了几个冷战。爸爸心疼地跑过来,搂着我说:“别着凉,以后爸爸一定早点回来。我那朋友真是太热情了,不停地让我喝酒……”他还在说着什么,我没有在意,只觉得那些飞舞的雪花好美好美……(选自江苏中考作文,有删改) 1.文章第②段的画线句子属于什么描写?有什么作用?(4分)___________________________________________________________________________________________________________________________________________________________________________2.“外面的雪似乎又大了起来,爸爸还在灯下徘徊,他不停地咳嗽着,咳得很厉害”一句中“似乎”一词能否删去?为什么?(4分)__________________________________________________________________________________________________________________3.文章结尾处写道:“我没有在意,只觉得那些飞舞的雪花好美好美……”这里对雪花的描写有什么作用?(4分)__________________________________________________________________________________________________________________环境描写。渲染风雪之大,天气之冷,烘托父亲为了“我”能安静学习而愿意遭受风雪折磨的伟大形象,突出父爱的深沉和伟大。不能。“似乎”表明是“我”主观上的感受,渲染了故事的氛围,反衬出父爱的深沉、伟大。通过环境描写,表现了“我”对父亲的理解,深化了主题,讴歌父爱的无私。 4.“那些飞舞的雪花好美好美”,为什么“我”觉得“雪花好美好美”?(5分)____________________________________________________________________________________________________________________因为洁白的雪花是纯洁的象征,它正像父亲关爱子女、为他人着想的高尚的品质一样,所以“我”觉得今天的雪花格外地美。 2019秋部编版九年级语文上册第五单元第十七课中国人失掉自信力了吗 名师导学17中国人失掉自信力了吗一、课前自学知识储备1.议论文的论证方式——立论和驳论:(1)立论:就一定的事件或问题,提出并阐明自己的见解或主张。(2)驳论:就一定的事件或问题发表议论,批驳片面的、错误的,甚至反动的见解或主张。2.批驳对方的论点主要有三种方式——驳论点、驳论据、驳论证:(1)直接批驳(驳论点):通过论述敌论据不能证明敌论点,即论证不合逻辑驳斥对方论点,这是直接批驳。 (2)间接批驳(驳论据、驳论证):从正面提出论点,与对方针锋相对,证明这个论点的正确性就等于宣告对方论点不成立,这是间接批驳。一课一法1.掌握课文的批驳方式:《中国人失掉自信力了吗》是一篇驳论文,作者逐层深入地批驳——先直接批驳对方的观点是错误的,再以正面立论的方法间接批驳对方的观点。2.体会课文的批驳的语言特色——富有战斗性和讽刺意味:课文在语言上巧用排比、反问、仿拟等修辞方法,增强语言的说服力和感染力。(1)“总”“一味”“只”……这一系列词语栩栩如生地勾勒了国民党官僚和所谓社会名流的邪恶脸谱。 (2)“自信力”“他信力”“自欺力”这三个概念都有共同的语素——“力”,作者采用仿拟的修辞方法,使后两个新造词与原词“自信力”一对照,显出极其深刻的讽刺意义。二、内容梳理(根据课文填空,理清文脉)作者以敌论据不能证明敌论点为“突破口”,运用反驳对方论证的方法,通过正面立论和间接反驳相结合的手法,一举击中要害,反驳强劲有力。论证思路上采用揭示对方谬误——直接反驳——间接反驳——得出结论的顺序逐层推进,语言具有辛辣的讽刺特色,全文可分为三个部分: 中国人失掉自信力了吗摆谬论(第1~2段)敌方论据:自夸——只希望着国联——求神拜佛,怀古伤今敌方论点:①____________________________中国人失掉自信力了驳论点(第3~5段):②_________批驳直接失掉的是“③__________”他信力发展着“④__________”自欺力驳论证(第6~8段):⑤_________批驳正面立论:⑥______________________________________________________摆出论据:⑦______________________________________________________间接我们有并不失掉自信力的中国人在从古至今满怀自信的“中国的脊梁”得结论(第9段):自信力的有无,状元宰相的文章是不足为据的,要自己去看地底下 名师导练基础训练1.根据拼音写出相应的词语。(1)现在是既不夸自己,也不信国联,改为一味求神拜佛,huáigǔshāngjīn()了——却也是事实。(2)信“地”和“物”,还是切实的东西,国联就miǎománg(),不过这还可以令人不久就省悟到依赖它的不可靠。一到求神拜佛,可就xuánxū()之至了,有益或是有害,一时就找不出分明的结果来,它可以令人更长久的mázuì()着自己。怀古伤今渺茫玄虚麻醉 (3)我们从古以来,就有máitóukǔgàn()的人,有拼命硬干的人,有为民请命的人,有shěshēnqiúfǎ()的人。(4)他们在qiánpūhòujì()的战斗,不过一面总在被cuīcán(),被抹杀,消灭于黑暗中,不能为大家所知道罢了。(5)要论中国人,必须不被搽在表面的zìqīqīrén()的脂粉所kuāngpiàn(),却看看他的筋骨和脊梁。自信力的有无,状元宰相的文章是bùzúwéijù()的,要自己去看地底下。埋头苦干舍身求法前仆后继摧残自欺欺人诓骗不足为据 2.下列句子中加点的词语使用不恰当的一项是()A.梦境,星星点点,支离破碎;梦想,美如笑靥,遥不可及;追梦,坚持不懈,不再渺茫。B.可是请你告诉我,一种暂时忘记你的方法,即使是刹那的麻醉也好,这样我就不会在有任何一点愁思的时候立刻想到你。C.当699万高校毕业生深陷“史上最难就业季”时,更多前仆后继走出国门的准留学生们不得不开始思考,什么专业在将来会有更好的就业前景。D.中学生课外阅读要有理性,要坚定方向,要有选择,要读那些真正能体现人文和社会价值的书,“畅销书”和“排行榜”是不足为据的。C 3.下列对病句的修改不正确的一项是()A.食物总有相互抵触发生反应的时候,所以牛奶中切忌不要添加果汁等酸性饮料。(在“食物”前加“因为”)B.据中国社科院相关部门预测,今年中国网络购物总额接近5000多亿元,占社会消费品零售总额的3%。(删去“接近”或“多”)C.难道我们能否认获得诺贝尔奖的屠呦呦不是中国人的骄傲吗?(删去“不”)D.当前,宏观调控将更加注重有保有压,对那些过剩和落后的产能要坚决淘汰,绝不允许死里逃生。(把“死里逃生”改为“死灰复燃”)A 4.仿照画线句子,在后面续写两句话,要求内容相关,句式整齐。初中三年,光阴荏苒。忆同学少年,良多趣味。我们曾谈曹操青梅煮酒,纵论天下英雄;___________________________,___________________;_______________________________,____________________……如今,这些都如片片枫叶,珍藏在你我青春的诗集里。我们曾谈李白举头望明月细诉思乡情怀我们曾吟诵周敦颐爱莲篇章立下君子之志 课内精读5.赏析课文中精妙的句子。(6分)(1)课文开头,“总自夸着‘地大物博’”“只希望着国联”“一味求神拜佛,怀古伤今了”,充满着嘲讽意味,请结合课文内容赏析加点字词的表达效果。__________________________________________________________________________________________________________________________________________________________________________________________________________________________________________________________________________________________________“总”,一副自我炫耀、夜郎自大的样子;“只”,一个仰人鼻息、抓救命稻草的形象;“一味”,一副沉迷于其中不能自拔、顽固不化的样子。这些尖锐泼辣的语言,栩栩如生地、漫画式地画出了悲观论者的脸谱,给读者留下深刻印象。 (2)“状元宰相的文章是不足为据的,要自己去看地底下”一句中“状元宰相”和“地底下”分别指的是什么?这样表达有什么作用?____________________________________________________________________________________________________________________________________________________________________________________________________________________________________6.“‘自欺’也并非现在的新东西,现在只不过日见其明显,笼罩了一切罢了。然而,在这笼罩之下,我们有并不失掉自信力的中国人在。”这个句子在内容和结构上有什么作用?(4分)__________________________________________________________________________________________________________________“状元宰相”喻指当时能“公开的”国民党发动政客及其御用文人的言论;“地底下”喻指当时还处于地下斗争的中国共产党及其领导下的革命力量。这些充满嘲讽的语言充分表明了鲁迅先生立场坚定,爱憎分明。内容上是从正面立论,使对方的论点不攻自破;结构上是个过渡段,承上启下。 7.下列对课文的理解,不正确的一项是()(3分)A.文章有直接反驳论据,也有从正面立论使对方的论点不攻自破的间接批驳,立驳结合,增加了文章的说服力。B.文章运用了多种论证方法:第3、4段结合事实讲道理,第7、8段主要运用了事实论证的论证方法。C.“这就是中国的脊梁”运用了比喻论证的论证方法。D.“中国人失掉自信力了吗”是个反问句,既点明本文要反驳的论题,又表明了作者的观点。D 议论文阅读技法专练——驳论文阅读训练安贫乐道法鲁迅①孩子是要别人教的,毛病是要别人医的,即使自己是教员或医生。但做人处世的法子,却恐怕要自己斟酌,许多别人开来的良方,往往不过是废纸。②劝人安贫乐道是古今治国平天下的大经络,开过的方子也很多,但都没有十全大补的功效。因此新方子也开不完,新近就看见了两种,但我想:恐怕都不大妥当。③一种是教人对于职业要发生兴趣,一有兴趣,就无论什么事,都乐此不倦了。当然,言之成理的,但到底须是轻松一点的职业。且不说掘煤、挑粪那些事,就是上海工厂里做工至少每天十点的工 人,到晚快边就一定筋疲力倦,受伤的事情是大抵出在那时候的。“健全的精神,宿于健全的身体之中”,连自己的身体也顾不转了,怎么还会有兴趣?——除非他爱兴趣比性命还利害。倘若问他们自己罢,我想,一定说是减少工作的时间,做梦也想不到发生兴趣法的。④还有一种是极其彻底的:说是大热天气,阔人还忙于应酬,汗流浃背,穷人却挟了一条破席,铺在路上,脱衣服,浴凉风,其乐无穷,这叫作“席卷天下”。这也是一张少见的富有诗趣的药方,不过也有煞风景在后面。快要秋凉了,一早到马路上去走走,看见手捧肚子,口吐黄水的就是那些“席卷天下”的前任活神仙。大约眼前有福,偏不去享的大愚人,世上究竟是不多的,如果精穷真是这么有趣,现在的阔人一定首先躺在马路上,而现在的穷人的席子也没有地方铺开来了。 ⑤上海中学会考的优良成绩发表了,有《衣取蔽寒食取充腹论》,其中,有一段:“……若德业已立,则虽饔飧不继,捉襟肘见,而其名德足传于后,精神生活,将充分发展,又何患物质生活之不足耶?人生真谛,固在彼而不在此也。……”(由《新语林》第三期转录)⑥这比题旨更进了一步,说是连不能“充腹”也不要紧的。但中学生所开的良方,对于大学生就不适用,同时还是出现了要求职业的一大群。⑦事实是毫无情面的东西,它能将空言打得粉碎。有这么的彰明较著,其实,据我的愚见,是大可以不必再玩“之乎者也”了——横竖永远是没有用的。 8.通读全文,概括说说作者批驳了什么观点。(2分)__________________________________________________________9.请简要分析本文的论述思路。(4分)____________________________________________________________________________________________________________________________________________________________________________________________________________________________________________________________________________________________________________________________________________________________劝人安贫乐道的方法是不妥当的。首先由别人所教的为人处世之道并不适用于自己来引出观点:劝人安贫乐道的方法是不妥当的;然后针对两种劝人安贫乐道的方法,指出其不妥和错误;接着列举中学生作文考试中的不当言论,指出其表现是言行不一的;最后进一步点明自己的观点,即不要再用冠冕堂皇的空话劝人安贫乐道了,在现实面前,如此做是毫无作用的。 议论文阅读模拟训练重拾汉语的母语自信纪秀生①母语通常是指一个人所属民族的群体语言,是一个民族最为鲜明的符号标志,它能把一些人凝聚为一个共同体。母语自信则是对母语品质和母语生命力的坚定信念,体现了一个国家和民族对自身母语价值的充分肯定和尊重。②每个民族对自己的母语都有一种天然的认同感和自豪感,视其为最美的语言。汉语是中国人的母语,是中华民族的文化徽章。汉语拥有最为丰富的书面文献和自公元前841年以来不曾中断的历史记录,在相当长的时间内,它因其承载的文化先进性而对世界产生了不小的影响。这让我们自豪,也增强了我们对汉语的中考考点实训 自信。然而,在近现代,随着社会的发展,加之西方语言和文化的冲击,我们对汉语的自信发生了动摇,这给汉语带来了危机,造成了伤害。五四运动期间,汉语严重欧化,有学者甚至主张用拼音来代替汉字;20世纪80年代,汉语的计算机输入方式成为难题,汉字又蒙受了“笔画复杂、输入困难”的责难;即使是现在,国人对外语的重视程度依然远远超过汉语……因此,重拾汉语的母语自信是每个中国人必须正视的问题。③对母语的不自信,究其本质,是对自身文化的不自信。在当今形势下,我们更应清醒地认识到,中华文化是我国生存和发展的软实力,是中华民族的命脉和灵魂。而汉语既是这种文化的产物,也是这种文化的繁荣与发展,就不能再对汉语生发那种“自贬、自侮、自戕”的感觉,而应重新拾回对汉语的自信。 ④事实上,从语言学角度看,汉语也的确有值得我们自信的理由。汉语是世界上历史最悠久、发展水平最高的语言之一。在世界语言中,汉语具有独一无二的简洁而富有想象力的特点,精通多国语言的季羡林先生称汉语是世界语言里最简练的一个语种,同样表达一个意思,如果英语要60秒,汉语5秒就够了。汉语在表达的自由度和创造性上具有无可比拟的优越性。美国语言学家范诺萨说,汉语充满动感,不像西方语言那样被语法、词类规格套死。汉语有声调,音韵铿锵,和谐美丽。瑞典语言学家高本汉认为,在这方面,汉语比西方语言略胜一筹。⑤进入21世纪,随着我国影响力的不断增强,汉语在全球快速传播,国际地位显著提高,不断升温的“汉语热”不仅强化了人们对汉语的认同,旺盛的需求也为汉语“走出去”增添了信心。 据英国语言学家格雷多尔研究预测,到2050年,汉语将上升到国际语言分类金字塔的最高一层。因此,我们更应重拾汉语的母语自信,做一个根基牢固的中国人。(选自中国社会科学网,有删改) 1.下列说法与原文不相符的一项是()(3分)A.汉语是中国人的母语,是中华民族的文化徽章,是中华民族的命脉和灵魂。B.汉语是世界上历史最悠久、发展水平最高的语言之一。C.汉语在表达的自由度和创造性上具有无可比拟的优越性。D.进入21世纪,随着我国影响力的不断增强,汉语在全球快速传播,国际地位显著提高。A 2.请简要分析第②段的论证思路。(4分)_____________________________________________________________________________________________________________________________________________________________________________________________________________________________________________________________________________________________3.下面两则材料分别可以用作本文哪一段的论据?请选择其中一则材料简述理由。(3分)材料一:国学大师钱穆说过,汉语之美在于“精”。“语种山果落,灯下草虫鸣”,仅10个字,不仅使我心与天地大生命融凝合一,又兼及自然科学。着语不多,会心自在深微处。首先提出“汉语是中国人的母语,是中华民族的文化徽章”的分论点;接着从正面举了自公元前841年以来汉语在历史上不曾中断的记录以及对世界的影响来论证论点;然后从反面举出近现代以来汉语遭受的冲击以及面临的危机的史实;最后强调了“重拾汉语的母语自信是每个中国人必须正视的问题”。 材料二:德国语言学家洪堡特说,语言是一个民族所必需的“呼吸”,通过一种语言,一个人类群体才得以凝聚成民族,一个民族的特性才能获得完整的映照和表达。________________________________________________________________________________________________________________________________________________________________________________________________________________________________________________________________________________________________________________________________________________________________________________________________________________________________________________________________________示例一:材料一适合放在第④段。因为第④段的分论点是“汉语是世界上历史最悠久、发展水平最高的语言之一”,强调了汉语简洁而富有想象力;而材料一也强调了“汉语之美在于‘精’”的观点,与第④段分论点相吻合。示例二:材料二适合放在第①段。因为第①段强调的是“母语通常是指一个人所属民族的群体语言,是一个民族最为鲜明的符号标志,它能把一些人凝聚为一个共同体”;而材料二也强调了“通过一种语言,一个人类群体才得以凝聚成民族”,两个观点相吻合。 2019秋部编版九年级语文上册第五单元第十八课怀疑与学问 名师导学18怀疑与学问一、课前自学知识储备中心论点和分论点:(1)分论点:一篇文章有时可以有几个论点,其中一个论点是主要的,即中心论点;其余的论点就是分论点,分论点要服从中心论点,从几个方面来论证中心论点。(2)分论点之间的关系:一般有并列、正反对照和层进三种。(3)分论点的作用:有了分论点,可以使文章的层次更丰富,说理更充分,脉络更清晰。 一课一法体会课文中论据的作用——道理论据和事实论据:(1)道理论据及作用:(2)事实论据及作用:道理论据作用开头引用古代两位著名的大学问家的话。引出论点,运用道理论据证明中心论点,增强论点的说服力。中间引用孟子的“尽信书则不如无书”的名言。引用古代大学者的话作为论据,证明了怀疑精神的重要性,对于增强文章的说服力起到重要的作用。事实论据作用①听说的口头消息。②听说古代有三皇、五帝。③听说“腐草为萤”。具体典型地论证了怀疑精神的重要性。戴震善问的例子。论证“怀疑是建设新学说、启迪新发明的基本条件”这个分论点。 二、内容梳理(根据课文填空,理清文脉)《怀疑与学问》论述了“学者先要会疑,学则须疑”的观点,强调了怀疑精神在治学过程中的重要作用,并提倡学者应有怀疑精神,不要随便盲从或迷信,是层层深入的递进式结构,全文可分为三个部分: 怀疑与学问第一部分(第1~2段):引用名言提出中心论点——①___________________________________________怀疑的精神是做一切学问的基本条件第二部分(第3~5段):论证分论点——②____________________________________________(消极方面)怀疑是辨伪去妄的必须步骤第三部分(第6段):论证分论点——③_____________________________________________(积极方面)怀疑是建设新学说、启迪新发明的基本条件 名师导练基础训练1.根据拼音写出相应的词语。(1)譬如在国难危急的时候,各地一定有许多口头的消息,说得如何xiōngxiǎn(),那便是别人的传说,不一定可靠;要知道实际的情形,只有靠自己亲身shìchá()。(2)这一番事前的思索,不随便qīngxìn()的态度,便是怀疑的精神。(3)我们若能这样追问,一切xūwàng()的学说便bùgōngzìpò()了。(4)否则便是mángcóng(),便是迷信。凶险视察轻信虚妄不攻自破盲从 (5)对于别人的话,都不打zhékòu()地承认,那是思想上的懒惰。(6)一切学问家,不但对于liúsú()传说,就是对于过去学者的学说也常常要抱怀疑的态度。(7)若使后之学者都mòshǒu()前人的旧说,那就没有新问题,没有新发明,一切学术tíngzhì(),人类的文化也就不会进步了。折扣流俗墨守停滞 2.下列句子中加点的词语用语得体的一项是()A.先生辱临寒舍,甚感荣幸。B.明早您在舍下等候,我准时前往请教。C.下午四点,我陪同来访的外国友人拜访您的家父。D.他大学毕业后,曾在一所山区小学就教十余年。A 3.下列对病句的修改不正确的一项是()A.逢进必考,取消加分规则,奠定了平民子弟进入公务员行列的通道。(把“奠定”改为“打开”)B.乌克兰危机的根本原因是国内民众在“向东走”还是“向西走”问题上的深度对立造成的。(删去“的根本原因”或“造成的”)C.改变“亚学习”状态,要合理安排时间,切忌不要打“疲劳战”。(删去“切忌”或“不要”)D.博物馆里,有些参观者衣衫不整、高声喧哗、在展厅里吃东西、乱丢垃圾和不文明行为,对博物馆里的其他参观者和展品都是一种不尊重的表现。(把“展品”和“其他参观者”对调)D 4.仿照画线句子,在后面续写一组句子。面对苦难,我们应该有足够的勇气,勇气是打败一切难题的有力武器,拿出勇气,勇敢地面对苦难,定会拨开云雾;面对困难,我们应该有足够的________,__________________________________,___________,_____________________,____________________。毅力毅力是消解一切困难的催化剂拿出毅力坚定不移地直面困难定会迎来灿烂的阳光 课内精读5.本文的中心论点是怎么提出来的?有什么作用?(4分)______________________________________________________________________________________________________________________________________________________________________________6.课文中“怀疑不仅是消极方面辨伪去妄的必须步骤,也是积极方面建设新学说、启迪新发明的基本条件”一句有什么作用?(3分)________________________________________________________________________________________________________________________________________________________________________________________________________________________________________本文的中心论点是借用古代学者程颐和张载的名言提出来的。这种写法,既开门见山地提出了论点,同时名言本身也是证明中心论点的有力论据,增强了论点的说服力。此句是承上启下的过渡句,也是文章的两个分论点,“怀疑不仅是从消极方面辨伪去妄的必须步骤”是总结上文,“也是积极方面建设新学说、启迪新发明的基本条件”既是文章的分论点,又自然地引出下文。 7.下列对课文分析理解,不正确的一项是()(3分)A.“经过‘怀疑’‘思索’‘辨别’三步以后,那本书才是自己的书,那种学问才是自己的学问”中,这三个步骤是按人们认识事物的过程,先对事物产生怀疑,再进一步思索疑点,辨明正确与错误,前后顺序不能调换,体现了议论文语言的严谨性。B.课文第4段主要运用了对比论证、举例论证的论证方法。C.课文引用孟子的话是道理论证,有力地证明了怀疑精神的重要性,从而证明中心论点,增强说服力。D.作者举出戴震治学善问的事例从反面论述了许多大学问家都是从怀疑中锻炼出来的。D 议论文阅读技法专练——理清中心论点和分论点的关系品读经典朱万曙①林语堂曾说:“读书,开茅塞,除鄙见,得新知,增学问,广识见,养性灵。”读书可以使人增长学问见识,领悟为人处世的道理,即“开茅塞,除鄙见”,这是阅读大多数书籍都可以带来的好处。读书还可以“养性灵”,这种好处,则非品读经典而不可得。品读经典可以养成知识丰富、道德高尚、情趣健康的性灵,可以让人生从浮躁走向宁静,从浅陋走向优雅。总之,品读经典可以培养高尚心灵。②何谓经典?唐代史学家刘知几说:“自圣贤述作,是曰经典。”他认为,古代圣贤所述所作的就是经典。这个解释有些道理。 圣贤的思想往往能够洞穿古今,必然是经典。但问题又来了,圣贤何以成为圣贤?实际上,还是靠其著述留存后世,为后人所接受、认可、推崇。有学者曾总结过经典的特性:传世性、权威性、耐读性、累积性。可以说,经典之所以成为经典,关键在于其蕴含的思想精髓能触及人们的心灵。它们代表着时代精神,能够穿越时空、启迪后人。因而,经典是历史选择出来的最具价值的书籍。③品读经典,人们可以沐浴思想的光华,感受圣贤哲人的思考。__________又如,马克思、恩格斯的著作历来为共产党人所推崇,百读不厌,回味无穷。品读马克思、恩格斯的经典著作,不仅有助于我们用辩证唯物主义和历史唯物主义的立场、观点、方法来研究人类社会的各种现象和问题,而且可以引发我们对世界观、人生观、价值观的深入思考,启迪智慧,荡涤心灵。A ④品读经典,人们可以穿越到遥远的古代,仿佛身临历史现场,感受古人的风采。__________《史记》中荆轲刺秦王,一番悲壮,一幕惊险。从《史记》中可以看出,作者司马迁的追求是“究天人之际,通古今之变,成一家之言”。他将失败的英雄项羽列入本纪,为匈奴作列传,还专辟《货殖列传》记录那些在当时为世人所不屑的商人。品读这部历史经典,我们可以读出并且学习司马迁的大气魄、大胸怀。⑤品读经典,可以给人以美的享受。“所谓伊人,在水一方”,人美景也美;“七八个星天外,两三点雨山前”,让人感到清新、舒旷,意境悠长。阅读叙事文学作品,就如同步入历史人物长廊:刘兰芝、焦仲卿的悲剧让人感叹,巾帼英雄花木兰的形象令人叹奇,梁山泊一百零八好汉的忠义令人回肠荡气,等等。文学经典往往也B 是历史经典。例如,杜甫的“三吏三别”,书写了安史之乱的历史场景。__________又如,一部《红楼梦》通过对贾家的细致描写,揭示了封建社会大厦将倾的历史趋势。⑥品读经典未必能让人学会一项生存技能,但可以培养高尚的心灵。__________经典蕴含的深刻哲思、美妙文辞,给人带来的并非单一的启迪,而是多元的文化熏陶,使我们在潜移默化中气质得到提升、心灵得到洗礼、心胸变得开阔、见识更加高远。(选自《人民日报》,有删改)CD 8.请找出文章的中心论点和分论点。(4分)____________________________________________________________________________________________________________________________________________________________________________________________________________________________________9.文章第④段主要运用了什么论证方法?有什么作用?(3分)____________________________________________________________________________________________________________________________________________________________________________________________________________________________________中心论点:品读经典可以培养高尚心灵。分论点:①品读经典,人们可以沐浴思想的光华,感受圣贤哲人的思考;②品读经典,人们可以穿越到遥远的古代,仿佛身临历史现场,感受古人的风采;③品读经典,可以给人以美的享受。举例论证。以《史记》中的史实为例,充分论证了“品读经典,人们可以穿越到遥远的古代,仿佛身临历史现场,感受古人的风采”这一分论点。 10.下面这句话放在文中A、B、C、D哪一处最为合适?请简述理由。(3分)孔子所强调的仁者爱人,一直是中华民族宝贵的精神财富,在今天依然具有不可替代的价值。____________________________________________________________________________________________________________________________________________________________________________________________________________________________________放在A处最为合适。因为孔子是古代圣贤,仁者爱人的思想富有哲理性,是经典论据,能论证“品读经典,人们可以沐浴思想的光华,感受圣贤哲人的思考”的分论点。 中考考点实训议论文阅读模拟训练学贵质疑王菊①我们说有成就的人有“学问”,既然是“学问”,那么就要既“学”且“问”。“问”从何来?从疑而来。只有多疑、善疑、质疑、探疑,才能获得渊博的学识,用之于我们的事业。所以我们说:学贵质疑。明朝学者陈献章说:“学贵知疑,小疑则小进,大疑则大进。疑者,觉悟之机也。”说的正是这个道理。②人们常常把知识比作海洋,海洋是无边无际的,知识也是无止境的。一个人,无论他有多大的学问,总会有无知的地方,而多疑、善疑、质疑、探疑则是获取新知识的途径。正是基于这 一点,法国伟大作家巴尔扎克说:“打开一切科学大门的钥匙毫无疑义的是问号,而生活的智慧,大概就在于逢事都问个为什么。”的确如此,如果没有达尔文对“神创论”的怀疑,就不会有“自然选择学说”的确立;如果没有哥白尼对“地心说”的怀疑,也不会有“日心说”的创立。所以说,只有疑才能使我们的智慧之树开出艳丽的花,结出丰硕的果。③但是,我们必须明白,疑是建立在丰富的知识和认真思考之上的,绝不是无端的猜疑或随便的怀疑。达尔文对“神创论”的怀疑,并不是一时的心血来潮,而是在于他随“贝格尔”号帆船环球旅行五年,观察和采集了大量的动植物标本,考察和研究了无数的地质资料,经过综合探讨之后,才向根深蒂固的“神创论”发出了强有力的挑战。这是一场真理对谬误的挑战,其结果自然是真理胜利。可见,任何有效的怀疑,都依赖于对事实的仔细分析和对理论的深入研究。 ④可是我们有许多青年,他们不善于怀疑,不善于发现。他们相信,凡是书上写的便是正确的。凡是前人说的便是真理。他们迷信书本,崇拜前人,不敢越雷池一步。这样的人,自然不会有什么发现,更不可能有什么创见。这样的人多了,势必会影响社会的进步和文明的发展。所以我们必须提倡怀疑精神。⑤地质学家李四光曾对他的学生说:“不怀疑就不能见真理。”这句话对我们也同样适用。我们要增长知识,寻求真理,就必须多疑、善疑,而且质疑、探疑。这才是我们打开知识大门的金钥匙。年轻的朋友们,勇敢地拿起这把钥匙,去打开科学的大门吧!(选自《中国中学议论文写作大全》,有删改) 1.下列对文章的分析理解,不正确的一项是()(3分)A.文章的中心论点是学贵质疑。B.作者先从阐述“学”与“问”的关系入手,然后以一个设问句搭桥,自然地引出了全文议论的话题,并引用陈献章的话来证明论点。C.文章先指出“疑”的重要性,再进一步阐述了“疑是建立在丰富的知识和认真思考之上的”观点,接着列举青年不善于怀疑和发现的事例从反面论述观点,这样层层深入论述,条理清晰,说理全面。D.“这才是我们打开知识大门的金钥匙”一句运用比喻论证的方法,生动形象地论证了质疑精神的重要性,从而增添了议论文语言的严谨性。D 2.下列事例能作为“学贵质疑”的事实论据的一项是()(3分)A.苹果从树上掉下来,牛顿就有了疑问:苹果为什么落地呢?他据此深思,终于发现了万有引力。B.数学家陈景润平常走路时也思考一些疑难问题,有一次竟撞在树上,把眼镜也弄破了,就凭着这股精神,他终于在数学领域取得了突出成就。C.清代学者阎若璩从小愚钝,记忆力差,还患有口吃的毛病,但他毫不气馁,不懂就问,长期刻苦勤奋,终于成为著名的考据学家。D.孔子曰:“学而不思则罔,思而不学则殆。”A 3.选文第②段主要运用了什么论证方法?有什么作用?(4分)________________________________________________________________________________________________________________________________________________________________________________________________________________________________________举例论证。选取达尔文“自然选择学说”的确立和哥白尼“日心说”的创立两件事例,具体典型地论证了文章中“多疑、善疑、质疑、探疑则是获取新知识的途径”这一观点,从而证明“学贵质疑”的中心论点。 2019秋部编版九年级语文上册第五单元第十九课谈创造性思维 名师导学19*谈创造性思维一、课前自学知识储备1.设问:一种无疑而问、自问自答的修辞方法,又是一种常用句式。2.议论文中设问句的作用:(1)内容上:提醒读者注意,引导读者思考,突出某些内容,使文势有变化,波澜起伏。(2)结构上:起承接、过渡的作用,使文章结构紧密,条理清楚。 一课一法理解课文中设问句的作用:(1)文章的开头,先列出四个图形及思考题,随即提出设问:“对上面这个问题,你是怎么回答的呢?”——引出“事物的正确答案不止一个”的观点。(2)第4段末尾提出设问:“创造性的思维必须具备哪些条件呢?”——激发思考,引出下文。(3)第9段:“这种创造性的思维是否任何人都具备呢?是否存在富有创造力的人和缺乏创造力的人的区别呢?”——激发思考,自然过渡。 二、内容梳理(根据课文填空,理清文脉)《谈创造性思维》开头非常精彩,以一个有趣的问题为例,告诉我们“正确答案不止一个”——一种重要的思维方式;接着又以设问的形式告诉我们创造性思维必须具备的条件;然后告诉我们人人都具有创造力,只要不断积累知识,不满足于一个答案,而去探求新思路,一旦产生小的灵感,相信它的价值,并锲而不舍地把它发展下去,就能成为一个富有创造性的人。全文可分为三个部分: 谈创造性思维第一部分(第1~3段):引论——举数学分析题证明①_________________________正确答案不止一个推论②______________________________不满足于一个答案,不放弃探求第二部分(第4~12段):本论如何寻求“第二种答案”——③___________________有赖于创造性的思维创造性思维的条件④________________________________汲取知识,使自己学识渊博⑤__________________________________________________________________有探求新事物并为此而活用知识的态度和意识⑥________________________________持之以恒地进行各种尝试创造性思维是否任何人都具备心理专家的调查——⑦_______论证道理贝多芬、爱因斯坦等——⑧_______论证举例由此看来:⑨____________________________________拥有创造力的人留意自己细小的想法第三部分(第13段):任何人都拥有创造力 名师导练基础训练1.根据拼音写出相应的词语。(1)“正确答案只有一个”这种思维模式,在我们头脑中已不知不觉地ɡēnshēndìɡù()。(2)由于情况的变化,原来xíngzhīyǒuxiào()的方法,到了现在往往不灵了。(3)因此,不满足于一个答案,不放弃tànqiú(),这一点非常重要。(4)富有创造力的人总是zīzībùjuàn()地汲取知识,使自己学识渊博。根深蒂固行之有效探求孜孜不倦 (5)因为这些知识随时都可能进行组合,形成新的chuàngyì()。(6)在此基础上,chízhīyǐhéng()地进行各种尝试。(7)他将原来毫不相关的两种机械——葡萄yāzhà()机和硬币打制器组合起来,开发了一种新机械。(8)bùyánéryù(),在创造的宇宙里,这些人是光辉灿烂的明星。(9)然而在大多数情况下,即便是他们,也并非qīnɡéryìjǔ()就能获得如此非凡的灵感。创意持之以恒压榨不言而喻轻而易举 2.下列句子中加点的词语运用不恰当的一项是()A.马云认为,做企业不仅仅是做创意,创意是企业运营中的重要的一环,但它只是一环,更重要的是把每项工作落到实处。B.群众中有丰富的生产上的各种经验,我们认真学习和汲取,对我们的工作有很大的意义。C.陈老师课堂上孜孜不倦的诉说,课外无微不至的关怀,抚慰了单亲孩子孤寂的心。D.一座书店对于城市的意义不言而喻,世界著名钢琴家鲁宾斯坦有一句话:“评价一座城市,要看它拥有多少书店。”C 3.下列对病句的修改不正确的一项是()A.通过设立交通安全宣传站,发放宣传材料,讲解安全常识,使市民增强了安全意识。(删去“使”)B.导致青少年营养不良的原因主要是偏食挑食、吃零食过多、为追求身材过度节食造成的。(删去“造成的”)C.很多人喜欢运动,但专家提醒,适量运动才能增加人体的免疫力,运动要讲究科学性。(把“提醒”改为“警告”)D.除公益放鱼环节外,本届太白湖放鱼节,还有超模大赛、航空模型展演等。(在句末加“活动”)C 4.阅读下面的材料,按要求作答。本报讯(实习记者唐剑)端午节当日,记者在西宁市街头看到,各种颜色鲜艳、做工精美的手工香包,颇受市民的青睐。30日上午8时,记者在西宁市城中区南川东路早市看到,许多市民围着一个老奶奶的香包摊,争相购买各色香包,元宝、口袋、莲花等多种造型的香包非常吸引人的眼球。“这种是‘元宝香包’,有招财的意思;这是‘福袋香包’,有带福的意思!”在老奶奶的讲解宣传下,一位中年女性一下就买了10个。在采访中,很多市民表示,端午节是我国传统的节日,虽然很多传统习俗已经慢慢变淡,但西宁人的端午节依然要过得古色古香:粽子要吃,尤其手工香包一定要戴。人们相信端午节戴香包,可以祛邪祈福。所以,端午节这天老奶奶手工香包的生意格外好。 (1)请用一句话概括以上新闻的内容,不超过30字。_________________________________________________________(2)根据材料内容,以“端午节”为描述对象,用上一种修辞方法,写一句话。___________________________________________________________________________________________________________________端午节手工香包颇受西宁人青睐。端午节,人们吃粽子,是在咀嚼遥远鲜活的记忆;人们戴香包,是在祈求美好幸福的未来。 课内精读5.下列句子中加点的词语能否删去?为什么?(6分)(1)富有创造力的人总是孜孜不倦地汲取知识,使自己学识渊博。______________________________________________________________________________________________________________________________________________________________________________(2)发挥创造力的真正关键,在于如何运用知识。________________________________________________________________________________________________________________________________________________________________________________________________________________________________________不能删去。因为“总是”强调了“富有创造力的人”都这样“孜孜不倦地汲取知识”,无一例外,如果删去了就没有强调的意味,表示的范围也不明确。不能删去。“真正关键”强调了“如何运用知识”是发挥创造力的最重要的要素,如果删去这个词语,则表明该要素是一般的,得不到突出和强调。 6.请阅读下面的材料,尝试着用课文中的观点分析池田菊苗发现味精的原因。(4分)味精的发明,纯属一种偶然。1908年的一天,日本帝国大学的化学教授池田菊苗在品尝海带黄瓜片汤时,发现汤的味道特别鲜美。“也许海带里有奥妙。”职业敏感使他细细研究起海带来。半年后,池田菊苗教授从海带中成功提取出谷氨酸钠这一化学物质——味精的主要成分。__________________________________________________________________________________________________________________①池田菊苗精通化学知识;②他敢于探求新事物、活用知识进行尝试;③他关注一闪念的想法也就是所谓的灵感。 议论文阅读技法专练——品析重点语句的作用①创新从哪里来?首先就需要有那么一种生存与发展的紧迫感与危机感。500年来,世界经济中心几度迁移,背后的重要力量正是创新。创新能力不足,“卖一台电脑只能赚一捆大葱钱,卖10亿件衬衫才能换一架波音飞机”的故事就会不断上演。看当下转型升级,谋长远富民强国,创新都处于核心位置。当年,钱学森为让祖国“挺直腰杆子”,执着于“两弹一星”研制,立下卓著功勋;焦裕禄为让乡亲们过上好日子,宵衣旰食,在漫天风沙漫地碱的兰考创造出多种治沙新招。从根本上说,正是这种报效祖国、发展国家的理想与抱负,使无数人焕发出执着创新的勇气与锐气,踏平坎坷以成大道。②然而,创新之路上,失败是最亲密的伙伴,理想与热情往往 会遭遇无数次失败的无情打击。在相当意义上说,创新能力取决于这种抗打击能力。大量科研创新的事例显示,科研创新的成功率仅为10%左右。如果没有对190多次失败的无畏,屠呦呦怎么能提取到青蒿素?倘若不能善待一次次失败,诺贝尔如何能发明炸药?所谓大师,其实是失败最多的人。创新是壮丽的探险,只有不惧困难和挫折,视失败为“在成功路上”,并以“失败虐我千万遍,我待创新如初恋”的精神砥砺前行,方有可能踏着失败拾级而上,直至梦想的顶峰。(节选自李秦卫的《涵养创新的精气神》,有删改) 7.选文第①段的画线句子在文中有什么作用?(3分)__________________________________________________________________________________________________________________8.选文第②段中的画线句子有什么表达效果?(3分)__________________________________________________________________________________________________________________第①段的画线句子是设问句,既能激发读者思考,又能自然地引出下文,使得行文条理清晰。第②段的画线句子是反问句,强调了“抗击失败才能在创新之路上取得成就”的观点,增强语气。 ①漫漫岁月,茫茫人海,生活道路上无不充满坎坎坷坷,如生活困难、高考落榜、转干不成、升职无望、体质不佳、借贷无门、办事受阻、无端受控等天灾人祸。不管你喜欢不喜欢,不管你愿意不愿意,挫折随时都可能“翩翩而来”。②应该怎样看待挫折,怎样去面对挫折呢?③“自古英雄多磨难”。历史上许多仁人志士在与挫折斗争的过程中做出了不平凡的业绩。司马迁在遭受宫刑之后,发愤著书,写出了被鲁迅誉为“史家之绝唱,无韵之离骚”的名著《史记》。音乐家贝多芬,一生遭遇的挫折是难以形容的。他17岁失去母亲,32岁耳聋,接着又陷入了失恋的痛苦之中。对一个音乐家来说,这打击是多么的巨大啊!可贝多芬不消沉、不气馁,他在一封信中写道:“我要扼住命运的咽喉,它妄想使我屈服,这绝对办不到!”他始终顽强地生活,艰难地创作,最终成为世界不朽的音乐家。(节选自刘熙的《善待挫折》,有删改) 9.选文第②段是一个设问句,它在文中有什么作用?(3分)__________________________________________________________________________________________________________________10.请简析选文第③段的画线句子的作用。(3分)_________________________________________________________第②段的设问句在结构上起承上启下的过渡作用,在内容上吸引读者注意,激发读者思考。第③段的画线句子总起下文,并自然地引出下文的事实论据。 中考考点实训议论文阅读模拟训练交友之道张保振①俗话说:“在家靠父母,出门靠朋友。”这说明朋友的重要。《诗·小雅》言:“虽有兄弟,不如友生。”这说明朋友的至要。《孟子·滕文公上》载:“出入相友。”这说明朋友的必要。朋友的重要、至要、必要,在古人看来,是“友也者,友其德也”的。所以,与朋友交,就要只取其长,不计其短。也就是说,交友要有选择,看重并学习朋友的优良品德。②那么在现代生活中,我们又该怎样交友呢?③交友之道首先是择友。不是什么人都可以成为朋友的。 对朋友,古人们看得很重,认为是“五伦之一”,是“平居可与共道德,缓急可与共患难”。并认为,“友则两相关切。若酒肉饮博,相与往还,此党也,非友也”(蒲松龄语)。所以,择友,就要或道同相助,或品德相亲,或学问相成,或气节相感,或然诺相信,或才技相合,或诗文相尚等,要两相共赢,昭昭可鉴。毕竟,择友只有志趣相投,互为“人镜”,补充提高,才能获取增加一倍的智慧,放大一倍的力量,才能如同读一本圣贤经传,或一篇名人诗篇,使身心得到滋补,境界得以升华。而这,是从一般人那里难以得到的。④交友之道关键是取友。世间尽管有“一见如旧识,一言知道心”的交友、取友之道,但取友毕竟是取优,故不可等闲视之,草率取之。自古至今,为人称颂的取友之道,“慢热型”的居多。这种慢热型,往往是先淡后浓,先疏后亲,先远后近。初次见面,就亲热得不得了,未必是真友、至友,很可能是遇上了“见面熟”。 这种“见面熟”式的朋友,当你遇到挫折,甚至将要掉进深渊时,他的双手未必来拉住你;当你一帆风顺,乐不思蜀,甚至误入歧途而执迷不悟时,他的双手未必来指引你。取友,就是要智可以砥砺,行可以辅弼,有了错误可以指正。司马迁在《史记·汲郑列传》中说:“一死一生,乃知交情;一贫一富,乃知交态;一贵一贱,交情乃见。”这“六个一”,全在人生的关节点,道出了取友的最佳点。所以,对贫贱之交、生死之交,不仅终生不可忘,而且要始终一契之。⑤交友之道重点是结友。结友要结真友。无疑,在生活中,友有面友、真友之分。面友者,友而不心也。真友者,生无请言,死无托词也。交友就要交真友。真友是需要风浪的考验、时间的检验的。人的一生不可能年年顺意,事事行时。在遇到人生的逆境、背时,才能对面友或真友看得清,分得明。这方面的教训很 多。如战国中期有一对同习兵法、一师之徒的同窗朋友孙膑与庞涓,由于孙膑的才能略高一点而招致庞涓嫉恨在心,以至利用自己先到魏国并深得魏王重用的机会,密招孙膑入魏,口言善,说是要向魏王力荐;身行恶,实则借刀杀人,对孙膑施以膑刑①,“断其两足而黥之”,使孙膑成为一个“刑余之人”而不能进入仕途。这就警示人们,饮酒要饮醇,结友要结真。否则,不仅自己会吃尽苦头,而且连残至死都不知是什么原因。⑥交友要交优。只有多看朋友的长处,把朋友的长处吸收为自己的长处,从而让朋友的优点、优势也能体现在自己身上,才能在人生路上肩并肩手挽手,共谱新篇章。(选自《中学生阅读》,有删改)【注释】①膑刑:古代剔除膝盖骨的酷刑。 1.下列能准确概括本文中心论点的一项是()(3分)A.朋友是很重要的。B.与朋友交,就要只取其长,不计其短。C.交友之道首先是择友。D.结友要结真友。B 2.下列对文章的分析理解,不正确的一项是()(3分)A.文章开头用俗语以及古籍中的语录做道理论据,既表明我国自古以来都十分重视交友之道,又自然地引出并论证了本文的中心论点,同时也为开启下文做引导和铺垫。B.文章第③段画线句子是道理论证,引用古人的语录,进一步论证了“交友之道首先是择友”的观点。C.文章举出孙膑与庞涓的事例,论证了“在遇到人生的逆境、背时,才能对面友或真友看得清,分得明”的观点,从而进一步正面论证“结友要结真”的观点。D.作者认为交友首先要择友,关键是取友,重点是结友,这样层层深入论述,行文条理清晰。C 3.文章第②段在文中有何作用?(4分)________________________________________________________________________________________________________________________________________________________________________________________________________________________________________内容上,作者用设问句进一步提出了自己要论述的问题,起到了引导读者思考的作用;同时在文章结构上起到了过渡衔接的作用。 2019秋部编版九年级语文上册第五单元第二十课创造宣言 名师导学20*创造宣言一、课前自学知识储备1.宣言:国家、政府、政党、团体或其领导人为了说明自己的政治纲领或对某重大问题表明自己基本立场和态度而公开发表的文件。2.宣言的特点:宣言是一种演讲词。既是演讲,那它一定具备演讲稿的特点——文辞优美,感情充沛,说理生动,鼓动性强。 一课一法1.宣言文体:陶行知用宣言的形式表明自己的教育主张,表达出自己坚定的信念。2.课文先破后立的议论文的结构:对敌论先加以批判驳斥,然后在此基础上再表明自己的观点。3.课文主要的论证方法——举例论证:作者引用了古今中外大量的事例,形象有力地反驳了“不能创造”的错误观点。例如八大山人朱耷挥毫画他几笔便成为一幅名贵的杰作;法国企业家雷塞布竟能在沙漠中造成苏伊士运河;不识字的慧能偶听人讲经,顿悟佛理,后来成为禅宗南宗创始人;遭遇八十一难之玄奘,毕竟取得佛经等。 二、内容梳理(根据课文填空,理清文脉)《创造宣言》是一篇驳论文,作者运用大量典型的事例与名言警句驳斥了五种“不能创造”的错误观点,提出了“处处是创造之地,天天是创造之时,人人是创造之人”的观点,并发出了“只要有一滴汗,一滴血,一滴热情,便是创造之神所爱住的行宫,就能开创造之花,结创造之果,繁殖创造之森林”的创造宣言。全文可分为三个部分: 第一部分(第1~3段):对比引出论题——教育者要创造①__________的活人艺术家造石像——否定无自信的崇拜真善美教育家造人——肯定创造的重要性②__________论证正反对比第二部分(第4~10段):举例驳斥了五种“不能创造”的错误观点借口反驳论据客观环境③____________——八大山人、飞帝亚斯、米开朗基环境太平凡④_____________——《易经》《正气歌》、苏联国歌、《尼赫鲁自传》、雷塞布、平老静生活太单调主观因素⑤__________——莫扎尔特、爱迪生、帕斯加尔⑥__________——曾参、慧能年纪太小太无能⑦____________________——玄奘、哥伦布、莫扎尔特、歌德走投无路,陷入绝境举例论证 第三部分(第11~16段):提出自己的正确观点并论证提出观点(第11段):⑧_________________________________________________________________________________________________列举事例(第12~15段):举东山樵夫因循守旧的例子,论证要从点滴做起,让创造之神常驻心中处处是创造之地,天天是创造之时,人人是创造之人引用名言(第16段):论证不能停止创造,发出宣言,激励人们用“汗”“血”“热情”去创造 名师导练基础训练1.根据拼音写出相应的词语。(1)在刀法之交响中,投入一丝一毫的杂声,都是zhòngshāng()整个的和谐。(2)刀法如果用得不对,可以万像同毁;刀法如果用得对,则一笔下去,huàlóngdiǎnjīng()。(3)可见平凡单调,只是懒惰者之dùncí()。(4)但是lǔdùn()的曾参,传了孔子的道统……我们岂可以zìbàozìqì()呀!中伤画龙点睛遁词鲁钝自暴自弃 (5)有人说:shānqióngshuǐjìn(),zǒutóuwúlù(),陷入绝境,等死而已,不能创造。(6)绝望是nuòfū()的幻想。(7)大自然雕刻的奇峰、怪石、瀑布,huànyǎng()的飞禽、走兽、小虫和几千年来农人为后代种植的大树,于他无用,都等于没有看见。(8)当你看见满山的树苗在你监护之下,得到我们的汗、血、心、生命的guàngài(),一根一根地都长成参天的大树,你不高兴吗?山穷水尽走投无路懦夫豢养灌溉 2.下列句子中加点的词语使用不恰当的一项是()A.一个有智慧的人应该是真诚的,不傲慢,不欺骗,不会造谣中伤,也不会令人讨厌。B.我的眼泪流了下来,灌溉了下面柔软的小草,不知道来年,会不会开出一地的记忆和忧愁?C.生命教育必须贯穿教育的始终,让学生不但有崇高的生命意识,而且有应急救险的技能,灾难突发时,他们才能从容应对,而不至于惊慌失措、内外交困。D.在作文的结尾或开头适当地添加一些好词,便可以起到画龙点睛的作用。C 3.下列句子中有语病的一项是()A.武大、华科大效仿牛津、剑桥两校划船比赛的做法,在东湖举行“同城双星”龙舟赛。B.家庭和学校对孩子安全问题的过度关注,反而会降低孩子自我保护的意识和能力。C.我们要像节食减肥一样,减少使用数码产品的时间,借以缓解对数码产品的心理依赖。D.“组队读书”能改变阅读浅层化、碎片化的现状,让人们学会深度阅读和系统阅读。B 4.请从下面这则报道中提炼出两条最主要的信息。2018年3月31日,陷入“瘦肉精”危机的双汇集团在其总部所在地河南省漯河市体育馆召开了“双汇万人职工大会”,参会人员包括双汇集团管理层、漯河本部职工、经销商、新闻媒体等。会上,双汇集团董事长代表集团向消费者致歉,并将3月15日定为“双汇食品安全日”。随后,双汇与中国检验认证集团签订长期战略合作协议,由中检集团作为独立的第三方质量检验机构,全方位监督双汇的质量安全。(1)信息一:________________________________(不超过20字)(2)信息二:_____________________________________________(不超过20字)双汇召开万人大会向消费者致歉。双汇引入第三方检测机构,全面监督质量安全。 课内精读5.赏析课文中精妙的句子。(6分)(1)刀法如果用得不对,可能万像同毁;刀法如果用得对,则一笔下去,画龙点睛。______________________________________________________________________________________________________________________________________________________________________________(2)但是遭遇八十一难之玄奘,毕竟取得佛经;粮水断绝、众叛亲离之哥伦布,毕竟发现了美洲;冻饿病三重压迫下之莫扎尔特,毕竟写出了《安魂曲》。______________________________________________________________________________________________________________________________________________________________________________“刀法”比喻教育方法,“万像”比喻众多教育对象,“画龙点睛”比喻使众多教育对象成才。全句的意思是:教育方法不当,可能使众多受教育者被毁;教育方法得当,可以使众多受教育者成才。运用举例论证的论证方法,具体有力地论证了有志者排除万难也可取得巨大成就的观点;运用排比的修辞方法,使论证内容充实丰富,增强了语言的气势,充分体现了宣言的特点。 6.课文中作者主要针对哪几种错误观点来进行反驳?作者是怎样对错误观点进行反驳的?(5分)_____________________________________________________________________________________________________________________________________________________________________________________________________________________________________________________________________________________________作者围绕五种“不能创造”的借口,进行了针锋相对的批驳。这五种错误观点是:①环境太平凡;②生活太单调;③年纪太小;④太无能;⑤走投无路,陷入绝境。作者主要运用了典型事例与名言警句来进行反驳。(或:此文主要运用举例论证和引用论证这两种论证方法进行反驳) 议论文阅读技法专练——分析举例论证的作用勤出智慧。传说古希腊有一个叫德摩斯梯尼的演说家,因小时口吃,登台演讲时,声音含混,发音不准,常常被雄辩的对手所压倒。可是他不气馁,不灰心,为克服这个弱点,为战胜雄辩的对手,他每天口含石子,面对大海朗诵。不管春夏秋冬,他坚持五十年如一日,连爬山、跑步也坚持练习演说,终于成为全希腊最有名气的演说家。宋代学者朱熹讲过一个故事:福州有一个叫陈正之的人,反应相当迟钝,每次读书只能读50个字,一篇小文章也要读一二百遍才能读熟。但他不懒不怠,勤学苦练,别人读一遍,他就读三遍四遍,天长日久,知识与日俱增,后来读了很多书,成了博学之士。这表明,即使天资比较差,反应比较迟钝,只要勤,同样也是可以变拙为巧的。(节选自林家箴的《说勤》,有删改) 7.选段运用了什么论证方法?有什么作用?(3分)__________________________________________________________________________________________________________________运用了举例论证的方法,具体典型地论证了“勤出智慧”的观点,增强了文章的说服力。 ①古往今来,有恒者事竟成。一个人未必具备出众的天赋,但如果拥有异于常人的执着,“千磨万击还坚劲”“咬定青山不放松”,就能把“恒”字写进自己的品格,守得云开见月明,成就不凡事业。词学宗师夏承焘专注于词学研究和教学,以毕生之力旁搜远绍、取精用宏,终成现代词学的开拓者和奠基人。革命战士吴玉章40年如一日,再苦再难也不忘教书育人,坚持革命、坚持办教育、坚持做好事,造就了一大批人才。事实上,无论何种行业或领域,不管是做学问还是做事情,倘若没有水滴石穿的韧性,缺乏坐得住、沉得下的脾性,都难有经得起时间检验的成绩。②然而现实中,也有一些人贪图安逸、缺乏毅力,往往让事情无疾而终。譬如,有的人遇到挫折就乱了阵脚,逃避矛盾,蛰伏起来;有的人干工作浮在表面,看上去勤勤恳恳,但其实心里根本就没有恒定的目标;还有的人习惯东一榔头西一棒子,做不到一锤接 着一锤敲,累积不出实绩。古人说得好:“贵有恒,何须三更起五更眠;最无益,只怕一日曝十日寒。”要做成一件事情重要的是专注于目标,毫不动摇,勇往直前。心态一旦浮躁,注定难以走出浅尝辄止、兜兜转转的人生迷宫。(节选自王艺侨的《用一生写好一个“恒”字》,有删改) 8.选文第①段主要运用了什么论证方法?有什么作用?(3分)___________________________________________________________________________________________________________________________________________________________________________9.选文第②段主要运用了什么论证方法?有什么作用?(3分)___________________________________________________________________________________________________________________________________________________________________________举例论证。列举夏承焘执着于词学研究和教学,终有所成和吴玉章坚持教书育人,造就大批人才的事例,从正面具体论述了“古往今来,有恒者事竟成”的观点。举例论证。列举生活中一些贪图安逸,缺乏毅力而终无所成的人的事例,从反面论证了“有恒者事竟成”的观点,使得论述更全面。 中考考点实训议论文阅读模拟训练创造力比知识更重要王松石①美国中学生奥林匹克比赛中有一道竞赛题,要求参赛学生设计一种水上运载工具,但要打破常规造型,强调求异思维,体现创新精神。许多学生绞尽脑汁,设计了各种造型的运载工具,可总摆脱不了大家熟知的船的形状和结构,唯独有一位学生构思奇特,他设计的作品像一只硕大的“水蜘蛛”,不像船那样在水上航行,而是像水蜘蛛那样在水面上“爬行”。这件作品在所有参赛作品中独树一帜,引人注目。虽然这一设计最后在实际操作中失败了,但几乎所有的评委都给他亮了最高分。 ②这是培养学生创造能力的一个实例。社会进入信息时代,新技术革命风起云涌,各国都把培养学生的创新精神和创造能力放到了异常突出的地位。因为创造活动是人类生存与人类文明持续发展的重要保证,是人类知识进化的源泉。创造力比知识更重要。③那么,怎样培养创造力呢?④科学巨匠牛顿说,他之所以取得伟大的成就,是因为他站在巨人的肩膀上。这“巨人”可以理解为无数前人所创造的知识的化身。积累知识是基础,融通知识更重要。现代科学技术正朝着既不断分化又不断综合的方向发展,新知识的生长点往往出现在学科的边缘和学科之间的交叉处。学文科的学生应懂一些理科知识,学理科的学生也应涉猎文学艺术。法国化学家利希腾贝格说过:“一个只知道化学的化学家,他未必真懂化学。”广泛涉猎,博学多识,学贯古今,触类旁通,应该成为当代有志中学生共同的追求。⑤我们掌握的知识越多,就越容易产生新的联想、新的见解、 新的创造;但我们对某一事物的传统意义知之太多,又会阻碍思维的灵活性,使我们不由自主地被前人牵着鼻子走,从而形成智力屏障,导致创造能力的僵化。古今中外有不少人勤奋刻苦,但终其一生,有积累而无创造,为知识所累,为知识所困。同学们要学会把心智的“杯子”空出来,为思路的开拓变化留有充分的余地,使知识能灵活地聚合、置换、跳跃、碰撞,迸发出创造的火花。⑥善于捕捉热线,随时记录灵感。“热线”就是酝酿成熟了的想法和思路,一旦有热线闪烁就要抓住不放,深入挖掘。当然也要善于抓住转瞬即逝的“一闪之念”,对于那些突然闯入脑际的新思想、新概念、新形象,要随时摘记,定期整理,深入思考,激发创造。化学家诺贝尔就是受到笔记本中“硝化甘油掉在沙地上随即凝结起来”这句话的启发,成功地解决了硝化甘油的运输 问题。每一位学生都要为自己准备一本思想记录本,当新的思想、新的灵感在头脑中闪现的时候,及时把它记下来,长期坚持,养成习惯,敏捷的思维品质和出众的创造力才能就能逐渐培养出来。(选自《课外语文》,有删改) 1.下列与原文内容不相符的一项的是()(3分)A.作者认为创造力比知识更重要。B.只要随时摘记转瞬即逝的“一闪之念”和突发的新思想、新概念、新形象,我们就能激发创造的灵感。C.作者认为应该广泛涉猎,博学多识,使得知识能灵活地聚合、置换、跳跃、碰撞,才能迸发出创造的火花。D.要培养创造力,就要善于捕捉热线,随时记录灵感。B 2.下列对文章的分析理解,不正确的一项是()(3分)A.文章从奥林匹克竞赛题写起,自然地引出关于创造力的论题,激发读者的阅读兴趣。B.文章采用“总—分—总”的写作结构,先提出中心论点,接着论述了如何培养创造力的问题,最后总结全文。C.文中第③段的设问句,既能激发读者的思考,又能使行文条理清晰。D.文中第⑤段“学会把心智的‘杯子’空出来”可以理解为不被知识束缚,在头脑中留有创造的空间。B 3.文章第⑥段主要运用了什么论证方法?有什么作用?(4分)______________________________________________________________________________________________________________________________________________________________________________举例论证。举出诺贝尔的事例,具体有力地论证了“善于记录灵感,才有利于激发创造”的观点,增强了文章的说服力。 2019秋部编版九年级语文上册第六单元第二十一课智取生辰纲 名师导学21智取生辰纲一、课前自学知识储备1.古典章回体小说:章回体小说为中国长篇小说的一种传统形式。源于宋代平话,确立于元末,以长篇小说《水浒传》和《三国演义》为成熟标志。其特点是将全书分为若干章节,称为“回”。2.伏笔和铺垫:(1)伏笔:文学创作中叙事的一种手法,就是上文看似无关紧要的事或物,对下文将要出现的人物或事件预先做出的某种提示或暗示。 (2)铺垫:也称铺叙衬垫,它是为了突出主要的人物或事物而铺叙另外的人物或事物以做衬垫。运用铺垫写法是为了蓄积气势,突出文章主旨。(3)伏笔和铺垫的区别:A.从目的上看:铺垫描述的是次要人物或事件,目的却在为表现主要人物或事件而蓄势。伏笔是对将要在作品中出现的人物或事件,预做提示或暗示,以求前后呼应,它常常与照应配合使用,即所谓前有伏笔,后有照应。B.从形态上看:为了达到衬托的目的,铺垫对起陪衬作用的部分往往大肆渲染,唯恐读者不见,因此,铺垫可以说是“显性”的。而伏笔通常比较隐蔽,是“隐性”的。巧妙的伏笔,在没有看到“照应”之前,貌似“闲笔”。 C.从笔墨上看:铺垫所使用的笔墨往往较多;而伏笔通常只是一两笔,点到为止,否则就失去了“伏”的意义。D.从位置上看:铺垫一般在文章开头,伏笔常见于文章中间。一课一法课文中的伏笔和铺垫:从全文的结构来看,课文可以分为两个部分,前一部分主要写天气酷热难当,写杨志与老都管、虞候及众位军士的矛盾,为后文生辰纲被劫埋下了伏笔。第二部分主要写杨志与晁盖等人在黄泥冈上斗智斗勇,是全文的高潮。第一部分处处铺垫,处处蓄势;第二部分则以第一部为基础,展示了事物的结局。二、内容梳理(根据课文填空,理清文脉)《智取生辰纲》节选自中国古典名著《水浒传》第十六回“杨志押送金银担吴用智取生辰纲”。文章从杨志开始上路写起,通过对杨 志押送生辰纲时的谨慎、急功近利等描写反衬出梁山好汉的智谋,反映了封建统治下尖锐的阶级矛盾,表现了起义英雄在反封建斗争中的智慧和力量。文章内容可分为两个部分:第1~7段写杨志押送生辰纲;第8~14段描写晁盖、吴用等人智取生辰纲,其中第14段是补叙。智取生辰纲谨慎押送谨慎防范改时间不满酷热强行遭众怒①________忍耐暗度陈仓走小路②________怨怅③_______________反抗黄泥冈机警防范用计智取智用天时——以逸待劳智用④__________——黄泥冈松林智用⑤__________——卖酒智用计谋——一稳(扮客商)、二诱(佯争酒)、三麻(⑥__________)地利矛盾巧下药杨志的行动变化众人的心理变化 名师导练基础训练1.根据拼音写出相应的词语。(1)天气热了,行不得,见着林子便要去xiēxi()。(2)你两个好不晓事!这gānxì()须是俺的!你们不替洒家打这夫子,却在背后也慢慢地挨。(3)老都管道:“你们不要yuànchàng(),巴到东京时,我自赏你。”(4)杨志道:“你也没fēnxiǎo()了,如何使得!这里下冈子去,兀自有七八里没人家。甚么去处,敢在此歇凉!”歇息干系怨怅分晓 (5)抬举你做个提辖,比得草芥子大小的官职,直得恁地chěngnéng()。(6)那七人道:“你diāndǎo()问,我等是小本经纪,那里有钱与你。”(7)到来只顾吃嘴,全不晓得路途上的gòudàng()艰难。(8)那七个客人从松树林里推出这七辆江州车儿,把车子上枣子都丢在地上,将这十一担金珠宝贝,却装在车子内,叫声:“guōzào()!”一直望黄泥冈下推了去。逞能颠倒勾当聒噪 2.下面语境中,用语得体的一项是()李编辑,您好!我是一名在校中学生,寄上拙作《秋月》,望拜读。敝人有吟诗雅趣,学习之余,凑成几句,错处颇多。但自己不能斧正,您是大手笔,望工作之暇给予修改,不吝赐教。A.拙作B.拜读C.雅趣D.斧正A 3.下列对病句的修改不正确的一项是()A.“实践十号”卫星的成功发射、在轨运行和回收,将极大提高我国微重力科学及空间生命科学研究。(在句末加“水平”)B.未成年学生玩手机的危害应该得到有效引导和管理。(把“引导和管理”改为“管理和引导”)C.我们只要与自然和谐相处,才能保护好我们赖以生存的家园。(把“只要”改为“只有”)D.通过考古发掘、征集和捐赠,甘肃省博物馆获得历史文物藏品近82000多件。(删去“近”或“多”)B 4.阅读下面的材料,按要求作答。某翻译家在《文艺报》上撰文指出:有人说中国人称自己的国家为“中国”,表示自己是坐镇在世界中央的天朝,说明中国人自傲。但从国名的中文翻译来看,译名却能够表达中国人的情感。例如,“英国”为什么不译作“阴国”?“美国”为什么不译作“霉国”?“德国”为什么不译作“歹国”?这是因为中国人要从同音字中选出具有最美好含义的字来命名这些国家。用什么字呢?用“英雄”的“英”、“美丽”的“美”、“道德”的“德”、“法理”的“法”、“芬芳”的“芬”、“祥瑞”的“瑞”……而外国,比如英国,用英文翻译别国的国名,只用音译,译名中不含有任何褒贬意义。 (1)请用一句话概括该翻译家的观点,不超过30字。_________________________________________________________(2)根据材料内容,以“汉字”为描述对象,用上一种修辞方法,写一句话。____________________________________________________________________________________________________________________汉字是人与人之间沟通的桥梁,传递了中国国民的善良友好,传承了中国文化的博大精深。对别国国名的翻译,中文能表达美好的情感,而外文不能。 课内精读那汉子口里唱着,走上冈子来,松林里头歇下担桶,坐地乘凉。众军看见了,便问那汉子道:“你桶里是甚么东西?”那汉子应道:“是白酒。”众军道:“挑往那里去?”那汉子道:“挑去村里卖。”众军道:“多少钱一桶?”那汉子道:“五贯足钱。”众军商量道:“我们又热又渴,何不买些吃?也解暑气。”正在那里凑钱。杨志见了,喝道:“你们又做甚么?”众军道:“买碗酒吃。”杨志调过朴刀杆便打,骂道:“你们不得洒家言语,胡乱便要买酒吃,好大胆!”众军道:“没事又来捣乱。我们自凑钱买酒吃,干你甚事,也来打人。”杨志道:“你这村人理会的甚么!到来只顾吃嘴,全不晓得路途上的勾当艰难。多少好汉,被蒙汗药麻翻了。”那挑酒的汉子看着杨志冷笑道:“你这客官好不晓事,早是我不卖与你吃,却说出这般没气力的话来。”正在松树边闹动争说,只见对面松林里那伙贩枣子的客人,都 提着朴刀走出来问道:“你们做甚么闹?”那挑酒的汉子道:“我自挑这酒过冈子村里卖,热了在此歇凉。他众人要问我买些吃,我又不曾卖与他。这个客官道我酒里有甚么蒙汗药。你道好笑么?说出这般话来!”那七个客人说道:“我只道有歹人出来,原来是如此。说一声也不打紧。我们倒着买一碗吃。既是他们疑心,且卖一桶与我们吃。”那挑酒的道:“不卖,不卖!”这七个客人道:“你这汉子也不晓事,我们须不曾说你。你左右将到村里去卖,一般还你钱。便卖些与我们,打甚么不紧。看你不道得舍施了茶汤,便又救了我们热渴。”那挑酒的汉子便道:“卖一桶与你不争,只是被他们说的不好。又没碗瓢舀吃。”那七人道:“你这汉子忒认真,便说了一声打甚么不紧。我们自有椰瓢在这里。”只见两个客人去车子前取出两个椰瓢来,一个捧出一大捧枣子来。七个人立在桶边,开了桶盖,轮替换着舀那酒吃,把枣子过口。无一时,一桶酒都吃尽了。七个客人道:“正不曾问得你多少价钱?”那汉道: “我一了不说价,五贯足钱一桶,十贯一担。”七个客人道:“五贯便依你五贯,只饶我们一瓢吃。”那汉道:“饶不的,做定的价钱。”一个客人把钱还他,一个客人便去揭开桶盖,兜了一瓢,拿上便吃。那汉去夺时,这客人手拿半瓢酒,望松林里便走。那汉赶将去。只见这边一个客人从松林里走将出来,手里拿一个瓢,便来桶里舀了一瓢酒。那汉看见,抢来劈手夺住,望桶里一倾,便盖了桶盖,将瓢望地下一丢,口里说道:“你这客人好不君子相!戴头识脸的,也这般啰唣。” 5.用一句话概括选段中第1段的主要内容。(2分)_________________________________________________________6.这半瓢酒可是智取生辰纲的关键处,它反映出好汉们发挥智慧,充分利用了客观条件:____________、____________和主观条件:_______________________________来达到目的。(3分)7.作者笔墨细致地对杨志进行语言、心理、动作的描写,从中可以看出该人物怎样的性格特点?(3分)_________________________________________________________杨志禁止众军士买酒喝。天气炎热地方偏僻彻底消除了对手防备之心精明强干,处事精细,谨慎。 8.杨志时时谨慎,处处提防却丢失生辰纲。请结合文章说说杨志失生辰纲的主要原因有哪些?(4分)_____________________________________________________________________________________________________________________________________________________________________________________________________________________________________________________________________________________________①晁盖、吴用计策高妙,无懈可击;②杨志急功近利、粗暴蛮横,导致内部矛盾激化,杨志也确有“不智”之处;③梁中书用人却疑,只用其“艺”而不信其“人”,派老都管、虞候跟随。老都管斥责杨志的一番话,顺应了军汉、虞候的心愿,而使杨志最终妥协,同意军汉们买酒、喝酒,自己也身受其害。 记叙文阅读技法专练——理清伏笔和铺垫的区别和作用①汽车进入了山区,山路崎岖不平,颠得人五脏六腑都翻腾出来。车上只有十几个乘客,坐在后几排的乘客,因为颠得吃不消,都挪到了前排。②他却主动移到了最后一排,五个座位连在一起,正好可以躺下。他太需要休息了。这段日子,他工作丢了,谈了好几年的女朋友也吹了,整个人完全处在心灰意懒中,连续十几天吃不下睡不着。他觉得自己走到了人生的绝境,自己是那么渺小,存不存在都不重要。此行,他想回老家看看父母,年迈的双亲培养出他这个大学生很不容易。他决定在了断自己之前,再看一眼可怜的双亲。③汽车颠簸着前进,乘客都昏昏欲睡。他也恍恍惚惚进入梦乡。 ④突然,在一阵剧烈的撞击后,汽车猛地停了下来。⑤所有的乘客,都被惊醒了,有人头撞在了前排椅子扶手上,有人被震碎的窗玻璃割伤,有人被抛出了座位……躺在后排的他,也被高高地弹起,又重重地摔了下来——出车祸了!(节选自孙道荣的《你有多重要》,有删改)9.阅读选文,说说第①段有什么作用?(4分)______________________________________________________________________________________________________________________________________________________________________________选文第①段对山路崎岖难走的描述为下文车祸的发生埋下了伏笔;车上其他乘客都挪到了前排则为后文车祸发生后车内的惊险状况做了铺垫。 中考考点实训名著阅读模拟训练再说林冲踏着那瑞雪,迎着北风,飞也似奔到草场门口,开了锁,入内看时,只叫得苦。原来天理昭然,佑护善人义士,因这场大雪,救了林冲的性命。那两间草厅已被雪压倒了。林冲寻思:“怎地好?”放下花枪、葫芦在雪里,恐怕火盆内有火炭延烧起来,搬开破壁子,探半身入去摸时,火盆内火种都被雪水浸灭了。林冲把手床上摸时,只拽得一条絮被。林冲钻将出来,见天色黑了,寻思:“又没打火处,怎生安排?”想起离了这半里路上,有个古庙,可以安身。“我且去那里宿一夜,等到天明却做理会。”把被卷了,花枪挑着酒葫芦,依旧把门拽上锁了,望那庙里来。入的庙门,再把门掩上,傍边止有一块大石头,掇将过来, 靠了门。入的里面看时,殿上做着一尊金甲山神,两边一个判官,一个小鬼,侧边堆着一堆纸。团团看来,又没邻舍,又无庙主。林冲把枪和酒葫芦放在纸堆上,将那条絮被放开,先取下毡笠子,把身上雪都抖了,把上盖白布衫脱将下来,早有五分湿了,和毡笠放在供桌上。把被扯来盖了半截下身。却把葫芦冷酒提来便吃,就将怀中牛肉下酒。正吃时,只听得外面必必剥剥地爆响。林冲跳起身来,就壁缝里看时,只见草料场里火起,刮刮杂杂地烧着。……当时张见草场内火起,四下里烧着,林冲便拿枪,却待开门来救火,只听得前面有人说将话来。林冲就伏在庙听时,是三个人脚步声,且奔庙里来。用手推门,却被林冲靠住了,推也推不开。三人在庙檐下立地看火,数内一个道:“这条计好么?”一个应道:“端的亏管营、差拨两位用心。回到京师,禀过太尉,都保你二位做大官。这番张教头没的推故了。”那人道:“林冲今番直吃我们 对付了。高衙内这病必然好了。”又一个道:“张教头那厮,三回五次托人情去说‘你的女婿殁了’,张教头越不肯应承。因此衙内病患看看重了,太尉特使俺两个央浼二位干这件事,不想而今完备了。”又一个道:“小人直爬入墙里去,四下草堆上点了十来个火把,待走那里去!”那一个道:“这早晚烧个八分过了。”又听得一个道:“便逃得性命时,烧了大军草料场,也得个死罪。”又一个道:“我们回城里去罢。”一个道:“再看一看,拾得他一两块骨头回京,府里见太尉和衙内时,也道我们也能会干事。”林冲听那三个人时,一个差拨,一个是陆虞候,一个富安。林冲道:“天可怜见林冲,若不是倒了草厅,我准定被这厮们烧死了。”轻轻把石头掇开,挺着花枪,一手拽开庙门,大喝一声:“泼贼那里去!”三个人急要走时,惊得呆了,正走不动。林冲举手肐察的一枪,先戳倒差拨。陆虞候叫声:“饶命!”吓的慌了手 脚,走不动。那富安走不到十来步,被林冲赶上,后心只一枪,又戳倒了。翻身回来,陆虞候却才行的三四步,林冲喝声道:“奸贼!你待那里去!”批胸只一提,丢翻在雪地上,把枪搠在地里,用脚踏住胸脯,身边取出那口刀来,便去陆谦脸上阁着,喝道:“泼贼!我自来又和你无甚么冤仇,你如何这等害我!正是杀人可恕,情理难容。”陆虞候告道:“不干小人事,太尉差遣,不敢不来。”林冲骂道:“奸贼,我与你自幼相交,今日倒来害我,怎不干你事!且吃我一刀。”把陆谦上身衣服扯开,把尖刀向心窝里只一剜……入庙里来……穿了白布衫,系了搭膊,把毡笠子带上,将葫芦里冷酒都吃尽了,被与葫芦都丢了不要,提了枪,便出庙门投东去。 1.草厅被雪压倒后,林冲探半身入去摸“火盆”,到山神庙后又特地搬石头顶住庙门。这两个细节在文章中有何作用?(3分)____________________________________________________________________________________________________________________________________________________________________________________________________________________________________2.陆谦等三人的对话对林冲性格发展起到了什么作用?(3分)_________________________________________________________________________________________________________________这两个细节在文章中起伏笔作用。前一个细节表明草料场的大火绝非无意间的失火,而是有人故意纵火;后一个细节则为陆谦等人被阻于庙门之外,使林冲能从他们的对话中得知阴谋,从而下定杀死仇敌的决心的情节埋下伏笔。陆谦等三人的对话对林冲从逆来顺受、委曲求全到被迫杀人、逼上梁山的反抗性格的转变,起最后触发的“催化剂”作用。 3.林冲“轻轻把石头开,挺着花枪,一手拽开庙门,大喝一声……”中的动词有怎样的表达效果?(3分)_______________________________________________________________________________________________________________________________________________________________________________________________________________________________________________________________________________________________“开”“挺”“拽”“喝”一系列动词用得十分传神,形象地写出林冲与仇敌决战时机警异常,坚决果断,动作干净利索。一个“挺”字表现出人物的万丈怒火,一个摆脱了种种羁绊的顶天立地的英雄形象出现在读者面前。 2019秋部编版九年级语文上册第六单元第二十二课范进中举 名师导学22范进中举一、课前自学知识储备讽刺手法:运用比喻、夸张等手段和方法对人或事物进行揭露、批判和嘲笑。其作用是加强文章的深刻性和批判性,使语言辛辣幽默。 一课一法赏析课文中的讽刺手法:(1)漫画式的外形描写:“屠户横披了衣服,腆着肚子去了。”通过漫画式的外形描写,表达出作者鲜明的爱憎感情。作者主要是通过日常生活情节的提炼和典型化,把讽刺的现象显示出来。(2)运用夸张进行讽刺:范进中举发疯,一出门就让他摔了一跤,故意出这个新中举人的洋相——“走出大门不多路,一脚踹在塘里,挣起来,头发都跌散了,两手黄泥,淋淋漓漓一身的水,众人拉他不住,拍着笑着,一直走到集上去了。”作者善于对人物的最富特征的细节进行夸张的描写,即把某种需要否定的东西延伸放大,以揭示出这个人物的真实面目。(3)运用对比手法进行讽刺:①胡屠户对范进中举前后态度的对 比描写就尤为鲜明;②众乡邻对范进中举前冷漠无视和中举后的送酒送肉对比;③中举前范进的卑怯懦弱和中举后范进的圆滑世故对比。作者往往让同一个人在不同的情况下对待同一对象采取不同的甚至完全矛盾的态度,造成强烈对照,从而产生喜剧效果。(4)运用细节描写直接写实进行讽刺:如范进中举后,“屠户见女婿衣裳后襟滚皱了许多,一路低着头替他扯了几十回”“屠户把银子攥在手里紧紧的,把拳头舒过来,道:‘这个,你且收着。我原是贺你的,怎好又拿回去?’”的细节描写,表现了胡屠户的市侩;又如范进卖鸡的表现,让我们认识到封建科举制度下文人的迂腐可笑。白描式的细节描写,生动细致地表现出人物的性格特征。 二、内容梳理(根据课文填空,理清文脉)小说以范进中举为中心事件,对范进中举前后的境遇进行了对比描写,着力刻画了范进这个热衷科举的下层知识分子的典型形象,同时生动地刻画了他周围的人,尤其是对范进的老丈人胡屠户,在范进“中举前”和范进“中举后”的不同表现进行细致的刻画,反映了当时社会的世态和人心,深刻揭露并辛辣地讽刺了封建科举制度的腐朽及其对读书人的腐蚀和毒害。全文可分为两个部分,第一部分(第1~2段):写范进进学回家到中举前的情况;第二部分(第3~11段):写范进中举后的情景。 范进中举范进:生活贫困,地位卑微,性格①__________范进中举前范进中举后懦弱胡屠户:态度④______________________________________百般辱骂,恶言训骂众乡邻:不予理会,毫不同情张乡绅:不识范进,不曾往来丑态百出,地位②__________,性格③__________提高世故阿谀奉承,市侩嘴脸⑤________________________送酒送肉,趋炎附势⑥________________________巴结拉拢,送金赠房 名师导练基础训练(1)若是家门口这些做田的、扒粪的,不过是平头百姓,你若同他拱手zuòyī(),平起平坐,这就是坏了学校规矩,连我脸上都无光了。(2)范进道:“岳父jiànjiào()的是。”(3)你问我借pánchɑn(),我一天杀一个猪还赚不得钱把银子,都把与你去丢在水里,叫我一家老小嗑西北风!(4)说着,往后一交跌倒,牙关咬紧,bùxǐngrénshì()。(5)他爬将起来,又拍着手大笑道:“噫!好!我中了!”笑着,bùyóufēnshuō(),就往门外飞跑,把报录人和邻居都吓了一跳。作揖见教盘缠不省人事不由分说 (6)他只因欢喜狠了,痰涌上来,迷了xīnqiào()。(7)虽然是我女婿,如今却做了老爷,就是天上的xīngxiù()。(8)张乡绅先攀谈道:“世先生同在sāngzǐ(),一向有失亲近。”(9)范进道:“晚生jiǎoxìng(),实是有愧。却幸得出老先生门下,可为欣喜。”心窍星宿桑梓侥幸 2.下列句子中加点的词语使用不恰当的一项是()A.我们第一代的华侨,寄籍外洋,生儿育女,却世代翘首神州,不忘桑梓之情,当祖国需要的时候,他们都会慷慨奉献。B.德国诗人歌德作《浮士德》延续60年之久,曹雪芹写《红楼梦》“批阅十载,增删五次”,所以说,一切优秀的作品,都是不辞艰辛侥幸偶得之物。C.那出租车司机见一位不省人事的老人倒在路边,便立即把他送往医院救治。D.男孩瞪大眼睛,又惊又喜,高兴地扑到我身边,不由分说地就摘下我的帽子往窗外远远地扔去。B 3.下列句子中没有语病的一项是()A.在王宇璇明白了事情真相以后,就一直被浓浓的暖意包裹着,促使她不断反思着自己的错误。B.《范进中举》中有个市井人物胡屠户,此人在文中有许多生动形象、个性鲜明的语言和行动。C.能否切实形成艰苦奋斗、勤俭节约的新风尚,关键在于全面提高国民的基本素养。D.为了避免类似问题的发生,我们一方面思想上要重视,另一个方面要进一步加大管理力度。D 4.阅读下面的材料,按要求作答。①端午节是我国民间的一个传统节日,又称端阳节。②端午节的时间是在每年的农历五月初五这一天。③端午节的起源说法不一,但大多认为源于纪念投汨罗江自沉的楚国爱国诗人屈原。④报国无门的屈原投身汨罗江中,以死明志,汨罗江收容了一个忠贞而高尚的灵魂,而人们也以自己特有的方式抚慰着这一缕忠魂。⑤欢庆端午节,人们通常要赛龙舟,今年广东就举行了龙舟竞赛活动,香港、澳门也派了代表队参加。⑥端午节期间,南方各省区人们通常要吃粽子,这是用粽叶包裹糯米而煮成的一种食品。 (1)提取材料中的要点,给“端午节”下定义,不超过35字。__________________________________________________________________________________________________________________(2)根据材料内容,以“屈原”为描述对象,用上一种修辞方法,写一句话。____________________________________________________________________________________________________________________________________________________________________________端午节是我国农历五月初五以吃粽子、赛龙舟等形式纪念屈原的一个民间传统节日。屈原坚贞爱国的情怀如一粒饱满的种子,播散在历史中,长出一棵橘树,开出一朵兰花,带着两千年前遥远的芬芳,熏养着一代又一代的中华儿女。 课内精读范进不看便罢,看了一遍,又念一遍,自己把两手拍了一下,笑了一声道:“噫!好了!我中了!”说着,往后一交跌倒,牙关咬紧,不省人事。老太太慌了,慌将几口开水灌了过来,他爬将起来,又拍着手大笑道:“噫!好!我中了!”笑着,不由分说,就往门外飞跑,把报录人和邻居都吓了一跳。走出大门不多路,一脚踹在塘里,挣起来,头发都跌散了,两手黄泥,淋淋漓漓一身的水,众人拉他不住,拍着笑着,一直走到集上去了。众人大眼望小眼,一齐道:“原来新贵人欢喜疯了。”老太太哭道:“怎生这样苦命的事!中了一个甚么举人,就得了这个拙病!这一疯了,几时才得好?”娘子胡氏道:“早上好好出去,怎的就得了这样的病!却是如何是好?”众邻居劝道:“老太太不要心慌。我们而今且派两个人跟定了范老爷。这里众人家里拿些鸡蛋酒米,且管待了报子上的老爹们,再为商酌。” 5.范进欢喜疯了的过程写得很有层次,请分别用简洁的语言概括范进发疯的四个阶段:第一层写__________,第二层写_________,第三层写__________,第四层写_____________。而四层中,“__________”的动作,“_______”的表情几乎一以贯之,又加以漫画式的丑化:“_______________________________________________”,包含了极其强烈的讽刺意味。(7分)6.从对范进疯态的描写,可以看出作者对范进持什么样的态度?(3分)__________________________________________________________________________________________________________________昏厥疯跑跌倒疯走上集拍手笑头发都跌散了,两手黄泥,淋淋漓漓一身的水表现了作者强烈的讽刺和批判,作者也以此鞭挞科举制度的弊端和知识分子的精神堕落。 7.作者描写范进的疯态时又极力表现老太太的惊慌以及报录人、众人的表现各有什么作用?(4分)____________________________________________________________________________________________________________________________________________________________________________________________________________________________________8.结合课文分析范进发疯的原因。(4分)___________________________________________________________________________________________________________________________________________________________________________用侧面烘托的手法,写老太太的惊慌,烘托出范进昏厥得怕人;用报录人和邻居都吓了一跳的表情,烘托出范进飞跑的疯狂;用众人拉他不住的动作,烘托出范进的那股疯劲。每一处衬笔都突显了中心人物。一是科举制度对知识分子的直接危害;二是趋炎附势的社会环境的压力;三是范进热衷科举考试,到了痴迷境地,读死书、死读书,顽固不化的个人因素。 记叙文阅读技法专练——体会讽刺手法的作用我向河岸望去,过去生长稻米和油菜的田野,如今长出了连绵的高楼。如果对比20年前后的照片,你会发现,对岸升起了一个钢筋水泥的星球。那里是小镇的拆迁房集中地,失去土地的农民陆续搬到这里。前年大涨一波后,此地房价破万。开发商欢天喜地盖楼,农民欢天喜地拆迁——种地能赚几个钱?不仅是农田在消失,农民也在消失。农家子弟或是读书,或是打工,以各种因由离开土地。几代农民的梦想,是当个城里人。(节选自路明的《关于桥的事》,有删改) 9.“开发商欢天喜地盖楼,农民欢天喜地拆迁”一句中“欢天喜地”一词连用两次有何用意?(4分)___________________________________________________________________________________________________________________________________________________________________________该句运用了反复的修辞方法,突出表现了开发商赚取利润的满足感和失地农民获得补偿的喜悦之情,含有讽刺意味,强调了对人们拆桥功利性的批判,表达了“我”对“桥”消失的惋惜之情。 中考考点实训名著阅读模拟训练那一阵风过处,只听得乱树背后扑地一声响,跳出一只吊睛白额大虫来。武松见了,叫声:“呵呀!”从青石上翻将下来,便拿那条梢棒在手里,闪在青石边。那个大虫又饥又渴,把两只爪在地下略按一按,和身望上一扑,从半空里撺将下来。武松被那一惊,酒都做冷汗出了。说时迟,那时快,武松见大虫扑来,只一闪,闪在大虫背后。那大虫背后看人最难,便把前爪搭在地下,把腰胯一掀,掀将起来。武松只一躲,躲在一边。大虫见掀他不着,吼一声,却似半天里起个霹雳,振得那山冈也动;把这铁棒也似虎尾倒竖起来,只一剪,武松却又闪在一边。原来那大虫拿人,只是一扑,一掀,一剪, 三般提不着时,气性先自没了一半。那大虫又剪不着,再吼了一声,一兜兜将回来。武松见那大虫复翻身回来,双手轮起梢棒,尽平生气力,只一棒,从半空劈将下来。只听得一声响,簌簌地将那树连枝带叶劈脸打将下来。定睛看时,一棒劈不着大虫。原来慌了,正打在枯树上,把那条梢棒折做两截,只拿得一半在手里。那大虫咆哮,性发起来,翻身又只一扑,扑将来。武松又只一跳,却退了十步远,那大虫却好把两只前爪搭在武松面前。武松将半截棒丢在一边,两只手就势把大虫顶花皮肐地揪住,一按按将下来。那只大虫急要挣扎,早没了气力,被武松尽气力纳定,那里肯放半点儿松宽。武松把只脚望大虫面门上、眼睛里只顾乱踢。那大虫咆哮起来,把身底下扒起两堆黄泥,做了一个土坑。武松把那大虫嘴直按下黄泥坑里去,那大虫吃武松奈何得没了些气力。武松把左手紧紧地揪住顶花皮,偷出右手来,提起铁锤般大小拳头,尽平生之力,只顾打。打得五七十拳,那大虫眼里、口里、鼻子里耳朵 里都迸出鲜血来。那武松尽平昔神威,仗胸中武艺,半歇儿把大虫打做一堆,却似躺着一个锦布袋。……当下景阳冈上那只猛虎,被武松没顿饭之间,一顿拳脚,打得那大虫动掸不得,使得口里兀自气喘。武松放了手,来松树边寻那打折的棒橛,拿在手里,只怕大虫不死,把棒橛又打了一回。那大虫气都没了,武松再寻思道:“我就地拖得这死大虫下冈子去。”就血泊里双手来提时,那里提得动!原来使尽了气力,手脚都疏软了,动掸不得。武松再来青石坐了半歇,寻思道:“天色看看黑了,倘或又跳出一只大虫来时,我却怎地斗得他过?且挣扎下冈子去,明早却来理会。”就石头边寻了毡笠儿,转过乱树林边,一步步捱下冈子来。(节选《水浒传》) 1.请你以“地点+人物+情节”的格式概括选文内容。原著中有另一处打虎情节,请按同样格式写在答题线处。(2分)_________________________________________________________2.请再概括出两个有关武松的故事情节,并结合选文分析武松的性格特点。(4分)___________________________________________________________________________________________________________________________________________________________________________3.武松在打虎过程中,一开始就把梢棒打断了,这个细节有何妙处?(4分)__________________________________________________________________________________________________________________________________________________________________________景阳冈武松打虎。沂岭李逵杀四虎。示例:斗杀西门庆、醉打蒋门神、大闹飞云浦、血溅鸳鸯楼等。从武松拼尽全力也要打死老虎的情节中可以看出,武松具有勇敢无畏的英雄本色和为民除害的决心。作者借梢棒打断的细节,充分表现了武松全神贯注的紧张神态,渲染了这场恶斗的气氛,也为后文武松赤手空拳打虎的情节做了合理安排,从而突出了武松的神力和勇武。 2019秋部编版九年级语文上册第六单元第二十三课三顾茅庐 名师导学23*三顾茅庐一、课前自学知识储备1.正面描写:用生动形象的语言,把人物或景物的状态直接具体地描绘出来。2.侧面描写:作者通过对周围人物或环境的描绘来表现所要描写的对象,以使其鲜明突出,即间接地对描写对象进行刻画描绘。3.烘托:一种从侧面渲染来衬托主要写作对象的表现技法,突出表现写作对象的特点。写作时先从侧面进行描写,然后再引出主题,使要表现的事物更加鲜明突出。 一课一法体会课文刻画人物的多种手法:(1)正面描写:作者对诸葛亮的肖像描写、语言描写从正面表现了诸葛亮的雄才大略。(2)侧面烘托:①课文通过描写刘备两次拜访未见,反复渲染寻访孔明之不易,侧面烘托出了诸葛亮的神秘莫测和雄才大略,为下文的正面描写做铺垫。②作者采用侧面烘托的手法刻画刘备,多次写到张飞的急躁鲁莽来侧面烘托刘备礼贤下士、求贤若渴的仁君形象。二、内容梳理(根据课文填空,理清文脉)《三顾茅庐》一文由我国古典长篇小说《三国演义》第三十八回“司马徽再荐名士刘玄德三顾茅庐”改写而来。课文记叙了刘备三顾茅庐,诚心诚意请诸葛亮出山,辅佐他完成统一国家大业的故事, 赞扬了刘备求贤若渴、礼贤下士的精神,反复渲染了诸葛亮出山前的神秘莫测的特点以及他的雄才大略。课文可分为两个部分: 三顾茅庐第一部分(第1~2段):刘玄德三顾茅庐张飞的莽撞无礼(侧面烘托)刘备下马、徐步、拱立(正面描写)突出刘备①____________________________的特点求贤若渴、礼贤下士第二部分(第3~4段):诸葛亮隆中决策第一步:②____________________________________分析政治形势,争取荆、益二地第二步:利用魏、吴矛盾,造成鼎足之势第三步:谋图中原,实现统一,恢复汉室表现孔明盖世才能、③______________________的特点惊人智慧(雄才大略) 名师导练基础训练1.根据拼音写出相应的词语。(1)兄长两次亲往bàiyè(),其礼太过矣。(2)玄德曰:“汝若同往,不可shīlǐ()。”(3)这先生如何àomàn()!见我哥哥侍立阶下,他竟高卧,推睡不起!(4)汉室末胄,涿郡愚夫,久闻先生大名,rúléiguàněr()。(5)孔明曰:“南阳野人,shūlǎn()性成,屡蒙将军枉临,不胜kuìnǎn()。”拜谒失礼傲慢如雷贯耳疏懒愧赧 (6)玄德曰:“司马德操之言,徐元直之语,岂虚谈哉?望先生不弃bǐjiàn(),曲赐教诲。”(7)大丈夫抱jīngshìqícái(),岂可空老于林泉之下?(8)将军既帝室之胄,信义著于四海,总揽英雄,sīxiánrúkě()。(9)百姓有不dānshíhújiāng()以迎将军者乎?(10)先生之言,dùnkāimáosè(),使备如拨云雾而睹青天。鄙贱经世奇才思贤如渴箪食壶浆顿开茅塞 2.下列句子中加点的词语使用不恰当的一项是()A.作者怀着崇敬的心情,到成都郊外去拜谒他的祠堂,祠堂门口高耸入云的柏树枝繁叶茂、郁郁葱葱,古柏光明正大与诸葛亮的忠心耿耿交相辉映。B.世上不知有多少人,为着疏懒误了自己的人生。C.作者能站在时代的制高点,用入木三分的文笔,针砭世道人心,揭示生活真谛,从而起到如雷贯耳的作用。D.先生三言两语的指点,使我顿开茅塞,找到了研究工作难于突破的关键所在。C 3.下列对病句的修改不正确的一项是()A.近年来,工地施工升降机坠落事故时有发生,事故原因均牵涉到现场安全管理混乱,设备维修、安装和保养不到位,施工单位和政府部门监管缺位等问题。(把“维修、安装”改为“安装、维修”)B.“一带一路”的倡议得到沿线各国的理解、认同和支持,中国和这些国家之间的交流领域不断增加。(把“交流”改为“交往”)C.在那壮美的青藏高原上,是我祖祖辈辈生活的地方。(删去“在”和“上”)D.西宁市快速公交的正式运营,极大地缓解了市内交通拥堵。(在“交通拥堵”后加“的状况”)B 4.阅读下面的材料,按要求作答。2018年,海南一跃成为全球最大的自贸区(港),万众瞩目。作为最新设立也是前景最为广阔的自贸区(港),海南有着巨大的优势。首先,海南的地理位置得天独厚,从亚洲范围来看,海南位于东亚和东南亚的中心位置,是泛南海合作的中心,海上丝绸之路的关键节点。其次,海南的土地面积最大,即使与新加坡和迪拜等老牌国际知名自贸港相比,未来海南也毫不逊色。再者,海南是驰名中外的国际旅游岛,环境优美,是镶嵌在中国浩瀚南海上的一颗明珠。海南空气清新,散发着醉人的芬芳;阳光明媚,绽放着火一般的热情;_____________________________。因此,海南非常适合居住、度假和开展商贸活动。建立自贸区(港),必将使海南飞速发展,风靡全球。 (1)请用一句话概括以上内容,不超过35字。__________________________________________________________________________________________________________________(2)仿照画线句,再补写一个句子,使之与前两句构成一组排比句。___________________________________________________________________________________________________________________海南作为自贸区有着地理位置得天独厚、土地面积大、自然资源丰富等巨大优势。示例一:沙滩松软,呵护着人们的肌肤示例二:海浪翻滚,撩动着弄潮儿的心弦 课内精读却说玄德访孔明两次不遇,欲再往访之。关公曰:“兄长两次亲往拜谒,其礼太过矣。想诸葛亮有虚名而无实学,故避而不敢见。兄何惑于斯人之甚也!”玄德曰:“不然。昔齐桓公欲见东郭野人,五反而方得一面。况吾欲见大贤耶?”张飞曰:“哥哥差矣。量此村夫,何足为大贤!今番不须哥哥去;他如不来,我只用一条麻绳缚将来!”玄德叱曰:“汝岂不闻周文王谒姜子牙之事乎?文王且如此敬贤,汝何太无礼!今番汝休去,我自与云长去。”飞曰:“既两位哥哥都去,小弟如何落后!”玄德曰:“汝若同往,不可失礼。”飞应诺。于是三人乘马引从者往隆中。离草庐半里之外,玄德便下马步行,正遇诸葛均。玄德忙施礼,问曰:“令兄在庄否?”均曰:“昨暮方归。将军今日可与相见。”言罢,飘然自去。玄德曰:“今番侥幸得见先生矣!”张飞曰:“此人无礼!便引我等到庄也 不妨,何故竟自去了!”玄德曰:“彼各有事,岂可相强。”三人来到庄前叩门,童子开门出问。玄德曰:“有劳仙童转报:刘备专来拜见先生。”童子曰:“今日先生虽在家,但今在草堂上昼寝未醒。”玄德曰:“既如此,且休通报。”吩咐关、张二人,只在门首等着。玄德徐步而入,见先生仰卧于草堂几席之上。玄德拱立阶下。半晌,先生未醒。关、张在外立久,不见动静,入见玄德犹然侍立。张飞大怒,谓云长曰:“这先生如何傲慢!见我哥哥侍立阶下,他竟高卧,推睡不起!等我去屋后放一把火,看他起不起!”云长再三劝住。玄德仍命二人出门外等候。望堂上时,见先生翻身将起,——忽又朝里壁睡着。童子欲报。玄德曰:“且勿惊动。”又立了一个时辰,孔明才醒,口吟诗曰:大梦谁先觉?平生我自知。草堂春睡足,窗外日迟迟。孔明吟罢,翻身问童子曰:“有俗客来否?”童子曰:“刘皇叔在此,立候多时。”孔明乃起身曰:“何不早报!尚容更衣。” 遂转入后堂。又半晌,方整衣冠出迎。玄德见孔明身长八尺,面如冠玉,头戴纶巾,身披鹤氅,飘飘然有神仙之概。玄德下拜曰:“汉室末胄、涿郡愚夫,久闻先生大名,如雷贯耳。昨两次晋谒,不得一见,已书贱名于文几,未审得入览否?”孔明曰:“南阳野人,疏懒性成,屡蒙将军枉临,不胜愧赧。”二人叙礼毕,分宾主而坐,童子献茶。茶罢,孔明曰:“昨观书意,足见将军忧民忧国之心;但恨亮年幼才疏,有误下问。”玄德曰:“司马德操之言,徐元直之语,岂虚谈哉?望先生不弃鄙贱,曲赐教诲。”孔明曰:“德操、元直,世之高士。亮乃一耕夫耳,安敢谈天下事?二公谬举矣。将军奈何舍美玉而求顽石乎?”玄德曰:“大丈夫抱经世奇才,岂可空老于林泉之下?愿先生以天下苍生为念,开备愚鲁而赐教。”孔明笑曰:“愿闻将军之志。”玄德屏人促席而告曰:“汉室倾颓,奸臣窃命,备不量力,欲伸大义于天下,而智术浅短,迄无所就。惟先生开其愚而拯其厄,实为万幸!”孔明曰:“自董卓 造逆以来,天下豪杰并起。曹操势不及袁绍,而竟能克绍者,非惟天时,抑亦人谋也。今操已拥百万之众,挟天子以令诸侯,此诚不可与争锋。孙权据有江东,已历三世,国险而民附,此可用为援而不可图也。荆州北据汉、沔,利尽南海,东连吴会,西通巴、蜀,此用武之地,非其主不能守:是殆天所以资将军,将军岂有意乎?益州险塞,沃野千里,天府之国,高祖因之以成帝业;今刘璋暗弱,民殷国富,而不知存恤,智能之士,思得明君。将军既帝室之胄,信义著于四海,总揽英雄,思贤如渴,若跨有荆、益,保其岩阻,西和诸戎,南抚彝、越,外结孙权,内修政理;待天下有变,则命一上将将荆州之兵以向宛、洛,将军身率益州之众以出秦川,百姓有不箪食壶浆以迎将军者乎?诚如是,则大业可成,汉室可兴矣。此亮所以为将军谋者也。惟将军图之。”言罢,命童子取出画一轴,挂于中堂,指谓玄德曰:“此西川五十四州之图也。将军欲成霸业,北让曹操占天时,南让孙权占地利,将军可占人和。先取 荆州为家,后即取西川建基业,以成鼎足之势,然后可图中原也。”玄德闻言,避席拱手谢曰:“先生之言,顿开茅塞,使备如拨云雾而睹青天。但荆州刘表、益州刘璋,皆汉室宗亲,备安忍夺之?”孔明曰:“亮夜观天象,刘表不久人世;刘璋非立业之主:久后必归将军。”玄德闻言,顿首拜谢。 5.细读课文,说说文中哪些描写表现了刘备求贤若渴、礼贤下士的至诚精神。(4分)______________________________________________________________________________________________________________________________________________________________________________________________________________________________________________________________________________________________________________________________________________________①出发前的语言描写:叱责张飞的鲁莽,反衬刘备的大度和尊重人才的品质;②将到时的细节描写:“离草庐半里之外,玄德便下马步行”,表现了刘备礼贤下士的品质;③到达时的动作描写:“徐步而入”“拱立阶下”,耐心等候,表现了刘备求才之诚。这些描写生动细致地刻画出一个求贤若渴、胸怀大志的君主形象。 6.赏析下列句子在文章中的作用。(6分)(1)玄德见孔明身长八尺,面如冠玉,头戴纶巾,身披鹤氅,飘飘然有神仙之概。__________________________________________________________________________________________________________________(2)先生之言,顿开茅塞,使备如拨云雾而睹青天。__________________________________________________________________________________________________________________肖像描写,生动地表现出诸葛亮具有君子之相貌、儒士之才华和隐士之风度。运用了比喻的修辞方法,生动形象地写出了刘备对诸葛亮隆中对策的赞赏,从侧面烘托出诸葛亮谋划天下的盖世才略。 7.选文主要采用了什么手法塑造诸葛亮的形象?请结合选文简析。(4分)_____________________________________________________________________________________________________________________________________________________________________________________________________________________________________________________________________________________________8.选文中张飞具有怎样的性格特点?他在行文中起到什么作用?(3分)__________________________________________________________________________________________________________________①直接描写隆中对中的语言,诸葛亮为刘备献三分天下之计,滔滔不绝,既表现了他惊人的口才,对时局的洞察,更表现出他的高瞻远瞩和非凡韬略;②侧面烘托,刘备听到三分天下之计后“避席拱手谢”的动作描写,表现出刘备对诸葛亮才智的敬佩,从而烘托出诸葛亮的形象。作者反复突出了张飞的莽撞、无礼、急躁的性格特点,反衬出刘备宽容大度、求贤若渴、礼贤下士的仁君形象。 记叙文阅读技法专练——品味侧面烘托的作用我山里的汉子敦实威风,他们脱了棉袄,穿着对襟夹袄,舒张着血脉,喷红着脸膛,挥舞着铁臂,在一面五尺大鼓的指挥下把小鼓、腰鼓、铜钹、铜锣敲得地动山摇,惊神泣鬼。我看得正入神,只见掌大鼓的壮汉把双槌一收,抹了一把额头上的汗水,用眼光把人群扫了一遍,最后锁定我身边的一个老头,“张伯,你来耍一把。”“别,别……”这个叫张伯的老头长相瘦弱邋遢,穿着一件破旧的黑布棉袄,双手操在袖管里,听到喊叫,甚是羞涩地往后缩。这么个蔫老头能敲出什么道道来?但姑娘媳妇儿却揪住他,直往前推搡,“鼓神,来一个!”打鼓的汉子一脸虔诚,那些打小鼓、腰鼓、铜锣、铜钹的后生们也是满脸殷切……_这倒引起了我的好奇。 只见张伯像运气似地抡了抡双臂,脱袄,扔衣,蹿奔,大步流星,就在大鼓前站稳了。他拉开架势,将双槌高擎过头,布满皱纹和胡茬的脸上顿时豪气万丈。一个在黄土中长大的像黄土一般普通的汉子忽然间像天神降临高耸在鼓阵中间;一个在群山中滚爬像山石一样平凡的山民一瞬间如金刚发威挺立在群峰之上。他双眼生辉,一脸神圣,在轻敲几下鼓沿后,双臂一挥,劈下一串天雷,直教石破天惊。我像被魔语诅咒了似的,双目紧盯着他,只见他急敲慢叩——急如狂风暴雨,缓似岚岗生月;重擂轻掠——重如万马奔腾,轻似惊鹿踏雪,把一面鼓指派得出神入化。(节选自《鼓神》,有删改) 9.品读选段中的画线句子,简要分析其作用。(3分)____________________________________________________________________________________________________________________________________________________________________________________________________________________________________姑娘媳妇们口嚷“鼓神”往前推搡张伯,打鼓汉子“一脸虔诚”地看着张伯,后生们“满脸殷切”地望着张伯,“我”被表演震慑了,这些都是侧面描写,从而烘托出鼓神表演技艺的高超和震撼人心的精神。 中考考点实训名著阅读模拟训练宋江就忠义堂上与众弟兄商议立梁山泊之主。吴用便道:“兄长为尊,卢员外为次,其馀众弟兄各依旧位。”宋江道:“向者晁天王遗言:‘但有人捉得史文恭者,不拣是谁,便为梁山泊之主。’今卢员外生擒此贼,赴山祭献晁兄,报仇雪恨,正当为尊,不必多说。”卢俊义道:“小弟德薄才疏,怎敢承当此位!若得居末,尚自过分。”宋江道:“非宋某多谦,有三件不如员外处:第一件,宋江身材黑矮,貌拙才疏;员外堂堂一表,凛凛一躯,有贵人之相。第二件,宋江出身小吏,犯罪在逃,感蒙众弟兄不弃,暂居尊位;员外出身豪杰之子,又无至恶之名,虽然有些凶险,累蒙天祐,以免此祸。第三件,宋江文不能安邦,武又 不能附众,手无缚鸡之力,身无寸箭之功;员外力敌万人,通今博古,天下谁不望风而降。尊兄有如此才德,正当为山寨之主。他时归顺朝廷,建功立业,官爵升迁,能使弟兄们尽生光彩。宋江主张已定,休得推托。”卢俊义恭谦拜于地下,说道:“兄长枉自多谈,卢某宁死,实难从命。”吴用劝道:“兄长为尊,卢员外为次,人皆所伏。兄长若如是再三推让,恐冷了众人之心。”原来吴用已把眼视众人,故出此语。只见黑旋风李逵大叫道:“我在江州,舍身拚命,跟将你来,众人都饶让你一步。我自天也不怕,你只管让来让去做甚鸟!我便杀将起来,各自散火!”武松见吴用以目示人,也发作叫道:“哥哥手下许多军官,受朝廷诰命的,也只是让哥哥,他如何肯从别人?”刘唐便道:“我们起初七个上山,那时便有让哥哥为尊之意。今日却要让别人!”鲁智深大叫道:“若还兄长推让别人,洒家们各自都散!”(节选自《水浒传》) 1.请用简洁的语言概括选文的主要内容。(3分)_________________________________________________________2.选文主要通过什么描写手法表现人物性格?请分别概括选文中宋江、吴用和李逵的性格特点。(限用四个字)(3分)__________________________________________________________________________________________________________________3.选文的画线句子集中体现了宋江怎样的思想?联系原著,概述作品中能体现他这一思想的其他情节。(4分)__________________________________________________________________________________________________________________________________________________________________________众好汉议立梁山之主。语言描写。宋江:谦虚谨慎;吴用:机智灵活;李逵:粗鲁率直。画线句子集中表现了宋江浓厚的忠君思想,他希望建功立业,青史留名。作品中宋江最终选择接受招安,归顺朝廷,这也是其忠君思想的表现。 2019秋部编版九年级语文上册第六单元第二十四课刘姥姥进大观园 名师导学24*刘姥姥进大观园一、课前自学知识储备设置悬念的方法和作用:(1)设置悬念:在文章的某一部分设置一个疑问或矛盾冲突,以造成读者某种急切期待或热烈关心的心理的一种写法。有以下几种方法:①倒叙法:利用倒叙法设置悬念,就是把故事的结局先写出来,给读者以强烈鲜明的印象,让人们带着悬念去阅读下文。②疑问法:在叙述故事的过程中,故意地设置一些疑问,以引起 读者的深思。③误会法:利用作品中人物之间的猜疑或误解来激化矛盾,掀起波澜,不断推动情节的发展变化,最终释疑解扣。④省略法:叙述故事时,故意地省略一些内容,让读者一时难以琢磨,引起他们的种种猜疑和推想。(2)设置悬念的作用:使文章情节曲折生动;激发读者阅读兴趣;突出文章主旨;刻画人物形象;达到震撼人心的效果。一课一法课文采用省略法来设置悬念,激发读者阅读兴趣,不断为下文铺本张力:第一处:只简单交代凤姐和鸳鸯“二人便如此这般商议”。第二处:“鸳鸯便忙拉刘姥姥出去,悄悄的嘱咐了刘姥姥一席话”。 二、内容梳理《刘姥姥进大观园》生动地刻画出刘姥姥是位没有见过世面的乡野村妇,进而和贾府人形成了鲜明对比;同时透过刘姥姥的眼也映射出贾府“朱门酒肉臭”的景象,对封建统治阶级的腐败豪奢进行了深重的谴责。课文内容可分为三个部分:第一部分:写凤姐、鸳鸯等人设局取笑刘姥姥。第二部分:写刘姥姥上演“笑”剧的过程。第三部分:写刘姥姥上演“笑”剧后的感慨及凤姐、鸳鸯等人以实情相告。 名师导练基础训练1.根据拼音写出相应的词语。(1)tiáotíng()已毕,然后归坐。(2)众人先还fāzhèng(),后来一想,上上下下都一齐哈哈大笑起来。(3)黛玉笑chàqì(),伏着桌子只叫“嗳哟!”(4)贾母笑道:“这定是凤丫头cùxiá()鬼儿闹的!快别信他的话了。”(5)贾母又说:“谁这会子又把那个筷子拿出来了,又不请客摆大yánxí()!都是凤丫头支使的!还不换了呢。”调停发怔岔气促狭筵席 2.下列句子中加点的词语使用不恰当的一项是()A.但心中如海浪一般,汹涌澎湃的悲伤,一叠比一叠迅猛,直接将他心中残留的理智淹没,只能继续地发怔。B.有些刁钻促狭、迂腐愚蠢的人,喜欢吹毛求疵,对小说家的每一个细节都要追根究底。C.她善于发挥每个同学的长处,大家各得其所,各尽所能,为班级做出自己的贡献。D.初春时节,乍暖还寒,虽然身着冬装,漫步田野,仍然觉得不寒而栗。D 3.下列对病句的修改不正确的一项是()A.通过参加“优秀读书人物”评选活动,使他的学习热情更高涨了。(删去“通过”或“使”)B.房屋倒塌,主要原因是使用了劣质材料造成的。(删去“造成的”)C.在路况不好的公路上驾驶汽车,时速最好不要超过60千米左右。(删去“左右”)D.我校针对安全问题,采取了学生进出校门要打卡,保证学生的人身安全。(把“人身”改为“生命”)D 4.阅读下面的材料,按要求作答。北京故宫是中国明清两代的皇家宫殿,旧称为紫禁城,位于北京中轴线的中心,是中国古代宫廷建筑之精华。北京故宫以三大殿为中心,占地面积72万平方米,建筑面积约15万平方米,有大小宫殿70多座,房屋9000余间,是世界上现存规模最大、保存最为完整的木质结构古建筑之一。北京故宫于明成祖永乐四年(1406年)开始建设,以南京故宫为蓝本营建,到永乐十八年(1420年)建成。它是一座长方形城池,南北长961米,东西宽753米,四面围有高10米的城墙,城外有宽52米的护城河。紫禁城内的建筑分为外朝和内廷两部分。外朝的中心为太和殿、中和殿、保和殿,统称三大殿,是古代王朝举行大典礼的地方。内廷的中心是乾清宫、交泰殿、坤宁宫,统称后三宫,是 皇帝和皇后居住的正宫。1961年,国务院宣布故宫为第一批“全国重点文物保护单位”。从20世纪五六十年代起对其进行了大规模的修整。1988年故宫被联合国教科文组织列为“世界文化遗产”。 (1)阅读材料,提炼最核心的信息介绍故宫,不超过60字。__________________________________________________________________________________________________________________(2)根据材料内容,以“故宫”为描述对象,用上一种修辞方法,写一句话。___________________________________________________________________________________________________________________故宫是明永乐四年建于北京中轴线中心的皇家宫殿,是世界上现存规模最大的木质结构古建筑,被列为“世界文化遗产”。故宫,如一位饱经沧桑的老者,幽幽诉说着中华悠久的历史传说,见证着中华民族千百年来的兴衰荣辱。 课内精读正说着,只见贾母等来了,各自随便坐下,先有丫鬟挨人递了茶,大家吃毕,凤姐手里拿着西洋布手巾,裹着一把乌木三镶银箸,按席摆下。贾母因说:“把那一张小楠木桌子抬过来,让刘亲家挨着我这边坐。”众人听说,忙抬过来。凤姐一面递眼色与鸳鸯,鸳鸯便忙拉刘姥姥出去,悄悄的嘱咐了刘姥姥一席话,又说:“这是我们家的规矩,要错了,我们就笑话呢。”调停已毕,然后归坐。薛姨妈是吃过饭来的,不吃了,只坐在一边吃茶。贾母带着宝玉、湘云、黛玉、宝钗一桌,王夫人带着迎春姐妹三人一桌,刘姥姥挨着贾母一桌。贾母素日吃饭,皆有小丫鬟在旁边拿着漱盂、麈尾、巾帕之物,如今鸳鸯是不当这差的了,今日偏接过麈尾来拂着。丫鬟们知他要捉弄刘姥姥,便躲开让他。鸳鸯一面侍立,一面递眼色。刘姥姥道:“姑娘放心。”那刘姥姥入了坐,拿起箸来,沉甸甸的不伏手,——原是凤姐 和鸳鸯商议定了,单拿了一双老年四楞象牙镶金的筷子给刘姥姥。刘姥姥见了,说道:“这个叉巴子,比我们那里的铁掀还沉,那里拿的动他?”说的众人都笑起来。只见一个媳妇端了一个盒子站在当地,一个丫鬟上来揭去盒盖,里面盛着两碗菜,李纨端了一碗放在贾母桌上,凤姐偏拣了一碗鸽子蛋放在刘姥姥桌上。贾母这边说声“请”,刘姥姥便站起身来,高声说道:“老刘,老刘,食量大如牛:吃个老母猪,不抬头!”说完,却鼓着腮帮子,两眼直视,一声不语。众人先还发怔,后来一想,上上下下都一齐哈哈大笑起来。湘云掌不住,一口茶都喷出来。黛玉笑岔了气,伏着桌子只叫“嗳哟!”宝玉滚到贾母怀里,贾母笑的搂着叫“心肝”,王夫人笑的用手指着凤姐儿,却说不出话来。薛姨妈也掌不住,口里的茶喷了探春一裙子。探春的茶碗都合在迎春身上。惜春离了坐位,拉着他奶母,叫“揉揉肠子”。地下无一个不弯腰屈背,也有躲出去蹲着笑去的,也有忍着笑上来替他姐妹换衣裳的。独有 凤姐鸳鸯二人掌着,还只管让刘姥姥。刘姥姥拿起箸来,只觉不听使,又道:“这里的鸡儿也俊,下的这蛋也小巧,怪俊的。我且得一个儿!”众人方住了笑,听见这话,又笑起来。贾母笑的眼泪出来,只忍不住;琥珀在后捶着。贾母笑道:“这定是凤丫头促狭鬼儿闹的!快别信他的话了。”那刘姥姥正夸鸡蛋小巧,凤姐儿笑道:“一两银子一个呢!你快尝尝罢,冷了就不好吃了。”刘姥姥便伸筷子要夹,那里夹的起来?满碗里闹了一阵,好容易撮起一个来,才伸着脖子要吃,偏又滑下来,滚在地下。忙放下筷子,要亲自去拣,早有地下的人拣出去了。刘姥姥叹道:“一两银子也没听见个响声儿就没了!”众人已没心吃饭,都看着他取笑。贾母又说:“谁这会子又把那个筷子拿出来了,又不请客摆大筵席!都是凤丫头支使的!还不换了呢。”地下的人原不曾预备这牙箸,本是凤姐和鸳鸯拿了来的,听如此说,忙收过去了,也照样换上一双乌木镶银的。刘姥姥道: “去了金的,又是银的,到底不及俺们那个伏手。”凤姐儿道:“菜里要有毒,这银子下去了就试的出来。”刘姥姥道:“这个菜里有毒,我们那些都成了砒霜了!那怕毒死了,也要吃尽了。”贾母见他如此有趣,吃的又香甜,把自己的菜也都端过来给他吃。又命一个老嬷嬷来,将各样的菜给板儿夹在碗上。 5.刘姥姥进大观园后上演了一场“笑”剧,这“笑”剧是谁导演的?她们为什么要导演这场“笑”剧?(3分)____________________________________________________________________________________________________________________________________________________________________________________________________________________________________6.刘姥姥傻吗?为什么明明知道别人作弄她还要配合,甘愿给别人当笑料?这体现了刘姥姥怎样的性格特点?(4分)____________________________________________________________________________________________________________________________________________________________________________________________________________________________________刘姥姥进大观园后表演的这场“笑”剧,是由凤姐、鸳鸯等人导演的。她们觉得刘姥姥的言行举止与大观园有些格格不入,甚至滑稽可笑,于是就拿刘姥姥来开开玩笑,目的就是为了巴结讨好贾母。刘姥姥配合王熙凤上演这出“笑”剧,一是为了报答贾府曾经资助自己的恩情,同时也是为了刻意取悦讨好众人。刘姥姥是个善良淳朴的农妇,她诙谐幽默,自娱娱人,将自己作为笑料以供贾府众人玩乐;她也是一个精明乐观、知恩图报的人。 7.体会文章中描写手法的精妙。(6分)(1)贾母素日吃饭,皆有小丫鬟在旁边拿着漱盂、麈尾、巾帕之物。___________________________________________________________________________________________________________________________________________________________________________(2)刘姥姥便伸筷子要夹,那里夹的起来?满碗里闹了一阵,好容易撮起一个来,才伸着脖子要吃,偏又滑下来,滚在地下。__________________________________________________________________________________________________________________细节描写,写出贾母日常生活排场的奢华,生动地表现了贾母的富贵威严,与刘姥姥形成对比,突出人物形象,也展示了贾府的繁荣盛景。动作描写,生动传神地写出了刘姥姥吃鸽子蛋时的滑稽可笑而又憨态可掬的神态,突出了刘姥姥的淳朴诙谐的性格特点。 8.文中细致地描写了各具情态的笑,给人留下了难忘的印象。请选取两个人物加以分析,说说反映了他们怎样的性格特点。(4分)______________________________________________________________________________________________________________________________________________________________________________________________________________________________________________________________________________________________________________________________________________________示例:①史湘云最不会控制自己的情绪,最先喷饭,反映出她率真、爽朗、不受拘束的性格特点;②林黛玉身体娇弱,所以一笑便“岔了气”,林黛玉的笑又极力控制,反映出她含蓄、有教养而又谨慎的性格特点;③宝玉笑时钻到贾母的怀里,反映出他的天真、孩子气的性格特点;④惜春笑得肚子疼,让奶母给揉肠子,反映出她娇气、孩子气的性格特点。 记叙文阅读技法专练——分析悬念的设置看到北坡两块麦田泛黄,村主任老王心里火烧火燎。回到家,便给在城里的女儿小菊打电话,让她回来帮这两户割麦。女儿不肯,说旅行社最近忙。老王说:“再忙也要回来,不然爹就自己去收麦,累死也去。”说完啪地挂断电话。一会儿,女儿打回电话说:“我星期天组织些人来割麦,但您要答应我一个条件。”“只要你回来割麦,100个条件都答应。”“那好,您准备40把镰刀,40顶新草帽。”老王说:“镰刀可以去借,草帽得买。”女儿说:“您买吧,到时我给钱。”一顶草帽好几块呢,老王本想不答应,可想到熟了的麦子,只有点头。星期天,老王一早来到北坡。八点了,人没来。九点了,人还没来。 看看越来越热的太阳,老王往手心猛啐两口唾沫,抄起镰刀,咔咔地割起来。这时,一辆豪华大巴卷着黄土,停在了坡边。车门打开,女儿小菊施施然地走下来。一身套裙,扎着领结,还提个电喇叭。老王一看,鼻子快气歪了,这是来割麦的吗?接着下来一帮城里青年,有说有笑的。小菊拿起电喇叭说:“我们从小就学‘锄禾日当午,汗滴禾下土’,但却没干过农活,今天就让大家亲自体验,自由组合,一块麦田算一组,咱们来个割麦比赛!”“好!”青年们摩拳擦掌,你追我赶割起了麦子。小菊没割,对着电喇叭说:“唐代诗人白居易有首著名的《观刈麦》,诗中说,‘田家少闲月,五月人倍忙。夜来南风起,小麦覆陇黄……’”(节选自刘怀远的《在唐诗中割麦》,有删改) 9.选文开篇写老王焦急的等候有什么作用?(3分)__________________________________________________________________________________________________________________10.悬念是写作常用的手法,请说说选文中两处主要的悬念设置。(4分)___________________________________________________________________________________________________________________________________________________________________________通过写老王焦急的等候,为下文写小菊带领旅行团一行回乡割麦做铺垫。本文设置的悬念主要包括如下两处:一是通过写女儿提出回家割麦的条件设置悬念;二是通过描写女儿的穿着设置悬念。 中考考点实训名著阅读模拟训练正说之间,只见差拨过来,问道:“那个是新来的配军?”林冲见问,向前答应道:“小人便是。”那差拨不见他把钱出来,变了面皮,指着林冲便骂道:“你这个贼配军,见我如何不下拜,却来唱喏?你这厮可知在东京做出事来,见我还是大剌剌的。我看这贼配军满脸都是饿文,一世也不发迹。打不死,拷不杀的顽囚,你这把贼骨头好歹落在我手里,教你粉骨碎身,少间叫你便见功效。”林冲骂的一佛出世,那里敢抬头应答。众人见骂,各自散了。林冲等他发作过了,却取五两银子,陪着笑脸告道:“差拨哥哥,些小薄礼,休嫌小微。”差拨看了道:“你教我送与管营 和俺的都在里面?”林冲道:“只是送与差拨哥哥的。另有十两银子,就烦差拨哥哥送与管营。”差拨见了,看着林冲笑道:“林教头,我也闻你的好名字,端的是个好男子,想是高太尉陷害你了。虽然目下暂时受苦,久后必然发迹。据你的大名,这表人物,必不是等闲之人,久后必做大官。”林冲笑道:“皆赖差拨照顾。”差拨道:“你只管放心。”又取出柴大官人的书礼,说道:“相烦老哥将这两封书下一下。”差拨道:“即有柴大官人的书,烦恼做甚!这一封书值一锭金子。我一面与你下书,少间管营来点你,要打一百杀威棒时,你便只说你一路有病未曾痊可。我自来与你支吾,要瞒生人的眼目。”林冲道:“多谢指教。”差拨拿了银子并书,离了单身房自去了。林冲叹口气道:“有钱可以通神,此语不差。端的有这般的苦处。”(节选自《水浒传》) 1.选文中作者采用了什么表现手法刻画差拨怎样的形象特点?(3分)____________________________________________________________________________________________________________________________________________________________________________________________________________________________________2.选文着力刻画差拨的形象有何作用?(3分)__________________________________________________________________________________________________________________________________________________________________________作者采用对比手法刻画人物,差拨先是骂林冲为“贼配军”“贼骨头”,后来夸林冲为“好男子”“久后必然发迹”,先骂后夸的对比手法突出刻画了差拨利用职权诈取钱财的势利虚伪的小人形象。差拨的跋扈羞辱反衬了林冲遭受迫害却无奈隐忍的形象,侧面反映了黑暗腐朽的社会现实,突出了“官逼民反”的主题思想。 3.林冲是《水浒传》中受官府迫害最深的人物之一,联系原著,概述林冲遭受迫害的其他情节。(4分)______________________________________________________________________________________________________________________________________________________________________________________________________________________________________________________________________________________________________________________________________________________①高太尉设计让林冲误入白虎堂,导致林冲被判携刀私入白虎堂,刺配沧州;②林冲被陷害刺配沧州途中,高俅收买公差董超、薛霸,图谋杀死林冲,幸得鲁智深“大闹野猪林”相救;③林冲到达沧州后被分到看守草料场的工作,陆虞候又火烧草料场,意图杀死林冲。 部编版九年级语文上册教学课件第一单元1.祖国啊我的祖国 日积月累 课文助读走近作者舒婷,中国当代女诗人,朦胧诗派代表人物,著有诗集《双桅船》《会唱歌的鸢尾花》《舒婷顾城抒情诗选》等。 写作背景1978年12月,中国迎来了具有重大历史意义的十一届三中全会,开启了改革开放历史新时期。1979年4月,诗人面对祖国摆脱苦难、正欲奋飞的情景,以自己独有的抒情方式写下了此诗。 主要内容《祖国啊,我亲爱的祖国》是一首深情的爱国之歌,诗中将个体的自我放在祖国的大形象里,并承担起使祖国繁荣富强的重任,表达了强烈的爱国之情和深沉的历史责任感。 基础过关 2.下列诗句,朗读节奏标示有误的一项是()A.我/是你河边上/破旧的老水车B.照你/在历史的隧洞里/蜗行摸索C.我/是新刷出的/雪白的/起跑线D.是“飞天”袖间/千百年来未落到/地面的花朵3.在这首诗中,诗人抒发了对祖国怎样的感情?这种感情又是通过哪些事物来表现的?这些事物有什么特点?D诗人抒发了与祖国血肉相连、荣辱与共的感情,表达了强烈的爱国之情和历史责任感。诗人通过一些具体事物来表现这种情感,如“老水车”“矿灯”“稻穗”“路基”“驳船”等,象征着祖国饱经风雨而依然具有顽强活力的特点。 5.(育才中学中考模拟)下列句子中,有语病的一项是()A.读了恶书、烂书,等于给脑海中输送了负能量,还不如不开卷,任凭风霜雪雨来教导。B.在伟人传记的字里行间汲取精神养分,让自己的身心变得充盈富足起来,也就有了对抗社会风浪袭击。C.要是在人生观、价值观上好好培育,除草、驱虫、施肥、灌溉,“种心”也就收获了“心正”。D.如果生命的长度没法把控,那么我们就应该尽量延伸生命的宽度。B 6.(西大附中中考模拟)下列句子排列顺序正确的一项是()①短短3个月间我国就发生了12次沙尘暴,波及大半个中国。②草原牧场不断地被过度放牧,又不进行补偿性保护种植,大大加重了草场退化。③近几十年来,我国由于人口急剧增长,不少地方便以超垦、过牧和滥伐获取必要的生活资料。④于是导致去年一场场席卷而来的沙尘暴频频袭击了我国北方大部分地区。⑤结果粮食没打多少,反而造成了土壤盐碱化和荒废了更多的土地。⑥大片的树林草原被开垦成了农田。A.①②③④⑥⑤B.⑥④①⑤③②C.②⑤③④①⑥D.③⑥⑤②④①D 7.(重庆一中中考模拟)将汉字的某些形近字放在一起,常常会引发有趣的联想,让人悟出一些哲理,请仿照示例,自选两个形近字写一句有哲理的话。示例:“上”与“下”:追求的方向不一样,也就决定了人生境界的高低。我选择的是:“人”与“大”:承担了应该承担的责任,人才会变得伟大。 1.(独家原创试题)下列诗句的诵读节奏划分有误的一项是(  )A.——祖国啊,我亲爱的/祖国!B.我/是你簇新的理想,刚从神话的蛛网里/挣脱C.我/是新刷出的/雪白的起跑线;是/绯红的黎明D.我/是你雪被下古莲/的胚芽;我/是你挂着眼泪的/笑涡答案D 正确的划分是:我/是你雪被下/古莲的胚芽。 2.将下列意象按其象征意义进行归类。(只填选项)意象:A.破旧的老水车   B.熏黑的矿灯C.干瘪的稻穗  D.失修的路基E.雪白的起跑线  F.绯红的黎明G.雪被下古莲的胚芽H.千百年来未落到地面的花朵象征意义:(1)祖国的未来和希望:。(2)祖国长期以来贫穷与落后的状态:。(3)人民心中渺茫、难以实现而又从未消失的对未来幸福的希望:。 答案(1)EFG (2)ABCD (3)H解析回答本题时,一要根据课内所学进行选择,二要根据意象名称及其修饰词确定。“起跑线”“黎明”和“胚芽”象征着希望和未来;“破旧的老水车”“熏黑的矿灯”“干瘪的稻穗”和“失修的路基”描绘了祖国贫穷落后的状态;“千百年来未落到地面的花朵”揭示出蕴藏在中华民族灵魂中的希望之花从未消亡,灾难虽重,理想却不曾消亡,只是暂未实现。 3.2019年10月1日是中华人民共和国成立七十周年的日子。让我们一起创作一首小诗向祖国献礼。请仿照上下节把下面这首小诗补写完整。寄祖国你是一条大河,奔流不息,在你的岸边,我美好地生长。我就是那羞涩的小花,为你绽放是我的心愿。 你是一棵大树,郁郁葱葱,你是我的祖国,美丽富饶,在你的怀抱,我美好地生长。我就是那意气风发的少年,为你奋斗是我的心愿。 答案(示例1)在你的枝头,我美好地生长。我就是那快乐的小鸟,为你歌唱是我的心愿。(示例2)在你的周围,我美好地生长。我就是那腼腆的小草,为你守候是我的心愿。解析此题考查仿写能力。通过观察可知,应按照“你是……,+(形容词),在你……,我美好地生长。我就是……,为你+(动词)+是我的心愿”的格式仿写。 1.关于这首诗的体裁,判断正确的一项是(  )①自由体新诗  ②现代格律诗  ③叙事诗④抒情诗  ⑤哲理诗  ⑥近体诗A.①④  B.①③⑥  C.①④⑥  D.②④⑥答案A 自由体新诗:运用现代白话写作,不拘泥于外在韵律和音节等,诗体不受任何框式的束缚,段、行和字数都不固定。现代格律诗是用白话依据固定的格式和严密的韵脚进行创作的诗体。叙事诗是以叙述者的口吻来刻画人物、叙述事件的诗体。抒情诗是以作者的口吻抒发主观情绪情感的诗体。哲理诗:表现诗人的哲学观点、反映哲学道理的诗。近体诗也叫“今体诗”,是指在唐初形成的一种以讲究平仄、对仗为特点的格律诗。 2.阅读下面诗歌,请从文学欣赏的一两个角度进行赏析。祖国啊,我要燃烧叶文福当我还是一株青松的幼苗,大地就赋予我高尚的情操!我立志做栋梁,献身于人类,一枝一叶,全不畏雪剑冰刀!不幸,我是植根在深深的峡谷,长啊,长啊,却怎么也高不过峰头的小草。我拼命吸吮母亲干瘪的乳房,一心要把理想举上万重碧霄!我实在太不自量了:幼稚!可笑! 蒙昧使我看不见自己卑贱的细胞。于是我受到了应有的惩罚——迎面扑来旷世的风暴!啊,天翻地覆……啊,山呼海啸……伟大的造山运动,把我埋进深深的地层,——我死了,那时我正青春年少。我死了!年轻的躯干在地底痉挛,我死了!不死的精灵却还在拼搏呼号:“我要出去!我要出去!我要出去啊——我的理想不是蹲这黑暗的囚牢!” 漫长的岁月,我吞忍了多少难忍的煎熬,但理想之光,依然在心中灼灼闪耀。我变成了一块煤,还在舍命呐喊:“祖国啊,祖国啊,我要燃烧!”地壳是多么的厚啊,希望是何等的缥缈!我渴望:渴望面前闪出一千条向阳坑道!我要出去,投身于熔炉,化作熊熊烈火:“祖国啊,祖国啊,我要燃烧——”答: 答案①诗歌运用拟物化的手法,借助青松幼苗、地下煤块等意象,叙写了“我”的人生轨迹,抒发了作者不甘于现状,为实现理想,将积极进取、热情奉献的思想感情。全诗主题鲜明,激情洋溢,气势磅礴。即使身处低谷,埋进地层,吞忍煎熬,“我”也在渴望投身熔炉。诗中一次次的“祖国啊,祖国啊,我要燃烧!”的反复呐喊,足以表现出赤子对母亲的殷殷报效之心和拳拳奉献之意,那一腔忠诚和热血,真可谓动人心弦,撼人心魄。②诗歌结构跌宕起伏,回环曲折。 诗歌通过结构上跌宕起伏、回环曲折的精妙设计,把“我”乐观、积极、坚毅、不屈的性格和不屈从命运,努力抗争,执着追求崇高理想,即使天不恋我,也将洒尽热血、一心报国的忠贞决心表现得淋漓尽致,真可谓使苍天动容,让世人震撼。这种特有的结构形式一改平淡的表白,能更好地触动读者的思绪,撞击读者的心扉,展现出诗歌强大的艺术感染力,使作品气 韵延绵,回味悠长。③诗歌的遣词也很有特点,许多动词、形容词用得生动形象,恰如其分,值得品味,细腻地表现了作者的思想感情,如“蒙昧、旷世、痉挛、吞忍、灼灼、舍命、熊熊”等。另外,第一人称的口吻也显得亲切自然,能很好地感染读者;尤其是反复修辞手法的运用,更使得诗歌情意真切,气贯长虹,动人心弦,引人共鸣。(只要抓住一至两个赏析点,能做正确的赏析评价,符合情理,内容具体,表达通顺即可) 解析《祖国啊,我要燃烧》写于1979年,诗歌的语言很有激情,诗人以煤为抒情主人公,深切地表达了要冲破漫长的岁月,与坚硬的地壳搏斗,把缥缈的希望变成现实的愿望。这种不怕牺牲,渴望为人民燃烧,为祖国贡献热能的呼喊,正是广大知识分子的共同心声。诗中坦露的为祖国就是烧成灰烬也在所不辞的殷切之情,感人至深。这首诗主题深刻,抒情独特,是一首优秀的现代诗。赏析的时候可以从内容、情感、手法等角度入手,注意语言通顺。 阅读提高(巴蜀中学中考模拟)纸上留香包利民①记得多年前,在舅舅家的墙上看到一幅字画,四个篆体大字“梅馨入梦”,虽然当时尚是冬天,我们那里也没有梅花,却依然从四个字间感受到了一种若有若无的香气。那不是墨香,而是少年的心中第一次生发出来的意境和想象,从此那四个字便印在了我的心上。②读初中时,有一阵子很盛行一种带有香气的信纸。那时我们常常写信,或是给远方的亲友,或是在杂志上看到的作者,而且那些好看且带有香味的信纸,被折叠成不同形状,蕴含着不同的意思。那时也曾收到过这样的信,展读之际,淡香盈然,伴着字里行间的温暖,心儿便无比的宁静和欣喜。③现在想来,那是纯真年代最朴素的一种香味,却遥远得不可追溯。回忆那些书来信往的时光,即使是最简单的信纸、最简短的问候,也在生命中氤氲着无尽的香气,淡雅悠长,一 如那些如月澄澈的年华。那些写满思念与思绪的信,就像我们的青春一样,一去不回。在这个通讯没有距离的年代,我们失去了等候的味道,也失去了在小窗前,在阳光下,捧读远方来信的芬芳心境。④后来读书渐多,知道了唐代女诗人薛涛,也知道了她发明的“薛涛笺”。那是一种深红色的纸,可写信,亦可题诗。又叫“浣花笺”,就像李贺诗中说“浣花笺纸桃花色,好好题词咏玉钩”,想来就让人神飞无限。我觉得,薛涛制于浣花溪畔百花潭边的红笺,虽美在其色,更重要的,是其所蕴清馨,未题字句而先成意境,所以历来为人所钟爱。⑤中学时有一阵子疯狂迷恋上书法,因为当时有个老师是书法家,给我们看过他的不少获奖作品,毛笔字各体皆佳,一下子镌进心里。当时有几个伙伴一起练,找来许多旧报纸,闲时便写,乐此不疲。那时满室充盈着墨香,还有我们欢快的笑声。随着学业的加重,书法渐渐地远去。闲来写上几笔,却是无由的烦躁。那么多年过去,有时想起曾经泼墨挥毫的岁月,便有着一种沧桑感,我知道,我不可能再有那么单纯而无忧的心境。也许,在走过的成长之路上,除了悠悠的墨香,什么也 没有留下。有一天,在网上看到当年一起练书法的同学的博文,她却是迥然的心境,她写道:岁月和心情都远去,可是,我却没有辜负当年的那些旧报纸。⑥没有辜负当年的旧报纸,是啊,那些纸上,曾写下我们多少青春的梦想,留下我们多少稚嫩的情愫,一如初开的花儿,清香满溢。那些香气,那些梦想,那些时光,只有曾经的旧报纸知道。⑦有一次在一家旧书店买回一箱子书,翻看时才发现,其中竟混有一个古老的日记本,塑料皮儿,中间还有彩色插画。上面的字迹已经变色模糊,就像隔着岁月的尘烟。我便饶有兴致地阅读,那是一个女生的日记,记录着少年的心事,多么简单的时光,多么朴素的成长。是的,那个时候,我们就是用笔来和自己说话,对着日记,将满腹之言倾吐。于是想起自己曾经记过的几十本日记,它们就放在故乡的老家里,那一刻,有着一种回头看看的冲动。⑧前一阵子回老家,翻箱倒柜地找自己的那些日记,却是杳无踪迹。可能父母搬家时,不知失落于何处。满心的怅然失落,那是我从小学到大学的所有日记,现在,想从当年的心事 中重温一遍成长也空如一梦。那些年留在日记上的字,也会有着一种香气吧?就像偶然得到的那本女孩的日记,虽隔着漫长的岁月,却依然洇染着我的心境。我希望,我的那些日记,也会偶尔温暖一个人的回望,好能在这个纷扰嘈杂的樊笼里,感到丝丝的清凉。⑨忽然发觉,似乎已经许久不曾提笔写字了,习惯了触摸键盘的手,对纸笔有着畏惧与陌生。那个夜里,偶然一梦,自己仿佛还是少年时,拿着毛笔在旧报纸上写字,写下的每一个字都开出了一朵花,就像当年那些纯真的笑颜,于是梦里一片芬芳,于是醒来时的眼中,有着浅浅的濡湿。 1.请分析标题“纸上留香”有何作用。2.请简要概括这篇文章写了哪几件与“纸”有关的事。概括文章主要内容,暗示文章主题,表达了作者对曾经拥有的美好时光的怀念之情,引起读者阅读兴趣。①初中时用带香气的纸通信;②回忆自己知道的“薛涛纸”;③中学时用旧报纸写字;④寻找自己曾写的日记。 3.文章第⑤段中,练书法的同学为什么说自己“没有辜负当年的那些旧报纸”?4.请理解下面句子的含义。我的那些日记,也会偶尔温暖一个人的回望,好能在这个纷扰嘈杂的樊笼里,感到丝丝的清凉。因为那些纸上曾写下了他们青春的梦想,留下了他们稚嫩的情愫,记录了曾经美好的时光。“我”的那些日记,也会偶尔让看到它的人忆起往事感到温暖,让他能在这个喧嚣的世界里,心灵得到一些宁静与安慰。 5.“馨香青春”丛书的编辑欲将此文编入丛书,于是对文章的结尾(第⑨段)作了如下修改。你认为是作者的结尾好,还是编辑的结尾好,为什么?今夜,又想起舅舅家墙上“梅馨入梦”这四个篆体大字。我似乎嗅到了岁月的气息,翻箱倒柜地找出一沓泛黄的旧稿纸,拿起笔来在上面写下了大大小小的文字。这些如花般的文字在昏黄的灯光下散发着墨香,它们氤氲着,跳跃着,再次映红了我有些苍白的脸庞。我知道,这叠稿纸将伴我走过今后的道路,让我的人生重新溢满香气……示例一:我觉得作者的结尾好。因为这段文字写了自己梦到自己少年时代再次写毛笔字,照应了第⑤段作者中学时练书法的事,表达了作者许久不再提笔写字的无奈以及对曾经美好回忆的怀念之情。示例二:我觉得编辑的结尾好。因为这段文字写了作者重新拿起笔来在纸上书写文字的事,照应了第①段多年前在舅舅家的墙上看到四个篆体大字“梅馨入梦”,表达了作者重新提笔写字的坚定决心和对美好时光的向往,编入“馨香青春”丛书更显出青春的正能量。 非连续性文本阅读(重庆一中中考模拟)5G来了!究竟能做什么4G做不了的?5G来了!让人充满想象。近日,记者搜集了一些大家对5G最感兴趣的话题,采访了10余位业内权威专家,提取干货,以期大家快速、准确了解5G。5G就是比4G多了1个G?5G速度更快、能实现分流。常说5G是大带宽、低时延。一部10G的视频,4G下载需15分钟,5G仅需9秒;无人驾驶刹车智能控制反应距离,4G下是1.4米,5G下是2.8厘米。如果把2G、3G、4G几代移动通信技术比作把路不断修宽,让更多车可以跑,5G则是利用技术在继续修宽高速路的同时,对路进行规划,实现分流,提高利用效率。比如切片技术,大唐电信集团总工程师王映民介绍:“在一个大楼内,有人用网络进行远程医疗,有人微信聊天,5G网络设计会自动让远程医疗获得更快、更可靠的网络资源,5G可以根据不同行业、用户需求,自动实现分门别类分配网络。” 再比如利用边缘计算技术,数据不用再传到遥远的云端,在边缘侧就可以计算,这实现最近距离、最短时间,把网络上需要的资源推到用户面前。用5G要不要换手机?答案是肯定的。4G手机接收不了5G信号,5G手机依然可以接收4G信号。展望5G时代的手机,造型将更多元,折叠屏、曲面屏等,在一段时间内都将成为手机厂商尝试的方向。目前,华为、小米、三星等手机厂商都已推出5G手机样机。业内预计,今年下半年,国内可能出现5000元左右的5G手机,5G手机大规模上市可能在明年。5G对你有什么影响?业内常说,4G改变生活,5G改变社会。5G来了,普通人先接触到的可能是4K、8K直播,VR、AR游戏等,体验“全场景”“沉浸式”效果。目前,基于5G网络下的远程医疗、自动驾驶、无人机作业等也开始探索应用。4G时代,人与人的连接已经差不多完成,5G将实现人与物、物与物的连接,也就是家庭、办公室、城市里的物体都将实现连接,走向智慧和智能。5G下物联网每平方公里连接数可超过一百万。万物互联时代将到来。 “4G改变了不少服务业,5G在改变工业方面潜力巨大,尤其是工业机器人应用会进一步拓展。”曹磊说。5G之后将怎样?国内已开始研究6G,共识是基于人工智能。走向智能是通信技术、网络技术的必然趋势,6G的网络构架一定是为了满足人类更深层次智能通信需求。对于未来,需要打开想象。比如,6G下的网络传感设备,可能会实时采集物品的画面、气味、温度、湿度、光线等信息。再比如,北京邮电大学信息与通信工程学院教授张平等的一篇研究文章提出,6G不仅包含5G下人、机、物这三个元素,还将设计存在于虚拟世界的第四元素。这是人的人工智能超级助手,它会将客观和主观结合进行决策建议。总之,未来不远。 1.第3、4两部分内容可以换顺序吗?请简述理由。2.文中多处引用业内权威专家的话,请你任选一处,说一说它的作用。不能换顺序;第3部分介绍的是5G即将给生产生活带来的变化,第4部分则是设想(预测)5G之后的6G的前景,两者之间有时间(发展)的先后顺序。示例:文中引用王映民的介绍,具体说明5G能实现分流,提高利用效率。并且这样引用,能增强本文所介绍内容的可信度,提高文章的权威性。 3.如何理解结尾句“未来不远”?4.对人工智能的发展,有人欢喜有人忧,你怎么看待人工智能的发展?请作阐述,50字左右。指能满足人类更深层次需求的人工智能世界已不遥远;表达了对未来人工智能世界的憧憬与自信。示例:应当明确人工智能正成为潮流,用更开放的心态拥抱智能化变革。但人工智能不是万能的,应该冷静客观看待,多角度思考和审视,人工智能会给生活工作带来改变,但技术不能取代人的兴趣、感受和情怀等等。 朋友是快乐日子里的一把吉他,尽情地为你弹奏生活的愉悦;朋友是忧伤日子里的一股春风,轻轻地为你拂去心中的愁云。朋友是成功道路上的一位良师,热情地将你引向阳光的地带。朋友是失败苦闷中的一盏明灯,默默地为你驱赶心灵的阴霾。美文欣赏·友情 谢谢观看 部编版九年级语文上册教学课件第一单元2.梅岭三章 日积月累 课文助读走近作者陈毅(1901—1972),字仲弘,四川乐至人,中国共产党党员。伟大的无产阶级革命家、军事家、诗人。著有诗集《陈毅诗词选集》。 写作背景1934年10月,江西中央红军开始长征,陈毅因伤留在江西担任军事指挥,并主持政府工作。1935年春,他在油山和梅山地区开展游击战争,直到1937年抗日战争爆发才离开。这三首诗就写于三年游击战争时期。 主要内容在这三首诗里,作者用革命乐观主义的态度表现了自己献身革命的崇高精神和对革命必胜、共产主义理想必定实现的坚定信念。全诗表现了诗人视死如归的气概和誓与反动派血战到底的革命精神。 基础过关 3.下列说法有误的一项是()A.诗前小序说明作者在身临绝境时用诗的形式留下了自己的遗言。B.诗的小序可有可无。C.三首诗可看成一个整体,又各表达不同的意思。D.“血雨腥风应有涯”“人间遍种自由花”“取义成仁今日事”这三句分别运用了借喻、借代、引用的修辞手法。B 5.(育才中学中考模拟)下列句子排序正确的一项是()①你会发现,烦恼正渐渐消散。再次启程,你会一身轻松。②停下来,在大自然的怀抱里感受叶的光芒、花的辉煌、草的力量。③面对他人的质疑,你是否苦恼过?面对学业的受挫,你是否沮丧过?④那么,何不选择偶尔停下来呢?A.③④②①B.②①③④C.③②①④D.②④③①A 1.第三首诗中有两个成语,请指出并写出其含义。(1):(2):答案(1)血雨腥风 形容残酷屠杀的景象。(2)取义成仁 为了成全仁义,不惜牺牲生命。这里指为了人民的解放事业而勇于牺牲。解析根据所学所记,理解诗歌内容,直接写出成语,并联系平时积累写出其大意即可。能力提升全练 2.下列诗句中与“此去泉台招旧部,旌旗十万斩阎罗”意境相同的一项是(  )A.生当作人杰,死亦为鬼雄。(李清照)B.僵卧孤村不自哀,尚思为国戍轮台。(陆游)C.人生自古谁无死?留取丹心照汗青。(文天祥)D.生命诚可贵,爱情价更高。若为自由故,二者皆可抛。(裴多菲)答案A 根据所学所记,理解“此去泉台招旧部,旌旗十万斩阎罗”表达的内容,即誓死与反动派做斗争,A项中“死亦为鬼雄”与其意境相同。 3.阅读课文内容,回答问题。(1)理解下列句子的含意。①捷报飞来当纸钱。答:②血雨腥风应有涯。答:(2)赏析“人间遍种自由花”一句。答: 答案(1)①诗人借用烧纸钱纪念死者的民间习俗,生动形象地道出了他对生者、对战友寄予的最大期望。②表明诗人从“血雨腥风”的茫茫黑夜中已看到了黎明的曙光。(2)作者用“自由花”比喻革命胜利的前景,“遍”字充分表现了作者对革命前途的展望,表达了作者胜利的信心。此句想象丰富,充分表现了诗作的浪漫主义风格。解析题(1),根据全诗内容理解“捷报飞来”的含义和“血雨腥风”的比喻用法。题(2),赏析诗句,要从内容、写作方法、语言风格等方面进行分析,重点分析其中生动的字词,说明其精妙的作用。 1.本课的第三首诗从格律来看,是。诗中押韵的字是。答案绝句 家、涯、花解析根据知识积累直接解答即可。 2.依据语段内容,对画线句修改最恰当的一项是(  )我国历史上,曾出现过一大批忧国忧民的仁人志士和坚强不屈的民族英雄。他们积极改革、励精图治以顺应历史潮流,他们反抗民族压迫、抵御外来侵略以捍卫国家主权、民族尊严,中华民族的爱国主义精神从不同侧面得以体现,谱写了壮丽的爱国诗篇,铸就了不朽的民族之魂。A.修改:中华民族不同侧面的爱国主义精神得以体现B.修改:不同侧面中华民族的爱国主义精神得以体现C.修改:从不同侧面体现了中华民族的爱国主义精神D.修改:体现了不同侧面的中华民族的爱国主义精神 3.赵、钱、孙、李四个同学读了下面这首诗后各自发表了看法,请找出其中错误的言论,并说明理由。坚 壁[注]田间狗强盗,你要问我么:“枪、弹药,埋在哪儿?”来,我告诉你:“枪、弹药,统埋在我的心里!”1943年6月作 [注]诗题“坚壁”是“坚壁清野”的简称。“坚壁清野”是抗战时期我国军民对付日军“三光”政策的方法之一,指把粮食、枪弹等埋藏起来,不让敌人得到。赵:开头直呼“狗强盗”,三个字把“我”烈火般的愤怒和仇恨倾泻无遗。钱:“你要问我么:‘枪、弹药,埋在哪儿?’”这个问句表示“我”已把敌人看透了,突出了“我”的自信。孙:“来,我告诉你”,是“我”面对敌人搜查时的嘲弄、斥责之词,也是全民抗战的真实写照。李:全诗采用虚拟手法塑造了“我”的形象,而对敌人却有意冷落,不费笔墨,表现了诗人对国民党反动派的极端蔑视和仇恨。答: 答案(示例)李同学的分析有误。这里的敌人指日本侵略者,而不是国民党反动派;诗人这样剪裁主要是给读者留下想象的空间,强化了诗作的表现力。解析根据诗歌下方[注]中的内容可以明确“狗强盗”指的是日本侵略者。全诗假设了一段对话,采用的是虚拟的手法,除表达了强烈的情感,也为读者留下了想象的空间。 阅读提高(川外附中中考模拟)人间四月天钱红莉①小区隔三差五会有卖东西的人进驻。经过物业同意,在南门附近的空地搭一个棚子,白日卖货,夜间关起门来歇息,长则个把月,短则一周。卖稀奇古怪的牛角梳子、弹弓、痒痒挠;卖锅碗瓢盆、矿泉水、煤气灶;洗衣机、冰箱以旧换新……这些跟生活息息相关的日用品,走出家门便可买到,挺受欢迎。②最近,进驻了一对夫妇——为顾客现场加工蚕丝被。进出间,惹人停下脚步,细细打量。他们带来了无数蚕茧,以及一台神奇的机器。把蚕茧倒入机器,合上盖子,一会儿工夫,囫囵的蚕丝抽出来,白练一般亮眼,湿漉漉的……妻子将一匹匹的蚕丝挂上衣架,晾在池塘边的绳子上。春天风大,一匹匹蚕丝徐徐地荡过来荡过去,诗意盎然。 ③昨日买菜回来,经过他们的棚屋。许是时间尚早,生意寥落,只有几匹蚕丝挂在屋内,丈夫不知去了哪儿,妻子则闲闲坐在池塘边的草地上,一棵一棵,耐心地择一堆蒲公英……她背后是一排柳树,春风微微地吹,万千垂碧悠来荡去。坐在草地上的她,像极一幅静物画,脸上流泻着娴雅与安宁。这一幕,让我的心略动一下,有一份感动,隐隐地来。④人在春天里的这份安宁,多值得珍惜啊——这对夫妇自千里之外的外乡来,将孩子丢在老家,就为出门挣点钱,一年的收入就靠这些蚕丝了。生意清淡的时候,也不急,趁空闲在小区挖点蒲公英,焯焯水,凉拌,便是一道菜。挖蒲公英的时候,顺便赏赏景儿。⑤四月了,小区里的辛夷、桃、梨、海棠、紫荆、晚樱一齐在开花。他们的小屋边,有一棵木瓜海棠、一棵梨树,一树红,一树白,开得新鲜洁净,好看得很。她每天低头絮着一床床蚕丝被,再穿针引线地缝进被套里,颈椎酸了,抬头看看花,低头挖挖蒲公英,也是一种放松,顺便在心里想想乡下的孩子…… ⑥一个母亲在春天里想念远方的孩子,阳光也变得柔润了。⑦这对夫妇纵然漂泊着,有四海为家的动荡,但也可看出来,两人感情深笃,配合默契,从不相互埋怨。中国乡下有许多这样子的夫妇,一起出门闯荡,相互扶持。一年年地辛苦着,但心里是有满足的。生来恬淡,便不贪心。人无贪念,便会减少许多痛苦,不会身陷焦躁不安之中。一贯知足,自闲。⑧如今,在人们的脸上,很难捕捉到那一份遥远的闲适之色,尤其在城市,人们总是匆匆来去,上下班高峰,没有哪一条路不被堵得水泄不通,脸上普遍流露出焦虑、烦躁,不能有片刻的安宁——匆匆去接孩子,回家烧好晚饭,急迫地吞下,孩子最后一口饭尚在嘴里,便催促着他去到书桌前……做大人的,连散步的空都挤不出,忙东忙西的,转眼夜深,躺倒于床,每一条骨头缝都疼,日复一日,总是陷入孤独无援的精神困厄……⑨在我们小区这对夫妇的脸上,那份“闲”似一只白鸽,久违地飞回来了,即便每天忙碌着,但还是让你感受到一种恬淡安适——他们的目标,也许就是在乡下屋基上起一座三层小楼,将来娶个好媳妇……养蚕是辛苦的,一夜无数次喂食桑叶, 好不容易把蚕养大,收获了蚕茧,原本可偷懒拿去镇上收购站卖掉的,但为了能卖上好价,就再辛苦一次,勤勉地拉到城里,为人现做蚕丝被。⑩这一阵,进进出出间,看见他们,无比心安。他们相互爱惜着,朴素的感情如微风,吹得旁观者的心上起了涟漪,荡了又荡。春风吹在脸上,有时还有点儿寒意,但,也不碍事的啊,身边有那么多的花开,尤其红叶李,如烟如霞,阳光一直笼着你,有人世的寂静安宁,如此平凡,又如此珍贵。⑪再过几日,他们将所有的蚕丝卖完,便要赶回家乡,重新养蚕,摘不尽的桑叶,喂不够的蚕宝……一年的生计都在这里了。生命就是这样的,一年一年的花开草长,一年一年的平凡日子,急不来,须一日一日地过。⑫人世的安稳,值得珍视。一年年里,春去春回——坐在南窗前,听一听柴可夫斯基的《第一钢协》,久违的安宁重新回来。⑬窗外的香樟发了新叶,斑鸠领着一群儿女在学飞,戴胜鸟在枝头跳跃来去,红叶李无声地开着,风来,浅粉的花落了一地……(选自《光明日报》2019年4月,有改动) 1.作者以“人间四月天”为题好在何处?2.阅读下列语句,按要求回答问题。(1)坐在草地上的她,像极一幅静物画,脸上流泻着娴雅与安宁。(从修辞的角度赏析)①题目“人间四月天”化用林徽因《你是人间四月天》的篇名,富有诗意,能够激发读者的阅读兴趣;②以此为题,点明故事发生的时令,四月万物复苏,春意正浓,生机勃勃,以此表达作者对美好未来的憧憬;③用“人间四月天”为题,表达作者希望人们能从焦躁、匆忙中走出,放松自己,拥抱自然,感受美好,拥有一份“闲”的好心境;④以此为题,以景衬情,以四月的美表现年轻夫妇勤勉、恬淡的美好品格,表达对他们的喜爱和赞美。运用比喻的修辞,将“她”比作一幅静物画,生动形象地表现了“她”在春景的衬托下美丽的姿态和娴雅、平和的心境。 (2)一个母亲在春天里想念远方的孩子,阳光也变得柔润了。(赏析加点词的表达效果)3.第⑧节中,写城市人的“忙碌”,有何作用?“柔润”原意是柔和润泽,这里有怜爱之意。该句借物写人,表现了一位母亲思念远方的孩子时,流露出来的浓浓暖意和温馨甜美。表现城市人生活的快节奏和焦躁、疲倦的精神状态,与文中这对夫妇形成强烈的对比,更衬托出这对夫妻质朴的品质和闲适的生活状态的难能可贵。 4.阅读下面链接材料,思考本文结尾采取的方式与下列材料有何不同?【链接材料】说什么“山不在高,有仙则名”,我却知道“山不在名,有泉则灵”。孕育生机,滋润万木,泉水就是鼎湖山的灵魂。这一夜,只觉泉鸣不绝于耳,不知是梦,是醒?梦也罢。醒也罢。我愿清泉永在,我愿清泉常鸣。(节选自《鼎湖山听泉》)不同点:《人间四月天》结尾采用白描的手法,融情于景,含蓄点明主题,以描写窗外恬静、优美的风景,表达作者希望人们能够从焦躁、匆忙中走出,融入自然,拥有闲适的心境,从而尽享人生美好的愿望。《鼎湖山听泉》结尾直抒胸臆(或直接抒情的手法),直接点明主旨,总括一夜听泉的感受,表达了作者对鼎湖山泉水的喜爱、赞美,抒发了他对祖国大好河山的热爱。 非连续性文本阅读(北大附中中考模拟)如何成为一个“学霸”“学霸”有很多含义,其中一种含义是指那些会学习,学习成绩优秀的学生。虽然学霸在先天方面可能有一些优势,但是,在心理学家眼中,学霸之所以为学霸,正确的努力——良好的学习习惯和方法,是最重要的原因。本文想为各位呈现一些经心理学研究证实普遍有效的学习方法。一、详尽发问法:连接新知识与旧知识研究发现:在学习过程中使学生思考“为什么”的问题能够非常显著地提升学习成绩。很多时候我们认为自己读懂了,实际上印象并不深刻,也没有真正理解。当我们对所学知识进行各种“为什么”的发问时,我们会把新的知识与旧的知识联系起来,用旧的知识来解释新的知识,从而让新知识融入已有的知识体系,这样才是真正的“理解”,因而也会记得更牢。所以,我们应打破教条式接受知识的习惯,对学到的每个知识点都要多问几个“为什么”。 二、心理意象法:用心作画印象深人的想象力是无穷的。心理意象法的关键就是把抽象的东西形象化。在心理学界早就流传一句话:只要能付诸比喻和联想,使其“可见”,就没有记不住理解不了的东西。我们要做的就是把抽象的学习内容放到一个心理图像里,这个心理图像可能是一个具体的物品、一幅画面、一个动态的故事。心理意象法把新的知识与生活中常见的事物和场景联系起来,实现知识整合,提升我们的记忆和理解。三、习题检测法:从内部主动回忆大部分学生都很不喜欢习题测验。但是,习题测验的确能够提升学生对学习内容的掌握和记忆。1906年,美国著名心理学家桑代克说过:“一般而言,从内部进行主动回忆比从外部吸收的印象要深。”所以,我们能给学生的建议是适量地做高质量的习题,而这种习题最好是依据自己的学习情况编写。习题测验的过程必然是存在心理煎熬的,然而要看到它对学习的巩固作用。四、分散学习法:不要“临时抱佛脚”如果你想在考试中发挥出色并能长期掌握知识,那就要用分散学习法了。对于学习来说有计划有间隔地完成学习任务是值得推荐的,因为有些人虽然可以考前突击去记住大部分内容,但是没有前者来得牢固和持久。从长远的效果考虑,应把“少量多次” 作为首要策略。分散学习10小时相比集中学习15或20小时,对长期记忆而言效率更高。而且,研究还发现有意识地进行这种分散学习的效果比偶然出现这样的分散学习要好得多。当然,这种方法的运用还是要考虑所学内容的特点。五、交叉学习法:促进思维快速转换交叉学习,是在一个学习阶段内学习多个技能,进行穿插练习。如果我们利用交叉学习的方法,会得到比长时间学习同一种内容更好的学习效果。由于进行交叉学习,在做下一类习题的时候,前面的知识点和习题种类仍然停留在学生的记忆中,因而学生能够更加清晰地区分不同问题用何种方法来解决。这样,下次遇到这些问题时,学生能够更快更准地把问题归类到不同知识点上。因此,我们不妨把不同类型的学习材料交叉地排在学习计划中,这样能让我们在快速转换思维的过程中对知识系统进行灵活地调整和明确地区分。(选自《百科知识》2014,09A,有删改)【链接材料1】罗马房间记忆法是一种古老的记忆术,当时的政治家为了发表长篇演讲,要记下大量的资料,为方便记忆,便创造了这个方法。其特点就是利用自己的房间作为记忆的“档案柜”,把记忆的对象与熟悉的生活环境连结起来。 1.下列说法不符合选文意思的一项是()A.真正的“理解”需让新知识融入已有的知识体系。B.习题测验是用内部主动回忆的方式来巩固学习。C.有意识地分散学习对长期记忆而言效率更高。D.交叉学习是利用前面的知识来解决后面的问题。2.【链接材料1】中的“罗马房间记忆法”与选文中哪种学习方法类似?请指出其相同点和不同点。D心理意象法。相同点:两者都是将记忆的对象与熟悉的事物建立联系。不同点:心理意象法建立联系的事物范围很广;而罗马房间记忆法建立联系的事物仅限于房间中的事物。 3.阅读【链接材料2】,具体分析学霸何晓羽用到了选文中哪些学习方法。4.老师要求背诵《桃花源记》,小峰大为苦恼,请你根据选文,推荐两种方法帮助他背诵。并结合《桃花源记》的内容作简要阐述。①分散学习法,利用早餐、午餐、午休前后的3个10分钟和2个20分钟的时间来分散学习语文、英语、数学;②交叉学习法,利用早餐后的20分钟,交叉学习了英语和数学;③习题检测法,挑战数学易错题。①详尽发问法。通过用“为什么”提问加深对《桃花源记》的结构、思路、关键性语句的理解,背诵起来会记得更牢。②心理意象法。想象自己是“桃花源”的游人,将游人的行踪变成一幅幅图画(或变成一个游人探险的故事),把内容串联起来。③分散学习法。将课文分成几个部分,这几个部分又可分散时间来背诵。 友情是人最宝贵的财富,无论你走到哪,身处何方,都会有一段温馨的回忆伴随着你。友情不像铁,越炼,越氧化,最后化成一缕轻烟;它像块金子,越炼,越纯,永远闪烁着金色的光芒。友情是一瓶很纯的葡萄酒,越陈,就越醇,也越甜;友情是一缕很淡的花香,越淡,使人越依恋,也越能持久,越沁人心脾。美文欣赏·友情 部编版九年级语文上册教学课件第一单元3.短诗五首 日积月累 课文助读走近作者沈尹默(1883—1971),原名君默,别号鬼谷子。浙江绍兴人,我国杰出的学者、诗人、书法家。戴望舒(1905—1950),字朝安,浙江杭州人。中国现代派象征主义诗人,又称“雨巷诗人”。其作品有《望舒草》《我的记忆》《灾难的岁月》等。卞之琳(1910—2000),江苏海门人。我国著名的诗人、文学评论家、翻译家,主要诗集有《三秋集》《鱼目集》《十年诗草》等。芦荻,原名陈培迪,1912年生,现代诗人。亦有诗歌理论、鉴赏文章和著作行世。聂鲁达,智利当代著名诗人。1923年发表第一部诗集《黄昏》。1971年获得诺贝尔文学奖。主要作品有《二十首情诗和一支绝望的歌》《漫歌》等。 主要内容《月夜》全诗仅四行,却清晰地勾勒出一幅和谐统一的“月夜”场景:霜风,月光,高树和“我”。这首诗表现了诗人(即是五四前后的一代知识分子)独立不倚的坚强性格和追求思想自由与个性解放的奋斗精神。《萧红墓畔口占》是近代诗人戴望舒于1944年所作的一首悼亡诗。这首诗是诗人在萧红墓前凭吊时的口占之作,它包含了诗人对朋友的真挚怀念和对当时社会现实的深沉感慨。 《断章》写于1935年10月,原为诗人一首长诗中的片段,后将其独立成章,因此标题名之为“断章”。这是中国现代文学史上文字简短、意蕴丰富而又朦胧的著名哲理短诗。《风雨吟》写于上个世纪四十年代,当时是一个风雨飘摇的年代。诗人平日熟悉的景象发生了巨大改变,诗人也因此产生了年轻舵手的“忧怀”。他将自然的风雨、社会的风雨以及心中的风雨融合在一起,写下了这首直抒胸怀的战士般的诗。《统一》这首简短的诗歌蕴含着深刻的道理,它告诉我们不要被眼睛蒙蔽内心,不要相信谎言,要看到事物的本质。 基础过关 3.下列句子排序正确的一项是()①诗这种文学样式,偏重于抒怀言志。②诗与其他文体相比,能更为充分地显示作者的品格和情怀。③优秀的诗篇承担着丰富和美化人们精神生活的使命,它向我们展示世界,表达心灵,并启示真理。④情绪和感情是诗的基础。⑤但它一般不直接告诉读者应如何如何,而是以潜移默化的方式净化人们的灵魂。A.①③④②⑤B.③④①⑤②C.②①③⑤④D.①④②③⑤D 4.(重庆一中中考模拟)综合性学习。中华文化源远流长,传统美德薪火相传。“信”文化意蕴丰厚,深入人心。请你参与“走进传统文化,传承‘信’之美德”主题实践活动。根据要求完成下列任务。(1)许慎在《说文解字》中说:“信,诚也。从人,从言。”由此推测。“信”字的本义是:(2)成语是浓缩的文化,它言简意赅,深刻隽永,今天依然有强大的生命力。根据你的积累,写出一个含“信”字的成语。(3)“信”是中华民族的传统美德之一,也是社会主义核心价值观之一。在现代社会,诚信是公民的第二张身份证,诚信老人江新波的事迹感人至深,阅读下面材料,仿照示例,为他拟一则颁奖辞。(50字左右)语言真实示例:信以为真。 江新波,男,84岁。2009年他的小儿子罹患癌症去世,当初为了给小儿子治病花光了家里所有积蓄,还欠下20多万元外债。老伴长年卧病不起,小儿媳没有工作,追悼会上,江新波抚摸着小儿子的遗像老泪纵横:“儿子,向亲戚们借的救命钱,我们不能赖,你放心,欠下的债,我一定会还清!再苦再累,也不能丢掉诚信!”10年来,他每天早上四点钟起床,靠着种菜、养牛,偿还了20多万元债务。【颁奖辞示例】少小离家,乡音无改。曾经勇冠巾帼,如今再让世人惊叹。你点滴积蓄,汇成大河灌溉一世的乡愁。你毕生节俭,只为一次奢侈。耐得清贫,守得心灵的高贵!(2019年度“感动中国”组委会授予马旭的颁奖辞)示例:披星戴月,劳碌十载,只为偿还巨额债务;失去儿子的痛,化为替子还债的愿。他用老百姓最朴素的方式,诠释了“诚信”的含义。他用自己的实际行动,书写了大写的“诚信”二字。 1.(独家原创试题)下列关于作家作品的说法有误的一项是(  )A.《统一》一诗的作者是匈牙利诗人聂鲁达,他曾获1971年诺贝尔文学奖。主要作品有《二十首情诗和一支绝望的歌》《漫歌》等。B.《萧红墓畔口占》一诗的作者戴望舒被称为“雨巷诗人”。主要作品有诗集《望舒草》《望舒诗稿》《灾难的岁月》等。C.《风雨吟》一诗的作者芦荻,原名陈培迪,诗人。主要作品有诗集《桑野》《驰驱集》等。D.卞之琳是“新月派”代表诗人,与李广田、何其芳合称“汉园三诗人”。主要作品有诗集《鱼目集》《慰劳信集》《雕虫纪历》等。答案A 聂鲁达是智利诗人。能力提升全练 2.阅读下面的诗歌,回答问题。月 夜霜风呼呼的吹着,月光明明的照着。我和一株顶高的树并排立着,却没有靠着。(1)这首小诗描绘了一幅月夜图景,请你用自己的语言将这幅月夜图描绘出来。答:(2)如何理解“我和一株顶高的树并排立着,却没有靠着”这句诗的深刻含意?答: 答案(1)刺骨的寒风呼呼地刮着,明亮的月亮洒下光辉,我与一棵顶天立地的大树并排站立在旷野上接受寒风的洗礼,我却没有靠在大树身上,让它为我遮蔽风寒。(2)一个“却”字,既没有贬低树的高大,又凸显出“我”的傲然自立。身处艰难困境,诗人“并排立着”“没有靠着”大树,形象地表现出诗人独立不倚的坚强性格和奋斗精神。解析(1)抓住诗中主要物象“霜风”“月光”“我”“树”,以及四个物象之间的关系描绘画面。注意语言流畅、优美、有画面感。(2)结合诗的写作背景,扣住“自由”或“独立”答出诗人的品格。言之成理即可。 3.阅读下面的诗歌,回答问题。萧红墓畔口占走六小时寂寞的长途,到你头边放一束红山茶,我等待着,长夜漫漫,你却卧听着海涛闲话。(1)首句强调“走六小时”“长途”表明了什么?答:(2)谈谈你对“到你头边放一束红山茶”一句含意的理解。答:(3)赏析结尾句中“却”字的表达效果。答: 答案(1)强调“走六小时”“长途”既是写实,更隐含着诗人对已故者友情的深厚与诚挚。(2)此句不说“墓畔”而说“头边”,蕴含着对萧红早逝的痛惜。“茶花”一直被赋予高洁、自然、清纯、朴素、秀逸等内涵,“放一束红山茶”,以寄托对她的深情悼念,也象征她山茶花般热烈、红艳的生命,传达了诗人对萧红的赞美与欣赏。(3)一个“却”字表示天人永隔,表达了诗人的伤悼之情,也表明诗人体悟到了逝者自有其看透世事的安详与达观。解析(1)从时间之长与距离之远的角度体会诗人对逝者的情感。(2)理解句子含意要注意抓关键词“头边”“红山茶”,体会诗人这样做的原因及内心情感。(3)赏析用词要结合语境,尤其要结合上句“我等待着”,体会“你却卧听着海涛闲话”传达出的隐隐哀伤与对逝者心境的揣测。 1.下列对《月夜》和《萧红墓畔口占》的分析不恰当的一项是(  )A.《月夜》以动词“照”“靠”押韵,却在每行诗末缀以一个轻声词尾“着”,使语气委婉,节奏舒缓自然,富于散文之美。B.《萧红墓畔口占》中“口占”,说明这是出口吟成。古人作绝句,常说:“口占一绝。”可见戴望舒是把这首诗当作绝句来写的。C.《萧红墓畔口占》第三句中的“长夜漫漫”写出“等待”的漫长、痛苦、难以忍受,也隐喻萧红的生命,正是在这漫漫长夜中被摧残、窒息的。D.《月夜》运用铺陈渲染的手法,使诗中“我”的卓然独立的形象,在“霜风”“明月”的“一吹”“一照”中,越发显得清晰、生动,写意传神,颇有韵致。答案D“运用铺陈渲染的手法”有误,应为“运用白描手法”。 2.对下面这首诗的赏析,不恰当的一项是(  )偶 然徐志摩我是天空里的一片云,偶尔投影在你的波心——你不必讶异,更无须欢喜——在转瞬间消灭了踪影。你我相逢在黑夜的海上,你有你的,我有我的,方向;你记得也好,最好你忘掉, 在这交会时互放的光亮!A.这首诗把“偶然”这样一个极为抽象的时间副词形象化,充满情趣,富有哲理,不但朗朗上口,而且余味无穷,意溢于言外。B.此诗写的是两件比较实在的事情,一是天空里的云偶尔投影在水里,二是“你”“我”(都是象征性的意象)相逢在海上。C.如果我们用“我和你”“相遇”之类的标题,会更富有诗味。D.“云”“波”“你”“我”“黑夜的海”“互放的光亮”等意象及其之间的关系构成,都可以因为读者个人情感阅历的差异及体验强度的深浅而产生不同的理解。答案C 若用“我和你”“相遇”之类的标题,会使诗显得平淡、缺少内涵。 3.对下面这首诗的解说,不恰当的一项是(  )天真的预示[英]布莱克一颗沙里看出一个世界,一朵野花里有一座天堂,把无限放在你的手掌上,永恒在一刹那里收藏。A.第一句是说以小见大,“窥一斑而知全豹”。B.第二句是说一个个体生命里藏着偌大的自由与自信。C.“无限”与“永恒”可由自己创造。D.“天真”预示着美好,希望与永恒靠大家努力才能实现。 答案D 诗中说“把无限放在你的手掌上,永恒在一刹那里收藏”,所以“靠大家努力”在诗中未提及。诗歌想告诉我们的是:希望与永恒要靠自己努力才能实现。 阅读提高(南开中学中考模拟)被圈住的母亲乔叶“我这一辈子都叫大山圈着……”无数次,母亲略带伤感地说。在她的意念里,大山是个圈。小时候,姥姥是圆心,她是半径。她十二三岁就一个人赶着花面毛驴,走几十里山路给她的姥姥送煤。她穿着自己织的布、自己缝制的印染着白蝴蝶的衣服,留着月牙头,走在弯弯曲曲的山路上,走过一沟一梁,走过美丽的桃树坡,一切的一切都成为她半径上的风景。成家后,家和孩子是圆心,她是半径。大哥二哥嗷嗷待哺时,正赶上没粮吃的年代。一丛丛的灌木像汹涌的波浪,她就在这波浪里挖野菜。猛然间,她发现一棵山葡萄树,上面一串串的小葡萄让她牢记在心,掐着指头总算盼到葡萄成熟的日子。她往山上赶,采到葡萄的欢喜让她忘了空空的肚腹和颤抖的膝盖。进门后迫不及待先送到孩子们的手上,笑着看吃得欢天喜地的孩子,然后钻进灶棚里开始煮野菜。 父亲在外面教书,母亲一个人扛着家里所有的体力活。上山拾柴让母亲更加忙碌。山上的橡壳、香栗等野果,母亲都用来填过全家人的肚子。每年夏秋,打山桃、山杏,更是母亲不可少的大事。她总是第一个上山,拾掇得筐满袋满。然后,她就开始没日没夜地蜕核,核蜕完了再敲仁儿。一灯如豆,锤子板凳叮当响,常忘了夜已三更。仁儿够上一回石碾了就到碾上推,推完了再上火熬油。记忆中,母亲炒菜哪里肯用勺子舀油,就是一截筷子上扎一小块白布,从小油罐里蘸一下,在锅底上蹭几下就开始炒菜。一日三餐就是凭着这些油花,点缀了一家人贫穷的日子。老家缺水,收雪、打冰块是母亲每年冬天最重要的事,一大早她就拿上笤帚、簸箕、箩筐出去收雪,收一筐雪不知要摔倒多少次。四里地以外有个地方叫狼窝沟,那里有一眼浅浅的小井,山崖上常年有山涧水流下,冬天这里会结很多冰。母亲要么挑上半担水,要么打上一担冰,回去放在大水缸里。一滴水里有母亲的百滴汗,看着炕上欢雀乱跳的孩子,她不觉得累。她想着,孩子们很快就长大了。 我们逐渐长大,母亲的圆圈在逐渐缩小。在煤矿打工的二哥是母亲最扯心挂肠的人。如遇雨天,她就提着一箩筐一箩筐的煤灰和柴灰给二哥垫路,一直垫到村口和大路接上,说二哥回来不好走。雪天,她就会站在雪地里,下一层雪,扫掉一层,直到二哥在大路上出现,这才放心地回家暖一下冻得麻木的手。母亲的圆圈在一点点缩小。有一次她到附近的地里刨土豆,不足一腿高的地堰她手脚并用才爬了上去,但刨了半袋的土豆却怎么也拿不动,她坐在地上伤心地痛哭起来……母亲把力气已经都交给了大山和土地,她还想给,可是她没有了。母亲的圆圈在一点点缩小。她开始经常摔倒。家人都劝她拄上拐棍,她就是不拄,最终还是拗不过自己的腿,拄上了。即便这样,母亲只要觉得自己有点力气,就会做各种吃的,等着孩子们回到老屋享用。她还要站在公路边的榆树下,看着班车停下,载走她的儿孙。 母亲的圆圈在一点点缩小。她连公路边的榆树下也去不了,只能走到院子下边的柴堆旁,坐下来,把拐棍放在脚边,眼巴巴地往公路上看。再后来,她只能坐在炕边的椅子上,看着她的儿孙走出家门。这时候,母亲成了圆心,儿孙、重孙们都是她的半径。可常年守在老屋的母亲无法看到每一个半径上的景致。一年中最热闹的日子就是她的生日、中秋节、祭祖日和过年。但这样的日子如一阵风,来得快,去得也快,热闹之后更冷清……母亲坐上轮椅之后,三哥时常推着她在老屋外晒太阳。她无力的目光时不时投向灰蒙蒙的远山,像是和身边的人说,又像在自语:“又梦见不知道在山上做甚哩,树叶绿汪汪哩,我跑得可快哩……”母亲半闭着眼睛,声音拉得悠远,那是对梦境的无限留恋。(选自《三晋都市报》,有删改) 1.通读全文,概括文章写了母亲的哪几件事?2.文中的“圆心”不断发生变化,这说明了什么?①为她的姥姥送煤;②在家为孩子做饭;③上山采野果;④冬天收雪、打冰块;⑤雨天为二哥垫路;⑥上了年纪的母亲仍然想为子孙做点事。圆心是亲情的纽带。它说明无论母亲是圆心,还是孩子们是圆心,亲情在岁月的更迭中始终不变,温暖着全家人的心。 3.赏析下面的句子。(1)她穿着自己织的布、自己缝制的印染着白蝴蝶的衣服,留着月牙头,走在弯弯曲曲的山路上,走过一沟一梁,走过美丽的桃树坡,一切的一切都成为她半径上的风景。(2)进门后迫不及待先送到孩子们的手上,笑着看吃得欢天喜地的孩子,然后钻进灶棚里开始煮野菜。外貌或细节描写,写出了母亲心灵手巧,勤劳能干,为下文母亲操劳了一辈子做铺垫。运用精准的动作描写,“迫不及待”“送”“笑”“钻”,形象生动地写出了母亲对孩子的爱和她忙碌的身影。 4.读完全文,你看到了一位怎样的母亲?请概括她的人物形象。5.文末最后一句话有怎样的含义?她是大山里的一位普通的农家妇女。她勤劳、朴实、善良、坚韧,承担起生活的重担,在儿女的成长中慢慢老去,无怨无悔。表达了母亲对往事的怀念与依恋,真希望自己不是圆心,而是半径。 我开始了一次很远很久的旅行,一路上经历了千难万险。我穿越寸草不生的大沙漠,友情为我降下了一场甘霖;我穿越水流湍急的大河,友情为我支起了一叶扁舟;我穿越断壁残垣的大峡谷,友情为我建起了一座大桥;我穿越阴雨连绵的热带雨林,友情为我撑起了一把小伞……这一路,友情伴我同行,为我护航,才使我的征途如此精彩。美文欣赏·友情 谢谢观看 部编版九年级语文上册教学课件第一单元4.海燕 日积月累 课文助读走近作者高尔基(1868—1936),前苏联无产阶级作家,社会主义现实主义文学的奠基人。主要作品有自传体长篇小说三部曲《童年》《在人间》《我的大学》。 主要内容《海燕》是一篇著名的散文诗,也是一篇战斗宣言,表达了革命者自信豪迈的战斗情怀和高昂的革命乐观主义精神。这也是革命者号召人民行动起来,去迎接一场伟大的“暴风雨”般的革命。 基础过关 2.破折号的用法有以下几种:A.解释说明;B.插说;C.话题转换;D.声音的延长。请选出下列句中破折号的用法。(1)海鸭也在呻吟着,——它们这些海鸭啊,享受不了生活的战斗的欢乐:轰隆隆的雷声就把它们吓坏了。()(2)看吧,它飞舞着,像个精灵,——高傲的、黑色的暴风雨的精灵,——它在大笑,它又在号叫……()AB 4.(重庆八中中考模拟)下列句子中没有语病的一项是()A.五四青年节前夕,身残志坚的蒋萌,被共青团中央授予2019年“全国向上向善好青年”。B.为了提升学生的身体素质,多年来我校坚持开展“阳光体育”活动,广泛得到社会的关注和家长的支持。C.世界各国都非常重视培养青少年的实践能力和知识水平,希望有更多的年轻人爱科学、学科学。D.一个国家的文化底蕴是否深厚,不仅在于它曾经拥有多少优秀文化,更在于它今天还能拥有多少优秀文化。D 5.(西大附中中考模拟)下列句子排序正确的一项是()绿茶茶艺表演的第九道程序是“春波展旗枪”。似乎是有生命的绿精灵在跳舞,十分生动有趣。①尖尖的叶芽如枪,展开的叶片如旗②杯中的热水如春波荡漾,在热水的浸泡下,茶芽慢慢地伸展开来③这道程序是绿茶茶艺的特色程序④千姿百态的茶芽在玻璃杯中随波晃动⑤直直的茶芽称之为“针”,弯曲的茶芽称之为“眉”,蜷曲的茶芽称之为“螺”A.①②④③⑤B.①④③⑤②C.③②①⑤④D.③②④①⑤C 6.(重庆一中中考模拟)综合性学习。二十四节气中,既是节气又是节日的,就只有清明。《历书》有云:“万物皆齐洁而清明,盖时当气清景明,万物皆显。”清明这个节气,因此得名。重庆一中初三年级某班举行“话清明,思传统”的清明主题活动,请按要求回答以下题目。(1)【诗话清明】艾钟依同学收集了关于清明节的背景资料。请认真阅读链接材料,完成题目。【链接材料】中华民族传统的清明节大约始于周代,距今已有两千五百多年的历史。清明节在仲春、暮春之交。扫墓祭祖和踏青郊游是清明的两大主题。这天,人们除了扫墓祭祀先祖,还有许多“踏青访春”活动,如在郊外放风筝、荡秋千、蹴鞠,还有插柳植树,祈求长寿等。寒食在清明之前。根据以上链接材料,下列诗句不是描写清明节活动的是()A A.杨柳枝,芳菲节,所恨年年赠离别B.马穿杨柳嘶,人倚秋千笑,探莺花总教春醉倒C.风雨梨花寒食过,几家坟上子孙来?D.梨花风起正清明,游子寻春半出城。(2)【寄语清明】请仿照例句,自己创作一句清明寄语,表现清明节的特点。示例:端午节:龙舟竞发千帆舞,汨罗江畔吊屈原。中秋节:月到中秋圆,亲人心相连。清明节:(3)【探访清明】艾钟依打算在学校进行采访,以了解大家对清明节的熟悉情况,并对传统节日的继承出谋划策。请帮助他完成以下采访问题设计。采访对象:同学采访问题:采访对象:校长采访问题:春风抚柳晃秋千,子孙扫墓忆旧人。我们在清明节应该做些什么?我们该如何将清明这种传统节日传承下去? 1.下列句中加点词语含贬义的一项是(  )A.它们这些海鸭啊,享受不了生活的战斗的欢乐。B.雷声轰响。波浪在愤怒的飞沫中呼叫,跟狂风争鸣。C.在乌云和大海之间,海燕像黑色的闪电,在高傲地飞翔。D.看吧,狂风紧紧抱起一层层巨浪,恶狠狠地把它们甩到悬崖上,把这些大块的翡翠摔成尘雾和碎末。答案D A.“享受”指物质上或精神上得到满足,不含贬义。B.“愤怒”是中性词。C.“高傲”在这里不是通常所说的“自以为了不起”的意思,而是“意气风发”的意思,表现了海燕藐视恶劣环境的英雄气概,属于褒义词。D.“恶狠狠”用来形容沙皇反动统治的猖獗与残暴。含贬义。·········能力提升全练 2.下列句中修辞手法判定有误的一项是(  )A.暴风雨!暴风雨就要来啦!(反复)B.海燕像黑色的闪电。(比喻)C.它笑那些乌云,它因为欢乐而号叫!(夸张)D.大海抓住闪电的箭光,把它们熄灭在自己的深渊里。(拟人)答案C 该句运用的是拟人的修辞手法。 3.下列句中破折号的运用判断不正确的一项是(  )A.海鸥在暴风雨来临之前呻吟着,——呻吟着,它们在大海上飞窜……(表示声音延长)B.它飞舞着,像个精灵,——高傲的、黑色的暴风雨的精灵……(表示转折)C.它叫喊着,——就在这鸟儿勇敢的叫喊声里,乌云听出了欢乐。(表示解释说明)D.海鸭也在呻吟着,——它们这些海鸭啊,享受不了生活的战斗的欢乐:轰隆隆的雷声就把它们吓坏了。(表示解释说明)答案B 该句中的破折号表示解释说明。··· 1.文中把海燕比喻为“黑色的闪电”,意在突出海燕的(  )A.勇敢、高傲   B.敏捷、矫健C.敏感、聪慧  D.热情、乐观答案A“黑色的闪电”比喻准确,勾勒出海燕矫健高傲的雄姿。运用比喻的写法,写活了海燕行动时的自信与勇敢。 2.“它从雷声的震怒里,早就听出了困乏”这句话说明海燕所象征的无产阶级革命者具有(  )A.英勇无畏的战斗精神。B.乐观的态度和战斗的激情。C.夺取胜利的百倍信心。D.高度的预见性和敏锐的洞察力。答案D 运用拟人的修辞手法,说明了无产阶级高度的预见性和敏锐的洞察力。 3.下面的话是围绕高尔基如何走上文学创作道路而展开的,但两段话在语意上不连贯,请你在横线上写出必要的过渡性语句,将两段连接起来。高尔基在童年时期就尝到了生活的艰难。他捡过废纸,当过学徒,做过扫院人、面包师、守夜人,换过许多职业。在这种最困难的生活环境里,高尔基的性格受到了锻炼,他的经验日渐丰富,同人民的联系日渐紧密。高尔基的外祖母聪明、慈祥,而且精通俄罗斯语言,她知道许多童话歌谣,常常生动地把这些童话和歌谣讲给孩子们听。外祖母的童话和歌谣,引起了高尔基要做一个艺术家的愿望,为他以后的文学创作埋下了种子。 答案(示例)如果说艰难的生活为高尔基后来的创作打下了坚实的生活基础,那么,高尔基的外祖母就是他走上文学道路的引路人。解析这个过渡句在结构上要起到承上启下的作用。在内容上,前半句是对第一段话的概括,即高尔基童年的生活经历对他的影响,后半句引出第二段话的内容,即外祖母对高尔基的影响。 非连续性文本阅读(西大附中中考模拟)可燃冰是天使还是魔鬼①21年前的夏天,德国科学家在北太平洋海底800米深处,第一次取出可燃冰样品,并使人类第一次看到,冰雪般的东西被点燃后,发出魔幻般淡红色的火焰,耗尽能量后,硕大的冰块竟变成了一摊清水……可燃冰的由来②可燃冰的学名叫甲烷水合物,它的形状像石蜡遇热未融化前的样子,洁白而绵软。它是甲烷气体和水分子在高压低温作用下的特殊产物,它像地毯一样覆盖了10%的海底,总面积达4000万平方公里,深度在数百米到数千米不等。据科学家估计,全球可燃冰总储量高达2.5万万亿立方米,是石油、天然气和煤总储量的两倍,假如能够安全开采,至少可供人类使用数百年。③按美国科学家的说法,可燃冰是海洋微生物和海底油气在海洋板块作用下的产物。因为占地球2/3的生命是由生活在海床下微生物构成的,那里没有氧气,一片黑暗,亿万年来,不计其 数的微生物从不停歇地制造着甲烷。另外,当海洋板块下沉时,海底油气随板块边缘喷涌而出,在接触到冰冷的海水后,天然气与海水发生化学作用,日积月累,在高压低温的作用下,就形成了浩瀚厚实的可燃冰,像地毯一样沉睡在海底。④可燃冰储量最多的国家是俄罗斯,其次是日本、加拿大和美国。据美国地质勘探局估计,美国的可燃冰蕴藏量为9600万亿立方米,是国内传统天然气储量的200倍,丰厚的利润自然引起投资者的关注。可燃冰延迟开发的原因⑤时值NASA(美国国家航空航天局)进行“阿波罗计划”时,NASA的燃料专家正在苦寻可以替代液氧的质量更轻、热值更大的燃料。因为体型庞大的“土星”火箭进入太空后,最多时每秒需消耗13吨燃料,极大地压缩了宇宙飞船的有效载荷。得知可燃冰的存在后,美国科学家兴奋不已,立刻确定为国家级科研项目,全力以赴地投入分析研究。⑥对可燃冰的研究,美国科学家一直处于世界领先水平,但他们迟迟没有进行后续开采动作。有人猜测,美国人重点进行理论研究,因为他们希望在未来的星际旅行时,开发其他星 球上甲烷水合物,用作飞船能源,在即将到来的星际大开发中确保难以撼动的领先地位。其实,情况远非那么简单。就在各国科学家纷纷建议政府投资开发可燃冰,以解煤炭、石油等能源告罄之虞时,美国科学家却告诫世人审慎开发可燃冰,因为它可能加剧温室效应,即使是极少的可燃冰被释放到大气中,后果可能都是灾难性的。可燃冰是天使还是魔鬼?⑦目前,科学界对可燃冰开发有两种观点,以美国俄勒冈大学的地质学家格雷沃里为首的一派认为,甲烷水合物是自然界中隐身的恶魔,会不定期地向地球发难,应是人类今后重点防范的自然灾害之一。⑧远在2.5亿年前,在二叠纪末期,一种突然降临的灾害,短时间内毁灭了地球上的大部分生物。科学界普遍认为是一颗小行星撞击地球所致,但格雷沃里认为是海底积存的巨量的甲烷发生爆炸,导致气候温度骤然上升,泛滥至地面的大火消耗了大气中的氧,所以多数地球生物相继毙命,甚至包括很多鱼类。他估计,北冰洋可能是今后甲烷水合物泛滥的导火索。因为那里可燃冰大多在浅海,并且得益于低温而非高压,所以一旦发生溢出爆炸,就是上帝降临也束手无策。 ⑨另一种观点是,可燃冰就像陆地的森林,生生息息,自有规律,即使作用于自然,也不是很严重的,完全不必杞人忧天。这种观点的代表人物是加利福尼亚大学圣芭芭拉分校的海洋地质学教授海格尔。⑩海格尔说可燃冰的变迁会给地球带来灾难性后果还没有直接的证据,即使有甲烷溢出,也是非常少的,根本不足以影响气候。他做过观察试验,发现有相当多的可燃冰会自行溶解在海水里,而没有释放出来。这充分说明,可燃冰释放的甲烷也是海洋食物链的一环——海水里的需氧菌会消耗甲烷,而进入大气的部分却是甲烷消费者释放的二氧化碳。(船舷《大自然探索》,有删改) 3.下列说法与原文的内容表述一致的一项是()A.美国之所以没有开发可燃冰是因为可燃冰会加剧温室效应,如果可燃冰释放到大气中,即使极少,后果也可能是灾难性的。B.可燃冰是海洋生物释放的甲烷气体和水分子在高温低压作用下的特殊产物。C.用可燃冰作燃料比液氧热值更大,质量更轻,可减少火箭运载负荷。D.甲烷水合物是自然界中隐身的恶魔,不定期地向地球发难,是人类今后重点防范的自然灾害之一。C 4.怎样理解“可燃冰是天使还是魔鬼”这句话?可燃冰对人类益处大还是害处大尚不确定。说它是“天使”是因为它可作燃料,热值大,质量轻,储量大,估计可供人类使用数百年;说它是“魔鬼”是开发它可能会加剧温室效应,也可能溢出引起爆炸,毁灭地球。 打开记忆的匣子,那温暖的画面依旧停留在那段美好的时光里。我们牵手上学,我们高唱歌曲,我们笑颜如花……真的很感谢,感谢在生命最青春的那段时光里,让我遇见了你,遇见你的美好,遇见你的包容,遇见我们最纯粹的友谊!美文欣赏·友情 部编版九年级语文上册教学课件第二单元5.孔乙己 日积月累 课文助读走近作者鲁迅(1881—1936),原名周树人,字豫才,浙江绍兴人。中国现代伟大的文学家、思想家、革命家和新文化运动的奠基人。1918年5月,首次以“鲁迅”为笔名,发表了中国现代文学史上第一篇白话文小说《狂人日记》,奠定了新文学运动的基石。 主要内容课文通过对孔乙己后半生几个悲惨生活片断的描述,成功地塑造了封建末期备受科举制度摧残的下层知识分子的形象,控诉了封建制度的罪恶,揭示了国民当时昏沉、麻木的状态。 基础过关 2.下列关于课文的说法不正确的一项是()A.《孔乙己》写于五四运动前夕,是鲁迅先生的第一篇白话小说,用孔乙己的悲剧讨伐了封建制度和封建文化。B.小说以酒店小伙计“我”的见闻为线索,撷取孔乙己人生中的几个典型片段构成情节,布局精巧。C.小说中用阵阵笑声与孔乙己的不幸遭遇形成对比,揭露了社会的病态,批判了群众的麻木。D.本文中既有社会环境描写,也有自然环境描写。A 4.(育才中学中考模拟)将下面句子组成语意连贯的一段话,填写在横线处,排列正确的一项是()了解自然,便是非走路不可。。这又何苦呢?并且只有自己发现的美景对着我们才会有贴心的亲切感觉,才会感动整个心灵,而这些好景却大抵是得之偶然的,绝不能强求。①并且天下的风光是活的,并不拘泥于一谷一溪,一洞一岩。②旅行的人们也只得依样葫芦一番,做了万古不移的传统的奴隶。③但是我觉得有意的旅行倒不如通常的走路那样能与自然更见亲密。④旅行的人们所看的却多半是这些名闻四海的死景、人人莫名其妙地照例赞美的胜地。⑤旅行的人们心中只惦着他的目的地,精神是紧张的。实在不宜于裕然地接受自然的美景。A.⑤①④③②B.③①④⑤②C.③⑤①④②D.③④⑤②①C 1.下列句子运用的修辞手法不同于其他三项的一项是(  )A.多乎哉?不多也。B.要是不偷,怎么会打断腿?C.你怎的连半个秀才也捞不到呢?D.我想,讨饭一样的人,也配考我么?答案A A项是设问,其他三项是反问。能力提升全练 2.选词填空,并赏析所选词语的表达效果。(1)他不回答,对柜里说:“温两碗酒,要一碟茴香豆。”便(掏 排)出九文大钱。选,赏析:(2)他从破衣袋里(摸 拿)出四文大钱,放在我手里。选,赏析: 答案(1)排 勾画出孔乙己拿钱买酒时的得意神气,也表达出他对短衣帮的蔑视。(2)摸 表明孔乙己的钱已经极少,生活已到了穷困不堪的地步。解析要结合上下文,尤其是孔乙己当时的处境进行选词、赏析。句(1)中,孔乙己当时境遇相对好一点,面对短衣帮的嘲笑,他用“排”这个动作表示反抗,也暗含着对短衣帮的蔑视。句(2)中,孔乙己当时已被打折了腿,生活更加窘迫,用“摸”可以形象地表现出他的钱少。 3.(独家原创试题)根据下列情境,按要求答题。2019年6月8日下午,高考最后一场考试即将结束。安徽合肥十中考点大门外,已聚集了多名考生家长。下午5时许,英语考试结束,考生陆续走出考场。一位身着黄色T恤的高个小伙从大门走出,脸露微笑,步伐轻盈,他径直走向人群。母亲冲着他说了句:“结束了!”小伙回:“妈,我有话和你讲。”这位母亲还没来得及反应,儿子突然跪了下去,高声说:“妈,谢谢您!这些年您辛苦了!”母亲见状,双手抱住儿子的头,哭了出来。假如你目睹了这一幕,你会对小伙说些什么?答: 答案(示例)同学,你的孝心、你的做法值得点赞!我应该向你学习,勇敢表达内心的情感,让多年辛苦付出的父母知道自己的孩子懂得了感恩!解析本题考查口语交际能力。注意进入情境,抓住小伙勇于表达内心情感、有孝心、懂感恩的特点,表达对小伙的敬佩与赞赏。 1.表明本文开头两段的作用的一项是(  )A.交代“我”的职务及身份,表明“我”在故事中的重要作用。B.揭露酒店内幕,披露酒店掌柜弄虚作假的事实。C.勾画“穿长衫的”的趾高气扬,批评“短衣帮”的斤斤计较。D.写明鲁镇酒店顾客的等级,点明人物生活的社会环境。答案D 开头两段交代了“我”的职务和身份,但不是为了表明“我”在故事中的重要作用,而是为了交代社会背景,同时为下文展开情节做铺垫。酒店掌柜弄虚作假的事实与本文主题关系不大,作者交代掌柜在酒里羼水也是为了点明社会背景。介绍“穿长衫的”和“短衣帮”的目的是表现顾客的等级,为突出表现孔乙己身份的特殊做铺垫。 2.下面两个句子哪个表达效果更好?为什么?原句:孔乙己立刻显出颓唐不安模样,脸上笼上了一层灰色。改句:孔乙己显出颓唐不安模样,脸上笼上了一层灰色。答:答案原句表达效果更好,因为“立刻”一词表现了孔乙己痛苦的神情出现之快,也表现了“短衣帮”对孔乙己的心灵伤害之重。解析此题考查对语言的比较赏析能力。首先比较原句与改句的差别:原句比改句多了“立刻”一词。然后分析“立刻”的表达效果:“立刻”表示听到“你怎的连半个秀才也捞不到呢?”的话后,孔乙己的表情很快发生变化,间接反映出孔乙己当时内心的痛苦,可见“短衣帮”揭孔乙己的伤疤、伤害孔乙己心灵的话语是多么残酷无情。 3.请仿照下面句子,在横线处再写一个句子,使之构成排比。“横眉冷对千夫指,俯首甘为孺子牛”,这是鲁迅先生人生的写照,也是他作为一个伟大作家全部人格的体现;“鞠躬尽瘁,死而后已”,这是诸葛亮许身社稷的誓言,也是他作为两朝辅臣忠心报国的见证; 答案(示例1)“问君能有几多愁?恰似一江春水向东流”,这是李煜对强烈的亡国之痛的描述,也是他作为一个薄命君王悲苦境遇的反映。(示例2)“王师北定中原日,家祭无忘告乃翁”,这是陆游临终的嘱咐,也是他作为一位爱国诗人终生追求的梦想。解析此题考查仿写及对名言名句积累、运用的能力。首先分析例句,确定仿写句式为:“‘……’,这是××……的……,也是他作为……的……。”然后选择自己熟悉的名言名句进行仿写即可。注意语意连贯,对名句的分析正确。 阅读提高(北大附中中考模拟)墙万吉星小区有些年头了,密密麻麻的防盗笼,杂乱无章的网线、电话线蜘蛛网似的爬满了每个单元门楼前的空墙上,除了偶尔有几张水电费催缴通知外,大多贴满了代疏通下水道的牛皮癣广告。小王两口子带着刚满月的孩子搬进来时,总感觉很陌生。月子里,父母从乡下老家带了几百个土鸡蛋给儿媳补身体,直吃得小王媳妇看见鸡蛋就想吐,看着几大箱土鸡蛋还大多都没动,小王心里也着急,天气太热,时间一长就放坏了。媳妇说:“要不楼上楼下左邻右舍每家送点吧,远亲还不如近邻呢,我们刚搬来以后少不了要麻烦人家的。”小王觉得这主意不错,便提了一篮鸡蛋去敲对面邻居家的门,厚厚的冷冰冰的防盗门阴沉着脸,“笃—笃—笃”,连应门声也是阴沉沉的,不像在乡下,用系头把门擂得“咣—咣—咣”山响。敲了几下后,小王把耳朵贴在门上,听到从里面传来一阵轻微的 脚步声,他连忙后退了一步,用手捋了捋头发,可门并没有开,他听到门后传来一阵“窸窸窣窣”的响动,小王知道这是主人家正对着猫眼往外观察呢,城里人都兴这套,他便对着门里说:“我是刚搬来对面的邻居,家里有些乡下带来的土鸡蛋,给你们送几个过来。”门内沉默了几秒钟,传来一个女人冷冰冰的声音:“谢谢,不用了,我们家都不喜欢吃鸡蛋。”话音一落,便有脚步声渐渐远去。小王感到有些失落,他抬腿上了楼,敲了敲楼上邻居家的门。门开了,但只虚掩了一道不大的缝隙,从里面探出来一颗男人的脑袋,粗声粗气地问道:“你找谁?”小王忙将手里的篮子往面前送了送,把刚才在楼下的话又重复了一遍,男人一愣,用疑惑的眼光打量了一下小王,又看了看篮子里的鸡蛋,依然冷冰冰地说:“谢谢,不用了,我们家冰箱里还有很多。”说完便“砰”的一声关上了门。小王站在阴冷的楼道里,感觉很冷。没过几天,孩子变得不安生了,一到深夜就哭闹不停,搅得楼上楼下都睡不好觉。小王媳妇说:“这样下去也不是个办 法啊,都吵得左邻右舍无法休息了,要不你再去给大家说一声道个歉,伸手不打笑脸人,与其等人家骂上门来,不如先去赔个笑脸。”小王一想到前次吃的闭门羹,心里就一万个不愿意。最后两口子一合计,想到楼下单元门前不是有一块贴小广告的空墙吗?一商量,决定写张小纸条贴在那儿统一给大家表示一下歉意。清晨天一亮,人们出门时,在单元门前的空墙上看到了这样一张手写的纸条给邻居:“我们刚搬到这个小区,由于孩子太小,晚上吵闹,影响了大家的休息,深表歉意,同时也感谢大家这些天来对我们以及孩子的理解和包容,在此真诚地谢谢大家!”在小纸条的最后,还印上了孩子可爱的小脚印。这张小纸条在那些广告传单中间显得十分的温暖。第二天,纸条旁边多了一张小纸条,上面写着:“谁家都有孩子,我们也是过来人能理解!”纸条的后面,画了一颗小小的爱心。第三天,又多了一张小纸条:“是不是尿不湿让孩子不舒服,我家用的是××牌的,孩子晚上睡得很香,你们可以试试!” 第四天,再次多了一张纸条:“是不是奶水不够,孩子没吃饱?花生炖猪脚能催奶,我女儿刚断奶,家里还有很多花生,我给你们拿一点来吧!”第五天,第六天……最后有一天,在那面贴了无数张爱心小纸条的墙上,又多了这样一张小纸条:“我们在这个小区住了很多年了,但大家相互都不认识,中秋节快到了,要不我们就一起走出家门,在院子里搞一次联欢吧!”关了电视,放下书本,二楼的抱来一箱苹果,三楼的提了一袋花生,四楼的拿出袋核桃,院子里顿时热闹起来,一轮明月慈祥地挂在夜空,皎洁的月光洒在每个人开心的笑脸上,“好长时间没有这么悠闲地赏月了,真美啊!”“哦,原来那张纸条是你写的啊!”“你就住在我对面?搬进来几年了,愣不知道我对面住的是什么人。”爽朗的笑声打破了小区原来的死气沉沉。夜深了,但大家还意犹未尽,临进家门,还不忘回头叮嘱一声:“这个周末每家出两个拿手菜,在院子里搞长街宴哦,不要忘了!”这个小区出名了!这面墙出名了!居委会主任来了,办事处主任来了,区长来了,最后连市长也来了。 1.小说开头写小区的环境描写有什么作用?2.结合全文概括哪些因素让小区人们之间的关系出现了转变。示例:小说开头小区的环境描写表明了小区生活环境的杂乱,以及人们之间的隔阂,为后面情节的展开做了铺垫。示例:①小王夫妇的真诚努力感动了小区居民;②小区居民本性善良;③小区居民内心对人与人之间交流沟通的渴望。 3.“墙”作为小说的标题,有怎样的含义?请结合全文简要分析“墙”这一标题的作用。4.小说前后文形成鲜明对比,贴小广告的空墙变成了贴小纸条的爱心墙,邻里间关系的变化都传达了一个怎样的生活道理?示例:含义:“墙”既是实指,又虚指人们的心灵隔膜以及消除了隔膜充满爱心的心灵。作用:①故事围绕“墙”展开,“墙”是小说的线索,小说围绕“墙”展现了小区居民关系的变化。②推动了情节发展,是故事发生转折的触发点。③反映了作品主题,给人以生活的启迪,隔膜人心的“墙”变成了“爱心墙”,表明人们相互之间真诚的沟通交流是消除心灵隔膜的有效途径。④折射出了小区居民的形象特点:小王夫妇的真诚,小区居民内心善良、渴望沟通。示例:只要真诚,冷人心的“墙”也会变成暖人心的“桥”。人与人之间内心都充满着对人情味的生活方式的向往。真诚需要践行才能迎来温暖的春天。 非连续性文本阅读(南开中学中考模拟)材料一:参考消息网7月8日报道台媒称,大陆正在世界“第三极”(青藏高原)建设第二条进入西藏的“天路”——川藏铁路。它将穿越地球上地质活动最剧烈的青藏高原东南部,成为世界上风险最高的铁路。川藏铁路起于四川成都,经雅安、康定后进入西藏境内,再经昌都、林芝、山南后至西藏自治区首府拉萨,新建线路约1700公里,总投资2500亿元人民币,设计时速为200公里,部分路段限速160公里。建成后,从成都到拉萨坐火车的时间将从48小时减少到约13个小时。中铁二院高级工程师、川藏铁路总体设计副负责人夏烈说,川藏铁路八起八伏,80%以上将以隧道和桥梁的方式建设,累计爬升高度达1.6万多米,相当于征服两座珠穆朗玛峰的高度。这就如同在艰险的高山峡谷当中修建“巨型过山车”,是人类铁路建设史上难度最大的超级工程。 报道称,规划设计中的四川康定至西藏林芝段是全线最长、最难的一段。预计这一段将于2019年动工,建设工期约7年。(《台媒关注大陆建设“史诗级”川藏铁路:史上最难超级工程》,有删改) 材料四:北京地铁网络穿越很多核心地带,对技术、安全和质量控制要求非常高。中铁十四局集团自2002年踏入北京地铁建设的门槛,转眼15年了,他们日夜攻关,高效优质地完成了一条条地铁建设。在北京地铁9号线工程建设中,军事博物馆站就是专家眼中难啃的“硬骨头”工程。建设者先后组织了近50次专家会,最终经过反复论证,解决了相关疑难问题。北京城市轨道交通建设过程中,始终坚持绿色发展道路,采用了再生制动能量回收系统、环保示范站、绿色施工系统等节能环保技术,推进城市轨道交通“绿色”建设。2016年12月底,设计时速最快的北京地铁新机场线动工建设,一举开启了国内轨道交通建设的“新时速”。按照近期规划,北京地铁将在2021年达到999公里。远景线网将由35条线路组成,总规模1524公里。届时,北京地铁网络将成为名副其实的中国“超级工程”。(选自《梦想在“超级工程”建设中放飞——北京地铁建设的成就与辉煌》,有删改) 1.阅读材料一,简要说说川藏铁路“是人类铁路建设史上难度最大的超级工程”的原因。2.川藏铁路、港珠澳大桥和北京地铁网络,都是我国的超级工程。根据以上材料,对“超级工程”解释不正确的一项是()A.指投资巨大,能带来巨大经济效益或功能效益的超大型建筑或建筑物。B.建设工期长,困难巨大,极具挑战性。C.建筑结构复杂,造型精巧,始终坚持绿色发展道路。D.科技含量走在世界前列,具有代表性和象征意义。①穿越地段地质活动最剧烈;②海拔高且起伏落差大;③修建的隧道、桥梁多(长)。C 3.根据以上四则材料,最能准确概括我国改革开放四十年建设成就的一项是()①科技发达②国防强大③人民富裕④经济繁荣A.①③B.②④C.②③D.①④D 寒冷的冬天过去了,温暖的春风吹遍了每一个角落,春天装着满袋的音乐种子来到人间。她把种子撒向林间,林间飞着百灵鸟动听的欢歌;她把种子撒向溪流,溪流荡着青蛙呱呱的鸣唱;她把种子撒向草地,草地飘着蟋蟀轻盈的歌唱……美文欣赏·春天 部编版九年级语文上册教学课件第二单元6.变色龙 日积月累 课文助读走近作者契诃夫是具有世界声誉的短篇小说大师,一生写了九百多篇短篇小说。《万卡》《小公务员之死》《套中人》均是脍炙人口的名篇。他的作品常取材于俄国中下等阶层的“小人物”,通过一些日常的平凡事情,描写小市民、小官吏的自私、虚伪、庸俗之丑态。 主要内容《变色龙》通过对见风使舵、欺下媚上的警官奥楚蔑洛夫这个沙皇专制统治的忠实走狗的刻画,巧妙地揭露了俄国警察制度的反动和虚伪,批判该制度反人民的实质。 基础过关 2.作者塑造奥楚蔑洛夫这个人物典型的社会意义是()A.揭露走狗奴才的丑恶灵魂。B.抨击腐败无能的沙皇统治。C.说明变色龙善于根据环境情况迅速变色以求得自身隐蔽的特征。D.揭露沙皇统治的黑暗,影射沙皇专制的虚伪和专横。3.文章以“变色龙”为题,试分析其妙处。A它与小说主人公奥楚蔑洛夫狡猾善变、见风使舵的个性特点相吻合,不仅生动形象,而且具有讽刺效果。 5.(巴蜀中学中考模拟)下列句子排序正确的一项是()①他虽然写古代,但是笔下主人公并不是行侠、报国、封荫做官的模式,人生理想也不是威逼、子女、玉帛的封建价值观念,而是渗透着个性解放和人格独立的精神,具有浓重的个性色彩。②金庸小说体现的现代精神,也是它超越于传统武侠小说、赢得一代一代新读者的地方。③再有,在我们这个多民族的国家怎样看待历史上的民族关系,能不能挣脱狭隘的民族观念束缚,也是考察作品有没有现代思想、现代精神的一个标志,金庸的民族平等、融合思想,表现得非常明显。④《射雕英雄传》里郭靖报完国仇家恨之后的复杂心情就是证明。⑤比如,对于旧式武侠小说“快意恩仇”的普遍观念,金庸小说从根本上是批评和否定的,他反对眦睚必报,反对滥杀无辜。⑥此外,金庸小说里人生理想、道德观念也是焕然一新的。A.⑥①②⑤④③B.⑥①②⑤③④C.②⑤③⑥①④D.②⑤④③⑥①D 6.(2019重庆A)参照示例,请从下列备选词语中任选一词,写一句话。要求:表意清楚,用上修辞,句式不限。示例:讲台讲台是一道独特的风景线,洁白的粉笔,在一片肥沃的黑土地上,耕耘着智慧和希望。备选词语:窗台操场林荫小道操场——操场是一方广阔的天地,如同一个舞台,供同学们在其中尽情地挥洒青春。 7.(重庆八中中考模拟)综合性学习。学校开展“我与祖国共奋进”主题德育活动,你班参与其中。(1)【树民族自信】为树立同学们的民族自信,老师拟写了上联:迎祖国七十华诞。请你结合教材中“长城”这一专题的学习感受,对出下联。(2)【学身边楷模】你推荐晓峰同学担任“学身边楷模”活动采访组组长,晓峰担心自己不能胜任。你鼓励晓峰说:树民族自信精神。自信是迈向成功的第一步,你的学识和能力,大家都是有目共睹的,都很认可你。你要相信自己,尽力去做,你一定能出色地完成这次活动的。 (3)【抒奋进豪情】采访后,你班准备评选优秀学习心得,班长发现某同学的表述中有两处语病,请你修改。有两处语病的表述:通过这次采访,激发了我勤奋进取的热情和报效祖国的志向。你修改后的句子:这次采访,激发了我勤奋进取的热情,坚定了我报效祖国的志向。 1.(独家原创试题)吕金凌同学为了解《变色龙》一文的作者契诃夫,搜集了一些资料,只是这些资料有语病,请你帮助修改一下。A.90年代是契诃夫创作达到高峰的时期,创作了不少杰作。B.除了契诃夫自己主动研究现实外,由于90年代俄国革命运动的发展和高涨,也给了他不少的影响,使他对现实的认识更加深刻了。他对沙皇专制制度的批判也更加深入。在著名小说《装在套子里的人》中,C.他编造了一个害怕一切新事物、顽固地维护旧制度的典型人物别里科夫的形象。契诃夫相信旧制度一定会灭亡,新的生活必然会到来。在剧本《樱桃园》里,他喊出了“新生活万岁!”能力提升全练 答案(1)创作了 他 (2)由于 (3)编造 塑造(1)A句成分残缺,应在“”前加“”。(2)B句成分赘余,应将“”去掉。(3)C句搭配不当,应将“”改为“”。解析A句缺少主语,应该在“创作了”前加主语“他”;B句成分赘余,应删掉“由于”;C句“编造”与“形象”不搭配,应把“编造”换成“塑造”。 2.请根据提示赏析下面的语句。(1)木柴厂四周很快就聚了一群人,仿佛一下子从地底下钻出来的。(从词语运用的角度赏析)答:(2)就连那手指头也像是一面胜利的旗帜。(从修辞的角度赏析)答: 答案(1)“钻”字写出了围观人群聚集之快,贴切生动。表现了人群以看热闹为乐的无聊心理。说明底层人们精神空虚、寂寞消沉。(2)比喻修辞手法的运用,形象地表现出赫留金把受伤的手指作为要求赔偿的证据的得意之态。解析本题考查语言品析能力。(1)句中动词“钻”用得好。结合上下文、社会背景及句意体会“钻”字体现了人们怎样的动作和心理。(2)根据“手指头”“旗帜”可以判断采用了比喻的修辞手法,表达效果要结合语境体会赫留金的得意之态。 阅读下面的文章,回答问题。名贵的狗契诃夫杜博夫,一个老兵出身、年纪不轻的中尉,和志愿入伍的克纳普斯正坐在一起喝酒。“好一条公狗!”杜博夫指着他的狗米尔卡对克纳普斯说,“名——贵——的狗啊!您注意它的嘴脸!光凭这嘴脸就值大钱了!遇上喜欢狗的人,冲这张脸就肯甩出二百卢布!您不信?这么说您是外行……”“我懂,不过……”“这可是长毛猎狗,英国纯种长毛猎狗!发现野物时它那副姿势别提多漂亮了,还有那鼻子……真灵!天哪,多灵的鼻子!当初米尔卡还是一条小狗崽 子,您知道我花了多少钱买下的?一百卢布!好狗啊!米尔卡,你这机灵鬼!米尔卡,你这小坏包!过来,过来,上这儿来……哎呀呀,我的小宝贝,我的小乖乖……”杜博夫把米尔卡招引过来,还在它的头上亲了一下。他的眼睛里涌出了泪水。“我谁也不给……我的小美人……小淘气。你是爱我的,米尔卡,是不是?……行了,滚一边去!”中尉突然喝道,“脏爪子尽往军服上蹭!说真的,克纳普斯,买这小狗我花了一百五十卢布!可见它很值钱,只可惜我没有时间打猎!这狗简直闲死了,也荒废了它的才能……所以我想把它卖了。您买吧,克纳普斯!您一辈子都会感谢我的!哦,要是您手头紧,我可以半价让给您……出五十就带走!您这是明抢呀!”“不行,亲爱的……我一个戈比也不出。一来我不需要狗,二来我也没有 钱。”“这话您早说不就好了。米尔卡,从这儿滚出去!”“行了,不要就不要……见您的鬼去!既不想买,也不想要……哎,您去哪儿?再坐一会儿嘛!”克纳普斯伸个懒腰,站起来,拿起帽子。“该走了,再见吧……”他打着哈欠说。“那您等一下,我来送送您。”杜博夫和克纳普斯穿上大衣,来到街上,默默地走了一百来步。“您看我把这狗送给谁好呢?”中尉开口说,“您有没有什么熟人?那条狗您已经看到了,是条好狗,纯种狗,可是……对我真是一点用处也没有!”“我不知道,亲爱的……再说我在这地方哪儿有什么熟人?”一直走到克纳普斯的住处,两位朋友再没有说一句话。克纳普斯握过中尉 的手,便打开自家的便门,这时候杜博夫咳了一声,有点迟疑地说:“您可知道本地的那些屠夫收不收狗呢?”“想必会收的……我也说不准。”“明天我就让瓦赫拉梅耶夫送了去……去它的!叫人剥了它的皮……这该死的狗!可恶极了!不但弄脏了所有的房间,昨天还把厨房里的肉全偷吃光了,下贱胚子……是纯种狗倒好了,鬼知道它是什么东西,没准是看家狗和猪的杂种。晚安!”“再见!”克纳普斯说。门关上了,中尉一人留在外面。(有删改) 1.杜博夫对他的狗米尔卡前后的态度发生了怎样的变化?答:2.阅读文中画线的句子,说说为什么杜博夫的眼睛里“涌出了泪水”。答:3.文章倒数第三段可以删掉吗?为什么?答:4.请结合文章内容,简要分析杜博夫这一人物形象。答:5.在写法上,本文和《变色龙》一文有哪些相同之处?(至少答出三点)答: 1.答案一开始极力称赞它是一条名贵的狗,是条好狗、纯种狗;后来骂它是下贱胚子,是该死的狗、杂种狗。解析本题考查对内容的梳理和概括能力。阅读文本,理清故事情节,根据题目要求筛选相关的词句概括作答。本题依据“‘好一条公狗!’杜博夫指着他的狗米尔卡对克纳普斯说,‘名——贵——的狗啊!’”“这可是长毛猎狗,英国纯种长毛猎狗!发现野物时它那副姿势别提多漂亮了,还有那鼻子……真灵!天哪,多灵的鼻子!……好狗啊!……哎呀呀,我的小宝贝,我的小乖乖……”“那条狗您已经看到了,是条好狗,纯种狗”可以看出杜博夫对他的狗米尔卡,一开始极力称赞它是一条名贵的狗,是条好狗、纯种狗。依据“明天我就让瓦赫拉梅耶夫送了去……去它的!叫人剥了它的皮……这该死的狗!可恶极了!不但弄脏了所有的房间,昨天还把厨房里的肉全偷吃光了,下贱胚子……是纯种狗倒好了,鬼知道它是什么东西,没准是 看家狗和猪的杂种”可以看出杜博夫对他的狗米尔卡,后来骂它是下贱胚子,是该死的狗、杂种狗。2.答案杜博夫装出对狗不舍的样子,是为博取克纳普斯的同情,从而把狗卖给他。解析本题考查对句子的理解能力。语句“杜博夫把米尔卡招引过来,还在它的头上亲了一下。他的眼睛里涌出了泪水”,采用动作和神态描写,叙写了杜博夫装出对狗的喜爱,其目的是把狗卖给克纳普斯。据此理解作答。3.答案不可以。因为这段话交代了杜博夫打算卖狗和送狗的原因,也说明了杜博夫开头极力赞扬狗的话,都是为了把狗卖给克纳普斯而编造出来的假话,从而暴露了他丑陋、表里不一的内心。 解析本题考查理解文中重要语段的作用的能力。要从内容和结构两方面进行分析作答。文章倒数第三段,在内容上,交代了杜博夫打算卖狗和送狗的原因。结构上,与前文杜博夫极力赞扬狗的话形成鲜明的对比。从而让读者明白了:杜博夫开头极力赞扬狗的话是为了把狗卖给克纳普斯而编造出来的假话,揭示了他是一个内心丑陋、表里不一的人。从而深化了文章的主题。据此理解作答。4.答案杜博夫为了把狗卖给对方,极力地称赞狗,见对方不愿意买,又提出送给对方,遭拒绝后又发誓要把狗送给屠夫,大骂狗的不是,可见其言不由衷、虚伪、厚颜无耻、自私狠毒、缺乏爱心、表里不一。 解析本题考查分析人物形象的能力。此类题的解答方法:要抓住刻画人物的各种描写方法来分析;从叙写的事件来分析;把人物放到一定的背景环境中去分析;注意分析文中对人物的介绍和评价。文章通过叙写杜博夫为了把狗卖给克纳普斯,就编造了他的狗是一条名贵的狗,是条好狗、纯种狗的假话,见对方不愿意买,又提出送给对方,遭拒绝后又发誓要把狗送给屠夫,大骂狗是下贱胚子,是该死的狗、杂种狗,可见杜博夫虚伪、狠毒、缺乏爱心、表里不一的性格特点。 5.答案①故事都是由“狗”引起的;②都通过语言、动作和神态等描写刻画人物的形象;③都运用了对比的手法;④都属于成功运用讽刺艺术的杰作。(答出任意三点即可)解析本题考查对写作手法的比较分析能力。从写作手法上看,两文都由“狗”引起。两篇文章都运用了对比的写法:《变色龙》中奥楚蔑洛夫对狗的态度不断变化,形成鲜明的对比;本文中杜博夫对狗的态度前后变化形成对比。两篇文章都使用了讽刺的手法,通过写奥楚蔑洛夫和杜博夫前后对狗的态度的变化,刻画出他们丑陋的嘴脸。从描写的角度看,两文都采用了动作、语言和神态描写的方法来刻画人物的丑恶灵魂,揭露了沙皇统治时期国家的腐朽、黑暗。据此概括作答。 阅读提高(南开中学中考模拟)新官上任李景文夏冰有晨练的习惯。健壮的体魄加上他的娃娃脸,让他显得比实际年龄要小许多。一早起来,看着窗外飘着的鹅毛大雪,他坐不住了。夏冰最近被提拔到城建局当一把手。本来,组织部门已跟他约好,明天就送他走马上任。但是,眼前的这场大雪,就如同在他头顶上垒起了一座雪山,随时都会崩塌。真的是悬念迭起呀!道路畅不畅通?自来水管道是否爆裂?种种想到或者没有想到的问题,就像眼前的大雪在他脑子里一片混沌……匆匆吃完早餐,一看表还不到7点,他决定自己的履新就从今天开始。既然是履新,就得有新气象,那这第一天上班就走过去好了,正好留心下一路的市容市貌有什么问题。一跨出楼梯,他就被纷纷扬扬的大雪吞没了,行走变得身不由己。然而,户外清新而凛冽的空气使他更加激昂,银装素裹的城市其实颇有诗意, 他的步伐更稳了。在途中,他看到身着橙色衣帽的环卫工人顶风冒雪在清除厚厚的积雪,心中就有一种别样的感动。走到城建办公大楼,他花了一个多小时。值班室里,一个年轻的保安正在埋头玩手机。他便憋不住地说,注意别伤着眼睛,铲铲雪还带锻炼呢。你是谁呀?有这么说话的?保安斜着眼睛扫了他一眼。我是新来的……站着说话不腰疼,门口有的是锹,那你铲呀!夏冰听出了话中的讥讽。他搓搓手,二话不说就拿了一把铁锹开始铲雪,从大楼的台阶一路铲下去,全身心投入,越干越欢,渐渐地,他的身后便铲出一条道来。连老天也仿佛被他的冲天干劲吓着了,雪在不知不觉中已经停了。当他欣慰地回望时,却看到保安也拿着一把锹快步朝门口走来……尚主任,您来这么早!保安一脸媚态。好,都把雪铲了。尚主任眯缝着眼打量夏冰,这位是……他说要跟您报到。心虚的保安说完就开溜了。哦,是小夏。尚主任以一种居高临下的口吻说,在机关,就是要见眼生情,能上能下。 夏冰点点头,我初来乍到,还望主任多提醒。既然进了一个庙,就不是外人了。尚主任拍拍夏冰的肩说,刚才在路上,我突然接到一个电话,话是说得哆哆嗦嗦,好像是给我们单位送空调的师傅出了车祸,在大转盘那儿吧,你就赶去看看,到底怎么回事?人命关天呀!夏冰心里一惊,忙问,师傅伤着没有?师傅的事,雇主是跑不脱的。尚主任撇了下嘴说,你看看空调摔坏了没有?要不要更换?商家什么时间能安装到位?这不是本末倒置嘛!夏冰的口气有些像争辩。俗话说,各扫自家门前雪,莫管他人瓦上霜。尚主任的脸色很不好看,现在,这个美差就交给你了,空调可是给新局长换的,今天不吃饭也要安装到位!新局长未必要吧!再说,局长室的空调是不是可以修一下,也许加点氟利昂就行了。夏冰想尽量让自己的情绪平复些,但是却越说越激动,老尚是不是这个理儿?一台空调跟人的生命比起来,可以说微不足道……你、你……教训我?尚主任气急败坏地说,简直目无领导!领导心中要有群众!夏冰微笑着说。 群众,满大街都是群众,你问得了吗?真是站着说话腰不疼。尚主任说着头也不回地走了。尚主任最后说的话,听上去很耳熟,但是夏冰没有工夫跟他计较,随即打车赶到出事地点。三个交警正在现场处理事故,肇事的师傅在一旁吓得不轻,原来不是送货的他受了伤,而是因雪天路滑,他开的电瓶车碰倒了一个老人……夏冰当即打120将老人送往医院。一做CT,三处骨折。很快,警方来电话说老人的亲属马上就到,夏冰迎上去,一看却是尚主任。夏局呀,我真是老眼昏花,有眼不识泰山!尚主任双手紧握着夏冰的手直摇,我把您当成……也是今天来报到的小夏了……我哪里有你说的那么年轻!夏冰说,当然,改正了就好,你不是也赶来了?唉,教训太深了!尚主任满脸羞愧地低下了头,被撞的,是我的老父亲…… 1.夏冰具有哪些优秀品质?请结合全文简要分析。2.小说中多次写到雪,有何作用?①责任心强,工作认真负责。表现在夏冰自己一人悄悄上任,只怕大雪会给民众的生活带来不便。②身体力行做出表率。表现在夏冰在保安讥讽后,毫无怨言地一人去铲雪。③一身正气,关心民众。表现在夏冰在听说送空调的师傅半路出了事,首先想到的是师傅的安危。④有一颗宽容的心。表现在尚主任向他道歉,他原谅了尚主任。①交代故事发生的时间;②贯穿全文的线索,推动情节的发展;③以“雪”的寒冷衬托夏冰对待工作与群众的火热的心。 3.小说结尾尚主任感叹“教训太深了”,请简要回答尚主任受到了哪些教训。4.本文善用对比手法,请找出三处并简要分析其作用。①没有弄清对方的身份,就开始大摆官架子,批评了自己的上司;②处理公事时,一心只想着讨好上级,忽视民众的利益,最终受害者反是自己的父亲。示例:①保安只顾玩手机而嘲讽夏冰铲雪是站着说话不腰疼,与他看到尚主任便拿着铁锹准备铲雪形成对比;②尚主任对夏冰“一台空调跟人的生命比起来,可以说微不足道”的观点表示愤怒,与后来发现自己弄错了夏冰的身份而主动道歉形成对比;③尚主任在知晓送空调的师傅出事时,只关心空调能否如期安上,与夏冰一心关心送空调师傅的安危形成对比。表现了尚主任与保安一心巴结上级,工作不务实的作风,突出了夏冰的为人的正直与工作的认真负责,让文章的主旨更为深刻,引人深思。 我眼里的春天又是一个淘气的小姑娘,她一会儿露出灿烂的笑脸,把一缕缕暖暖的阳光撒在大地上,让人觉得温暖舒适。一会儿又轻歌曼舞,吹起阵阵春风抚摸人们的脸庞,令人心旷神怡。偶尔她又会把泪水化为淅淅沥沥的春雨,滋润着大地万物。春雨中夹着的几声春雷,声音不大,却传递了春的信息,唤醒了沉睡的生物。美文欣赏·春天 部编版九年级语文上册教学课件第二单元7.溜索 日积月累 课文助读走近作者阿城,原名钟阿城,1949年出生于北京市,祖籍重庆市江津区,中国当代作家。1984年,发表处女作《棋王》。作品集《阿城文集》。编剧作品有《芙蓉镇》《小城之春》《刺客聂隐娘》等。 作品简介本文选自《遍地风流》。该书出于敬畏历史之心,本乎赏鉴文学之意,撷取20世纪短篇小说百篇精品,以颗颗璀璨的小珠,串成百年文学纪念的项链。 主要内容这篇小说写了一群过河的马帮汉子,一次溜索的经历。小说处处通过“我”的观察和感受来写,但从头到尾没有出现“我”字,好像每位读者都是这个“我”,给人以身临其境之感。 基础过关 3.阅读课文,回答下面问题。(1)文中用不少笔墨写牛,这对环境描写和人物描写各有什么作用?(2)结合本文内容简要分析马帮首领的形象。牛不肯挪动半步的恐惧和溜索时流泪发抖:①侧面表现怒江峡谷的高峻险恶;②与“我”“战战兢兢”互相映衬;又与领队及汉子的勇敢无畏形成反衬。①从容不迫,胸有成竹。“懒懒”地说话、“稳稳”地坐在马上。②认真负责,关爱部下。敲一敲溜索,吼“我”过江。③受人尊敬,与手下配合默契。瞟一眼,问一声。④英雄气概,粗犷豪迈。一声唿哨、最后一个过溜索。 5.(重庆一中中考模拟)下列各句中,没有语病的一项是()A.能否加强亚洲各国之间的文明交流互鉴,是携手共建亚洲命运共同体走向更为广阔前景的关键。B.我们要培育和践行社会主义核心价值观,提倡爱家爱国相统一,弘扬爱国主义、集体主义、社会主义。C.一项研究报告显示,中国学生近视呈现高发、低龄化趋势,学生的近视比例已超过50%以上。D.央视纪录片《大国工匠》讲述了当代中国工匠传奇的人生故事,展示了他们非凡的职业绝技。D 6.(重庆八中中考模拟)下列句子排序正确的一项是()①青少年正处于长身体、适应社会的过程中,所以提高他们的动商,进而带动智商、情商,就会有益于他们的身心更健康地成长。②在踢球时,球员要综合运用观察、注意、思维、想象、记忆等能力作出判断并采取相应动作,这就是对智商的培养与锻炼。③同样,在踢足球过程中,难免踢别人一脚,或者被别人踹一脚,在碰撞中可以学会说“对不起”和宽容别人。④玩转足球需要球员间的默契配合,而这正是情商培养的有效载体。⑤动商就是个人的运动商数,动商的提出重振了校园足球,那就是让体育回归教育,让校园足球发挥教育功能。⑥踢足球能使动商、智商、情商相互促进。A.①⑤③②④⑥B.⑥⑤②④③①C.⑤①③④②⑥D.⑤②④⑥③①B 7.(川外附中中考模拟)阅读下面材料,按要求回答问题。材料一:4月12日,由市委宣传部、市旅游发展委等部门共同主办的河池市2019年“壮族三月三·河池嘉年华”综合性民族文化展演活动在市体育广场举行。其中,铜鼓舞《瑶家有女初长成》、彩调《彩调欢歌》、原生态山歌《野欢》等节目,展示了河池浓郁古朴的民风民俗和独具特色的民族文化,赢得了观众的一致好评。材料二:铜鼓舞是壮族、瑶族最有代表性的道具舞蹈,它融祭祀文化与农耕文化于一体,集民族性、艺术性、文化性于一身,是壮、瑶两民族舞蹈艺术中的典范。(1)请用一句话概括材料一的主要内容(不超过20个字)。综合性民族文化展演活动在市体育广场举行。 (2)为宣传家乡的铜鼓文化,假如你是材料一中的活动组织者,请结合材料二的内容,为本次活动写一句宣传标语。(3)请你仿照下面画线部分的句式,将后面的句子补充完整。“壮乡三月三”,你以独有的风情征服了我。我想,如果我是作家,我一定会用华丽的词句描绘你美丽的容颜;,;,。示例:壮族铜鼓舞,舞动民族风。如果我是画家我一定会用多彩的画笔去描摹你独有的神韵如果我是歌唱家我一定会用美丽的歌喉歌唱你独特的气质 1.(独家原创试题)下列对《溜索》一文的分析理解有误的一项是(  )A.当“我”询问时,“首领也只懒懒说是怒江”中的“懒懒”表现了首领的心不在焉。B.文章多次写到鹰,既衬托出环境的奇险,又烘托了马帮汉子的形象。C.文章用了不少笔墨写牛,从侧面表现了怒江峡谷的高峻、险恶,同时与“我”的战战兢兢互相映衬,反衬出首领等人的勇敢无畏。D.“(瘦小汉子)脚一用力,飞身离岸,嗖的一下小过去”中的动作词“小”字,写出了瘦小汉子在溜索上快速渡过怒江峡谷时给“我”带来的视觉感受,富有表现力。答案A“懒懒”表现了首领的镇定从容、胸有成竹。··能力提升全练 2.请从写景视角的角度入手,赏析下面文字的表达效果。万丈绝壁飞快垂下去,马帮原来就在这壁顶上。转了多半日,总觉山低风冷,却不料一直是在万丈之处盘桓。怒江自西北天际亮亮而来,深远似涓涓细流,隐隐喧声腾上来,着一派森气。俯望那江,蓦地心中一颤,惨叫一声。……答: 答案这段话以壁顶为观察点,从视觉、听觉、内心感受多方面描写,多角度展示了峡谷壁顶的孤悬,突出了峡谷高峻、险恶的特点。同时,作者将远眺、俯瞰与听觉相结合,突出了当时的惊恐心情,使人如临其境。解析本题考查语句赏析能力。注意题干中的赏析角度“写景视角”;然后根据“万丈绝壁飞快垂下去,马帮原来就在这壁顶上”确定观察点;根据“怒江自西北天际亮亮而来,深远似涓涓细流”“俯望那江”可确定是视觉角度;根据“隐隐喧声腾上来”可确定是听觉角度;根据“俯望那江,蓦地心中一颤”体会“我”的内心感受。 阅读下面的选文,回答问题。壶口,壶口莫伸①在中国北方浩瀚的群山中,有无数条蜿蜒伸展的沟谷。由于地势的羁束,这些沟谷都多少显出一种挤手夹脚的尴尬和无奈,而地处陕西宜川和山西吉县一带的秦晋大峡谷却是个例外。②黄河有幸,倚身此中。③此前的黄河尽管强悍凶猛,却不得不委屈着自己,默默无闻地在峻梁雄峁中曲意周旋,它只能选择忍受和服从。但黄河是雄心勃勃的,它从来都不屑隐忍,不甘迁就,更不愿受辱。它像一条蓄爪待扑的巨龙,随时都在等待机会。而现在,秦晋大峡谷终于为它提供了一个释放的舞台,它没有丝毫犹豫 便腾空而起,狂奔劲舞,瞬间便造就出一个自然景观中的伟大和不朽!④这就是壶口瀑布。⑤壶口瀑布诞生在一个地地道道的壶口中——瀑布之前,黄河的河床足有上千米宽,而到了这里,两岸岩石陡然一缩,形成了一条只有二三十米左右的狭槽。不仅如此,槽道下又奇特地出现了一个高低悬殊、落差极陡的深谷。于是,漫天而来的黄河水流在这里聚为一股,齐心协力地朝狭槽里奔涌喷吐,由此形成了壶口瀑布。⑥壶口瀑布的壮美是难以用语言描述的。⑦离壶口瀑布足足还有几公里的距离,你便可以远远地看见峡谷里腾起一团团飘冉的云团,那是瀑布迸溅形成的飞雾。飞雾如霞似烟,弥漫在河床上空,形成了一道令人惊骇且永不消散的奇景。屏息凝听,空气中伴有一种不绝不灭的震吼。好像来自天际,又似乎源于地底;仿佛惊雷滚地,犹如万骏 疾走;声音撼天动地,包容万千。[A]那是一种捶胸哭天的苍凉,又是一种国难共赴的悲壮;是一种无坚不摧的刚烈,又是一种壮士扼腕的豪雄;是一种惊天泣鬼的神勇,又是一种众志成城的顽强。⑧在种种难以描述的感受中,最让人难忘的当数与瀑布贴身相逢又抬头仰望的时候。相信那一刻,任何人都难以摆脱一种灵魂出窍的感觉。那些在远处看去吼喊奔扑的洪流,在近处看去更有着一种如山倾倒的壮烈。那种疯狂和奋勇的喷扑让坚硬如钢的岩层不得不退让,无法不动容。狂放不羁的黄河水流在壶口的限制下,有了核心,得着凝聚,统一和团结使它们的力量排山倒海。瀑布从高处奔涌而下,每一滴水珠、每一朵浪花都不分彼此,都争先恐后,用一种近乎疯狂的急迫向“壶口”流泻和迸射。它们凝成一团,形成一根根粗大的浪柱,又由这浪柱组合成宽阔的扇面,齐刷刷地向下垂落。水浪砸在岩石上,迸出冲天的水柱,也迸成残酷的碎片。这种冲击和 对抗实在是太猛烈,碎片已完全不是我们想象中的小水珠或者小水滴,而是一种完完全全的肢解,直至肢解成用手无法捕抓、用舌无法舔触的水汽。一霎时,雾气漫天升腾,而脚下那些没有来得及牺牲的黄河水流前赴后继地、毫不间歇也毫不犹豫地继续疯狂朝前!⑨我们全看呆了。或许世界上还存在着许许多多的伟大力量,或许生活中还翻演着无数撼人心魄的壮烈冲击,但没有哪一种能像眼前这样,让我们感受到一种空前的震撼。那是愤怒的极致,激昂的顶端,凶猛的无限,力量的空前!那是须要我们永远体味和追求的精神的全部!⑩哦,千古黄河,伟大的河,我们民族的母亲河!我们几乎每天都行走在你的身边,也接受着你的泽润,那些日常的行走和接触使我们对你有了一种熟视无睹的平淡和麻木,但是只有此刻,我们才顿然醒悟:[B]为什么志士仁人都本能地要到你身边来一览姿容!为什么作家和歌者都虔诚地要到你身边来聆听涛吼!为什么你能够被众口一致地尊为我们伟大民族的不朽魂魄! 1.为什么说倚身于秦晋大峡谷,“黄河有幸”?答:2.壶口瀑布的壮美表现在哪几个方面?请分点概括。答: 3.结合语境,说说下面句子中加点的词语好在哪里。(1)那种疯狂和奋勇的喷扑让坚硬如钢的岩层不得不退让,无法不动容。答:(2)水浪砸在岩石上,迸出冲天的水柱,也迸成残酷的碎片。答:··· 4.本文运用了什么表现手法?请简要说明。答:5.请从文中[A][B]两处语句中任选一处,针对其中最鲜明的语言特点加以赏析,并概括该句的深刻内涵。答: 1.答案雄心勃勃的黄河得到了释放的舞台,造就出一个自然景观中的伟大和不朽!解析本题考查对内容的理解能力。先从文中找到“黄河有幸”;然后结合上下文,从第③段中摘取秦晋大峡谷对黄河的益处作为答案即可。2.答案(1)瀑布迸溅形成的飞雾形成了一道令人惊骇且永不消散的奇景;(2)瀑布发出的不绝不灭的震吼撼天动地,包容万千;(3)统一和团结使狂放不羁的黄河水流的力量排山倒海;(4)黄河水流具有前仆后继的献身精神和视死如归的英雄气概。解析本题考查对内容的概括能力。首先认真审题,抓住题干中的关键信息“壮美”。然后细读文章,根据第⑥段“壶口瀑布的壮美是难以用语言描述的”,确定答案应从第⑥段后面找。最后从景、声、势、体现出的精神气概等方面分条归纳壶口瀑布的壮美。尽量从文中摘取语句作答。 3.答案(1)形容词贬义褒用,生动形象地描述并赞美洪流勇往直前、义无反顾、不可阻挡的英雄气概。(2)动词,写出水浪的力量很大。生动形象地表现出水流视死如归、勇于献身的胆识、勇气和力量。解析本题考查词语赏析能力。要结合语境体会使用该词语写出了洪流、水流怎样的特点。4.答案象征(托物言志)。文章借“聚为一股,齐心协力地朝狭槽里奔涌喷吐”“每一滴水珠、每一朵浪花都不分彼此,都争先恐后”“前赴后继地、毫不间歇也毫不犹豫地继续疯狂朝前”等生动可感的壶口黄河水流形象,象征了拥有视死如归、勇往直前的豪情壮志的人民。解析本题考查对表现手法的鉴赏能力。根据文章结尾可确定表现手法为象征。然后结合对全文内容的理解,采用“文章借……象征了……”的基本模式组织语言作答。 5.答案(示例1)[A]处运用了排比的修辞手法,联想丰富,气势磅礴。该句揭示出瀑布撼天动地的声音所展现的勇猛刚烈的气势和悲壮豪雄的精神,表达了作者的崇敬之情。(示例2)[B]处运用了排比的修辞手法,使文章语言雄壮有力,使作者的赞美之情喷薄而出,连用三个“为什么”引人思考,突出了黄河精神作为我们伟大民族精神内核的强大影响力、感召力和凝聚力,揭示了文章的主旨。解析本题考查句子赏析能力。细读两处语句,首先选择自己理解最到位的语句具体分析该句的特点,如采用的修辞手法;然后分析该句写出了事物怎样的特点,表达了作者怎样的情感,该句在全文中有何作用,等等。 阅读提高(北大附中中考模拟)月亮村的月亮吴小军①月亮三年没回他那粤北深山里的家了,他已经在邻近深圳的一个小县城安了家,实现了进城的梦想。前几天突然接到村民小组长从家乡的县城打来的电话,说他爸有些不得劲了。月亮决定,趁着中秋请假回一趟,把母亲去世后一直不肯离开村子的老父亲接过去住。要不,还真拿不准啥时才回家。②一跨上山坳,他就真切地想起了老人讲的月亮村的名儿是怎么来的了。真是好大个月亮!就这样银光光、亮爽爽、突兀兀地从两座大山坳处撞入他的眼帘,撞入他的心房。③“我回来了!”坐了几个钟头的火车,坐了一个多钟头的汽车,坐了近一个钟头的小四轮,又走了一个钟头的山路的月亮,忘了所有的疲惫,心里豁然亮堂了起来。④转过这个坳,是一起做泥水工的广财的家,墙头有的塌了,瓦破了,屋顶上露着的椽子黑了。房门上还挂着几片纸,残的,月光虽然很亮,却看不出红白了。广财的孩子在城里上民办学校,前些年他爸也没了,他可能有四五年没回村了。 ⑤月亮沿着那条走了三十多年,眼下却觉有些陌生的小路下坳。陌生?是呀,怎么就陌生了呢?是三年没走了?哦,是杂草太盛了,反射着月光,银灿灿的一片,草间这条由乱石铺成的小路,显得瘦了。⑥下了坳,有一片稍缓的坡地,错落着几座屋子,分别是广发、兴利、黑牛、中坤等人的家,除了兴利是木工,其他几个原先也和月亮一起做泥水工、杂工,后来各自分开,也不知散落到哪处去了。一路上,经过了秋英、亚菊的家,屋子的窗都黑着,想是也没人在屋。这些年,村里的人像蒲公英的种子一样,陆陆续续飘进城里去了,有大城市,也有县城,最差的也到镇上去了。去了,也慢慢扎下了脚跟。大家都不愿意回来。⑦又转过一个山包,是一大片连着的庄稼地,现在全荒了。转过得水家,月亮看见祠堂了。守祠堂的孤老头七叔公会点香火。都是岁月,都是故事。前些年,祠堂香火很盛,除了年节、醮会,村里红事、白事都在那里上香鸣炮,禀告先人。祠堂,是一个村的中枢,是圣地哩。月亮想起了官厅前面两块石碑上刻着的字:月出满地水,云来一天山。今天是十四,往年这时候,祠堂已经开始祭祀的一些仪式了,而眼下,祠堂一片虚空,分明是一个孤独的老人,正在月光下品着这无边的寂寞呢。 ⑧还真有个孤独的人影。一身黑,佝偻着腰,像村民常用的钩刀。一勾一勾地,这个黑影,正从这一汪亮光里走到另一汪亮光里。⑨“爸!”月亮吃了一惊,“爸,你在这干啥哩?”⑩黑影稍稍抬起了头,还是钩刀一般站着。又低下了头,将手里一个东西放在祠堂与风水塘前面那块大坪的一汪亮光里。⑪顺着父亲的手,月亮发现,这一片亮汪汪的全是碗、盘、碟、缸之类的容器,全都装着水,从风水塘前一直排到大坪,整整齐齐地,足足有几千个。天,这是将村里每家的吃饭的家伙都收来了呀?⑫显然,父亲在用这些东西到塘里盛了水,又摆到了这地上。⑬“爸,你这是干啥呀?”⑭“月亮,那么多月亮。”父亲指着那些碗盘碟缸。是啊,每个碗盘碟缸里都有一个月亮!“月亮圆了,我儿子该回来了,全部人都该回来了……”父亲喃喃地说。 ⑮月亮一把抱住父亲,一阵心酸:“爸,我回来了!”⑯祠堂的侧门咿呀一声开了,出来一个黑影,像月亮父亲一样的钩刀般的身子,是村里的五保户七叔公,他就住在祠堂的侧房。他吃力地张了张嘴:“呀,是月亮回来了。”⑰七叔公说:“村里就剩我们俩了。”他望着天上的大月亮,“你带他走吧,就剩我和它了……”⑱银子似的月华笼罩着村庄,梦一般美丽。⑲月亮不由泪流满面。(有删改) 1.请用一句话概括文章的主要内容。2.赏析下面两个句子,按要求回答问题。(1)转过这个坳,是一起做泥水工的广财的家,墙头有的塌了,瓦破了,屋顶上露着的椽子黑了,房门上还挂着几片纸,残的,月光虽然很亮,却看不出红白了。(从描写角度)月亮村的月亮回乡接父亲一路上的所见所感。这个句子是对乡村的环境描写,形象地表现乡村一片破败的凄凉之景,反映乡村因人们外出谋生而逐渐败落的社会现实。 (2)祠堂一片虚空,分明是一个孤独的老人,正在月光下品着这无边的寂寞呢。(从修辞方法角度)3.请说说文章的标题“月亮村的月亮”有何妙处。这个句子用了生动的比喻,将祠堂比作是一个孤独的老人,一语双关,既象征了乡村老人的孤苦,又暗示了乡村如同老者走向末路,一片萧条。①一语双关,题目中的后一个“月亮”,既指山村的月亮,也指“月亮”这个人。②线索作用,“月亮”贯穿全文,以月亮这个人的行踪为线索,也以“月亮”这一物为线索。③新颖奇特,吸引读者阅读,激发阅读兴趣。④揭示主旨,暗示作者渴盼山村月圆人也圆的美好愿望。 4.请结合文章内容,联系现实生活,对文中人物月亮、广发、兴利、秋英等说说你的心里话。示例:月亮、广发、兴利、秋英,你们在城里住得很开心吗?城市繁华、拥挤,比不上你们曾经居住的乡村吧。国家关注民生,对农村人的农业生产、居住、医疗等有许多的新举措,乡村也是发家致富的好地方。你们回去吧,凭你们的勤劳、坚强,一定会将乡村建设得更加美丽,也一定会在乡村那片富饶的土地上走向富裕,实现“中国梦”。 我眼中的春天是一位细心的化妆师,她让小草破土而出,露出嫩绿的小芽,给大地染上了一片绿色。她让树枝长出毛茸茸的嫩芽,变成嫩叶,换上崭新的绿装。那些姹紫嫣红、各式各样的花朵,像五颜六色的地毯,把大自然点缀得更加美丽。美文欣赏·春天 部编版九年级语文上册教学课件第二单元8.蒲柳人家(节选) 日积月累 课文助读走近作者刘绍棠(1936—1997),主要作品有短篇小说集《青枝绿叶》《山楂村的歌声》《中秋节》和长篇小说《运河的桨声》。 主要内容《蒲柳人家》以独特的角度,通过充满稚气、机灵的何满子的眼睛,为我们勾勒出了几个世代栖息于北运河边上的农民的性格和命运。他们的义侠血性、慷慨豪宕和博大宽厚的灵魂,被写得情真意切、淋漓酣畅。 基础过关 2.下列词语中没有错别字的一项是()A.阔绰无原无故侍侯温顺B.隐匿无精打采吆喝腌臜C.戏谑不可救要驾驭伶俐D.置息心甘情愿帐蓬魁梧3.下列各句所用修辞手法不同于其他三项的一项是()A.他本是个整天跑野马的孩子,从早到晚关在家里,难受得屁股下如坐针毡,身上像芒刺在背。B.老秀才的眼睛尖得像锥子,一见他的身子动了动,就伸出斑竹白铜锅的长杆烟袋,敲他的光葫芦头。C.只有一个人能搭救何满子;但是,何满子望眼欲穿,这颗救命星却迟迟不从东边闪现出来。D.奶奶八样不放心,怕让狗咬了,怕让鹰抓了,怕掉在土井子里,怕给拍花子的拐走。BD 5.(西大附中中考模拟)下列句子没有语病的一项是()A.陆军某排雷大队战士杜富国,充分彰显了忠诚坚定的理想信念,为民奉献的国家情怀,敢于担当的进取精神。B.传统中国对如何读书有过很多讨论,其中,朱熹的《朱子读书法》流传甚广,成为许多人的读书指南。C.理想的中考成绩,不只取决于中考的发挥,还取决于平时在学习上是否刻苦努力,是否做到了无愧于心。D.《流浪地球》是中国电影发展已步入成熟阶段的标志性作品,赢得了国内外一致好评,创造了良好的票房业绩,填补了中国硬科幻电影的空白。B 1.结合语境,品味下列句中加点词句的表达效果。(1)一丈青大娘骂人,就像雨打芭蕉,长短句,四六体,鼓点似的骂一天,一气呵成,也不倒嗓子。答:(2)老大一个耳刮子抡圆了扇过去;那个年轻的纤夫就像风吹乍蓬,转了三转,拧了三圈儿,满脸开花,口鼻出血,一头栽倒在滚烫的白沙滩上,紧一口慢一口捯气,高一声低一声呻吟。答:·····································能力提升全练 答案(1)口语化的表述中加入书面语,诙谐幽默。(2)这句话中的动词用得非常生动传神,“抡圆了扇过去”充分写出了一丈青大娘的怒气和力气,纤夫“转了三转,拧了三圈儿”“栽倒”“捯气”“呻吟”,则写出了一丈青大娘这一巴掌的威力,读来令人如闻其声,如见其人。解析本题考查语言赏析能力。要抓住加点部分,从语体色彩(口语、书面语)、动词的运用等角度入手谈给读者的感受,或对表现人物态度、性格特点等方面的作用。 2.一丈青大娘是个刚直不阿、正气凛然的女中豪杰,与《水浒传》中著名女将一丈青扈三娘颇为神似。你知道《水浒传》中另外两位女将是谁吗?她们的绰号分别是什么?答:答案①顾大嫂,绰号:母大虫。②孙二娘,绰号:母夜叉。解析注意所写人物为“《水浒传》中另外两位女将”,要答出其姓名及绰号。 3.《蒲柳人家》中的一丈青大娘和右图漫画中的老奶奶一样,事事为孙子着想,请观察漫画,完成下面两道题。(1)用简洁的语言描述这幅漫画的内容。答:(2)如果你是漫画中“奶奶”的孙子,你将会怎样对“奶奶”说?答: 答案(示例)(1)两位小朋友在烈日下参加军训,其中一位小朋友的奶奶心疼孙子,想方设法从院墙外伸进一把伞,一边想着“我的孙子哟,这么热还军训,别晒坏了”,一边不顾自己汗流浃背,为正在军训的孙子遮出一片阴凉。(2)奶奶,请不要给我撑伞,军训能锻炼我的意志和吃苦耐劳的精神,对我以后的发展很有好处。天气虽然热,但我能坚持,回家吧,奶奶,您放心,我能受得了。解析本题考查读图和语言表达能力。题(1),要抓住画面中的人物及其言行,有条理地进行描述。语言要简洁流畅。题(2),要进入图中伞下孩子的角色,从军训对自身发展有好处方面劝说奶奶回家。语气要委婉、得体。 1.解释下列加点词语。(1)那个不知好歹的年轻纤夫,是个生愣儿,用手一推一丈青大娘,说:“好狗不挡道!”这一下可捅了马蜂窝。捅了马蜂窝:(2)何满子是一丈青大娘的心尖子,肺叶子,眼珠子,命根子。心尖子,肺叶子,眼珠子,命根子:················· 2.品析下列句子所表现的人物特点。(1)奶奶八样不放心,怕让狗咬了,怕让鹰抓了,怕掉在土井子里,怕给拍花子的拐走。答:(2)何满子就像春蚕贪吃桑叶,一册唱本不够他几天念的。答: 答案(1)“八样不放心”泛指事事不放心。接着连用四个“怕”字引领的排比句来表现一丈青大娘对孙子的溺爱。(2)“贪吃”写出了何满子对“唱本”的喜爱,这句话写出了何满子的聪慧灵秀。解析此题考查理解分析能力。(1)要抓住关键词“八样”“怕”体会人物的心情,体会一丈青大娘对孙子的爱。(2)采用比喻的修辞手法,“贪吃桑叶”体现了何满子对“唱本”的喜爱,“不够”“念”体现了何满子的“聪明”。 3.阅读下面的图画,回答问题。 (1)仔细阅读这组图画,用简洁的语言介绍图画的内容。(不超过28个字)答:(2)这组图画在社区宣传栏张贴后,很受居民欢迎。大家从中受到不同的启发,纷纷在社区网上发表感言。①从行为习惯的角度,八岁的牛牛说:②从教育方式的角度,珊珊的爸爸说:③从文化意义的角度,某大学的教授说: 答案(1)(示例)小孩看见妈妈给奶奶洗脚,也端来一盆水给妈妈洗脚。(2)①我要经常帮爸爸妈妈做些事。②孝顺长辈,身教重于言教。③孝道是家庭幸福、社会和谐的根本。解析题(1)考查读图及语言表达能力。首先按照顺序仔细观察四幅图画,明确“有什么人,做了什么事,结果如何”,然后用简洁、流畅的语言表述出来即可。题(2)考查口语交际能力。要进入不同角色,根据人物的不同身份,从问题提示的角度发表感言。语言一定要简洁。 阅读提高(2019重庆B)雪夜的老人叶骑①雪花平平仄仄落下来,散在我的脸上,像针,刺入肌肤。这痛告诉我,自己还在这个城市活着。②寒冷,是这个城市,在这个夜晚,留给我唯一真实的印记。③三年了,一千多个日夜,我告别父母,远走他乡,在这个城市开始自己的事业。但最终,三次创业换来的结局,是最初的壮志雄心成了今晚口袋里仅剩的十五块钱。我淡淡发笑,索性把这十五块钱,再换成三罐啤酒。一无所有,大概,不过如此吧。④我拿出手机,突然有倾诉的欲望。但打给谁呢?爸妈?万万不能。朋友?能说真心话的又有几人。不如,就跟眼前的夜相对无言吧,何必倾诉,谁愿倾听?我坐在公园的长椅上,看着这城市的灯火,突然忘了,在这个世界上有一种东西,你捂住嘴巴,它们就会从眼睛里跑出来。⑤夜越来越深,气温几乎跌至冰点,整个公园除了自己这个失意人,已经找不到其他行人。 ⑥我独自对着这茫茫夜色,雪花漫天飞舞。不知道什么时候,对面的长椅上来了一个老人。⑦老实说,一开始,我并没有注意到老人的到来。兴许,他恰巧从这里路过,或是心里也藏着一点儿事,而看到我,一个年轻人在这里坐着,他干脆也坐上片刻。这么冷的天,他不会待得太久。⑧我沉浸在三年创业的岁月里,分不清哪是雪,哪是泪。老人一直在我对面坐着,偶尔用目光打量一下我,像问候,像关怀,似乎也没有离开的意思。⑨我渐渐对这个老人有些好奇,不知道他这样跟我面对面坐着,是巧合,还是另有用意。⑩我前几天看报纸,说这附近的一个社区,成立了一个老年服务队,专门给需要帮助的陌生人提供力所能及的服务。莫非,他是这个服务队的成员,怕我一时做出什么傻事?⑪或者,是我长得像他的孩子,而他也曾在深夜里,看见自己的子女在生活面前声泪俱下、遍体鳞伤,眼前的这一幕勾起了他内心深处的回忆?⑫抑或,他是自己一个远方未曾谋面的亲人,我不认得他,而他记得我。在这孤独的夜晚,有他无声的陪伴? ⑬我黑色幽默般地放飞想象,希望求得一丝慰藉。但最终,悲伤如雪花般向我袭来,生活的痛楚,再次将我包围。⑭终于,夜色已深,是离开的时候了。眼前这个老人到底是谁,又何必在意。⑮我站起身,朝自己的出租房走去。⑯没走几步,我隐隐察觉到,身后的老人也起了身子。⑰我转过头,恍惚中,看见老人步履蹒跚地走到长椅旁,弯腰,捡起地上的三个空啤酒罐,微微向我致意,然后,安静地离开了公园。⑱我怔怔地站在原地,突然明白过来,却怎么也不敢相信:⑲这是一个拾荒老人,他用雪地一晚的守候,换来了三个易拉罐。⑳我望着老人远去的方向,蓦地记起他坐在公园长椅上,任由雪花飘落的那份倔强。或许,在他的一生中,还经历过无数个这样的夜晚,雪花可以落在他的头上,可以刺进他的肌肤,但大雪,从未将他掩埋。(21)四周风雪愈紧了。(22)我拨通母亲的电话,告诉她,今年生意没做好,但自己所在的城市下了一场大雪,家乡有句老话,瑞雪兆丰年,明年一定会是一个好年成。(选自《2018年中国微型小说精选》,有删改) 2.请联系上下文,理解下列句子含义。(1)寒冷,是这个城市,在这个夜晚,留给我唯一真实的印记。(2)雪花可以落在他的头上,可以刺进他的肌肤,但大雪,从未将他掩埋。我在这个城市里拼搏奋斗,最终却是一事无成,内心非常的痛苦与失落。生活的不如意与困苦让这位老人日渐苍老,但却没有击垮他的斗志与对生活的希望。 3.请联系上下文,阅读第④段画线句,具体分析人物的心理。但打给谁呢?爸妈?万万不能。朋友?能说真心话的又有几人。不如,就跟眼前的夜相对无言吧,何必倾诉,谁愿倾听?4.误会是小说在情节设置中常用的手法。请找出文中设置的误会,并简要分析其作用。打给父母,又怕父母担心。打给朋友,但是真心的朋友却没有几个。既然没有人诉说,那就给自己说,给面前的纷飞的雪花说吧。“我”一开始对老人的身份有多种猜测,直到老人拾起了易拉罐,“我”才知道他是一个拾荒者。这个误会让文章的情节起伏有波澜。 5.研读第(22)段中“我”打给母亲的电话内容,请说出其中揭示了哪些人生道理。无论在生活中遇到了多么巨大的挫折与失意,都不能失去对未来的美好希望。 非连续性文本阅读(巴蜀中学中考模拟)振兴老字号:守得住经典,当得了网红木桌凳、盖碗茶和悠闲的老茶客,灰黑的砖瓦和斑驳的墙体透露出独特的年代感。10多年前,导演宁浩在此喝了一个星期的茶,便诞生了一部电影《疯狂的石头》。之后,附近川美的学生、艺术家,都会来这儿寻找灵感。《工人日报》记者蓦然发现:这家重庆最有味道的老茶馆,在怀旧与现实中成为网红。老字号“网红”爆款目前全国有1200多个中华老字号品牌,许多老字号通过设计赋能、P合作、跨界合作、非遗传承、新零售等,推出了不少新产品和新服务、借助互联网销售成为“网红”。可是,随着市场竞争日趋激烈,传统老字号品牌的发展开始逐渐分化,这1000多家老字号中,仅有10%蓬勃发展。诞生于上世纪五六十年代的美加净与大白兔,是中国知名的两大品牌,其脑洞大开的跨界联姻出来的美加净大白兔奶糖味润唇膏一下就火了。据说开售不到1分钟,就在网上被抢购一空。 在自带情怀标签的老字号崛起之路上,“大白兔”并不孤独。据阿里研究院统计,近一年时间在天猫平台上,老字号品牌被搜索的总次数超过10亿次,老字号的消费者超过8600万人,购买老字号的90后消费者,也已超过了320万人。除了美加净大白兔奶糖味润唇膏外,还有六神花露水风味的鸡尾酒、周黑鸭的辣味唇膏、泸州老窖的顽味香水、福临门的卸妆油,跨界成为老字号年轻化的突破口,国货纷纷甩掉旧形象,以全新的姿态进入大众视野。“事实上,老字号所作的各方面探索,都是基于品牌文化传承,并非为了创新而创新。”北京稻香村有关负责人说,“盲目追求轰动效应和噱头的创新,会丢掉品牌的‘魂’。老字号不仅要做‘网红’,更要做‘天天红’‘百年红’。”奇葩新品受热捧去年的这个时候,上海老字号杏花楼创下了8个小时的排队纪录,其在清明节前推出的创新节令食品“咸蛋黄肉松青团”收获了意想不到的成功,被誉为当时最红的网红产品,有幸买到的消费者都忍不住在朋友圈炫耀一番。 去年夏天,老牌花露水六神牵手Rio鸡尾酒,推出了一款花露水口味新品。初一亮相,就成为消费界瞩目的“话题明星”,还在社交网络上引发热议。不少人直呼“奇葩”,“喝了能驱蚊吗”也成为网友们争相调侃的话题。随着95后、00后逐渐成为消费主力军,老字号开始一改墨守成规的路线,主动融入年轻消费层。“守得住经典,当得了网红”,成为当下众多老字号的自我修养,不仅要最大化地重现消费者脑中对老字号经典的记忆和认知,同时还要给他们带来新的体验和趣味。据调查,新奇特口味在今年将持续走红吸睛。像MM′S辣味巧克力豆、王老吉黑凉茶、奥利奥行走的表情包夹心饼干、海底捞懒人自煮微火锅等,一波波新奇特个性化产品持续点燃消费者的尝新欲,并引发年轻人的抢购热潮。得消费者得市场被网友戏称为“老汉撒娇,猝不及防”的泸州老窖粉色香水、登上纽约时装周的老干妈卫衣、与热播影视剧合作制造话题的百雀羚……无可否认,跨界、做IP确实好玩,但因产品新奇有趣带来的热度能维持多久仍未可知。 “得消费者,才能得市场。”上海交通大学安泰经济与管理学院副教授周颖评价说,“只有持续地生产优质的产品,给消费者更好的体验,老字号才能获得更为长久的生命力。”大数据统计分析,特色独家定制产品正日渐成为平台的消费趋势。依托京东大数据分析用户喜好和多元化需求,老字号进行灵活定制的产品越发受到消费者追捧。像百事星战IP定制罐、周黑鸭啃定团圆京东狗定制版、脉动全职高手京东定制款2.0、脉动XQ飞车QQ飞车十周年脉动纪念瓶、立顿京东“狗”年特供奶茶装等,都已上线京东,市场呼声极高。“技术创新对老字号而言,具有生死存亡的意义。现代社会科技发展日新月异,仅仅攥着祖传的老秘方是远远不够的。”中国社会科学院经济研究所研究员剧锦文认为,“要保护传承好老味道、老工艺,就必须有先进技术作支撑,这样才能再现老字号的辉煌。”(选自《工人日报》,2019年2月27日,有删减) 1.下列说法符合文意的一项是()A.1000多家传统老字号在日趋激烈的市场竞争中,大部分仍然能够得到蓬勃发展。B.在激烈的市场竞争中,老字号为了自身的发展,一定要追求轰动效应和噱头的创新。C.老牌花露水六神推出的花露水口味鸡尾酒,受到年轻的95后、00后网友们的热捧。D.老字号可依托京东大数据,分析用户喜好和多元化需求,为消费者定制特色独家产品。D 3.阅读全文,简述老字号怎样才能“守得住经典,当得了网红”。①推出新产品;②借助互联网,拓展销售渠道;③最大化地重现消费者对老字号经典的记忆和认知;④主动融入消费主力军;⑤持续地生产优质的产品,给消费者更好的体验;⑥通过名人宣传,扩大产品影响力。 4.湖州市商务局正在开展“百年老字号·中华行”活动,湖州丁莲芳食品有限公司为此准备拍摄宣传短片,请根据文章内容,续写宣传短片的文字稿。要求:①为百年老字号“丁莲芳”的振兴拟写两条措施;②语言得体,120字左右。“丁莲芳”的前世今生历史的车轮滚滚,它穿过时空的经,岁月的纬,从1878年的晨曦中走来。它摘下斗笠,打开挑子,用厚茧的手掌,郑重托起一碗热腾腾的千张包子粉丝头,以鲜猪肉、千张为原料,裹成长枕形的千张包子,引得路人闻香而来。它就是距今有着一百四十多年历史的中华老字号——丁莲芳。……百年倏忽过,时光来到2019。这是一个市场竞争日趋激烈的时代,这是一个科技发展日新月异的时代。湖州“丁莲芳”为了再现老字号的辉煌,推出了各式各样的新产品,蟹黄千张包鲜美多汁,水果千张包甜美清爽,衍化出的迷你款博得孩子青睐,爱心款最受情侣追捧。“丁莲芳”借助互联网平台,发展微商团队,拓展了销售渠道。古色古香的店铺,融入了时尚新潮的动漫元素。百年品牌“丁莲芳”已经成为湖州美食的金名片。 冬天过去了,春天又来了,世界犹如一幅美丽又美好的画卷,是她唤醒了大地。在郊外,柳枝抽出了新的芽,在春风中快活着欢乐着。春风吹落了一片又一片的桃花花瓣,美丽的花瓣飘来飘去,好像在向人们说春天来了。小燕子从南方赶来,他们唧唧地叫着,剪刀似的尾巴一翘一翘的似乎在歌颂着春天的美好!美文欣赏·春天 部编版九年级语文上册教学课件第三单元9.鱼我所欲也 日积月累 课文助读走近作者孟子(约前372—前289),名轲,邹(今山东省邹城市)人。战国时期伟大的思想家,他是继孔子之后儒家学派的又一位大师,被推尊为“亚圣”。 主要内容《鱼我所欲也》选自《孟子·吿子上》。本文指出“义”之价值高于生命,贤者在必要时应舍弃生命而“取义”;不辨礼义而贪求富贵的行为是不足取的。 基础过关 4.下列句子中没有语病的一项是()A.这位志愿者值守西湖边两平方米的“微笑亭”,多年来风雨无阻,志愿服务时数累计超过1.1万小时以上,让各地游客倍感暖心。B.如果吃饭时玩手机,血液就要分出一大部分供应给大脑和眼睛,这会严重导致胃肠的蠕动和消化液的分泌受到影响,造成消化不良。C.在围绕大运河文化带建设、推动制度创新方面,杭州积极发挥了试点市、示范市的引领作用,形成了可复制可推广的实践经验。D.以APP为主要形态的电子家庭作业或许具备即时上传、实时交互、大数据分析等优势,但归根结底由商业的利益在驱动是无疑的。C 5.(西大附中中考模拟)阅读材料,探究“大数据”发展趋势受哪些因素的影响。随着国家大数据战略推进实施以及配套政策的贯彻落实,大数据产业发展环境将进一步优化,社会经济各领域对大数据服务需求将进一步增强,大数据的新技术、新业态、新样式将不断涌现,产业规模将继续保持高速增长态势。大数据的技术发展与物联网、云计算、人工智能等新技术领域的联系将更加紧密,物联网的发展将极大提高数据的获取能力,云计算与人工智能将深度融入数据分析体系,融合创新将会不断地涌现和持续深入。随着2017年教育部公布第二批获准开设“数据科学与大数据技术”的高校名单,加上第一批获批高校获批开设该专业。今年开始,大数据需要的复合型人才将源源不断 形成。加之海外和传统行业跨界人才不断加入大数据行业,大数据产业将迎来创新发展。近年来,伴随着资本大量进入大数据行业,出现了创业公司估值过高的现象。泡沫期的大数据行业,许多企业的发展远远无法回归企业的本质,导致创业企业供给与市场需求之间脱节。随着资本的沉没,理性资本将引领大数据行业健康发展。一些在资本热潮褪去之后还能沿正常轨道发展的企业将占据新一轮的资本优势,得到进一步的发展和壮大。示例:①国家大数据战略的推进和配套政策的贯彻落实;②物联网、云计算、人工智能等新技术;③复合型人才的加入;④理性资本的投入和发展。 1.(独家原创试题)下列加点词语意义相同的一项是(  )A.乡为身死而不受   今为宫室之美为之B.所欲有甚于生者  万钟于我何加焉C.二者不可得兼  所识穷乏者得我与D.故患有所不辟也故不为苟得也答案D A.为了/做。B.比/对。C.得到/同“德”,感恩、感激。D.都是“所以”的意思。·······能力提升全练 2.(独家原创试题)下列句子的朗读停顿划分不正确的一项是(  )A.如使/人之所欲/莫甚于生B.由是/则生而有不用也C.立/天下之正位D.今为/妻妾之奉/为之答案D 根据句意“现在为了妻妾的侍奉却做了”可知应划分为:今/为妻妾之奉/为之。 3.根据提示默写填空。(1)本文的中心论点是:(2)文中体现“性本善”思想的句子是:(3)文中与“不食嗟来之食”的意思相一致的句子是: 答案(1)生,亦我所欲也;义,亦我所欲也。二者不可得兼,舍生而取义者也。(2)非独贤者有是心也,人皆有之。(3)呼尔而与之,行道之人弗受;蹴尔而与之,乞人不屑也。解析本题考查理解默写能力。一要用课文中的原句作答;二要仔细审题,筛选最恰当的语句;三要避免写错别字,漏字。 4.结合初中学习的文学作品,补全下联。上联:学《孟子》知舍生取义下联:答案(示例1)读《论语》懂不耻下问(示例2)看《水浒》明除暴安良解析本题考查根据学过的文学作品对对联的能力。要做到词性相对,意思相关;要与上联一样,体现出读文学作品所得到的收获。 1.文言文本来是没有标点的,我们现在所看到的文言文中的标点都是后人加的。有人将课文中的句子做如下断句,与课文断句比,哪一种断句更好?请说说理由。有人断句:鱼我所欲也,熊掌亦我所欲也。二者不可得兼,舍鱼而取熊掌者也。课文断句:鱼,我所欲也;熊掌,亦我所欲也。二者不可得兼,舍鱼而取熊掌者也。答: 答案课文断句更好。点明了主语“鱼”和“熊掌”,进而突出了本文讨论的“生”“义”这一话题,使读者的注意力集中在文章的中心论点上。解析本题考查对文言断句的比较鉴赏能力。比较两种断句方式的不同:课文断句将“鱼”和“熊掌”单独列出来,独立成为分句。回答时,首先要表明观点,然后从所选断句方式对表现文章内容、观点的作用角度分析作答。 2.阅读台湾漫画家蔡志忠先生的作品,判断漫画的内容符合下列选项中的哪一句,再列举一个符合这句话思想的社会行为,并说明理由。A.为政以德,譬如北辰居其所而众星共之。B.道之以政,齐之以刑,民免而无耻;道之以德,齐之以礼,有耻且格。 C.举直错诸枉,则民服;举枉错诸直,则民不服。D.君子周而不比,小人比而不周。答:(  )答案B (示例)由中央电视台打造的精品栏目《感动中国》,通过展现感动中国人物身上所具有的优秀精神品质,宣扬社会正能量,起到道德引领作用,使人心向善。这种行为,符合B项中孔子的思想。解析解答本题要读懂漫画内容,把握其中的关键词“政令”“刑罚”“德政”“没面子”等。对照选项,B项含有漫画中的关键词最多。举例要典型,要充满正能量,理由要从引导人向善的角度思考作答。 文言文阅读(重庆一中中考模拟)鱼我所欲也《孟子》鱼,我所欲也;熊掌,亦我所欲也。二者不可得兼,舍鱼而取熊掌者也。生,亦我所欲也;义,亦我所欲也。二者不可得兼,舍生而取义者也。生亦我所欲,所欲有甚于生者,故不为苟得也;死亦我所恶,所恶有甚于死者,故患有所不辟也。如使人之所欲莫甚于生,则凡可以得生者何不用也?使人之所恶莫甚于死者,则凡可以辟患者何不为也?由是则生而有不用也,由是则可以辟患而有不为也。是故所欲有甚于生者,所恶有甚于死者。非独贤者有是心也,人皆有之,贤者能勿丧耳。一箪食,一豆羹,得之则生,弗得则死。呼尔而与之,行道之人弗受;蹴尔而与之,乞人不屑也。万钟则不辩礼义而受之,万钟于我何加焉!为宫室之美、妻妾之奉、所识穷乏者得我与?乡为身死而不受,今为宫室之美为之;乡为身死而不受,今为妻妾之奉为之;乡为身死而不受,今为所识穷乏者得我而为之:是亦不可以已乎?此之谓失其本心。 2.下列句子翻译有误的一项是()A.故不为苟得也所以我不做苟且偷生的事B.贤者能勿丧耳贤人能够不丢掉罢了C.呼尔而与之没有礼貌地吆喝着给他D.乡为身死而不受乡邻因害怕死亡而不接受3.本文提出在“生”和“义”不能兼顾的情况下,应该“①”的观点,并进一步指出这是每个人都有的“②”。D舍生而取义者也本心 (育才中学中考模拟)鱼,我所欲也;熊掌,亦我所欲也。二者不可得兼,舍鱼而取熊掌者也。生,亦我所欲也;义,亦我所欲也。二者不可得兼,舍生而取义者也。生亦我所欲,所欲有甚于生者,故不为苟得也;死亦我所恶,所恶有甚于死者,故患有所不辟也。如使人之所欲莫甚于生,则凡可以得生者何不用也?使人之所恶莫甚于死者,则凡可以辟患者何不为也?由是则生而有不用也,由是则可以辟患而有不为也。是故所欲有甚于生者,所恶有甚于死者。非独贤者有是心也,人皆有之,贤者能勿丧耳。一箪食,一豆羹,得之则生,弗得则死。呼尔而与之,行道之人弗受;蹴尔而与之,乞人不屑也。万钟则不辩礼义而受之,万钟于我何加焉!为宫室之美、妻妾之奉、所识穷乏者得我与?乡为身死而不受,今为宫室之美为之;乡为身死而不受,今为妻妾之奉为之;乡为身死而不受,今为所识穷乏者得我而为之:是亦不可以已乎?此之谓失其本心。 7.翻译下列句子。(1)非独贤者有是心也,人皆有之,贤者能勿丧耳。(2)万钟则不辩礼义而受之,万钟于我何加焉!不仅贤德的人有这种天性,人人都有,只不过贤德的人能够不丧失罢了。对于优厚的俸禄(高位厚禄)不辨别是否合乎礼义就接受了,这样的优厚俸禄(高位厚禄)对我有什么好处呢? 8.作为21世纪的中学生,你怎样理解“义”?请选取两位我国历史上舍生取义的人物,简单叙述其事迹。示例:能见义勇为,为正义事业而献身,做于国于民有益的事是“义”。社会上的哥们义气,与小团体、个人私利相关的江湖义气和本文的“义”不同。我国历史上舍生取义的事例很多,如:唐雎出使秦国,不辱使命;苏武牧羊,不改初心;岳飞抗金,精忠报国。 非连续性文本阅读(重庆八中中考模拟)材料一:1958年到1982年,张家界林场职工垦荒整地、植树造林。到上世纪80年代初,林场的森林覆盖率达到95%以上,森林蓄积量由1958年建林初期的约4万立方米增长到约20万立方米。以前,由于路不通,林场“与世隔绝”。1979年后,媒体的推荐报道、吴冠中到此写生等,一次次向世界展示了张家界林场的美,来张家界的游客络绎不绝。1982年,中央批准建设全国第一个国家级森林公园——张家界国家森林公园。【亲历者说】张杰:原张家界国家森林公园管理处调研员,亲历张家界国家森林公园的建设发展。“三千奇峰、八百流水、十万森林,我们当地人称之为‘放大的盆景,缩小的仙境’。”张杰说,“为扩大宣传,我们背着电影机和传单走出大山,向外推广张家界国家森林公园。电影名为《奇山揽胜》,反响热烈,每一场都座无虚席。”“开发加宣传,1984年,公园接待游客共8万人次,到1988年,游客就达到了56万人次。” “游客多了,环境是否承载得起?2000年左右,为了减少污染,提质发展,森林公园15家酒店宾馆的煤锅炉全被叫停,换成了燃油锅炉。金鞭溪的上游建起了污水处理厂,老百姓家中排出的生活污水经过处理后,才能排入溪水中。景区的厕所外建起了技术先进的化粪池,粪污经过就地处理,能达到国家一级A类排放标准。”“无论是黄石寨,还是金鞭溪,现在依然是山清水秀林深。”这些年,张家界国家森林公园蜚声海内外,成为中国首批5A级旅游区,不仅被列入《世界遗产名录》,还被列为世界地质公园。(来源:《人民日报》有删改)材料二:森林公园大致分为国家级、省级和市县级,国家森林公园是其中的最高等级。据人民网报道,截至2019年2月,我国国家级森林公园已达897处。湖南省林业局专家表示,截至目前,湖南省已建立国家级森林公园64个,位居全国第一,经营总面积34.75万公顷。湖南省十分重视森林公园管理,出台了全国第一部森林公园管理方面的地方性法规《湖南省森林公园条例》。2018年,全省以森林公园为主体的森林旅游接待游客8600万人次,创旅游综合收入997亿元。(来源:《人民日报》有删改) 材料三:中国退耕还林工程始于1999年,是迄今为止我国政策性最强、投资量最大、涉及面最广、群众参与程度最高的一项生态建设工程,也是最大的强农惠农项目。仅中央投入的工程资金就超过4300多亿元,是迄今为止世界上最大的生态建设工程。2014年,为解决中国水土流失和风沙危害问题,增加中国森林资源,应对全球气候变化,中国批准实施《新一轮退耕还林还草总体方案》。2017年5月,国务院批准了国家林业局等部门提出的要进一步扩大退耕还林还草规模的请示,同意调减云南等18个省(区)3700万亩陡坡耕地基本农田用于退耕还林还草,从而使新一轮退耕还林还草总规模扩大了近一倍。(来源:百度百科有删改) 1.下列对三则材料内容理解不正确的一项是()A.张家界森林公园发展经历了三个阶段:第一个阶段是垦荒造林;第二个阶段开发宣传;第三个阶段提质发展。B.2018年,张家界以森林公园为主体的森林旅游接待游客8600万人次,创旅游综合收入997亿元。C.材料一中的画线句通过列数字和作比较的说明方法准确而突出地说明了张家界林场垦荒造林成效显著。D.从三则材料可以看出,我国从中央到地方,都十分重视保护环境,加强生态工程建设,实施绿色发展。B 2.材料三画线句“仅中央投入的工程资金就超过4300多亿元”中,“仅”字能否删去?为什么?不能删。“仅”表示范围,是“单单”的意思,表示单是中央的投入资金就超过了4300多亿元,如果加上其他各级的投入,资金更多,表现了政府对生态建设的高度重视。如果删去,意思就变为除了中央的超过了4300多亿元的投入资金外,没有了其他的投入,意思发生了变化。“仅”体现了说明文语言的严谨性。 3.读材料一,分条概述张家界森林公园为了减少污染所采取的措施。①煤锅炉换成了燃油锅炉;②上游建起污水处理厂,加强生活污水的处理;③厕所外建起先进的化粪池,粪污就地处理。 夏天是个变脸的季节,它时而哭,时而笑。一会儿晴空万里,天高云淡,骄阳似火;一会儿天低云暗,乌云密布,狂风怒吼;一会儿天公大发雷霆,电闪雷鸣;一会儿瓢泼大雨,倾盆滂沱;一会儿潇潇雨歇,虹桥飞架南北。夏天就如同孩子的脸一般,没有征兆地变换着。美文欣赏·夏天 部编版九年级语文上册教学课件第三单元10.唐雎不辱使命 日积月累 课文助读走近作者刘向(约前77—前6),本名更生,字子政,沛(今江苏沛县)人,汉皇族。西汉经学家、目录学家、文学家。 写作背景公元前221年,秦国统一了天下。安陵国是魏国的附庸小国。安陵在它的宗主国魏国灭亡之后,一度还保持着独立的地位。秦国想用诈骗手段吞并安陵,于是安陵君就派唐雎到秦国谈判。 主要内容这篇文章所记叙的是强国和弱国之间一场外交斗争的情况。文章用人物对话生动地塑造了唐雎的形象,表现了唐雎维护国土的严正立场和不畏强暴、敢于斗争的布衣精神,从而揭示了弱国安陵能够在外交上战胜强秦的原因。 基础过关 3.翻译下列句子。(1)虽千里不敢易也,岂直五百里哉?(2)布衣之怒,亦免冠徒跣,以头抢地尔。(3)以君为长者,故不错意也。(4)怀怒未发,休祲降于天。即使千里也不敢交换,难道仅仅是五百里吗?平民发怒,也不过是摘了帽子光着脚,把头往地上撞罢了。把安陵君看作忠厚长者,所以不打他的主意。心里的愤怒还没发作出来,上天就降示了征兆。 4.文中三次写了秦王的神态和一个简单的动作。请抄出原句,并结合课文分析秦王的心态变化。神态:秦王不说、秦王怫然怒、秦王色挠;一个简单的动作:长跪而谢之曰。刻画出了秦王由傲慢骄横到恼羞成怒再到吃惊折服的思想感情变化,反映了秦王是个扩张欲极度膨胀,但又色厉内荏、外强中干的纸老虎。 6.(重庆八中中考模拟)下列选项中没有语病的一项是()A.近来,栖霞区城市管理局积极开展“垃圾分类进校园”,让绿色低碳融入学生生活。B.随着电视剧《都挺好》的热播,剧中主人公的命运无时无刻都在牵动着广大观众的心。C.数字化转型对中国企业而言,不仅是一场认知与思维的革命,更是一种技术变革。D.推进传统媒体和新兴媒体的融合,关键在于转换观念,鼓起勇气,迎难而上。D 7.(南开中学中考模拟)依次填入下面语段横线处的语句,衔接最恰当的一项是()你去参加一场音乐会,,。为什么?从众心理。音乐会期间,。为什么?从众心理。音乐会结束后,,?恐怕也会留下一份小费。①你去更衣室取你的外套②一个人带头鼓起掌来,于是整个大厅里顿时掌声雷动,你也跟着鼓掌③你不假思索,也仰头观看④你看到前面的人将一枚硬币扔进一只碟子里,虽然更衣室的费用是包含在票价里的,但你会怎么做⑤在十字路口遇见一群人,他们一个个仰望天空⑥当表演到一个精彩段落时A.③⑤②⑥①④B.⑤③①④②⑥C.⑤③⑥②①④D.①④⑤③⑥②C 1.下列句子中加点词的意义和用法,相同的一项是(  )A.聂政之刺韩傀也/天子之怒,伏尸百万B.徒以有先生也/寡人以五百里之地易安陵C.臣未尝闻也/此三子者,皆布衣之士也D.安陵君因使唐雎使于秦/安陵君受地于先王而守之答案A A.都是助词,用在主谓之间,取消句子独立性。B.连词,因为/介词,用。C.语气词,表陈述/语气词,表判断。D.介词,到/介词,从。········能力提升全练 2.下列语句朗读停顿不正确的一项是(  )A.怀怒/未发,休祲/降于天B.安陵君/因使唐雎/使于秦C.寡人/欲以五百里之地/易安陵D.此庸夫/之怒也,非士/之怒也答案D 正确的朗读停顿应为:此/庸夫之怒也,非/士之怒也。 3.下列说法正确的一项是(  )A.“寡人”是臣民对古代帝王的敬称。B.《唐雎不辱使命》一文中,唐雎列举了平民中的四个有志之士的发怒。C.“岂直五百里哉?”一句中的“哉”是语气词,相当于现代汉语中的“呢”“吗”。D.《战国策》是宋代刘向根据战国时期的史料整理编辑的,共33篇,分国编次。答案C A.“寡人”是古代国君对自己的谦称。B.应是“三个有志之士”。D.“宋代刘向”不正确,应为“西汉刘向”。 4.下列句子全都表现秦王蛮横的一项是(  )①秦王不说。②秦王怫然怒。③秦王使人谓安陵君曰:“寡人欲以五百里之地易安陵,安陵君其许寡人!”④秦王曰:“天子之怒,伏尸百万,流血千里。”⑤秦王色挠。A.①③⑤  B.①②④  C.③④⑤  D.②③④答案D ②“怫然怒”直接描写了秦王的“蛮横”;③“秦王使人谓安陵君”中的“谓”字,使其蛮横态度毕现;④“天子之怒,伏尸百万,流血千里”中,秦王以“天子”自居,威吓唐雎,足见他的蛮横。 1.下列对文章内容理解错误的一项是(  )A.唐雎有胆有识,不为秦王的谎言所动,也不为他的威胁所屈,始终处于主动地位。B.秦王企图用“易地”的方法吞并安陵,由此引出安陵君派唐雎出使秦国一事。C.秦王的欺骗没有得逞,于是企图以“天子之怒”来恫吓唐雎。D.面对秦王的恫吓,唐雎起初有些惊慌,但最终还是驳斥了秦王,维护了安陵国的利益。答案D“唐雎起初有些惊慌”错误,唐雎自始至终镇定从容。 2.品读课内语段,回答问题。秦王使人谓安陵君曰:“寡人欲以五百里之地易安陵,安陵君其许寡人!”安陵君曰:“大王加惠,以大易小,甚善;虽然,受地于先王,愿终守之,弗敢易!”秦王不说。安陵君因使唐雎使于秦。从哪些地方可以看出秦王与安陵君易地实际是包藏祸心,恃强凌弱?请简要分析。答: 答案①从秦王“使人谓”可看出秦王对安陵君的轻视;②从“寡人欲以……安陵君其许寡人”可看出秦王口气强硬,并无商量余地;③从“秦王不说”可看出秦王名易实夺的心理。解析紧扣语段内容,思考秦王的言行,分析秦王的用心。 文言文阅读(川外附中中考模拟)甲客问元方:“尊君在不?”答曰:“待君久不至,已去。”友人便怒曰:“非人哉!与人期行,相委而去。”元方曰:“君与家君期日中。日中不至,则是无信;对子骂父,则是无礼。”友人惭,下车引之。元方入门不顾。乙康肃问曰:“汝亦知射乎?吾射不亦精乎?”翁曰:“无他,但手熟尔。”康肃忿然曰:“尔安敢轻吾射!”翁曰:“以我酌油知之。”乃取一葫芦置于地,以钱覆其口,徐以杓酌油沥之,自钱孔入,而钱不湿。因曰:“我亦无他,惟手熟尔。”康肃笑而遣之。丙秦王谓唐雎曰:“寡人以五百里之地易安陵,安陵君不听寡人,何也?且秦灭韩亡魏,而君以五十里之地存者,以君为长者,故不错意也。今吾以十倍之地,请广于君,而君逆寡人者,轻寡人与?”唐雎对曰:“否,非若是也。安陵君受地于先王而守之,虽千里不敢易也,岂直五百里哉?” 3.对文中画线句子翻译正确的一项是()虽千里不敢易也,岂直五百里哉?A.即使方圆千里的土地也不敢轻易(答应),哪里能直接用五百里的土地(交换)呢?B.即使方圆千里的土地也不敢交换,哪里只是用五百里的土地(就交换)呢?C.虽然方圆千里的土地也不敢轻易(答应),哪里只是用五百里的土地(就交换)呢?D.虽然方圆千里的土地也不敢交换,哪里能直接用五百里的土地(交换)呢?B 4.下列对文段理解和分析有误的一项是()A.三组对话皆聚焦冲突。三组选文中的人物对话,分别围绕“谁未守信”“本领是否值得炫耀”“要不要服从秦国要求”进行。B.三组对话均展现应答智慧。甲文元方有理有据,睿智回应;乙文卖油翁现身说法,类比说理;丙文唐雎据理反诘,沉稳应对。C.三组选文皆用对比。甲文客人冲动无礼对比元方有礼有节,乙文康肃神闲气定对比卖油翁骄傲自信,丙文唐雎不卑不亢对比秦王盛气凌人。D.三组选文语言简练。甲文仅“引”字就写出友人惭愧示好,乙文只用“安敢”就显出康肃居高临下,丙文以一“否”字就表现出唐雎态度鲜明。C (巴蜀中学中考模拟)秦王使人谓安陵君曰:“寡人欲以五百里之地易安陵,安陵君其许寡人!”安陵君曰:“大王加惠,以大易小,甚善;虽然,受地于先王,愿终守之,弗敢易!”秦王不说。安陵君因使唐雎使于秦。秦王谓唐雎曰:“寡人以五百里之地易安陵,安陵君不听寡人,何也?且秦灭韩亡魏,而君以五十里之地存者,以君为长者,故不错意也。今吾以十倍之地,请广于君,而君逆寡人者,轻寡人与?”唐雎对曰:“否,非若是也。安陵君受地于先王而守之,虽千里不敢易也,岂直五百里哉?”秦王怫然怒,谓唐雎曰:“公亦尝闻天子之怒乎?”唐雎对曰:“臣未尝闻也。”秦王曰:“天子之怒,伏尸百万,流血千里。”唐雎曰:“大王尝闻布衣之怒乎?”秦王曰:“布衣之怒,亦免冠徒跣,以头抢地尔。”唐雎曰:“此庸夫之怒也,非士之怒也。夫专诸之刺王僚也,彗星袭月;聂政之刺韩傀也,白虹贯日;要离之刺庆忌也,仓鹰击于殿上。此三子者,皆布衣之士也,怀怒未发,休祲降于天,与臣而将四矣。若士必怒,伏尸二人,流血五步,天下缟素,今日是也。”挺剑而起。秦王色挠,长跪而谢之曰:“先生坐!何至于此!寡人谕矣:夫韩、魏灭亡,而安陵以五十里之地存者,徒以有先生也。” 6.翻译下面的文言语句。布衣之怒,亦免冠徒跣,以头抢地尔。7.秦王最后是长跪而谢之,说明秦王内心状态怎样?同时从侧面表现了唐雎的什么精神?百姓发怒,也不过就是摘掉帽子,光着脚,把头往地上撞罢了。表现秦王内心的恐惧,侧面表现唐雎不畏强权的大无畏精神。 9.唐雎说“布衣之怒”时,为什么要引出专诸、聂政、要离行刺的故事?以专诸、聂政、要离行刺的故事表明自己的态度,说明自己将效仿他们,刺伤秦王。 非连续性文本阅读(2019重庆A)【文本一】刚刚过去的劳动节,家长们带着孩子们出去游玩,但又有多少家长在劳动节上好了“劳动”这一课呢?放学途中,书包在父母长辈的肩上,孩子们却两手空空;午餐时,小学一年级的众多孩子不会剥虾,导致无从下口;更有甚者,大学生不会自己洗衣服,每周带一大包脏衣服回家。对于劳动,家长们各有说法。家长1:儿子从2岁多,开始自己用微波炉热牛奶。4岁多,站在小板凳上煎鸡蛋。现在6岁,可以自己煮西红柿鸡蛋面。家长2:儿子10岁了,还不太会系鞋带。每次看他系得那么费劲,干脆所有的鞋子都买成带粘扣的。男孩就是没有女孩那么细心,生活能力差点儿就差点儿吧。家长3:作为80后,因为有超能干的妈妈,导致我的个人生活能力很差。我倒是挺想教会孩子一些生活技能,问题是我自己也不太会做。对此我也很无奈。家长4:我觉得孩子的任务就是读书。只要学习成绩好,长大了自然有出息。至于劳动教育,对孩子没什么意义。 【文本三】①劳动教育为孩子的幸福人生奠基,这已成为现代教育的共识。生活靠劳动创造,人生也靠劳动创造。劳动教育是提高中小学生综合素质、成就幸福圆满人生的有效途径。苏联教育家马卡连柯曾指出,“劳动永远是人类生活的基础,是人类创造幸福的基础”。②劳动教育对于立德树人,促进学生全面发展具有不可替代的作用。劳动教育可以促进学生形成勤俭节约、踏实肯干、意志坚定、团结协作的优良品质,使之成为有大爱大德大情怀的人。品德修养需要在长期的社会实践中、在日常生活的点点滴滴中踏踏实实地磨炼达成。③劳动可以促进学生形成基本的生活生产劳动技能、初步的职业意识、创新创业意识和动手实践的能力。劳动教育要在增长青少年的知识见识上下功夫,引导青少年在做中学、学中做,在社会劳动实践中增长见识、丰富学识、求真理、悟道理、明事理。 ④劳动可以促进学生强健体魄,形成健康身心和健全人格。劳动教育要引导学生在劳动中享受乐趣、增强体质、健全人格、锤炼意志。⑤劳动教育有利于加强和改进学校美育,形成以劳育美、以美育人、以文育人的育人模式,促进学生树立“劳动最光荣、劳动最崇高、劳动最伟大、劳动最美丽”的劳动审美观,让青少年学生在劳动创造中形成发现美、体验美、鉴赏美、创造美的意识和能力,从而提高学生审美能力和人文素养。⑥培养德智体美劳全面发展的社会主义建设者和接班人,离不开劳动教育的支撑。要把劳动教育融入劳动课程、校内劳动、校外劳动实践、家务劳动各环节中,整体构建起德智体美劳培养的教育体系。(选自《光明日报》2018年10月11日,有删节) 1.下列对文本一相关内容和文本三第③⑤段内容表述不正确的一项是()A.文本一的第二段列出了有关劳动的种种现象,意在引起大家对劳动教育的关注。B.“家长2”忽视孩子的劳动教育,这样做不利于培养孩子独立自主的生活能力。C.要增长青少年的劳动知识和见识,主要靠社会劳动实践,在做中学,学中做。D.青少年学生只有在劳动创造中才能形成发现美、体验美、鉴赏美、创造美的意识和能力。C 2.当前不少中小学生劳动能力较差的原因有哪些?请结合文本一和文本二作答。①家长原因:更多的家长不注重培养孩子的劳动意识。只注重学习教育,从而忽视了劳动教育,导致学生缺乏劳动锻炼。②学生原因:学生对劳动教育缺乏主动性,对劳动课的态度也是可有可无的。③学校原因:很多学校对劳动课的开设不够重视,大多学校认为劳动课程没有必要作为考核科目。 3.请写出文本三中画线句运用的论证方法,并分析其作用。4.针对文本一中“家长4”关于劳动教育的看法,请运用文本三的相关内容予以反驳。引用论证。引用了马卡连柯的话,有力地说明了劳动教育是孩子幸福人生的基础,增强了论证的说服力和权威性。劳动教育对于奠基孩子的幸福人生,全面发展,生活实践等都发挥着不可替代的作用。第一,一个孩子有没有出息不光是看学习成绩,还要立德树人,全面发展。若没有勤俭节约,踏实肯干等优良品质,也是庸才而已。第二,出息成才,不光是靠学习成绩。还有生产技能,职业意识等实践能力。缺乏这些,无异于纸上谈兵,误人误己。第三,出息成才,还需一个强健的体魄,健康身心和健全人格。综上所述,要成长成才,德智体美劳全面发展才行。因此,劳动教育是十分有必要的。 晴空万里,天上没有一丝云彩,太阳把地面烤得滚烫滚烫;一阵南风刮来,从地上卷起一股热浪,火烧火燎地使人感到窒息。杂草抵不住太阳的暴晒,叶子都卷成个细条了。每当午后,人们总是特别容易感到疲倦,就像刚睡醒似的,昏昏沉沉不想动弹。连林子里的小鸟,也都张着嘴巴歇在树上,懒得再飞出去觅食了。美文欣赏·夏天 部编版九年级语文上册教学课件第三单元11.送东阳马生序 日积月累 课文助读走近作者宋濂(1310—1381),初名寿,字景濂,号潜溪,浦江(今属浙江)人。明初著名政治家、文学家、史学家、思想家。其散文质朴简洁,或雍容典雅,各有特色。 写作背景明洪武十一年(1378),宋濂告老还乡的第二年,应诏从家乡浦江(浙江省浦江县)到应天(今江苏南京)去朝见,同乡晚辈马君则前来拜访,宋濂写下了此篇赠序,介绍自己的学习经历和学习态度,以勉励他人勤奋学习。 主要内容这篇赠序里,作者叙述个人早年虚心求教和勤苦学习的经历,生动而具体地描述了自己借书求师之难,饥寒奔走之苦,并与太学生优越的条件加以对比,有力地说明学业能否有所成就,主要在于主观努力,不在天资的高下和条件的优劣,以勉励青年人珍惜良好的读书环境,专心治学。全文结构严谨,详略有致,用对比说理,在叙事中穿插细节描绘,读来生动感人。 基础过关 3.下列朗读停顿划分正确的一项是()A.以是人/多以书/假余,余因得/遍观群书。B.又患/无硕师名人/与游,尝/趋百里外/从乡之先达/执经叩问。C.行/深山巨谷/中。穷/冬烈风,大雪/深数尺,足肤皲裂/而不知。D.余则/缊袍敝衣/处其间,略无慕/艳意,以中有/足乐者。4.下列语句有通假字的一项是()A.至舍,四支僵劲不能动。B.盖余之勤且艰若此。C.凡所宜有之书,皆集于此。D.岂他人之过哉?BA 5.下面对本文赏析不正确的一项是()A.文章从“嗜学”与“家贫”这一尖锐矛盾写起,初步展现了作者勤奋学习的态度。B.文章用学习态度和学习条件进行对比,进一步突出作者不畏艰辛、刻苦学习的精神。C.文章用先达的骄矜暴躁与自己的虔诚恭敬对比,反映了品德修为重于求学的道理。D.作者通过写自己的求学经历,动之以情,晓之以理,意在勉励青年学子要刻苦读书。C 7.(重庆一中中考模拟)下列各句中,没有语病的一项是()A.今年五一节前夕,发改委发出紧急通知,禁止空调厂商和经销商不得以价格战的手段进行不正当竞争。B.据报道,某市场被发现存在销售假冒伪劣产品,伪造质检报告书,管理部门将对此开展专项检查行动,进一步规范经营行为。C.随着个人计算机的广泛应用,互联网以不可阻挡之势在全世界范围内掀起了影响社会不同领域、不同层次的变革浪潮。D.打车软件为乘客和司机搭建起沟通平台,方便了市民打车,但出租车无论是否使用打车软件,均应遵守运营规则,这才能维护相关各方的合法权益和合理要求。C 1.下列加点字的注音完全正确的一项是(  )A.既加冠(guàn),益慕圣贤之道。B.穷冬烈风,大雪深数尺,足肤皲(jūn)裂而不知。C.当余之从师也,负箧(jiā)曳屣(lǚ)行深山巨谷中。D.余朝京师,生以乡人子谒(è)余,撰长书以为贽(zhì)。答案B A.冠guān。C.箧qiè,屣xǐ。D.谒yè。······ 2.下列句中加点的词解释有误的一项是(  )A.无从致书以观(得到)  媵人持汤沃灌(热水)B.弗之怠(不)走送之(行走)C.援疑质理(引、提出)缀公卿之后(跟随)D.俟其欣悦(等待)或遇其叱咄(有时)答案B“走”的意思为跑。········ 3.用现代汉语翻译下列语句。(1)以是人多以书假余。译文:(2)故余虽愚,卒获有所闻。译文:(3)凡所宜有之书,皆集于此,不必若余之手录,假诸人而后见也。译文: 答案(1)因此人们大多都会把书借给我。(2)所以,我虽然愚钝,(但是)最终有所收获。(3)凡是所应该具备的书籍,都集中在这里,不必再像我这样用手抄录,向别人借然后才能看到。解析注意句式的特点,重点实词和虚词的翻译要准确,如(1)中的两个“以”,(2)中的“卒”(最终),(3)中的“诸”(之于)。 4.根据要求默写。(1)揭示作者小时候学习就很勤奋的句子是:。(2)对太学生中的其他人“烨然若神人”,作者却“略无慕艳意”的原因是:,。答案(1)余幼时即嗜学 (2)以中有足乐者 不知口体之奉不若人也解析(1)由题干提示“小时候”“勤奋”可自然想到课文首句。(2)“略无慕艳意”之后的两句话就是原因。 1.下列各组句子中加点词的意义和用法不同的一项是(  )A.以衾拥覆以刀劈狼首B.当余之从师也  无鲜肥滋味之享C.俟其欣悦  亲旧知其如此D.足肤皲裂而不知  鸣之而不能通其意答案B A.“以”都是介词,“用”的意思。B.第一个“之”用在主谓之间,取消句子的独立性;第二个“之”是结构助词,的。C.“其”都是代词,“他”的意思。D.“而”都是表转折关系的连词,“却”的意思。········能力提升全练 2.下列句子中“以”字的用法与例句相同的一项是(  )例:余故道为学之难以告之A.无从致书以观   B.以衾拥覆C.人多以书假余  D.以中有足乐者答案A A.与例句中“以”一样,都解释为“来”。B.用。C.把。D.因为。 3.下面句子朗读节奏划分有误的一项是(  )A.余/因得/遍观群书。B.不知/口体之奉/不若人也。C.又患/无硕师/名人与游。D.尝趋百里外/从乡之先达/执经叩问。答案C 又患无/硕师名人/与游。 4.下列每句话都体现作者学习生活艰苦的一组是(  )①家贫,无从致书以观,每假借于藏书之家,手自笔录,计日以还。②天大寒,砚冰坚,手指不可屈伸,弗之怠。③或遇其叱咄,色愈恭,礼愈至。④故余虽愚,卒获有所闻。⑤穷冬烈风,大雪深数尺,足肤皲裂而不知。⑥寓逆旅,主人日再食,无鲜肥滋味之享。A.①②③④⑤    B.②③④⑤⑥C.①②④⑤⑥    D.①②③⑤⑥答案D 可采用排除法,④句写的是作者向人求学后的结果,而非生活的艰苦,这样直接将A、B、C三项排除掉。 5.作者谈及青少年时期“不知口体之奉不若人也”时,运用外貌描写,分别描述了自己和“同舍生”的服饰,这是怎样的写法?他借此赠给马生的精神礼物是什么?答:答案对比(或“反衬”)。锦衣玉食只是外在肤浅的享受,知识才是人生的至味。解析作者着力描写富家子弟衣饰的华美,目的是反衬自己粗劣的服饰,用自己俭朴的生活进行对比,进一步突出了文章的中心。 1.下面加点字解释完全正确的一项是(  )A.礼愈至(周到)     持汤沃灌(菜汤)B.预君子之列(预备)  岁有裘葛之遗(给予,赠送)C.缊袍敝衣(短的)  又患无硕师名人与游(担心)D.俟其欣悦(等待)腰(在腰间佩戴)白玉之环答案D A.汤:热水。B.预:参与。C.缊:乱麻。········ 2.下列句中加点的“之”字用法不同于其他三项的一项是(  )A.当余之从师也   B.岂他人之过哉C.戴朱缨宝饰之帽  D.坐大厦之下而诵诗书答案A B、C、D三项中“之”为结构助词,相当于“的”。A项中“之”是助词,用在主谓之间,取消句子的独立性。···· 3.下列加点词古今意义没有明显变化的一项是(  )A.不敢稍逾约  B.主人日再食C.录毕,走送之  D.媵人持汤沃灌答案AA.古今义都为“超过”。B.古义为“两次”,今义为“又一次”。C.古义为“跑”,今义为“人或鸟兽的脚交互向前移动”。D.古义为“热水”,今义为“食物煮后所得的汁水”。···· 4.以下加点词没有活用的一项是(  )A.手自笔录  B.渔人甚异之C.先达德隆望尊  D.主人日再食答案C A.名词做状语,用笔。B.形容词的意动用法,对……感到诧异。D.名词做状语,每天。···· 5.下列对文章内容理解不正确的一项是(  )A.常常手抄借来之书,隆冬时节,十指冻僵时仍不懈怠,足见“我”求学之“勤”。B.遇师叱咄,表情更恭敬,礼节更周到,没有争辩一句,足见“我”求学之“诚”。C.寒冬大雪,身背书箱,拖着鞋子,在深山巨谷中行走,足见“我”求学之“艰”。D.处于身着华丽服饰的同学之间,虽然羡慕,却坚守心中之乐,足见“我”求学之“精”。答案D 处于身着华丽服饰的同学之间,毫无羡慕之意,足见“我”求学之“乐”。 6.结合课文,谈谈你从作者的读书经历中悟出了哪些学习秘诀。答:答案(示例)①学习要勤奋;②求师要诚心;③求学必须不断克服困难。解析还可以从其他角度谈,只要能扣住文章即可。 文言文阅读(南开中学中考模拟)送东阳马生序(节选)余幼时即嗜学。家贫,无从致书以观,每假借于藏书之家,手自笔录,计日以还。①天大寒,砚冰坚,手指不可屈伸,弗之怠。②录毕,走送之,不敢稍逾约。以是人多以书假余,余因得遍观群书。既加冠,益慕圣贤之道。又患无硕师名人与游,尝趋百里外,从乡之先达执经叩问。先达德隆望尊,门人弟子填其室,未尝稍降辞色。余立侍左右,援疑质理,俯身倾耳以请;或遇其叱咄,色愈恭,礼愈至,不敢出一言以复;俟其欣悦,则又请焉。故余虽愚,卒获有所闻。当余之从师也,负箧曳屣行深山巨谷中。穷冬烈风,大雪深数尺,足肤皲裂而不知。至舍,四支僵劲不能动,媵人持汤沃灌,以衾拥覆,久而乃和。寓逆旅,主人日再食,无鲜肥滋味之享。同舍生皆被绮绣,戴朱缨宝饰之帽,腰白玉之环,左佩刀,右备容臭,烨然若神人;余则缊袍敝衣处其间,略无慕艳意,以中有足乐者,不知口体之奉不若人也。盖余之勤且艰若此。今虽耄老,未有所成,犹幸预君子之列,而承天子之宠光,缀公卿之后,日侍坐备顾问,四海亦谬称其氏名,况才之过于余者乎? 3.选择文中恰当的画线句,将其序号填写到对应横线上。作者家贫无书,却能“遍观群书”,离不开他“”的勤奋刻苦精神,离不开他“”的诚信守时品质。4.阅读选文结尾句,分析作者这样写的用意。5.学完本文后,小强同学想结合文章内容写一副对联,请你使用以下短语帮他将对联补充完整。圣贤之道德隆望尊色恭礼至君子之名上联:先达传。上联:弟子成。①②勉励马生要珍惜眼前优越的学习条件,以读书为乐,勤奋学习。德隆望尊圣贤之道色恭礼至君子之名 (育才中学中考模拟)①余幼时即嗜学。家贫,无从致书以观,每假借于藏书之家,手自笔录,计日以还。天大寒,砚冰坚,手指不可屈伸,弗之怠。录毕,走送之,不敢稍逾约。以是人多以书假余,余因得遍观群书。既加冠,益慕圣贤之道。又患无硕师名人与游,尝趋百里外,从乡之先达执经叩问。先达德隆望尊,门人弟子填其室,未尝稍降辞色。余立侍左右,援疑质理,俯身倾耳以请;或遇其叱咄,色愈恭,礼愈至,不敢出一言以复;俟其欣悦,则又请焉。故余虽愚,卒获有所闻。②当余之从师也,负箧曳屣行深山巨谷中。穷冬烈风,大雪深数尺,足肤皲裂而不知。至舍,四支僵劲不能动,媵人持汤沃灌,以衾拥覆,久而乃和。寓逆旅,主人日再食,无鲜肥滋味之享。同舍生皆被绮绣,戴朱缨宝饰之帽,腰白玉之环,左佩刀,右备容臭,烨然若神人;余则缊袍敝衣处其间,略无慕艳意,以中有足乐者,不知口体之奉不若人也。盖余之勤且艰若此。 8.作者为什么“不敢出一言以复”?这是写老师发怒时他求教的情景,作者“不敢出一言以复”是因为作者对师者的尊重与敬服,而且体现作者对求学的渴望,求学态度的诚恳。 (西大附中中考模拟)余幼时即嗜学。家贫,无从致书以观,每假借于藏书之家,手自笔录,计日以还。天大寒,砚冰坚,手指不可屈伸,弗之怠。录毕,走送之,不敢稍逾约。以是人多以书假余,余因得遍观群书。既加冠,益慕圣贤之道。又患无硕师名人与游,尝趋百里外,从乡之先达执经叩问。先达德隆望尊,门人弟子填其室,未尝稍降辞色。余立侍左右,援疑质理,俯身倾耳以请;或遇其叱咄,色愈恭,礼愈至,不敢出一言以复;俟其欣悦,则又请焉。故余虽愚,卒获有所闻。当余之从师也,负箧曳屣行深山巨谷中。穷冬烈风,大雪深数尺,足肤皲裂而不知。至舍,四支僵劲不能动,媵人持汤沃灌,以衾拥覆,久而乃和。寓逆旅,主人日再食,无鲜肥滋味之享。同舍生皆被绮绣,戴朱缨宝饰之帽,腰白玉之环,左佩刀,右备容臭,烨然若神人;余则缊袍敝衣处其间,略无慕艳意。以中有足乐者,不知口体之奉不若人也。盖余之勤且艰若此。 12.文中“穷冬”“烈风”“大雪”属于什么描写方法,有什么作用?环境描写。通过描写恶劣的环境,从侧面烘托了作者从师求学的旅途之艰。(侧面表现了作者不怕吃苦的精神。) 非连续性文本阅读(南开中学中考模拟)【材料一】央视网消息:2019年世界移动通信大会于2月28日在西班牙巴塞罗那落下帷幕,本届大会吸引了将近11万名参观者和2400多家企业,5G(第五代移动通信技术)是出现频率最高的关键词。中国5G领域的成果引发了广泛的关注,华为产品和技术广受好评,获多个奖项,多国企业与华为签订了合作协议。华为表示,在全球已和超过30家运营商签订了5G商用合同,5G基站全球发货量超过4万个,并持有2500多项5G专利。英国电信巨头沃达丰集团首席执行官里德在新闻发布会上说,禁用华为设备只会延迟欧洲5G时代的到来。如果不与华为合作,“不仅我们行业,而且国家的基础设施都会处于不健康的状态”。分析认为,各国纷纷对中企5G技术打开大门,正是对技术革新潮流,对互利共赢潮流的认同。在贸易自由化、经济全球化的大势面前,敞开胸怀,拥抱合作,将让人类技术的进步之路更加通畅,让人类共享更多互利共赢的成果。 【材料二】5G改变生活5G上网。每秒传输10GB,相当于1秒钟能下载一部高清电影,速度是4G网络的100倍……从4G到5G的升级,不仅是技术升级,更意味着用户上网体验的大升级。用户可以轻松在手机上看高清电影,玩VR(虚拟现实)游戏更加流畅。再如同样是通话,4G是视频,5G则可以实现VR立体场景体验,更加真实。5G出行。湖北一司机对无人驾驶技术既兴奋又忐忑。一方面,司机变乘客,解放了双手;另一方面,把生命安全拱手交给看不见、摸不着的人工智能,总是不太放心。今年3月,中国电信、中兴通讯、百度公司在雄安新区完成国内首个基于5G网络实况环境下的无人驾驶车测试。测试中,无人车转向、加速、刹车不在话下。未来,无人车还将针对红绿灯信息、天气信息、路面情况处理等接受一系列测试。5G家居。可以提升智慧生活家居体验。如人在室外便可用手机等设备远程操控家里的电饭煲、洗衣机、电视机等。业内人士指出,5G网络地址数量大,允许每个人都拥有很多台物联网设备,5G网络的功耗又相对较低,能保障较长的续航时间,加之网络延迟小、出错概率小,因此为智能家居等物联网发展提供了技术保障。 5G军事。首先是信息技术在作战体系中的延伸,军机、军舰之间数据链信息交互。一个安全稳定、传输速度极快的数据链网络,能让每一个士兵都通过5G技术实现人员间的互联,人员与武器间的互联,真正达到人器合一,上下指挥合一,精准军事行动指向,整个部队的作战潜力将得到极大的飞跃。【材料三】对5G未来的思考铺设5G网络,需要大量基础设施投入。为了获得更快传输速率,需要建设数量更多、密度更大的5G基站,基站设备、天线选址建设、优化调试等方面,都需要比4G高得多的资本投入。对于用户来说,目前的主流手机均不支持5G,用户想“尝鲜”,先得换一台新手机。预计我国将在2019年下半年生产出第一批5G手机。想要升级换代,还得耐心等待。5G时代的观影,为了高效运营可以通过大数据比对实现同场次电影差异收费。比如一部悬疑电影收A消费者10元,收B消费者 5元,原因是数据分析显示A爱看悬疑而B不爱。网友表示同一部电影,相同的场次,利用个人数据显示的喜好多赚钱是否公平?且爱不爱看悬疑电影还涉及个人隐秘。还有用户担心,到了5G时代,很多智能家居由手机操控。如果发生手机丢失、手机信息泄露,引发一系列连锁反应,后果不堪设想。在这个技术日新月异的时代,如何让技术更好地服务生活,值得我们每一个人思考。 1.《今日新昌》这份报纸想要转载材料一,还缺标题,请你帮忙拟一个。(不超过15个字)2.下列理解和分析不符合原材料意思的一项是()A.4G网络每秒传输0.1GB,是5G传输速度的1%,从4G到5G的升级,不仅是技术升级,更意味着用户上网体验的大升级。B.对于用户来说,目前的手机均不支持5G,用户想体验5G,得等到2019年下半年换一台新手机才行。C.如果基站数量少,密度小,网络传输速率就慢,网络延迟,很难提升智能家居的使用体验。D.华为的5G产品不仅对英国的电信行业产生影响,还会对英国的基础设施产生影响。示例一:中企技术扬大会,5G革新成趋势示例二:5G让世界共赢互利B 3.5G时代即将来临,有人赞成,有人反对,请结合三则材料,谈谈你的看法。4.5G手机即将走进我们的生活,请你用最通俗易懂的语言,合适的说明方法(至少一种)向你的爷爷介绍5G手机的优势,让他跟上时代的步伐。示例:赞成。①5G技术将成为未来发展的必然趋势。②5G对未来生活全方位的促进。虽然有些技术有待完善和发展,但5G技术一定能更好地服务我们的生活。示例:爷爷,5G手机来了,这种手机下载速度非常快,本来下载一部电影要几分钟,但5G手机就只要几秒,而且通话效果更好,以后我和你打电话就像我直接站在你面前一样。5G手机还可以操控家里的电器,人在室外就可以通过手机操控家里的电饭煲、电视机、洗衣机。5G手机这么方便,爷爷您一定要学着用噢! 夏天是多姿多彩的。池塘里的荷花,像一个个害羞的小姑娘红着脸,随着柔风翩翩起舞。红红的西瓜瓤,像一块红玛瑙。夏天是快乐的。白天可以与朋友玩耍,夜晚听蟋蟀断断续续的叫声,听风儿把树叶刮得沙沙作响。夏日的天空是美丽的。白天,云淡风轻,云姑娘穿着漂亮的衣裙,好像在欢迎我们来参观天空。夜晚,点点繁星,在天空中发出微弱的光芒,月亮似乎在向我们微笑。美文欣赏·夏天 部编版九年级语文上册教学课件第三单元12.词四首 日积月累 课文助读走近作者范仲淹(989—1052),字希文,汉族,苏州吴县人。北宋杰出的思想家、政治家、文学家。有《范文正公文集》传世。苏轼(1037—1101),字子瞻,又字和仲,号东坡居士,世称苏东坡、苏仙。北宋著名文学家、书法家、画家。其散文著述宏富,豪放自如,与辛弃疾同是豪放派代表,并称“苏辛”。辛弃疾(1140—1207),原字坦夫,后改字幼安,号稼轩,南宋豪放派词人。有词集《稼轩长短句》传世。秋瑾(1875—1907),女,中国女权和女学思想的倡导者,近代民主革命志士。 写作背景宋康定元年至庆历三年间,范仲淹任陕西经略副使兼延州知州,镇守西北边疆,这首《渔家傲·秋思》就是他身处军中的感怀之作。《江城子·密州出猎》这首词作于公元1075年(神宗熙宁八年),作者在密州(今山东诸城)任知州。这是宋人较早抒发爱国情怀的一首豪放词,在题材和意境方面都具有开拓意义。 《破阵子·为陈同甫赋壮词以寄之》这首词作于作者失意闲居信州(今江西上饶)之时。辛弃疾二十一岁时,参加了抗金起义。起义失败后,他回到南宋,当过许多地方的长官。他极力主张收复中原,却遭到排斥打击。后来,他长期不得任用,闲居近二十年。《满江红(小住京华)》此词是在词人与丈夫王廷钧吵架后所作。这年秋瑾三十岁,有感于民族危机,抛家别子,女扮男装,东渡日本,去追求别样的人生,去寻求民族振兴的道路。 主要内容《渔家傲·秋思》上片描摹出了一幅寥廓荒僻、萧瑟悲凉的边塞鸟瞰图;词的下片则抒发边关将士壮志难酬和思乡忧国的情怀。《江城子·密州出猎》是苏轼于密州知州任上所作的一首词。此词表达了强国抗敌的政治主张,抒写了渴望报效朝廷的壮志豪情。《破阵子·为陈同甫赋壮词以寄之》作者通过对早年抗金部队豪壮的阵容和气概以及自己沙场生涯的追忆,表达了作者杀敌报国、收复失地的理想,抒发了壮志难酬、英雄迟暮的悲愤心情。《满江红(小住京华)》反映了词人在封建婚姻家庭和旧礼教的束缚中,走向革命道路前夕的苦闷彷徨和雄心壮志的开阔胸襟。 基础过关 5.(重庆八中中考模拟)下列句子没有语病的一项是()A.课余时间我们要尽可能地多读书,因为广泛的课外阅读是提高学生语文素养的关键。B.为预防泥石流灾害之后不出现大疫情,四川省政府及卫生防疫部门采取了许多的措施。C.自从李娜参加了学校健美操队后,健美操就对李娜产生了浓厚的兴趣。D.徜徉在浩瀚的知识海洋中,使我们感到自己的深深不足。A 6.鲁迅先生在谈到《儒林外史》时说:“凡官师,儒者,名人,山人,间亦有市井细民,皆现身纸上,声态并作,使彼世相,如在目前。”课文中,现身纸上的儒者是,市井细民是,当时世态百相呈现在读者面前最集中的场面是。范进胡屠户范进中举后他家门前的场面 1.给下列加点的字注音。人不寐(   ) 右擎苍(   ) 锦帽貂裘(   )鬓微霜(   )麾下炙(   )  殊未屑(   )答案mèi qíng qiúbìn huī xiè解析注意“擎”字读二声。另外注音时要注意字母书写及声调标注的规范性。······拓展训练 2.默写填空。(1)四面边声连角起,,。(2)老夫聊发少年狂,,,,。(3)醉里挑灯看剑,。八百里分麾下炙,,。(4)算平生肝胆,。?答案(1)千嶂里 长烟落日孤城闭 (2)左牵黄 右擎苍 锦帽貂裘 千骑卷平冈 (3)梦回吹角连营 五十弦翻塞外声 沙场秋点兵 (4)因人常热 俗子胸襟谁识我解析注意字的正确写法,如“嶂”“苍”“锦”“冈”“襟”等。避免写成同音字或形近字。 3.下列作家均为同一朝代的一项是(  )①辛弃疾 ②范仲淹 ③温庭筠 ④柳宗元 ⑤苏轼 ⑥白居易A.①②④  B.①③④  C.③④⑥  D.②③⑤答案C 辛弃疾为南宋词人,范仲淹是北宋文学家,③④⑥同为唐代诗人,苏轼是北宋词人、文学家。 1.下面诵读节奏划分正确的一项是(  )A.俗子/胸襟谁/识我?B.浊酒/一杯/家万里,燕然/未勒/归无计。C.八百里/分麾/下炙,五十弦/翻/塞外声。D.酒酣胸胆/尚开/张。鬓微/霜,又/何妨!答案B正确的节奏划分应为:A.俗子胸襟/谁/识我?C.八百里/分/麾下炙,五十弦/翻/塞外声。D.酒酣胸胆/尚/开张。鬓/微霜,又/何妨!能力提升全练 2.根据对诗词的理解填空。(1)范仲淹的《渔家傲·秋思》中表现边地荒凉的句子是,。抒发征夫戍边难归的无奈和对家乡的眷念之情的句子是,。(2)苏轼的《江城子·密州出猎》中,抒发主人公杀敌报国、守卫边疆的坦荡胸怀和豪情壮志的句子是,,。(3)范仲淹的《渔家傲·秋思》中,与王维的“大漠孤烟直,长河落日圆”意境相似的句子是。(4)秋瑾在《满江红》中写自己虽有凌云壮志,但知音难觅,不觉泪湿衣襟的句子是?! 答案(1)塞下秋来风景异 衡阳雁去无留意 浊酒一杯家万里 燕然未勒归无计 (2)会挽雕弓如满月 西北望 射天狼 (3)长烟落日孤城闭(4)莽红尘何处觅知音 青衫湿解析本题考查理解默写能力。根据篇名和提示语中的关键词句以及标点符号确定所填词句。提示语中的关键词句有“边地荒凉”“戍边难归”“对家乡的眷念”“杀敌报国”“意境相似”“知音难觅”“泪湿衣襟”等。 3.请分别写出下列词句中词人借“酒”来表达的思想感情。(1)《渔家傲》中“浊酒一杯家万里”。(      )(2)《江城子》中“酒酣胸胆尚开张”。(      )(3)《破阵子》中“醉里挑灯看剑”。(      )答案(1)思乡之情 (2)胸宽胆壮 (3)渴望从军杀敌解析本题考查词句中蕴含的思想感情。 1.中国古代诗人常运用多种手法增强表达效果。下列诗句中,没有运用拟人修辞手法的一项是(  )A.轻风扶细柳,淡月失梅花。B.待到山花烂漫时,她在丛中笑。C.马作的卢飞快,弓如霹雳弦惊。D.草树知春不久归,百般红紫斗芳菲。答案C C项采用的修辞手法是比喻。 2.下列说法正确的一项是(  )A.秋瑾,别署鉴湖女侠,中国民主革命烈士,对妇女解放运动的发展起到了巨大的推动作用。B.《渔家傲·秋思》的作者是北宋时期的范仲淹,我们还学过他的《醉翁亭记》。C.《江城子·密州出猎》的作者是苏轼,他和哥哥苏洵、父亲苏辙同在唐宋八大家之列。D.《破阵子·为陈同甫赋壮词以寄之》的作者是北宋抗金将领辛弃疾,他和词中所提到的陈同甫同为抗战派。答案A B.《醉翁亭记》的作者是欧阳修。C.父亲苏洵、弟弟苏辙。D.辛弃疾是南宋人。 3.学校准备举办一次中国古典诗词朗诵活动,你认为下面四种字体哪一种最适合作宣传海报的标题用字?请先指出你所选的是哪一种书法字体,然后简要说明你选择它的理由。我选,这是字体。我选择它的理由是: 答案(示例1)A 楷书 该字体形体方正,书写工整规范,与我国古典诗词讲究形式整齐(对仗工整、结构精致)的特点相一致,且易于辨认,适于宣传用字。(示例2)B 草书 该字体笔画连带,流畅飘逸(灵动洒脱/如行云流水/更具有艺术性),与诗词的浪漫情怀相得益彰。(示例3)C 篆书(或“小篆”) 该字体笔画繁复(采用繁体字书写),古朴雅致,能体现古典诗词的厚重典雅和汉字文化的博大精深。(示例4)D 隶书 该字体形体扁平,庄重大气(笔画顺势变化,一波三折),字体美观,适合宣传用字,且与诗词整齐之中不乏变化(讲究格律而不拘泥于格律)的特点相呼应。解析选择自己喜欢的字体,并从其笔画特点,给人的感受等角度阐明理由即可。 4.(2018山东泰安中考,18—20)阅读下面的文字,完成(1)—(3)题。渔家傲·秋思范仲淹塞下秋来风景异,衡阳雁去无留意。四面边声连角起,千嶂里,长烟落日孤城闭。  浊酒一杯家万里,燕然未勒归无计。羌管悠悠霜满地,人不寐,将军白发征夫泪。(1)(★★☆)下列最能体现塞下秋天季节特点的一项是(  )A.衡阳雁去    B.四面边声C.长烟落日    D.孤城紧闭(2)(★★☆)下列对词的赏析错误的一项是(  )A.词人思念万里之外的家乡,却没有办法回去,“浊酒一杯”怎能排解对家乡亲人的思念之情? B.“燕然未勒”是借用东汉大将窦宪燕然勒石记功而还的典故,这里用来表现将士们建功立业的壮志和决心。C.“人不寐,将军白发征夫泪”,在悲怆的情调中,鲜明地表达了词人对朝廷腐朽软弱的控诉和批判。D.全词意境悲凉壮阔,形象鲜明生动,感情真挚深沉,语言质朴凝练,读来真切感人。(3)(★★☆)下列诗句中与“羌管悠悠霜满地”使用的表现手法相同的一项是(  )A.宁为百夫长,胜作一书生。(杨炯《从军行》)B.田家少闲月,五月人倍忙。(白居易《观刈麦》)C.无为在歧路,儿女共沾巾。(王勃《送杜少府之任蜀州》)D.露从今夜白,月是故乡明。(杜甫《月夜忆舍弟》) 解析(1)首句“塞下秋来风景异”点明地域是边塞,季节是秋天。词人特地用了一个“异”字,以统领全词景物的特点。接下来具体描写塞下秋景之“异”,即“衡阳雁去无留意”,在秋天,边塞的大雁过早地向衡阳飞去,毫无稍事逗留之意。故A项“衡阳雁去”最能体现塞下秋天季节特点。(2)“人不寐,将军白发征夫泪”,意思是将军和征夫都难以入睡,因守边辛苦,思念家乡,将军和征夫白了头发,流下了眼泪。这里的悲怆情调重在“思乡思亲”方面,对朝廷腐朽、软弱,不修武备、不重边功的愤懑不平表达较为含蓄。因此C项中“鲜明地表达了词人对朝廷腐朽软弱的控诉和批判”有误。答案(1)A (2)C (3)D (3)首先分析例句,“羌管悠悠霜满地”表现了边地的寒冷,侧面表达了戍守将士们艰苦的生活,渲染了一种悲壮的气氛,同时表达了词人的思乡之情。据此可以确定例句使用的是寓情于景的表现手法。D项“露从今夜白”,既写景,也点明时令;“月是故乡明”,也是写景,突出诗人深切的思乡之情。所以D项使用的也是寓情于景的表现手法。故选D。 阅读提高(川外附中中考模拟)形神兼备中国画①中国画历史悠久,在中国数千年的绘画发展史上,画家辈出,名作荟萃,在世界美术领域绽放出璀璨的光辉。②中国画有整套独特的表现手法,不追求形象的逼真,而着力于意境的表现,要求做到以形写神,形神兼备。中国画主要以笔墨纸砚为工具,有“书画同源”之说。中国画的颜料由天然的植物或矿物制成。画作以水墨为主体色,也因常用朱红色和青色,所以又称为“丹青”。③中国画按照题材主要分为以下三类。④人物画是以人物为主体的绘画。从传世的人物画作中,后人可以一窥中国古代帝王将相的面容,还能看到仕女图表现的宫廷生活和风俗画中展现的市井百态等。这些画作的人物形象惟妙惟肖,注重表现人物的精神风貌。例如明代曾鲸的代表作《王时敏像》,画作中人物画得不大,线条流畅遒劲,形象栩相如生;背景则大幅留白,突出了人物形象;同时配以书法题字,更彰显了人物的精神气度。 ⑤山水画是以山川自然景观为主要对象的画作。中国人自古以来就认为山水蕴含着天地宇宙间无穷的奥妙,由此形成了独到的山水观。中国的山水画常常运用散点透视法,利用远近、高低的对比,营造出江山万里、山谷深邃的立体效果,极具空间感。例如宋代山水画的代表作《千里江山图》,这是一幅恢宏壮丽、色彩斑斓的巨幅山水作品,画卷长11.915米、高0.515米,画幅几乎是同时期《清明上河图》的两倍。此画以略带俯视的角度横向展开全景式的大山大水,呈现出一派万象皆容的场景。画卷采用散点透视法,远近距离分明,空间层次清晰。在色彩表现上,该画以色调浓重的矿物颜料绘成,较同时期流传下来的其他青绿山水画作都要鲜艳。全画除青绿主色外,还以赭石来渲染山脚、天色,并以笔墨线条皴染山石,使山峰的立体感和明暗效果格外鲜明生动。元代书法家、鉴赏家溥光和尚在此画题跋中称其“独步千载,殆众星之孤月也”。⑥花鸟画主要以花草树木、禽鸟鱼虫为题材。花鸟画表现的是边、角、点的小景,注重笔情墨趣。中国花鸟画并不只是纯粹的写生,而是紧紧抓住动植物与人的关系,表达画家的思想情感。所以,中国花鸟画重视形似但不拘泥于形似,更崇尚意境与情趣。例如南宋赵孟坚的《春兰图》,描绘了两株生于杂草丛间的幽兰, 摇曳多姿,舒展飘逸。兰叶、花瓣均以淡墨一笔绘出,笔法潇洒灵动、柔中带刚,画面旷淡清幽、意蕴无穷。画中左方题诗一首,以兰喻人。全图诗、书、画、印完美结合,表现了画家洁身自好、清高脱俗的追求。⑦中国画体现出中华民族的哲学思想。人物画表现的是人类社会的生活风貌,山水画表现的是人与自然的关系,花鸟画则表现大自然的各种生命状态。三者的结合,构成了中国人对人生和宇宙的整体认识。画家们在创作时会自觉体现“天人合一”的观念,这是中国人对哲学思考的艺术表现,也体现了艺术的真谛。⑧中国画是一门综合性的艺术。它将诗、书、印等元素与画结合,这些元素丰富了画面的内容,提升了图画的境界。中国画有壁画、屏风、卷轴、册页、扇面等多种画幅形式,并以特有的装裱工艺装潢画幅,给人带来更多的审美感受。⑨领略中国画的魅力,不仅要欣赏画面,还要从古人的妙笔中感受他们的性情与哲思。(节选自《中国画》,有删改) 1.请根据选文内容填空。中国画按照题材主要分为A、、C三类。2.选文第④段画线句使用了哪种说明方法?有何作用?人物画山水画花鸟画画线句使用了举例子的说明方法,举了明代曾鲸的《王时敏像》的例子,具体真切地说明了人物画人物形象惟妙惟肖,注重表现人物精神风貌的特征。 3.下列对选文内容理解不正确的一项是()A.中国画主要以笔墨纸砚为工具,以水墨为主体色,也常用朱红色和青色,又称为“丹青”。B.宋代《千里江山图》是一幅巨幅山水作品,画幅是同时期《清明上河图》的两倍。C.中国花鸟画注重笔情墨趣,重视形似但不拘泥于形似,更崇尚意境与情趣。D.选文是一篇事物说明文,采用逻辑顺序,思路清晰、结构严谨地介绍了中国画。B 示例一:可以增添与梅花有关的诗句,诗句与梅花相得益彰,不仅提高了扇面的观赏性,且以梅喻人,还体现出人的精神追求。再盖上印章,将诗、书、画、印结合起来,可以丰富画面的内容,提升图画的境界。示例二:可以增添“凌寒独自开”这句诗,并盖上印章。这句诗能表现出梅花不畏严寒的品质,诗与画融为一体,突显出人的高洁品格。题诗盖印,使画面更丰富,意境更深远。 池塘里,告别了苞蕾的荷花,绽放着清丽的笑靥,在轻风拂送下,舞动着叠翠的裙裾,婷婷地妩媚着矜持的身姿,凝雾噙露着似若一婉约的女子在轻轻吟诵“棹拂荷珠碎却圆”的诗句,隽秀的枝干无不在向你展示着夏的风情,不由得让你在赞叹那绝佳的韵致间。于蓦然回首中,已是“接天莲叶无穷碧,映日荷花别样红”了。美文欣赏·夏天 部编版九年级语文上册教学课件第四单元13.短文两篇 日积月累 课文助读走近作者培根(1561—1626),英国哲学家、作家。培根的主要建树在哲学方面,《随笔》是培根在文学方面的主要著作。马南邨(1912—1966),原名邓拓,福建福州人,当代作家。他的作品涉猎很广,杂文独树一帜,有针砭时弊的批判性,并且把知识、趣味熔于一炉,具有学者杂文的独特魅力。 作品简介《谈读书》于1653年发表,收于《培根随笔》一书中,是其中一篇重要的文章。《培根随笔》是英国随笔文学的开山之作,以其简洁的语言、优美的文笔、透彻的说理、迭出的警句,在世界文学史上占据了非常重要的地位。内容主要是一些议论性质的短文,主要讲叙培根在不同的角度看待事物的态度和想法。《不求甚解》选自《燕山夜话》。《燕山夜话》是马南邨的杂文集。主要是知识性的杂文,内容丰富,涉及古今中外、天文地理;旗帜鲜明,议论风生,也能切中时弊;短小精练,语言流畅。 主要内容《谈读书》语言十分简练,大体上可以分为三个层次:第一,从开头到“全凭观察得之”,阐述读书的正确目的;第二,从“读书时不可存心诘难作者”到“始能无知而显有知”,阐述读书的方法;第三,从“读史使人明智”到结尾,阐述读书能塑造人的性格和弥补精神上的各种缺陷。《不求甚解》作者认为,“不求甚解”有两层意思:一是表示虚心,因为书不一定都能读懂,就承认“不求甚解”。二是说明读书的方法,不要固执一点,咬文嚼字,而要前后贯通,了解大意。 基础过关 4.(川外附中中考模拟)依次填入下列横线处的句子,语序恰当、语意连贯的一项是()“浅阅读”对于我们增长学问、开阔眼界无疑起到促进作用。,,,。,它使知识向精、深、专方向挺进。“深阅读”对传统文化的传承和国民素质的提升更加重要。①而“深阅读”就是深度阅读,它讲究反复咀嚼、品味、思考②但是,浅阅读本身存在无法克服的缺点和不足③那么对于国家和民族将是灾难性的④如果仅限于浅阅读,过分热衷于“浅阅读”⑤表现为走马观花,浅尝辄止,泛泛而读A.①②④⑤③B.②①④③⑤C.⑤④③①②D.②⑤④③①D 5.(重庆一中中考模拟)综合性学习。今年的10月1日是我们伟大祖国70华诞,为此我校九年级(5)班于本周五下午三点在班级举办以“实现中国梦·青春勇担当”为主题的诗歌朗诵会,请你完成以下任务。(1)请你为本次活动设计具体步骤。(2)活动开始前,班主任委派你邀请张校长参加本次活动,你将怎样邀请?示例:准备阶段:发动全班同学搜集或创作以“实现中国梦·青春勇担当”为主题的诗歌;实施阶段:请同学们将搜集或创作的诗歌以朗诵的形式展示;总结阶段:请班主任为本次活动做总结。示例:张校长,您好!我是九年级(5)班的学生,我班将于本周五下午三点在班级举办以“实现中国梦·青春勇担当”为主题的诗歌朗诵会,想邀请您参加,可以吗?谢谢! (3)参加本次活动后,请你写一句座右铭。激励自己在今后的人生道路上不断前行。(要求:不得借用名人名言)示例一:积累点滴知识,努力拼搏进取,将来报效祖国。示例二:青春因追梦而绽放华彩! 1.阅读下面文段,按要求答题。狡(xiá)者鄙读书,无知者(xiàn)读书,唯明智之士用读书,然书并不以用处告人,用书之智不在书中,而在书外,全凭观察得之。读书时不可存心拮难作者,不可尽信书上所言,亦不可只为寻章摘句,而应推敲细思。……读史使人明智,读诗使人A,数学使人B,科学使人C,伦理学使人D,逻辑修辞之学使人善辨:E(1)根据拼音写汉字,并改正加点词中书写有误的字,将它们依次工整地书写在下面的田字格内。拓展训练 (2)从下列选项中给文段中A、B、C、D四处选填合适的词语。(只填序号)选项:①周密 ②庄重 ③灵秀 ④深刻我会选:A.B.C.D.(3)请根据语境给文段中E处选填合适的语句。(  )A.凡有所学,定成性格。B.凡有所学,皆成性格。C.凡有所学,能不成性格?D.凡有所学,皆成性格哉? 答案(1)黠 羡 诘 辩(2)A.③ B.① C.④ D.②(3)B解析(1)“拮”和“诘”、“辨”和“辩”是形近字。(2)注意词语的搭配。要结合各学术领域的特点。(3)E处应是一个总结性的陈述句,据此排除C、D两项;由“皆”字的总结意味可确定选B项。 2.依次填入下面文段横线处,最恰当的两项是(  )“不求甚解”的含义有两层:一是表示虚心,目的在于学者不要骄傲自负……;二是说明读书的方法,不要一点,咬文嚼字,而要前后贯通,了解大意。A.告诫  B.警告  C.劝诫D.劝阻  E.固定  F.固执答案CF这里阐述“不求甚解”包含的两层含义。 1.下列各句不是比喻论证的一项是(  )A.读书费时过多易惰,文采藻饰太盛则矫。B.盖天生才干犹如自然花草,读书然后知如何修剪移接。C.否则书经提炼犹如水经蒸馏,味同嚼蜡矣。D.人之才智但有滞碍,无不可读适当之书使之顺畅,一如身体百病,皆可借相宜之运动除之。答案A A项只是直接进行陈述,不是比喻句,更不是比喻论证。能力提升全练 2.下列对课文理解正确的一项是(  )A.《不求甚解》和《谈读书》两篇短文都是驳论文。B.《谈读书》在运用论证方法时,主要采用道理论证和举例论证。C.《谈读书》一文按照读书的方法、目的和作用三个层次结构全篇。D.“不求甚解”有两层意思:一是表示虚心;二是说明读书的方法,不要固执一点,咬文嚼字,而要前后贯通,了解大意。答案D A.《谈读书》不是驳论文,是立论文。B.《谈读书》一文主要运用了比喻论证和对比论证。C.《谈读书》是按照读书的目的、方法和作用这一顺序写的。 3.仿句练习。[原句]天生才干犹如自然花草,读书然后知如何修剪移接。[仿句]天生才干犹如,读书然后。答案未琢之璞 成稀世之珍解析仿写的关键是找到合适的喻体。 4.一位同学读了培根的《谈读书》,做了一则读书笔记。请你参考①处画线句,帮他在②③处分别填上一个句子来概括后文内容。要求:用自己的话概括,句式不限。①读书价值颇多:足以怡情,足以傅彩,足以长才。②:狡黠者鄙读书,无知者羡读书,唯明智之士用读书。③:书有可浅尝者,有可吞食者,少数则须咀嚼消化。答案(示例)②不同品性之人对待读书的态度亦有所不同 ③不同的书读法亦不同解析②“狡黠”“无知”“明智”是形容人的品性的词,“鄙”“羡”“用”是形容态度的词,应据此概括。③“浅尝”“吞食”“咀嚼”“消化”是谈读书方法的词语,应据此概括。 1.仿照画横线的句子,补写两个句子,使之与画线句构成排比。读书是一个奇妙的过程,可以使软弱的性格变得坚强,,。答案(示例)可以使卑微的生命变得高尚 可以使单调的生活变得多彩可以使浮躁的内心变得平静(任写两个即可)解析仿句应符合“可以使……变得……”这一格式,内容是读书产生的正面作用。 2.仔细观察下面这幅漫画,回答问题。(1)用说明性的语言简要介绍画面的内容。答: (2)请谈谈你对“一本好书,一生财富”的认识。答:答案(1)(示例)画面的主体是一本打开的书,书的封面写有“一本好书”字样,封底写有“一生财富”字样。书的下面露出三双脚,分别代表成年男子、成年女子和一个孩子,他们共同阅读这本书。(2)(示例)一本好书能给人有益的知识,拥有知识就拥有了一生享用不尽的财富。解析(1)要求“用说明性的语言”,就不能写成描述性文字,注意要按照一定的顺序介绍。(2)理解句子,把握关键词“书”“财富”,并准确理解“好书”的深刻含义,在此基础上用通顺、流畅的语言表达出来。 3.品读课内句子,回答问题。读书足以怡情,足以傅彩,足以长才。(1)句中“怡情”“傅彩”“长才”三个词语的位置为什么不能互换?答:(2)这一句运用了什么修辞手法?在全文中起什么作用?答:(3)请你仿照句子,用“……足以……,足以……,足以……”的形式写一句话。答: 答案(1)“怡情”指使心情愉快;“傅彩”指给言辞增添光彩;“长才”指增长才干。这三个词语是按照由浅入深的顺序来排列的,所以不能互换。(2)排比。领起下文的论述。(3)(示例)书法足以让人心平气和,足以完善人的审美观,足以提高人的素质。解析(1)要在理解词义的基础上理清句子内部的逻辑关系。(2)句子或段落在文章的开头,作用一般是领起下文或总领全文。(3)注意格式一致,内容上连贯、由浅入深即可。 阅读提高(重庆八中中考模拟)生命因磨炼而美丽①平心而论,谁也不希望自己的生命经常忍受磨炼——折磨式的历练,哪怕真的是因此可以增加人的美丽,也不会有人欢呼着说“啊,我多么喜欢折磨式的历练呀”。人总是向往平坦和安然的。然而,不幸的是,折磨并不以人的主观愿望为依据,不论人们喜欢与否,它只管我行我素,甚至有时还要强加于人,谁奈它何?②既然如此,人们为什么不让自己振作起来去迎接这挑战呢?③人们为什么不能把它变作某种养分去滋润自己的美丽呢?人们回避磨炼,是因为不想忍受它。当回避不了时,人们又说磨炼原来是可以美丽人生的,两边皆有道理。④避开折磨是生命的最佳选择,一旦躲避不开,就让折磨变作美丽人生的养分,此亦是生命的最佳选择。之所以说此亦是生命的最佳选择,乃是因为人们在陷进折磨时,他们面对的选择不止一个,比如说痛苦、焦灼、失望、迷茫、束手无策或一蹶不振,而这些选择皆是对人生的消沉与颓废。比起这些选择,唯有选择让折磨变作美丽人生的养分,方才算是最佳。 ⑤生命因磨炼而美丽,关键在于人对磨炼认识的角度和深度。应该说,磨炼本身就具有美丽人生的功能,假若由于认识上的原因,反让磨炼把自己丑化了,这就有点雪上加霜的味道了,除了磨炼的起因之外,你只好谁也别怪。鉴于以上原因,所以也并不是说,谁的生命都会因磨炼而美丽的,生丑陋者也大有人在。⑥生命因磨炼而美丽,不仅仅因为生命需要在磨炼中成长;主要在于,磨炼对生命的不可回避性。人群之中,物欲横流,而且方向和力度又不尽相同,谁料得到何时何地就会滋生出一种针对自己的折磨来呢?料不到又必须跟随,跟随又不想使自己一蹶不振地消沉,这样,经过努力,使其转化为对自己有用的能量,就成为人之不选之选。这时候的磨炼对生命来说,已变作美丽的阶梯,虽然阶梯的旁边充满荆棘,但在阶梯尽处却充满鲜花,坦然走过荆棘,就必须置身于另外一重天地。⑦生命因磨炼而美丽,还在于它使人收获了用金钱也买不到的某种负面阅历。人生阅历,正面的居多;人生的教诲,善良的居多,这些东西,都构不成对人生的考验,唯有折磨具备这种特质。不是说“猪圈难养千里马,花盆难栽万年松”吗?为什么会是这样的呢?就是因为其缺乏考验的机会。不光如此,生活中的 其他事情也一样,凡没有接受过考验者,你就很难断言它是否完整和美丽,而这种考验,又不是有计划地出的考试题,它是不期然而然地就横亘在了人的面前,使人猝不及防。由于它的这种突发性质,所以它之于人考验的意味就足得很。经此一番挣扎磨炼,人没有颓废,反而更加精神了,这样的生命不走向美丽还能走向哪里呢?⑧固然,磨炼也是可以丑陋人生的。人生原本还有点美丽,经过数次折磨式的历练之后,非但没有使其成熟和美丽,反倒使它充满痛苦、迷茫、彷徨,甚至使人瞻前顾后,畏首畏尾,唯唯诺诺,没有一点棱角脾气了,这是不是有点丑陋呢?⑨对于这些人来说,所有的磨炼都不能称之为磨炼,而是灾难。总而言之,只要有点挫折和难受,就无不如同灾难临身。什么坐卧不安呀,魂不守舍呀,食不知味呀等,这些消耗情绪的东西就都来了。如此人生,让人如何从废墟中走向美丽呢?一颗心已被灾难二字占满,体会它尚且不够,如何能让他分出心来瞄一眼灾难背后的美丽?所谓的灾难,其本身已使人不堪忍受,再要以此种心态情绪去强化它对人的伤害,这不是越瘸越使棍打了吗?人生难美,是不是就这样被注定了呢?⑩这样对磨炼的感受,实际上大可不必有。⑪退一步说,假若你无力使折磨变作美丽生命的阶梯,却也不该使它变作生命的灾难之门。在美丽与灾难之间,保持个中立的态度如何?即以无所谓的心态来对待它如何?这样做,至少生命不会出现消极现象。 不消极不就说明其中有积极因素吗?这远比把磨炼说作灾难的认识事物的方法要乐观得多。⑫在某些时候,人生的精神财富比物质财富也许显得更重要,人们是不应该对之忽略的,精神财富的获得有许多方法,而不断地经受磨炼是其方法之一,或者说是最重要的方法之一。而人生之美丽与否,首先可看的也就是他的精神财富的多寡,而不是依据他的物质财富的多寡。生命因磨炼而美丽,美就美在此处。⑬不错,人总是希望平坦和安逸的,谁也不想要折磨式的历练,但是它却没有因此而不来。作为被动的承受者,又不想就此妥协,那么,就拿出你的智慧,化腐朽为神奇吧,人生将因此而走向美丽。虽然此举属于被迫的性质,也比无所作为好。歪打正着,亦弥足珍贵。(选自《读者文摘》) 1.下列对第⑤段运用的论证方法判断正确的一项是()A.举例论证对比论证B.举例论证比喻论证C.道理论证比喻论证D.道理论证对比论证2.下列的理解,符合作者观点的一项是()A.磨炼是指折磨式的历练,避开磨炼和选择磨炼都是最佳的选择。B.只要对磨炼认识的角度正确,深度足够,生命就会因磨炼而美丽。C.磨炼是对人生的考验,只有经受得起考验的人生才能美丽和完整。D.人生美丽与否取决于精神财富的多寡,经受磨炼是获取精神财富的方法之一。DC 3.下列对原文论证的分析,不正确的一项是()A.文章开头指出折磨是客观存在的,引出“振作起来迎接挑战”的观点。B.文章⑤⑥⑦段为并列关系,从三个方面论证了生命因磨炼而美丽。C.第⑦段引用谚语论证了如果缺乏考验就难以拥有美丽的人生。D.第⑪段论证了在美丽与灾难之间保持中立态度的意义。C 非连续性文本阅读(南开中学中考模拟)材料一:近年来,一系列的人口问题使计划生育的目标和理论依据都受到质疑。男女出生比例节节升高,从1980年的107.4:100上升到2004年的121.1:100,人口结构问题凸显。目前,我国已成为人口老龄化发展速度最快的国家之一,老年人口比例在不断上升,从2010年的13.3%上升到2014年的15.5%.另外,我国现在的独生子女家庭已经超过1.5亿户,独生子女养老负担沉重,而一些家庭的“失独”不仅成家庭灾难,也成社会之痛。2015年10月29日,党的十八届五中全会决定:坚持计划生育的基本国策,完善人口发展战略,全面实施一对夫妇可生育两个孩子政策,积极开展应对人口老龄化行动。材料二:我国从2016年1月1日起开始实施全面两孩政策。 【人物采访】市民胡女士,儿子刚满1岁,获悉新政策后,她坚定地表示,自己不会再生。“夫妻双方都是上班族,除了经济压力外,要把孩子教育好,太耗精力了。”她说,“儿子出生后,坚持母乳喂养、看早教书培养儿子,虽然父母搭把手,还是觉得精疲力尽。我只想集中精力把儿子培养成才,这样也是对孩子负责,即使想生还是要慎重考虑。”材料三:近年来,随着我国人口年龄结构的变化,育龄妇女人数呈现逐年减少趋势。2017年15—49岁育龄妇女人数比2016年减少400万人,其中20—29岁生育旺盛期育龄妇女人数减少近600万人。同时,随着经济社会发展,我国妇女初婚和初育年龄呈现不断推迟的趋势,妇女生育意愿也有所下降。在上述因素影响下,2017年一孩出生人数724万人,比2016年减少249万人。总的来看,“全面两孩”政策的实施,二孩出生人数的明显增加,在很大程度上缓解了一孩出生数量减少的影响,有利于改善人口年龄结构,促进人口均衡发展。(摘编自网络《“全面二孩”来了,你怎么看》) 1.下列表述,不符合以上材料内容的一项是()A.长期计划生育政策使我国处于低生育水平,男女出生比例节节升高,育龄妇女人数呈现逐年减少趋势。B.“全面二孩”政策的实施,二孩出生人数的明显增加,完全缓解了一孩出生数量减少的影响,促进我国人口均衡发展。C.我国全面实施一对夫妇可生育两个孩子政策,目的是改善人口年龄结构,降低老年人口的比重。D.随着经济社会发展,虽然我国妇女初婚和初育年龄呈现不断推迟的趋势,妇女生育意愿也有所下降。B 2.根据材料一的内容,简要概括国家“全面二孩”政策出台的原因。3.根据材料二的内容简述人们对“全面二孩”政策的态度及生养二胎持慎重态度的原因。①人口老龄化发展迅速;②男女出生比例失调;③养老负担沉重。经济压力和教育子女的压力。 礼物,多美好的一个词。仰望星空,地球是宇宙给人类的礼物;低头凝望,一花一叶,是大自然给世界的礼物;孩子是给父母的礼物;朋友是陪伴的礼物;回忆是时间的礼物。美文欣赏·朗读者 谢谢观看 部编版九年级语文上册教学课件第四单元14.山水画的意境 日积月累 课文助读走近作者李可染(1907—1989),江苏徐州人。中国近代杰出画家、诗人,齐白石的弟子。代表画作有《漓江胜境图》《万山红遍》《井冈山》等。代表画集有《李可染水墨写生画集》《李可染中国画集》《李可染画牛》等。 主要内容《山水画的意境》选自《李可染论文集》。课文为我们简述了山水画的灵魂——意境。他从意境出发告诉了我们什么是意境,怎样获得意境,最后以达到意匠结尾,告诉我们山水画创作的两个关键即“意境”和“意匠”。 基础过关 2.下列词语书写有误的一项是()A.描绘宇宙富丽堂皇金碧辉煌B.惜别惆怅朝朝暮暮浮光掠影C.熟悉赋予万里雪飘朝霞幕霭D.意境感悟身临其境胸有成竹3.写出下列句子所使用的论证方法。(1)毛主席的诗句,意境是很深的。如《十六字令三首》,每一首都是写景,每一字都是说山,但每一首、每一字又都充分表达了人的思想感情。()(2)经历过风暴、雷击,有一棵大树已横倒在地下,像一条巨龙似的,但是枝叶茂盛,生命力强,使人感觉很年轻的样子。()(3)杜甫说“意匠惨淡经营中”,又说“语不惊人死不休”。()C举例论证比喻论证引用论证 5.(重庆一中中考模拟)下列句子中没有语病的一项是()A.唱着毕业的骊歌,回眸初中生活的阴晴雨雪,七彩校园里回响着我们青春的誓言和浓浓的师生情谊。B.中国不仅有发展“一带一路”这一宏大倡议的决心,也有实现这一美好蓝图的能力。C.据业内专家估算,仅在我国,未来5年内大数据人才缺口就将超过130万人以上。D.由于电视剧《都挺好》以真情动人,使它取得了意想不到的成功,深受广大观众赞誉。B 6.(北大附中中考模拟)依次填入下面一段文字横线上的语句,衔接最恰当的一项是()马远画山水布局简妙,他善于对现实的自然景色做大胆地概括、剪裁。。。,。,。这真是“虚实相生,无画处皆成妙境”!①从而更加集中地刻画了渔翁专心于垂钓的神气,也给欣赏者提供了一种渺远的意境和广阔的想象空间②他的名作《寒江独钓图》,只画了漂浮于水面的一叶扁舟和一个独坐在船上垂钓的渔翁③他画山,常画山之一角;画水,常画水之一涯,其他景物也十分简练④然而,就是这片空白表现出了烟波浩渺的江水和极强的空间感,衬托了江上寒意萧瑟的气氛⑤四周除了寥寥几笔的微波之外,几乎全为空白⑥他的山水画,画面上常常留出大片空白,空旷渺漠,意境十分深远A.⑥②⑤①④③B.⑥③②⑤④①C.②⑤④①⑥③D.③⑥②⑤④①D 材料二:随着科技的发展,智能手机在生活中的应用越来越广泛。但是手机导致近视、“搜题”软件成为答卷神器等新闻层出不穷,智能手机的滥用对学生身心健康造成的影响已经成了一个社会问题。2018年9月21日,山东省十三届人大常委会第五次会议审议通过《山东省学生体质健康促进条例》规定:自2018年11月1日起中小学校应当加强学生在校期间电子产品使用管理,指导学生科学规范使用电子产品;严禁学生将个人手机等电子产品带入课堂。这是新时代全国首部为促进学生体质健康而立的省级地方性法规。(1)“材料一”中的这幅漫画是第二十八届中国新闻奖的获奖作品,请你简要指出该作品获奖的原因(不超过70字)。示例一:该作品反映了学生上课看手机的不良现象,借用孔夫子上课收手机的这一题材,角度新颖独特,既有幽默感又有讽刺性,能够引起人们对这种不良现象的深思。示例二:该作品聚焦学生上课看手机这一社会通病,借古喻今,角度独特,引人思考,是一幅将艺术性与思想性加以有机结合的漫画作品。 (2)阅读材料二,你将在“课堂远离手机,促进身心健康”的主题班会上向同学们阐述课堂远离手机的益处,请简要列出两条。示例一:①能够更好地保护视力,有效预防近视;②能够集中精力学习,养成良好的学习习惯。示例二:①能够避免网络不良内容侵害,促进身心健康;②能够维护课堂纪律,提高学习效率。 1.读下面的语段,回答问题。四川人说é眉天下秀,夔(  )门天下险,剑gé天下雄,青城天下yōu这话是有道理的。我们看颐(  )和园风景,则是富丽堂皇,给人金碧辉huáng的印象。(1)给加点字注音。(2)根据拼音写汉字。(3)在中标注恰当的标点符号。(4)解释加点词。富丽堂皇:······能力提升全练 答案(1)kuíyí(2)峨 阁 幽 煌 (3):“。”(4)形容建筑物华丽雄伟,也形容场面华丽而盛大。解析(1)结合语境确定字音。注意声调书写要规范。(2)根据拼音,结合语境确定字形。注意同音异形误写。(3)注意语段中“说”字后面用冒号;“这话”前的句号应在引号内。(4)根据“颐和园风景”及“金碧辉煌”推测词义即可。 2.在下面句中横线处填写恰当的关联词语。在我们的古诗里,往往有很好的意境。关于“人”一句也不写,,通过写景,充分表现了人的思想感情。答案虽然 但是 却解析根据句子内部的逻辑关系确定关联词。“关于‘人’一句也不写”与后面的内容间是转折关系。据此填写后采用通读法验证答案是否正确。 3.在课文中找出作者原话,说说什么是“意境”。答:答案意境就是景与情的结合;写景就是写情。解析细读文章,根据题干的提示“在课文中找出作者原话”作答即可。要注意,既然是问什么是“意境”,答案就应该是个判断句,即“意境是……”。 1.填入下面横线处的词语,与上下文衔接最恰当的一项是(  )在大雪纷飞中,在夕阳残照里,这些绝壁上的松树就像的战士,的渔夫,的隐者,站出了自己的风骨。大自然叩击着我们的心灵,让我们探求忠诚、勇敢、洒脱的真谛。A.搏击风浪  坚守阵地  远离世俗B.坚守阵地  远离世俗  搏击风浪C.坚守阵地  搏击风浪  远离世俗D.搏击风浪  远离世俗  坚守阵地答案C“坚守阵地”意思是“坚决地保卫着阵地,不让敌人占领”,“搏击风浪”意思是“与风浪作斗争”,“远离世俗”意思是“逃避现实,出家或隐居深山”,这三个词分别与“战士”“渔夫”“隐者”的身份特点相符。 2.下面对课文内容理解有误的一项是(  )A.意境是景与情的结合,是山水画的灵魂。B.要深刻认识观察对象,要有强烈、真挚的思想感情,才能获得意境。C.中国画不强调“光”,这是不科学的,因为画家要进行长期的观察,时间会影响观察对象的精神实质的表现。D.画画光有意境还不够,还要有意匠。答案C 中国画不强调“光”,这是科学的,因为中国画重在表现观察对象的精神实质,对“光”的要求并不高。 3.请同学们结合课文第一、二两段内容,说说在摄影技术如此发达的今天,山水画是否会被取代。答:答案山水画不是地理、自然环境的说明和图解,其更重要的是表现人对自然的思想感悟。可见,山水画具有其精神实质,这也就意味着它永不会为山水摄影所取代。解析本题考查内容理解能力。细读第一、二两段,从中摘取表明山水画独特性的语句,对其进行加工后即可作为答案。 阅读提高(育才中学中考模拟)①人贵立志。古往今来,能成就大事业者莫不先怀雄心壮志,并且矢志不渝。宋代学者程颢曾说:“治天下者,必先立其志。”明代学者王阳明也说:“夫学,莫先于立志。”可见,立志对于一个人的事业发展来说,起着十分关键的作用。②何谓立志、如何立志?答案见仁见智、人言人殊。一些人立志只注重个人利益,或者纯凭个人志趣,这显然过于狭隘和短视,不宜提倡;更多的人在立志时首先考虑国家和人民的需要,在这个大前提下寻找个人的兴趣点,然后确定志向。这样的立志,既有利于社会进步,又可充分发挥个人的积极性与特长,值得大力倡导。③在我国历史上,为国家和民族作出过重要贡献的人,大多立有大志。比如,被林则徐誉为“绝世奇才”的左宗棠,很早就怀有“身无半亩,心忧天下”之志。尽管他三次赴京会考皆名落孙山,直到40岁还是一介布衣,但救世济民之心毫不动摇,潜心研究中外军事战略,后来被人誉为“天才军事领袖”。在抵抗外国侵略方面,他先是抱病、带着棺材赴西北指挥,在新疆战胜了 英国支持的阿古柏军队后,又挫败了沙皇对中国西北的侵略阴谋,捍卫了中国的领土主权。反观与他同时代的一些官吏,一心只追求一己之利,在国家危难之时,贪生怕死临阵脱逃,何谈为国家和民族做出重要贡献?④关于立志问题,毛泽东同志在《致黎锦熙信》中有过深刻论述:“真欲立志,必先研究哲学、伦理学,以其所得真理,奉以为己身言动之准……尽力为之,以为达到之方,始谓之有志也。如此之志,方为真志,而非盲从之志。”这就是说,只有把立志建立在深明哲理的基础上,而不是简单盲从,才能使志向更加高远而切实。⑤毛泽东同志不但对如何立志提出了深刻见解,而且为我们如何实现高远志向做出了光辉榜样。他在求学时期,就立志救国救民。为了实现志向,他在学习上废寝忘食,寻找救国救民真理,最后终于找到了马克思主义。为了救国救民,他还利用一切机会开展社会调查。在1917年暑假,他带着一把雨伞、一双草鞋,步行900多里,历时一个多月,深入农村,访贫问苦,倾听村民的呼声。此外,他还先后多次去安源煤矿了解工人疾苦,向工人宣传马列主义。这些社会调查为他后来形成依靠工农、走农村包围 城市的革命道路,无疑起到了重大作用,也坚定了为民族独立、人民解放和国家富强、人民幸福而努力奋斗的意志。因此,在此后的革命征途中,不管遇到任何艰难险阻,毛泽东同志都矢志不渝,最终成为一代伟人。⑥在这个共筑中国梦的时代,我们要想为国家和民族的发展贡献自己的力量,实现自己的人生价值,就必须确立高远的志向,并为自己的志向不懈奋斗。(选自《人民日报》,有删改) 1.本文的中心论点是什么?2.第③段运用了什么论证方法?有什么作用?3.第④段与第⑤段能不能调换位置?为什么?立志对于一个人的事业发展来说,起着十分关键的作用。举例论证、对比论证。具体突出地论证了“在我国历史上,为国家和民族做出过重要贡献的人,大多立有大志”的观点。不能调换;因为第④段论述了毛泽东同志对立志提出了深刻见解;第⑤段论述了毛泽东同志为我们如何实现高远志向树立了榜样,第⑤段第一句是过渡句,承接了第④段内容,引出了第⑤段的论述。 4.对本文内容理解有误的一项是()A.古代学者程颢和王阳明告诉我们:无论治天下还是学习都要先立志。B.仅注重个人利益或者纯凭个人志趣来树立志向,是不宜提倡的。C.一个人,只要能立下大志,就一定能为国家和民族做出重要贡献。D.左宗棠心忧天下,面对外国侵略,赴西北指挥,捍卫了国家的领土主权。5.请结合自身实际,谈谈你对“立志”的理解。C示例:作为学生,我们要从小确立高远的志向,并为自己的志向不懈奋斗。 非连续性文本阅读(南开中学中考模拟)关于“诗词大会”的报道阅读【新闻报道】最近,杭州与诗词的缘分不浅。一周前,杭州外卖小哥雷海为获得了第三季《中国诗词大会》的总冠军;而昨天,华东师范大学教授、《中国诗词大会》出题专家之一的方笑一,带着他的新书《中华经典诗词2000首》来到杭州,与读者分享如何成为一名诗词达人。昨天的现场很是热闹,互动环节甚至出现抢话筒的场面,讲座结束时,方笑一被妈妈们团团围住,询问如何教小朋友背诗。《中国诗词大会》在全国掀起了一阵重读古诗词的热潮,第二季中的武亦姝,第三季中的雷海为,都是大热的选手。【记者采访】方笑一是第二季、第三季《中国诗词大会》海选面试的评审,他告诉记者,大多数选秀节目参赛选手水平每况愈下,可报名《中国诗词大会》的选手,水平却一届比一届高,一些选手水平高到令专家都震惊。 比如第三季决赛“诗词接龙”环节中,12岁的上海小朋友钱子昂用五代蜀后主王衍的《醉妆词》“者边走,那边走,只是寻花柳”,破解了第三季亚军彭敏的“必杀技”“后不见来者”,赢得专家们一片掌声。据方笑一的了解,熟记诗词并不是所有选手的“童子功”。不少人是看了节目之后,才被激发起对古诗词的兴趣,再经过一段时间的努力,也能在比赛中取得不错的成绩。“诗词之所以能在当下形成热潮,主要原因是,这种短小精练的文本,相较于其他的文学样式,更符合现代人利用碎片时间进行阅读的习惯。”方笑一举了刚刚拿下第三季《中国诗词大会》总冠军的雷海为的例子,他认为,这位外卖小哥1000多首的诗词储备。正是充分利用了工作间隙中大量的碎片时间。丰富的人生阅历,也会加深对诗词的理解与记忆。方笑一告诉记者:“其实,古诗一直在我们的身边,它所表达的真挚情感,所蕴含的人生体验,与你的心,你的人生,你的世界,息息相通。” 他个人比较喜欢的出题方向,就是将诗词与人们的生活相联系。他举了一道自己挺喜欢的题目:宋代朱敦儒《朝中措》词中“汤饼如丝”中的“汤饼”,是今天哪种面食的前身?A.汤面;B.汤包;C.汤圆。方笑一说,这道题的思考方式有很多,有知识储备的人可能会知道,汤饼类似于今天的汤面,还会想到魏明帝请何晏吃热汤饼的典故。假如不知道的话,从后面“如丝”的形容里可以推断出所描述的食物形状是细长的,就能得出汤面的答案。“我们总说中华文化博大精深,天文地理、饮食医药,而诗词正是打开中华文化这座宝库的钥匙。”方笑一说。【专家支招】想要成为“诗词达人”,你要学会下面几招。第一招:图。相比文字,图像更有助于加深记忆。明朝时,就出过《唐诗画谱》。在读到一些描写景致的诗句时,可以去网上搜搜图片,就算不能还原诗句中的意境,但能在脑海里形成场景,帮助你记忆。 第二招:唱。很多人都有课文记不住,但歌词却忘不掉的体验,这就是音乐旋律的功劳。诗词本身就有格律,诵读时朗朗上口,如果配上旋律,更能进一步加深记忆。第三招:用。古人的诗词不是凭空写的,都是有情境的,我们也不妨在合适的情境下,在朋友圈甩一两句古诗表达时下的感受。比如天冷下雪想喝酒,可以“晚来天欲雪,能饮一杯无”,有情有趣,心有灵犀的朋友自会来点个赞。教孩子背诗,可以将诗带入到现实情境中。比如出去玩看见色彩缤纷的小鸟,就可以背背“两个黄鹂鸣翠柳,一行白鹭上青天”。让孩子在这个过程中有愉悦感。第四招:脑补。如果说前三招是加深记忆的辅助之法,那么最后这招是成为诗词达人的根本,那就是读。读是有技巧的,可以大开脑洞,展开想象——比如跟随王维去“行到水穷处,坐看云起时”,看他看过的风景,走他走过的路,眼前是山涧涓涓细流,坐下抬头看到远处的云蒸霞蔚。这些只属于你想象的画面,会成为诵读诗词时最好的伴侣。 1.下列对材料内容表述有误的一项是()A.丰富的人生阅历,会加深我们对诗词的理解与记忆,从而能熟练掌握。B.《中国诗词大会》在全国掀起了一阵重读古诗词的热潮,其原因在于可以激发起对古诗词的兴趣,在比赛中取得好成绩。C.古诗词所蕴含的人生体验,与读者的心,读者的人生,读者的世界,息息相通。D.古人的诗词不是凭空写的,都是有情境的,结合情境的观察,可以加深记忆。B 2.结合【专家支招】部分内容。说说下面古诗适合哪种方法记忆,并阐述理由。【甲】使至塞上王维单车欲问边,属国过居延。征蓬出汉塞,归雁入胡天。大漠孤烟直,长河落日圆。萧关逢候骑,都护在燕然。【乙】子衿青青子衿,悠悠我心。纵我不往,子宁不嗣音?青青子佩,悠悠我思。纵我不往,子宁不来?挑兮达兮,在城阙兮。一日不见,如三月兮。①甲诗适合画图法,甲诗中描写的是塞外自然风光,诗中有画,画面感强,能给人留下深刻记忆。②乙诗适合吟唱,乙诗中的句子短小,富有音乐韵律,读来朗朗上口,吟唱便于记忆。 3.结合材料内容,说说杭州外卖小哥雷海为获得第三季《中国诗词大会》总冠军给我们带来的思考。①阅读需要充分利用工作间隙;②诗词可以充实我们的生活,改变我们自身。 说到眼泪,你会想到什么呢?软弱?眼泪有时候是软弱,有时候是坚强;有时候是忏悔,有时候是宽容;有时候是羞怯,有时候是勇气;有时候是失败,有时候是成功。就如同每一个人,都是哭着来到这个世界的,而当我们要谢幕的时候,也将会在别人的泪水里告别一样。美文欣赏·朗读者 谢谢观看 部编版九年级语文上册教学课件第四单元15.无言之美 日积月累 课文助读走近作者朱光潜(1897—1986),字孟实,安徽省桐城人。现当代著名美学家、文艺理论家、教育家、翻译家。主要著作有《悲剧心理学》《文艺心理学》《西方美学史》《谈美》等。 主要内容《无言之美》选自《朱光潜美学文集》。作者以文学、音乐、雕塑等各类艺术作品为例,分析了言不必尽意、无声胜有声、含蓄不露等所表现出来的“无言之美”,最后归纳出自己的观点:“说出来的越少,留着不说的越多,所引起的美感就越大越深越真切。” 基础过关 3.(川外附中中考模拟)下列句子中有语病的一项是()A.各参展企业不仅带来传统风味的猪肉蛋黄粽。还有鸡肉粽、牛肉粽等创新产品,让现场的消费者大饱口福。B.中国驻南非大使林松添表示,值此第二届“一带一路”国际合作高峰论坛成功召开之际,举办首批南非青年科学家赴华联合研究意义重大。C.我们中学生如果缺乏创新精神,就不能适应新时代的要求。D.在球场上攻城拔寨,摘金夺银,为国争光,这是多少足球运动员的梦想呀!B 4.(2019重庆A)综合性学习。近视是中学生绕不开的话题,为此,初三·1班将开展“未来之光—护眼行动”的主题活动,请你完成以下任务。【请你调查】(1)围绕本次活动主题,我们准备对初三部分学生进行问卷调查,请你为问卷设计两个问题。①②你主要通过哪些方式了解护眼?当你发现自己近视时,是否及时检查或佩戴眼镜? 【请你转述】(2)班主任王老师对金晶同学说“我们班想举办一次‘眼健康科普讲座’,听说你的邻居李叔叔是眼科专家,我想邀请他后天下午两点半到班上来做这次讲座,你问问他能来吗?”第二天,金晶应该怎样向李叔叔转述王老师的话?李叔叔,我们班想举办一次“眼健康科普讲座”,班主任王老师听说您是眼科专家,想邀请您明天下午两点半到我们班上做这次讲座,他让我问问您能来吗? 1.给文段中加点的字注音。此外像陈子昂的《登幽州台歌》:“前不见古人,后不见来者。念天地之悠悠,独怆(  )然而涕下!”李白的《怨情》:“美人卷珠帘,深坐颦(  )蛾眉。但见泪痕湿,不知心恨谁。”虽然说明了诗人的情感,而所说出来的多么简单,所含蓄(  )的多么深远!答案chuàng pín xù解析注意拼音和声调书写要规范。注意“怆”的声调。···拓展训练 2.下列词语书写完全正确的一项是(  )A.瞬息万变  心旷神怡  唯妙唯肖B.珠两悉称  轻描淡写  马鸣风萧C.包罗尽致  栩栩如生  信手拈来D.闲情逸至  目不忍睹  丝毫不爽答案C A.唯妙唯肖→惟妙惟肖。B.珠两悉称→铢两悉称。D.闲情逸至→闲情逸致。 3.下列句子加点词语解释有误的一项是(  )A.或者我们问得再直截一点。(径直,爽快,不绕弯子)B.但是要想明了无言的意蕴,宜从美术观点去研究。(这里专指绘画)C.和情绪意旨所附丽的语言,都要尽美尽善。(附着,依附)D.我们精神上就有一种沉默渊穆和平愉快的景象。(这里指极其美好)答案B“美术”在这里指具有美学意义的活动及其产物,如绘画、雕塑、建筑、文学、音乐、舞蹈等。········ 1.阅读下面的语段,回答问题。先从事实下手研究。pì如有一个荒村或任何物体,shè影家把它照一幅(  )相,美术家把它画一幅图。这种相片和图画可以从两个观点去比较第一,相片或图画,哪一个较“和自然逼(  )真不消说得,在同一视yù以内的东西,相片都可以包罗尽致,并且体积、比例和实物都两两相称(  ),不会有丝毫错误。(1)给加点字注音。(2)根据拼音写汉字。(3)解释加点词。尽致:(4)在处标注恰当的标点。·····能力提升全练 答案(1)fúbī chèn (2)譬 摄 域 (3)详尽细致,达到极点。(4):”?解析(1)注意“幅”字的声调;“称”是多音字,根据语境确定此处应读chèn。(2)注意“摄”字的偏旁不要写错。(3)可根据积累直接作答,也可根据语境推测词义。(4)第一处,后文是对“这种相片和图画可以从两个观点去比较”的具体阐释,因此应该用冒号;第二处的问号应在引号外。 2.指出下列句子采用的论证方法。(1)著名英国诗人济慈在《希腊花瓶歌》也说,“听得见的声调固然幽美,听不见的声调尤其幽美”,也是说同样道理。(     )(2)钱起的《省试湘灵鼓瑟》末二句“曲终人不见,江上数峰青”,也没有说出诗人的心绪,然而一种凄凉惜别的神情自然流露于言语之外。()(3)中国有一句谚语说:“金刚怒目,不如菩萨低眉。”(     )答案(1)道理论证 (2)举例论证 (3)道理论证解析本题考查对论证方法的判别能力。第(1)(3)两题借用他人作品中的话和谚语来说理,可确定为道理论证。第(2)题引用诗句并对其进行分析,从而证明一个观点,可确定为举例论证。 1.请你结合自身的阅读体验,举例说说你对“无言之美”的感受。答:答案正如作者探讨文学作品时举的数个例子所体现的,诗歌本是极其简短的几句话,但是其包含的意境却是极其博大的。如“大漠孤烟直,长河落日圆”,只有短短的十个字,但是读来却似看见大漠的壮阔宏伟之景,予人以悲凉雄壮的美感。然而,作者要详尽描写出这壮阔宏伟之景、悲凉之意,恐怕书万言都难以说尽,这不正意味着作者将它们寓于无言之中了吗?这就是古典文学中深蕴的无言之美。解析本题为开放性试题,答出自己的观感即可。 2.根据下面的语言材料,运用对联知识对出下联。有一座高山,山顶尖尖,像一支倒挂的笔在天上写字;高山脚下有一片梯田,像横着推过去的一片片树叶,铺满大地庄园,煞是壮观。上联是:尖山似笔,倒写青天一张纸下联为:答案梯田如叶,横推大地满庄园。解析阅读语言材料,结合材料找出所给上联的特点,然后根据上联以及后面的语言材料对出下联。如:“尖山”对“梯田”,“似笔”对“如叶”,“倒写”对“横推”,“青天”对“大地”,“一张纸”对“满庄园”。 阅读提高(重庆一中中考模拟)如何欣赏音乐?艾伦·科普兰每个人都会根据各自的能力来听音乐,在某种意义上说,人们是在三种不同的层次上来听音乐的:感官层次,表达层次,纯音乐层次。我把这个过程机械地拆分开来,这唯一的好处是便于分析。听音乐最简单的方式,就是觉得好听,这是感官层次。音乐有一种强大而原始的感染力,对每个正常人来说这是不言而喻的。比如,此刻你正读着这篇文章,如果有人在钢琴上轻轻弹奏一曲,房间里的氛围会顿时不一样。在这个层次听音乐,就只是听而已,不用任何思考,一边听着音乐,一边做着其他事情,对于音乐完全可以心不在焉,不少自认为合格的音乐爱好者沉溺于这个层次。然而,音乐的价值并不等同于它诉诸感官的愉悦程度。理解音乐所表达的意义,这就是听音乐的第二个层次,也就是表达层次。不过,这可是个难题:要用语言来解释某一音乐作品表达的含义,准确无误,还要让每个人都满意,只有老天爷才 知道这有多困难。但是,我们谁都不能否认音乐具有“表达”的力量,每个音符背后都有某种意义,作品总有想要言说的内容,这就是下面简单的问答:“音乐有意义吗?”“当然有。”“你能用语言把这个意义说清楚吗?”“不能。”人们对第二个问题的回答不会满意,他们总希望音乐有一个具体的意义:如果音乐能让听者的联想越熟悉,越明确,比如,一列火车,一场暴风雨,一个曼妙的舞姿……对他们来说,这样的音乐就越富有表现力。一位胆怯的女士曾向我坦陈,她担心自己的音乐欣赏力有严重缺陷,原因在于她听音乐的时候没法联想起某种明确的事物。这当然是把重点搞错了。要我说,一首乐曲无法变成简单而明确的解释或联想,它在不同时刻,不同情境会引发不同的感受:或安详宁静或朝气蓬勃;或懊悔不迭或心满意足;或暴怒或喜悦……它可以用无数细微的差别和变化来表达人类最微妙的情绪和感受,甚至传递任何语言都无法言说的意义。假定你很幸运,能用很多词汇来确切地描述你“听懂”的某一音乐,而且你很得意于自己的描述,但这并不 能保证别人会满意,别人也不需要满意,重要的是,每个人对同一首乐曲的理解会不同,就连经典作品,每次重新听的时候,你的感受很可能也会不同。我的意思是说,音乐的确可以表达某种意义,但欣赏音乐不是喋喋不休的解释。听者内心对音乐的感受、理解和触动,能体验音乐本身的魅力,这是最重要的。听音乐,能够,这就到了第三个层次,纯音乐的层次。的确,除了令人愉快的乐音和所能表达的意义之外,音乐存在于音符和对音符的处理方式中。一般来说,普通听众不太能达到这个层次,当然,专业音乐人士和业余爱好者的区别也在于此,但他们的误区在于,经常全神贯注地陷入琶音①和断奏②,以至于忽略了乐曲更为深刻的层面。我觉得,提高对乐曲的理解比克服纯音乐层次的坏习惯更为重要。不过,如果人们能懂得一些音乐曲式的原理,能跟上作曲家的思想脉络,能听出旋律、节奏等要素,就会在纯音乐的层次上慢慢听出门道,这对更深层次地理解音乐是没有任何坏处的。最后,我再重申一遍,为了便于分析,才把欣赏音乐分成三个层次,实际上,人们不会只在某个单一层面上听音乐,而是各个层面相互联接,相互融合的。从某种意义上来说,理想的听众在同一时刻既能进入音乐也能超脱音乐,既能欣赏音乐也能品评音乐,只有当你更自觉,更有意识地去听,才能加深对音乐的理解,学会欣赏音乐。 可是,读再多这样的文字也无法代替音乐本身带给你的体验,欣赏音乐最重要的是多听,要广泛涉猎,对经典作品反复听,听莫扎特,听巴赫,听贝多芬,听斯特拉文斯基③等等。我想,不仅是音乐,每一种艺术的欣赏都应如此吧。(原文有删改)【注释】①琶音:一种专门的音乐技巧。②断奏:钢琴的基本弹奏法之一。③莫扎特、巴赫、贝多芬、斯特拉文斯基:都是著名的音乐家。 1.下列对标题的回答不符合作者原意的一项是()A.听音乐不能只沉溺在音乐带来感官愉悦这个层次。B.听富有表现力的音乐要能产生明确而熟悉的联想。C.听者内心对音乐的感受、理解和触动是最重要的。D.懂得一些音乐的专业知识对深入理解音乐有好处。2.下列选项填入文中空缺处最恰当的一项是()A.从音符和对音符的处理方式来欣赏音乐B.有意识克服从琶音、断奏听音乐的误区C.听懂音乐曲式的原理和作家的思想脉络D.知道旋律、节奏等要素不一样的表现力BA 3.文章末段谈到“每一种艺术的欣赏都应如此”,请根据此段内容,就“如何欣赏艺术”这个话题,用下面句式作简要论述,要求观点明确,思路清晰,论证合理。欣赏艺术,最重要的是……,而不是……,欣赏文学艺术作品,就要……4.下面是鲁迅《社戏》中与音乐有关的句子,请比较本文与该句在写作内容和表达意图上有何不同。那声音大概是横笛,宛转,悠扬,使我的心也沉静,然而又自失起来,觉得要和他弥散在含着豆麦蕴藻之香的夜气里。——鲁迅《社戏》欣赏艺术,最重要的是欣赏艺术背后的美好,而不是只停留在艺术表现,欣赏文学艺术作品,就要欣赏文字背后的真诚与美好。从这句话可以看出,当时鲁迅对这种声音(音乐)的欣赏,只是停留在感官层次和表达层次,并没有深入到纯音乐的层次,这与他是一个文学家,并不是一个音乐家有关。 非连续性文本阅读(西大附中中考模拟)古人的名、字、号居然有这么多讲究顾明①古人非常重视人的名字,在取名字方面也有许多的讲究。而且,他们的命名方式与现在不同,一个人不仅有姓有名,还要另外取字和号。那么,古人的名、字、号之间有怎样的区别和联系呢?何谓名、字、号②古时候,一个人出生三个月后,长辈会给他起第一个名字,这就是“幼名”。③等到他长大成年之时,就要在名之外另取一“字”。《礼记·士冠礼》:“冠而字之,敬其名也。君父之前称名,他人则称字也。”在古人看来,一个人的“名”只在君王或长辈面前才能使用,而“字”是用来在平辈或晚辈间称呼的。④对于文人雅士,他们常常会为自己取个“别号”,在文章唱和时作为代称,也可以用来自称。⑤所谓“名以正体,字以表德,号以寓怀”,此三者包含着个人的全部品格。 古人怎样取名⑥每个人的名都将跟随他一生,因此取名是一件非常值得重视的事。为了慎重地给人命名,春秋时代还讲究“命名之道”。⑦关于古人取名的原则,《左传·桓公六年》写得明确:“名有五,有信、有义、有象、有假、有类。”信,指的是人与生俱来的一些标记和特点,比如某种特殊形状的胎记,与众不同的长相等。《红楼梦》中贾宝玉因含玉而诞,故名为“宝玉”,当然这有神话的色彩。义,指的是某人出生时伴随的祥瑞现象。象,是以相似之物来命名。孔子名“丘”,因为他出生时头顶是凹下去的,即所谓“圩顶”,父亲便以此为他命名。所谓假,是指假托万物之名。孔子有了儿子后,鲁昭公送给他一条鲤鱼,于是他就给儿子取名“鲤”,遵照的就是“取于物为假”的原则。所谓类,指的是可以取与父亲相似的名字。⑧此外,古人还提出了取名需避开的六条禁忌,分别是:不以国,不以官,不以山川,不以隐疾,不以畜牲,不以器币。如果人名与这几样东西重名,会造成诸多不便。晋国的僖侯名为“司徒”,宋武公名为“司空”,就废了司徒、司空这两个官名;而鲁献公名“具”,武公名“敖”,就废了具、敖两座山的山名。 ⑨从宋朝开始,取名的讲究就更多了,家族辈分、生辰八字、生肖属相等都纳入了考量的范围。著名历史学家陈寅恪,在家族中排“恪”字辈,他出生在寅时,因此取名“寅恪”。⑩古人的名字,显示着家族的血脉传承,寄托着长辈的殷殷期望,象征着人的身份、地位,是一个人安身立命的重要依托。古人的字有什么讲究⑪古人的“字”是“名外之名”,字数上可以取一到两个字。大多数情况下,字的选取与名有一定的联系。说起名与字的关系,大约有以下几种:⑫并列,也就是名和字的意思相同或相通,是并列的关系。比如屈平,字原,古语中“广平曰原”,所以两者意思相同。再比如孟轲,字子舆,轲、舆在汉语中都是“车”的意思。⑬辅助,是说字的意思和名相近,但不完全一样,可以互为补充。比如陆机,字士衡。机、衡二字都是北斗星宿的名,相辅相成。⑭矛盾式,即字和名两者意思相反。朱熹,字元晦。熹是“晨光”的意思,表示天亮;而晦则是“黑暗”的意思,表示天黑。 ⑮补充式,意思是字与名意思相顺承、互为因果或可以用来解释名。比如赵云,字子龙。《周易》说:“云从龙,风从虎。”属于同在一句话中,意思相顺。而于谦,字廷益,则来自《尚书》的“谦受益”,两者互为因果。⑯引申式。如李白,字太白。太白指太白金星,是对“白”字意义的延伸。⑰此外,古人的字男女皆可取,并非男性的独占。《红楼梦》中,贾宝玉问林黛玉:“表字如何?”黛玉答曰:“无字。”这一方面说明黛玉还未到十五岁,也未婚配,不到取字的年龄;另一方面也说明古代女子是可以取字的。别号怎样取⑱相比名和字,古人别号的选取显得更加自由,无论是字数、个数还是字词的选用,都没什么限制。⑲上文说过,别号一般是文人雅士之间用来互称或自称的,多寄托主人的情怀、品格、兴趣爱好和生活经历等,是人生追求的体现。⑳比如苏轼号“东坡居士”,居士表明他信仰佛教,而他曾在黄州城东打理过一片荒地,“东坡”就是那片荒地的代称。 (21)有时候,也可以用人的官职、故乡等为号,比如王维的号“王右丞”,柳宗元的号“柳河东”。(22)帝王、士大夫等死后还被冠以“谥号”“庙号”,也是别号的一种。如“范文正公”“曾文正公”“秦穆公”之类。(23)古人的名、字、号中,包含着他们的文化意蕴、生活态度。了解了这些讲究,闲来无事时,可以为自己取个别号,不仅怡情养性,更在方寸之间,自成雅趣。(选自《阅读时代》,有删改) 1.下列说法符合原文意思的一项是()A.古代只有男性才有字,女性没有。如《红楼梦》中,贾宝玉问林黛玉:“表字如何?”黛玉答曰:“无字。”B.古人取名有六条禁忌,分别是:不以国,不以官,不以山川,不以隐疾,不以畜牲,不以器币。晋国的僖侯名为“司徒”,就很好地避开了这些禁忌。C.在古人看来,一个人的“名”只在君王或长辈面前才能使用,而“字”只用于平辈间称呼。D.“号以寓怀”,别号一般是文人雅士自取的,用于自称或文友间互称,多寄托主人的情怀、品格、兴趣爱好和生活经历等,是人生追求的体现。D 2.下面句中加点的文字能否删去?为什么?大多数情况下,字的选取与名有一定的联系。不能删。“大多数情况下”从范围上加以限制,说明字的选取与名有一定联系,但不排除没有联系的情况存在。如果去掉,则字的选取与名有必然联系,与实际情况不符。这体现了说明文语言的准确性。 3.第⑳—(23)段主要使用了什么说明方法?请列举一例,并说明其作用。主要运用了举例子的说明方法。比如第(21)段举苏轼别号的来历这个例子,具体说明了别号“多寄托主人的情怀、品格、兴趣爱好和生活经历等”特点。(或:第(22)段列举王维和柳宗元别号的例子,具体说明别号“也可以用人的官职、故乡等”来取;第(23)段列举“范文正公、曾文正公、秦穆公”等别号的例子,具体说明了“帝王、士大夫等死后被冠的‘谥号’‘庙号’也是一种别号”。) 陪伴意味着有人愿意把最美好的东西给你,那就是时间。当然陪伴也是一个很平常的事,日复一日,年复一年。到最后陪伴就成了一种习惯。约定好的相逢,伴你天荒地老!美文欣赏·朗读者 谢谢观看 部编版九年级语文上册教学课件第四单元16.驱遣我们的想象 日积月累 课文助读走近作者叶圣陶(1894—1988),原名叶绍钧,字秉臣、圣陶,生于江苏苏州,现代作家、教育家、文学出版家和社会活动家,有“优秀的语言艺术家”之称。代表作品有童话故事《稻草人》,白话小说《春宴琐谭》,长篇小说《倪焕之》等。 主要内容《驱遣我们的想象》一文作者先通过论述作者、读者以及文字之间的联系来明确读者欣赏文艺作品的本质,即“接触作者的所见所感”,然后以赏析王维诗句为例,从正反两个角度论述了驱遣想象力的重要作用。 基础过关 2.下列词语字形完全正确的一项是()A.记栽荒凉舒适愉快B.静寂歌遥记录翅膀C.耀眼萌生汹涌海鸥D.解释震耳帐然凝望C 4.下列句子没有使用修辞手法的一项是()A.都是你自己找的,我怎么帮得了你的忙?B.只有那高傲的海燕,勇敢地,自由自在地,在泛起白沫的大海上飞翔。C.文字是一道桥梁。这边的桥堍站着读者,那边的桥堍站着作者。D.他们不时惊慌地扭头往后看,好像有人跟着他们似的。D 5.(北大附中中考模拟)综合性学习。社区开展“弘扬家风”活动,请你参与并完成以下任务。(1)说说下面古代育人小故事体现了什么家风?曾子杀猪:岳母刺字:(2)“孝顺”“邻里和睦”等都是我们应该传承、弘扬的家风。请将下面关于“孝顺”“邻里和睦”的名言补充完整。A:羊有跪乳之恩,B:远水难救近火,讲诚信精忠爱国鸦有反哺之义远亲不如近邻 (3)老王为儿子写了一幅书法作品,作品是一副对联。请你说说对联寄托了老王对儿子怎样的希望。立品定须成白璧读书毋忽过青年树立良好的品格,珍惜时间,趁年轻多读书。 1.(独家原创试题)下面加点字注音完全正确的一项是(  )A.油漆(xī)苟安(gǒu)掠过(lüè)B.拘泥(ní)怅然(chàng) 躲避(bì)C.鉴赏(jiàn) 桥堍(tù)  驱遣(qiǎn)D.一缕(lǚ)契合(qiè)即使(jí)答案C A.“漆”应读qī。B.“泥”应读nì。D.“契”应读qì。············拓展训练 2.下列词语书写全部正确的一项是(  )A.汹涌  大漠  蠢笨  畏缩B.白沫  慌凉  耀眼  歌谣C.激昂  忧惧  记载  既使D.宛如  审查  配合  海欧答案A B.“慌”应为“荒”。C.“既”应为“即”。D.“欧”应为“鸥”。 3.依次填入下面横线处的词语,恰当的一组是(  )对于作家而言,粉丝易得,知音难觅,粉丝,是为成名;知音,是为寂寞。知音之可贵,不但在于,能看出天才;而且在于,敢畅言所见。A.雪中送炭 锦上添花 慧眼独具 胆识过人B.锦上添花  雪中送炭  慧眼独具  胆识过人C.锦上添花  雪中送炭  胆识过人  慧眼独具D.雪中送炭  锦上添花  胆识过人  慧眼独具答案B“雪中送炭”比喻在别人急需的时候给以帮助。“锦上添花”比喻使美好的事物更加美好。“慧眼独具”指在眼力或洞察力方面有独到之处。“胆识过人”指有胆识,有气魄,超过一般人。根据词义,结合语境,从搭配的角度选择词语填空,确定答案即可。 1.结合语境解释下面句中加点的词语。(1)落日固然是圆的,难道朝阳就不圆吗?固然:(2)没有一丝的风,当然也没有风声,于是更来了个静寂的印象。静寂:(3)假如死盯着文字而不能从文字领会作者的意境,就无从得到这种受用了。受用:(4)要达到这个目的,不能够拘泥于文字。拘泥:········能力提升全练 答案(1)表示承认某个事实。(2)沉寂无声;寂静。(3)受益。(4)固执,不知变通。解析本题考查词义理解能力。要结合语境,采用换词法(如“固然”)或组词法(如“静寂”)理解。 2.指出下面句子采用的修辞手法。(1)落日固然是圆的,难道朝阳就不圆吗?(   )(2)在阴云和海的中间,得意扬扬地掠过了海燕……(   )(3)这胜利的预言者叫了:——让暴风雨来得厉害些吧!(   )答案(1)反问 (2)拟人 (3)比喻、拟人解析(1)根据“难道”可确定为反问。(2)根据“得意扬扬”可确定为拟人。(3)根据“胜利的预言者”可确定为比喻;根据“叫”及其后面的内容,可知也用了拟人的手法。 3.赏析下面句中加点部分的表达作用。(1)古今属于各种门类的文艺,我们所接触到的,可以说,没有一种不是文字的集合体。答:(2)读者也许是到过北方的,本来觉得北方的景物旷远、荒凉、静寂,使人怅然凝望。答:········ 答案(1)“没有一种不是”采用双重否定,语气强烈,更肯定地表达了“各种文艺无一例外都是文字的集合体”这一意思。(2)“也许”表猜测,表示不确定,符合实际,体现了文章语言的准确性、严谨性。解析本题考查语言赏析能力。要根据题意,抓住加点部分,理解其含义和特点,并结合句意分析加点部分突出了什么内容或体现了语言怎样的特点。 1.文中许多句子都有十分深刻的文艺观,它们或者有十分深刻的内蕴,或者有写作值得借鉴的实用价值。请阅读下面几句,谈谈你对它们的理解。(1)文艺的创作决不是随便取许多文字来集合在一起。答:(2)作者着手创作,必然对于人生先有所见,先有所感。答: (3)假如死盯着文字而不能从文字看出一幅图画来,就感受不到这种愉快了。答:答案(1)任何一篇文艺作品,都是文字集合而成的,但是这是一种有着内在逻辑顺序的结合,具有文本表现中的一般技法,既表现了内容,也传递着作者的思想感情。因此,这样的文章绝不可能随意拼凑,须由作者有意识有目的有逻辑地创造,完成之后作品中又传递出自然之感。(2)“见”字可指代材料,无“见”便会巧妇难为无米之炊,不可能有所创造;“见”字也可理解为见识、见解,没有思考与见解,纵然下笔也只能写就平庸文章,难见深度。“感”是体悟与情感,倘若下笔无情感寄寓其中, 写出来的文字必是冷冰冰的。这也提示了我们,在写作之前要学会多观察、多积累,同时要多思考、多感悟,这样才能写出优秀文章。(3)文艺作品一般都具有形象性,尤其是诗歌、散文等文艺作品,形象性尤其突出,且情感表达也尤其浓烈。这些情感常常寄寓在形象之中,所谓“言不尽意,立象以尽意”,就是此道理。因此,想要理解一篇文艺作品,就必须把握其情感,必须通过想象还原形象的描述,进而去体味。解析本题考查理解句子含意的能力。要结合语境,通过抓关键词的方式,或将句子意思表述得更加明白,或根据题干的提示回答自己从句中得到的写作方面的启示。 2.文章以王维的诗句“大漠孤烟直,长河落日圆”为例,论证了想象力之于鉴赏文学作品的重要性,你能在你读过的诗或散文中,再举一个例子来论证作者的观点吗?答: 答案(示例)以朱自清先生的《背影》为例:“他用两手攀着上面,两脚再向上缩;他肥胖的身子向左微倾,显出努力的样子。”这段描写父亲攀爬月台的背影的文字,能因我们的想象力变得更生动。他的身子微胖,我们可以想象两手上攀时定费了不少劲,甚至青筋凸起;还是因为胖,我们可以想象腿并不是缩了一次,而是挣扎着缩了好几次;更是因为想象,身子微微一倾,这形象便如拍照一般在脑中定格。有了想象中还原的这一幕,父亲的形象真切起来,切实地打动了读者的心。解析本题考查补写论据的能力。要紧扣要求证明的观点“想象力之于鉴赏文学作品的重要性”,选取自己最熟悉的“诗或散文”中的一篇,通过对具体语句的分析,证明要求证明的观点。 阅读提高(育才中学中考模拟)生活需要文学蜜饯一下阮直①作家与文学太需要一个像“护士节”“教师节”那样的节日了。文学需要民众的土壤为之滋补,作家的灵魂需在社会的舞台上高蹈,作家的情感更需要生活的蜜饯。②有人说现代化的都市不需要作家,可文学从来没有背叛过生活,都市只要还过着人的生活,你就无法摆脱文学。文学只是想告诉更多的人,大家在肉身疲惫之后,转身享受一下文学之美,用你的心灵过滤一下生活,你才能吧嗒出生活中有盐的滋味。③文学不是让人们向往虚构,作家也不是让大众舍弃财富,文学只是不想让人们的欲火太盛,作家只是让更多的人也能在傍晚的时光用文学的心态去看看树木花草,听听涛声拍岸。让忘记时间成为一种自觉的高贵,让一颗忙碌的心就此归隐。我们不妨假装一会儿懒惰,自觉地选择几件“没有意义”的事情去做,没用的文学就有这个用处。 ④没用的文学却有优雅的人生格调,因为文学有梦。高速、奢华、纷繁、喧嚣的都市已经让人们的灵魂疲惫,呼唤文学的回归迟早会成为更多人生活的目标。⑤放缓我们前行的脚步,为奔波之路点上一个休止符。停顿是美,停顿是诗,停顿是国画中的留白。农民你就走出田野,渔民就把船儿开回港湾,老板们放下几天的生意,不该我们奔波、不该我们干的事情在人生之中太多了,可人们却不知道。人们常说:能今天做好的事情,绝不拖到明天。作家吴亮也有名言:“凡是能拖到明天做的事情,今天就不要去干。”⑥大家以文学的名义举行一次精神的盛宴吧!灵魂也需要聚会。交流阅读文学的感悟,畅谈虚构的快乐,展望未来的美好……⑦如今的文学,已走进多元化的时代,它虽然没有繁花似锦,但也绝不会凋敝衰败。文学是能潜入有灵性人内心的,在不同的场合贴着不同的标签,成为可以多元选择的生活方式。在这样一个多元化的社会,谁也无法做到让全民族每一个人都去背诗写诗去热爱文学,但有梦想的人心中的文学情结就是一座活着的火山。⑧如果作家有了节日,文学就容易发酵。节日能让爱好文学的人们回归到享受精神欢愉的生活方式之中。节日能给作家传递 温暖,作家能以激情之火去燎原大众的情感。中国人即便不写诗也是诗人,就连一个姓氏的排名,国人都把它诗化、韵化成“赵钱孙李,周吴郑王,冯陈褚卫,蒋沈韩杨”。⑨中国人是活在亲情与人情中的,文学就是“人学”。这个世界无论哪个国家的文学消失,我们的文学也不会,文学是民族精神中永存的根脉,当下的文学由一度的“庙堂之高”,回到了民间的江湖,而文学总会在自己的土壤里长出属于它的花花草草。⑩当文学成为人们的一种生活方式,读者和作家都可以选择的时候,文学才算找到了它的根。但愿每个人心里都有一间房子,里面装颗自由畅想的心。(选自2019年5月10日《中国文化报》) 1.本文的中心论点是什么?2.第⑤段画线句子运用了什么论证方法?有何作用?放缓我们前行的脚步,为奔波之路点上一个休止符。生活需要文学。比喻论证。把“文学”比作“休止符”,生动形象地论证了文学在生活中的重要作用(或者论证了文学可以让人们拥有优雅的人生格调,让人们的生活变得更美好,回答论证了中心论点也可)。 3.结合选文内容,谈谈你对第⑧段加点词语的理解。如果作家有了节日,文学就容易发酵。4.下列对选文内容的理解和分析,不正确的一项是()A.“没用的文学”是作者说的反话,其实文学在人们生活中极其重要。B.呼唤文学的回归必将成为多数人的需要,热爱文学也会成为更多人的生活方式。C.作家吴亮的名言“凡是能拖到明天做的事情,今天就不要去干”与作者观点相反。D.中国人的文学即“人学”,它是民族精神永存的根脉,是不会消失的。“发酵”本义为复杂的有机化合物在微生物的作用下分解成比较简单的物质(或利用微生物的生长繁殖制造人们所需要的产品,或比喻事态持续发展)。此处的“发酵”是指利用“节日”,让爱好文学的人们能回归到享受精神欢愉的生活方式,作家能感受到温暖,并用自己的激情之火去燎原大众的情感。C 非连续性文本阅读(西大附中中考模拟)关于“中小学书法教育”的主题阅读【话题背景】教育部《中小学书法教育指导纲要》指出:义务教育阶段书法教育以语文课为主,也可在其他学科课程、地方和校本课程中进行。其中,小学3—6年级每周安排1课时用于毛笔字学习。普通高中可开设书法选修课。 71.4%的受访者表示一个人的字代表着一个人的形象气质;72.5%的受访者曾经有拿字帖练字的经历;42.5%的受访者认为自己的字拿不出手;74.7%的受访者认为,即便在互联网时代,写一手好字仍很重要。(摘自《中国青年报》)【精选案例】书法是戴敏的第一项特长。她回忆,小学放暑假,天天待在家里练字,开始也觉得苦,“但练着练着就练出感觉了,十分享受这过程”。戴敏练书法源于父亲一次不经意的鼓励,后来就渐渐爱上了书法,而老师也通常会在写得好的字上画圈,“每次看到自己字上的圈多了就会感到很开心”。对于用依帆来说,少年时代的练字更多的是痛苦的任务,“老师布置的练字作业每次都拖到暑假的最后一个星期。”他认为,互联网时代,真正用笔写字的时候越来越少,“也就剩下在商场刷卡签名时会写写字了,我身边许多人都这么认为。”(摘自中国教育新闻网) 【各方声音】网友冰水椰子:我是教语文的,又兼着书法课,常常觉得力不从心。好在一个学期上的书法课不多,期中和期末考试前就偷偷地改上语文课了。网友吞可吞:很喜欢书法教室和教室走廊陈列的名家作品。很羡慕写得一手好字的同学。但总是只有心动,没有行动。上了初中,作业多了,就更没有时间练字了。沈尹默(书法家):世人公认中国书法是最高艺术,就是因为它能显出惊人奇迹,无色而具画图的灿烂,无声而有音乐的和谐,引人欣赏……郭振有(中国教育学会常务副会长):书写,表达一种感情,与电脑写不是一种状态。如果一切都机械化了,就没有审美,变得贫乏而没有了想象力。沙如(中国教育学会书法专业委员会副秘书长):初中虽然学习压力加大了,但是练习书法能让我们静下心来,每天有半小时的时间练书法,之后学习效率会更高。【新闻现场】本报讯今天,“传承兰亭——绍兴市区中小学生‘兰亭雅集42人展’”在书法圣地兰亭右军祠启幕,这成为今年兰亭书法节的一个亮点。市内许多中小学校积极组织学生前来参观展览。一位带队老师说,他们学校有很多书法爱好者,他们是传承中国书法、传承中华文化的希望和未来。 1.下列选项中与文意不相符的一项是()A.教育部要求,小学3—6年级每周要安排1课时练习书法,普通高中要开设书法选修课。B.从调查数据可以看出,书法教育进中小学课堂有较好的社会基础。C.有反对者认为,互联网时代,写字的机会越来越少了,也就没有必要学写毛笔字了。D.中国书法是世人公认的最高艺术,它具有审美价值,能丰富人的想象力。A 2.根据以上几则材料,分条概述中小学生学习书法的意义。①提升一个人的形象气质;②使人静心,缓解压力,提高学习效率;③发展特长,培养自信;④传承书法艺术,传承中华文化。 有人说,我们这个时代不缺机会,所以也势必会让每个人面临很多的选择,那么,是遵从自己的内心,还是随波逐流;是直面挑战还是落荒而逃;是选择喧嚣一时的功利,还是恒久平静的善良。无论如何,希望每一个人,都能做出一个在日后回想不让自己后悔的选择。美文欣赏·朗读者 谢谢观看 部编版九年级语文上册教学课件第五单元17.屈原(节选) 日积月累 课文助读走近作者郭沫若(1892—1978),原名郭开贞,作家、诗人、历史学家、剧作家、考古学家、古文字学家、社会活动家。代表作品有诗集《女神》,历史剧《屈原》《棠棣之花》《虎符》《高渐离》《南冠草》等。 主要内容本文节选自《屈原》的第五幕的第二场,是全剧的高潮。该剧是一篇悲壮、慷慨、激昂的抒情独白。屈原召唤着风暴雷电等雄伟的自然力量,他与风暴雷电已完全融为一体!借此表达了对黑暗的愤激和对光明的礼赞和向往。借指斥神鬼偶像来抨击昏庸腐朽的当权者。 基础过关 4.(南开中学中考模拟)综合性学习。阅读下面三则材料,回答问题。材料一:人工智能技术注定会改变我们的世界。它注定会重新定义工作的意义以及财富的创造方式;它将带来前所未有的经济失衡现象,甚至改变全球的权力格局。随着它的进一步发展,会不可避免地对就业造成冲击。人工智能是一个“强者更强”的产业;数据越多,产品越好;产品越好,所能获得的数据就更多;数据更多,就更吸引人才;人才越多,产品就会更好。(摘编自李开复《人工智能对人类社会的真正威胁》)材料二:目前,人工智能正在以前所未有的速度“赋能”人类生活,硬件智能化是大势所趋,传统制造企业要抓住机遇,推动人工智能技术在制造业领域的创新应用,促进人工智能和实体经济深度融合。(摘编自刘坤《如何迎接人工智能热潮》) 材料三:商汤科技CEO徐立直言,国内人工智能创业大多扎堆在应用层面,创业者使用开源算法,找到某个垂直领域便套上“人工智能”概念扎进去,但真正从算法层出发做“原创技术”的人并不多。“而这块才是核心,是最需要厚积薄发的。”就国内而言,人才储备方面还相对薄弱。来自领英的数据显示,全球范围内,人工智能专业人才有195万,中国只占2%,排名第七。(摘编自张意轩、王威《人工智能需要翻越三道坎》)(1)请根据材料一概括“人工智能技术注定会改变我们的世界”的具体表现。①重新定义工作的意义和财富的创造方式;②带来前所未有的经济失衡现象;③对就业造成冲击。 (2)根据上述材料,分别从企业和个人两个角度,提出应对人工智能热潮的建议。从企业角度讲:要抓住机遇,推动人工智能与社会各领域的融合;避免扎堆应用层面,要抓住从“算法层”出发这一原创的“核心”。从个人角度讲:努力适应人工智能给生活带来的改变;努力学习,成为人工智能的专业人才。 1.下列句中有错别字的一项是(  )A.你坐在那马上私毫也不能驰聘。B.我们只有雷霆,只有闪电,只有风暴。C.我看见你挺身而出,指天画地有所争论。D.我思念那东海,那浩浩荡荡的无边无际的波澜呀!答案A 私→丝,聘→骋。拓展训练 2.从语段中括号内选择符合语境的词语填在相应的横线上。电,你这宇宙中的剑,也正是,我心中的剑。你劈吧,劈吧,劈吧!把这比铁还(坚挺 坚固)的黑暗,劈开,劈开,劈开!虽然你劈它如同劈水一样,你抽掉了,它又(围拢 合拢)了来,但至少你能使那光明得到暂时的一瞬的显现,哦,那多么灿烂的、多么(明亮 炫目)的光明呀!答案坚固 合拢 炫目解析注意结合语境进行判断。如“比铁还……”只能是“坚固”,因为“坚固”指结合紧密,不容易破坏。而“坚挺”则不符合语境。 3.下列句中加点词语使用正确的一项是(  )A.一连下了好几天雨,操场上拖泥带水。B.这山里的天气一会儿风,一会儿雨,真是风云突变啊!C.小屋粗制滥造,出自一个农村木匠之手,而不是由古希腊的能工巧匠建造起来的。D.因为我写字姿势不够端正,爸爸多次教育我:“写字时一定要正襟危坐,眼睛距离书本一尺远。”答案C A.拖泥带水:形容说话、写文章不简洁或做事不干脆。B.风云突变:用来比喻变幻动荡的局势。C.粗制滥造:制作粗劣,不讲究质量。也指工作不负责任,草率从事。D.正襟危坐:理好衣襟端端正正地坐着,形容严肃或拘谨的样子。A、B、D三项均属望文生义,对词义理解不正确,造成使用不当。················ 1.根据课文独白分析主人公形象,下列不恰当的一项是(  )A.从独白中我们可以看出,主人公是一个敢于否定神的人。B.主人公是一个富有革命理想、怀才不遇、视死如归的无产阶级革命战士。C.主人公是一位想象力极丰富的浪漫主义诗人。D.主人公是一个“举世皆浊我独清,众人皆醉我独醒”的人。答案B“无产阶级革命战士”有误,从文中无法判断出这一内容。能力提升全练 2.(独家原创试题)下列句子使用的修辞手法有误的一项是(  )A.哭,哭有什么用?眼泪,眼泪有什么用?(反问、反复)B.把这比铁还坚固的黑暗,劈开,劈开,劈开!(重复)C.虽然你劈它如同劈水一样,你抽掉了,它又合拢了来。(比喻)D.(雷)你把我载着拖到洞庭湖的边上去,拖到长江的边上去,拖到东海的边上去呀!(排比、拟人)答案B“重复”是一种语病,该句运用的是“反复”的修辞手法。 3.填入下面横线处的句子,最能表达屈原对天神蔑视和憎恨的一项是()把你这东皇太一烧毁了吧!把你这云中君烧毁了吧!你们这些土偶木梗,。A.你们高坐在神位上并没有德能,你们只是产生黑暗的父亲和母亲。B.你们高坐在神位上有什么德能?你们不能产生黑暗的父亲和母亲!C.你们高坐在神位上没有什么德能,你们只是产生黑暗的父亲和母亲。D.你们高坐在神位上有什么德能?你们只是产生黑暗的父亲和母亲!答案D 反问和感叹语气最能表达作者对天神的蔑视和憎恨。A、C项都是陈述语气。B项语意相反。故选D。 4.(独家原创试题)我们中国的传统节日承载着人们的纪念和祝福。请在清明节、春节和中秋节中任选一个节日,仿照例句写一个句子。例句:端午节——无论是赛龙舟,还是包粽子,都诉说着对故人久远的怀念。答:答案(示例1)清明节——无论是燃香烛,还是踏青游,都寄托着对逝去亲人的哀思。(示例2)春节——无论是放鞭炮,还是贴春联,都传递着对亲人安康的祝福。(示例3)中秋节——无论是赏明月,还是吃月饼,都表达着对阖家团圆的渴望。解析例句已经给出固定的格式“无论……还是……都……”;内容上要表达出特定的深意,表现出中华民族传统节日的意义,如题目中的“中秋节”,要表达出团圆的幸福,“清明节”要表达出对已逝亲人的思念。 1.下列句中画线部分的短语类型不同于其他三项的一项是(  )A.上野的樱花烂漫的时节,望去确也像绯红的轻云。B.我这熊熊地燃烧着的生命,我这快要使我全身炸裂的怒火,难道就不能迸射出光明了吗?C.我母亲待人最仁慈、最温和,从来没有一句伤人感情的话。D.暖国的雨,向来没有变过冰冷的坚硬的灿烂的雪花。答案B B项画线部分为动宾短语,A、C、D画线部分三项均为偏正短语。 2.下列解说有错误的一项是(  )A.风!你咆哮吧!咆哮吧!尽力地咆哮吧!分析:作者反复使用“咆哮吧”,强烈地表现了屈原对风的热切期盼和对黑暗势力的痛恨。B.我要看那滚滚的波涛,我要听那镗镗鞳鞳的咆哮,我要漂流到那没有阴谋、没有污秽、没有自私自利的没有人的小岛上去呀!分析:作者使用了排比的修辞手法,表现了屈原对“污秽”“自私自利”,玩“阴谋”的社会的憎恶,以及对光明、纯洁、无私的社会的追求。C.你们风,你们雷,你们电,你们在这黑暗中咆哮着的,闪耀着的一切的一切,你们都是诗,都是音乐,都是跳舞。分析:作者运用了排比、拟人、反复、比喻的修辞手法,表现了屈原对风、雷、电的热情歌颂。 D.我这熊熊地燃烧着的生命,我这快要使我全身炸裂的怒火,难道就不能迸射出光明了吗?分析:作者运用了反问的修辞手法,表现了屈原对自由的强烈追求。答案D 应该说“表现了屈原对光明的热烈追求”。 3.课文运用了象征手法,请你说说“风”“雷”“电”“洞庭湖”等形象的象征意义。答:答案风、雷、电:象征变革现实的伟大力量。洞庭湖、东海、长江:象征人民群众。解析象征是文学创作中一种重要的表现手法,它是根据事物之间的某种联系借助某一事物的形象(象征体),以表现某种抽象的概念、思想和情感(被象征的本体)。本文一些形象的象征意义,可以结合文章内容来理解。 非连续性文本阅读(川外附中中考模拟)“走两步”,认出你影片《碟中谍5》中有一个场景引起了人们对步态识别技术的关注,戴着面罩的间谍在安保系统的最后一道防线现形了,原来是他的步伐“出卖”了他。知道吗,影片中的步态识别技术如今已走进了现实。在生物识别技术领域,目标的有效识别距离以步态识别技术最远,可达50米。“步态识别技术”中的“步态”不仅包括体型特征,如高矮胖瘦等,还包括运动特征,如运动方式、姿态等。在“走两步”的背后是强大的自动步态识别系统,主要包括监控摄像机、计算机以及步态视频序列处理与识别软件,而建立步态数据库也是非常关键性的工作。这一系统在识别身份的功能基础上,能够存储和跟踪个人信息,关注个人健康发展,在运动员精准训练、疾病筛查、智能家居方面能发挥重要的作用。 炫酷新装备心情杯这款智能水杯,杯身颜色可以根据你正在经历的事情、你的心情而变化。例如,你支持的球队进球时,水杯内置的LED灯会立刻呈现蓝色,似乎它也在跟你一起为球队助威。“小纽扣”这款高清智能摄像机外形像纽扣,能通过语音、面部和步态识别,锁定你的拍摄对象,自动选择、录制最精彩的瞬间,并将其编辑成可共享的短视频。爱牙仪它是一款智能口腔自测仪,让你的口腔环境清晰可见,还能通过精准定位牙菌斑,准确检出蛀牙、牙周炎等口腔疾病的早期症状,有效预防、诊断各种口腔疾病。触感屏这是模拟实物触感的屏幕。当手指在鱼的图像上移动,能充分感受到鱼鳞特有的质感。若是将手指移动到鱼眼位置,你会发现手指的滑动没有丝毫阻力,就好像拨弄一颗小珠子。大白它是一种智能机器人。只要说出“大白”的名字就可以将其唤醒,进而完成你的各项指令。它可以陪着你出去或者在家玩耍,解答你天文、地理等方面的问题,还能预报天气状况,播放投影电影,甚至可以跟你聊天。 砰!音乐来了!语音识别技术主要解决30厘米到5米范围的语音交互问题,它的代表产品就是智能音箱。让音箱听懂你的语言,需要解决三个核心问题:听见、听准和听懂。从技术角度来看,就是拾音、读音和解音三个关键技术环节。拾音是最为基础的环节,必须保证音箱听见你的声音;读音是将符合要求的声音转换成文字;解音则是识别人类的指令甚至情感。距离音箱3米内,你只有使用普通话发出“播放‘歌手+歌名’”的明确指令,音箱才会精准播放歌曲。如说“播放周杰伦的《青花瓷》”,这首歌浓郁的中国风就会扑面而来。当距离超出5米,识别率就会下降甚至接收不到指令。在音乐方面,人工智能还有更广泛的应用。如在音乐订制网站,你可以根据所选择的主题、乐器、节奏、氛围和时长等多个元素订制乐曲。当按下“渲染”键后,砰!乐曲就诞生了!人工智能软件不仅完成了谱曲,同时也负责演奏和混音工作。(根据《大众科学》《今日科技》《环球科学》等期刊资料编辑) 1.根据情境完成题目。轩轩是科技馆的志愿者,小可在她那里看见了以上有趣的资料,对科技馆很好奇,轩轩便带她去玩。(1)输入个人相关信息后,小可开始体验馆内种种新奇的科技产品,她做不到的一项是()A.戴上面具,穿上轩轩的外套,步态识别系统仍然准确地“认出”了自己。B.用智能口腔自测仪轻松检测出蛀牙等早期症状,提醒自己注意保护牙齿。C.看到触感屏上鱼的图像,用手指轻轻地滑过,就像真的触摸到鱼鳞一般。D.对着智能音箱说:“来一首周杰伦的中国风!”音箱就播放出《青花瓷》。D (2)轩轩带小可走向一扇神奇的门,隔着约十米远,轩轩说:“芝麻开门!”门开了。这里发挥作用的是步态识别技术还是语音识别技术呢?请作出判断,并介绍其工作原理。示例:运用的是步态识别技术。它的工作原理是:首先,步态识别系统摄入步态和相应的身份信息;然后,系统提取步态特征并存储到步态数据库;当系统再次检测到相同步态的时候,便会从数据库中找出对应的信息,进而确认身份。 2.小可是个球迷,她想去看世界杯决赛。在科技馆这么多智能产品中,她想携带两件。如果你是轩轩,会推荐哪两件?为什么?示例:推荐“小纽扣”,因为这款高清智能摄像机能锁定小可喜欢的球员,自动选择、录制最精彩的瞬间,并将其编辑成可共享的短视频。推荐“大白”,因为这是一种智能机器人,它可以陪伴小可去看球,解答遇见的各种问题,甚至可以跟小可聊天。 3.轩轩播放了一首动听的乐曲,她说这是自己的作品,是送给小可的礼物。之前,她在音乐订制网站的主题栏选择了“友谊”,乐器栏选择了“长笛”,节奏栏选择了“舒缓”……按下“渲染”键,乐曲就诞生了。小可感谢之余,产生了疑惑:这个作品是轩轩的原创吗?请谈谈你的看法。示例:我认为是轩轩的原创。尽管乐曲是人工智能软件制作并演奏出来的,但乐曲的主题、乐器、节奏等核心元素都是由轩轩确定的。(言之成理即可) 我发现自己是如此的激动,以至于不能安坐或思考。我想只有那些重获自由即将踏上新征程的人们才能感受到这种即将揭开未来神秘面纱的激动心情。我希望跨越边境,与朋友相见握手。我希望太平洋的海水如同梦中一样的蓝。我希望。——电影《肖申克的救赎》美文欣赏·台词 谢谢观看 部编版九年级语文上册教学课件第五单元18.天下第一楼(节选) 日积月累 课文助读走近作者何冀平,中国剧作家。中央戏剧学院戏剧文学系毕业,毕业后从事专职戏剧创作,曾任北京人民艺术剧院编剧。1988年,何冀平创作的《天下第一楼》演出后轰动京城,演出场次仅次于《茶馆》,被誉为当代现实主义经典。 主要内容该剧描写了创业于清代同治年间,传至民国初年的老字号烤鸭店“福聚德”由入不敷出、势如累卵到东山再起、名噪京华而又面临倒闭的曲折发展历程。歌颂了卢孟实、玉雏姑娘、罗大头、常贵等人的聪明才智、事业心与实干精神;控诉、批判了游手好闲的败家子习气和黑暗腐朽的社会势力。 基础过关 4.(重庆八中中考模拟)下列各句中,没有语病的一项是()A.如期打赢脱贫攻坚战,是当前最紧要的民生工作,也是目前在中部地区崛起中实现湖南高质量发展的当务之急。B.在纪念“五四”运动100周年大会上,习总书记提出,我们要主动走近青年、倾听青年,做青年朋友的知心人。C.教育部在做出这项战略部署时,特别强调职业院校要加强对此项工作的引导,避免不再出现片面的“考证热”。D.好的演讲材料不是抄来的,而是演讲者对事实、数据等进行认真研究、整理、搜集的结果。B 5.(南开中学中考模拟)下列句子排序正确的一项是()①京绣对用料十分讲究,在上等的织物上,按照设计好的花纹和色彩,用绣针穿引彩线,佐以金丝银线,在绣料上刺缀运针,绣出图样。②而且,有些纹样的刺绣只能出现在京绣中,特别是皇帝龙袍上的纹样,如果用错便有篡位夺权之嫌。③明清时期,京绣大为兴盛,因其多用于宫廷,所以又称为“宫绣”,属于“燕京八绝”之一。④除京绣之外,民间的刺绣工艺发展也很迅速,形成了四大名绣,即苏州的苏绣,湖南的湘绣,四川的蜀绣和广东的粤绣。⑤京绣严格遵循“图必有意,文必吉祥”的宗旨,无论是服饰纹样,还是配饰小品,都充分体现端庄典雅、雍容高贵的皇家气派和尊严。A.①⑤②③④B.③④①⑤②C.①③④⑤②D.③⑤②①④D 6.(重庆一中中考模拟)综合性学习。栀子花开完了一季,又到了每年6月的毕业时节,我校正开展关于“毕业之后”的综合性学习活动。请阅读以下材料,回答问题。材料一:今年6月,拿到本科毕业证的大学生韩某因为一直没有找到工作,经济拮据的他进入教师办公室行窃,累计作案7起。几乎同一时间,辽宁一小伙失联数月,后于峨眉山跳崖,留下遗言表示跳崖是因为大学毕业找不到工作。人人网毕业季公布的就业调查统计显示,在距今年高校毕业不到100天的时间,仍有50.9%的大学生未收到用人单位的就业通知。——百度新闻网和中国青年网材料二:随着中国社会的发展,中国各大高校都在努力扩招人数,大学生人数增加导致人才供需关系不平衡,于是他们一毕业必将面临困境。而北京师范大学某教授也毫不留情地指出因为扩招,当今大学的含金量越来越低,很多在校学生面对社会上的诱惑和大学宽松的环境,立场不坚定,忽略了学习的重要性,导 致学习能力下降,自身能力欠佳。尽管如此,大学生们对待遇的期望却不低,在全国调查中,超过四成人对月薪的期望是8000—10000元,在重庆,这个比例是28.1%。——重庆晚报和腾讯网材料三:在近来高考填报专业之际,在读大学生甲和乙分别谈到了自己的专业选择。甲:读大学终究是为了找工作,所以我选读了会计专业。大二就开始实习了,为毕业工作筹备着。乙:我在大学里选了自己喜欢的文学专业,每天除了听课,就是泡图书馆,看了很多喜欢的书。 (1)材料一反映了什么问题?请根据材料二,简要说说材料一中问题产生的原因有哪些。材料一反映了大学生毕业就业难的问题。其原因有:高校扩招,导致人才供需关系失衡;很多在校大学生忽略学习,能力欠佳;大学生对待遇期望高。 (2)针对材料一反映的问题,有人在贴吧的毕业赠言板块写下这样的话,请你将它补充完整。临别时,我赠你一个美好的理想,让它成为你跋涉的手杖;临别时,我赠你一个温暖的拥抱,让它成为你远航的风帆;临别时,,。(3)请阅读材料三,说说你比较欣赏谁的做法,并说明理由。我赠你一个灿烂的微笑让它成为你登高的阶梯示例:我欣赏甲的做法。学习就是为了以后能更好地工作,甲同学比较务实,会计工作就需要这种求实不爱幻想的性格才可以做好。 1.下列加点字的注音全部正确的一项是(  )A.喧哗(huá)  眩晕(yūn)  玉雏儿(chú)B.瞅瞅(chǒu)嬉笑(xī)  贝勒府(lè)C.哄笑(hōng)煞白(shā)侍候(shì)D.忌讳(huì)落子(lào)呷了一口(yā)答案B A.“晕”是多音字,此处应读yùn。C.煞shà。D.呷xiā。············拓展训练 2.下列词语书写全部正确的一项是(  )A.凄惨  簇拥  耍威风  金璧辉煌B.老匾  账薄  歇歇脚  赫赫扬扬C.窝囊  鼎盛  鸭架桩  名噪京师D.冤枉  地锲  侦缉队  咬牙跺脚答案C A.璧→碧。B.薄→簿。D.锲→契。 3.下列句子中加点的成语使用不当的一项是(  )A.不是你俩还有谁!这班上就数你们最爱造谣生事,搬弄是非。B.圆明园是万园之园,建筑雕梁画栋、美轮美奂,堪称人间仙境。C.他自己虽然不知道是因为懒,还是因为无用,总之觉得是一个不肯运动、十分安分守己的人。D.本刊将洗心革面,继续提高稿件的编校质量,决心向文学刊物的高层次、高水平攀登。答案D“洗心革面”比喻彻底悔改。这个词只能用于人,不能用于物。················ 1.读下面的语段,根据要求回答问题。此时是福聚德的(繁盛 兴盛 鼎盛)时期。雕梁画dòng的大楼金碧辉huáng,门前那块黑底金字的陈年老biǎn泛着辉光。门前停的是汽车、马车、绿呢(  )大jiào,门里进出的是达官显贵、商贾(  )名流。福聚德已是赫(  )赫扬扬,名噪京师。(1)给语段中加点的字注音。(2)根据拼音写出汉字。(3)第一句横线处填写最恰当的一个词应为。(4)结合语境解释语段中加线词的意思。名噪:(5)语段中的“泛”字换成“闪”字好不好?为什么?答:···能力提升全练 答案(1)nígǔ hè(2)栋 煌 匾 轿 (3)鼎盛(4)(名声)广为传扬。(5)不好。因为“泛”字表示发出的辉光是持续的,而“闪”则是指光亮忽明忽暗,用于“老匾”不恰当。解析(1)注意“呢”是多音字,此处指一种布料的性质,应读ní。“贾”也是多音字,此处指经商之人,应读gǔ。(2)结合语境确定字形,注意避免误写成同音字。(3)首先辨析三个词语词义的异同。相同点是都有“兴旺”之意。不同之处在于“繁盛”还强调“多,茂盛”,不适用于形容烤鸭店;“鼎盛”的意思是“正当兴盛或强壮”;“兴盛”的意思是“蓬勃发展”。结合前文“此时”可确定选“鼎盛”。(4)结合前文描述,联系成语“名噪一时”可明确其含义。(5)首先明确表态——不好;然后从“泛”字“闪”字所表现的光亮的特点及使用对象角度阐明理由。 2.指出下面语句采用的描写方法。(1)他人到中年,衣着华贵,面容丰满,一脸威严。(    )(2)卢孟实向店里扫了一眼,坐在当年老掌柜的那把太师椅上。(    )(3)(卢孟实)脸色由青变得煞白,突然高声笑了起来。(    )答案(1)外貌描写、神态描写 (2)动作描写 (3)神态描写解析(1)根据“衣着”“面容”可确定外貌描写,根据“威严”可确定神态描写。(2)根据“扫”“坐”可确定动作描写。(3)根据“脸色”“笑”可确定神态描写。 阅读下面的文字,按要求作答。卖 猪在下棋声中,收猪人瞟了一眼画家,大意之中走错了一个棋子。卖猪人哎,我吃车。收猪人(突然按住卖猪人的手)等等!回一步。卖猪人咱还兴回棋?收猪人(冷冷地)那中,你吃吧!卖猪人(意识到)中,中,你回吧,你回吧。收猪人(得意地又换另一个棋子)我不走那一步啦。当卖猪人又走下一步棋时。收猪人(警告地)你敢动那个子儿,就叫你死! 卖猪人……叫我死?……对,赶快死。干脆,我自杀吧!(摆下那个棋子)收猪人你不能走那个子儿!卖猪人咋?……你不叫我死吗?收猪人不兴故意死!卖猪人(无奈地)你要是不叫我死,我就先活着。两人继续走棋。收猪人(发现一步好棋,强调地)咱可不能回啦!将!卖猪人(忽然高兴起来)你老弟的棋下得真好。(哭丧着脸恳求地)我那猪拉过来过磅吧?收猪人(扬扬得意地)你先举举手……卖猪人(不解地)举举手?[幕后声:“谁的猪跑啦?”] 卖猪人(向幕内一看)我的猪!我的猪跑啦!(跑下)卖猪人在幕内喊:“同志,来帮帮忙!”画家跑下。一阵猪嚎叫声。少顷,画家摊着沾有猪屎的双手上。卖猪人跑上,感激地用自己的上衣为画家擦拭手上的猪屎。卖猪人(稳住了精神,走近收猪人)我的猪逮住了,拉过来过磅吧?收猪人你还没举手呢。卖猪人举手?收猪人刚才那盘棋你输了,你就得举手投降!画 家(爆发地)你太过分了!(欲冲向收猪人)卖猪人(急拦住画家)你甭生气甭生气。我一个小老百姓,人家叫咋着就咋着。不就是举举手吗?(不待画家说话,转向收猪人)老弟,我投降我投降 ……(频频举手)画 家(极其生气地冲到收猪人面前)我说你这个同志办事也太过分啦!这位老同志来卖猪,你让他等一会儿他就等一会儿,他不愿意和你下棋,你非让他和你下棋,闹得他的猪差点儿跑了。这实在是令人……咱们都是国家工作人员,应当有点儿职业道德,总还要讲点儿精神文明吧?收猪人精神文明?画 家你……(激动地)好!精神文明你也不懂。可你百般戏弄一个老实巴交的乡下人,你还有没有良知?收猪人良知?画 家良知就是良心,良心你懂不懂?收猪人(皮笑肉不笑地)良心良心,就你有良心?噢!你是个老雷锋啊。(突然把脸一沉)办你的好事去吧! 画 家(更加气愤地)好!这些都不讲,现在咱们去找你的领导,看你在工作时间下棋对不对?收猪人我对!画 家你……收猪人咋着咋着?今天我非下棋不可!(故意摔打着棋子)我下,我就下!画 家(怒不可遏)你不能下!(掀翻棋盘)卖猪人(忽然意识到什么,上前将画家推了一个趔趄)唉!画 家(不可理解地)你?卖猪人与画家对视片刻。卖猪人(痛心疾首地)是我卖猪呀!我卖猪我还不急哩,你是急啥哩?画 家(完全糊涂了)我……收猪人强硬地摔着棋子。 卖猪人在砰砰的棋子声中走近收猪人,他思索片刻,忙殷勤地拾起掉在地上的棋子,轻轻地放在棋盘上,又诚惶诚恐地乞求收猪人……1.下列句子中括号内的文字叫。试分别说明它们的表达作用。(1)“收猪人(冷冷地)”(2)“卖猪人(意识到)”(3)“卖猪人(忽然高兴起来)”2.“……叫我死?……对,赶快死。干脆,我自杀吧!”句中三个加点词语的含义是什么?这几句台词反映了卖猪人怎样的心情?答:···· 3.“你甭生气甭生气。我一个小老百姓,人家叫咋着就咋着。不就是举举手吗?……老弟,我投降我投降……”对收猪人的行为卖猪人是否生气?他为什么劝画家不要生气?画线句反映了卖猪人怎样的心情?答:4.请你概括一下画家的性格特点。(10个字以内)答: 1.答案舞台说明 (1)因卖猪人有求于他却不让他回棋而不满,同时借此显示“威严”。(2)卖猪人专心下棋,故不相让,明白了收猪人的不满,才意识到自己有求于人。(3)因自己终于“合理”地输了棋,收猪人可能痛快地将猪收下而高兴。解析本题考查对文章内容的把握能力。文中括号里的内容,都是对人物心情、动作等的提示。(1)“冷冷地”说明收猪人对卖猪人态度冷淡。(2)“意识到”说明卖猪人想到了自己有求于收猪人。(3)“高兴”说明卖猪人此刻心情很好。 2.答案第一个“死”是卖猪人没明白对方的话,以为是让自己死掉;第二个“死”是卖猪人明白了对方的话,想“输掉”棋;“自杀”指卖猪人主动输棋的行为。这几句话反映了卖猪人无奈而又焦急的心情。解析本题考查对词语的理解能力。“叫我死”是卖猪人还没理解收猪人的意思而说的话,“赶快死”是卖猪人为迎合收猪人而说的话,“自杀”是卖猪人让收猪人的行为。据此分析作答即可。 3.答案卖猪人表面不生气,实际心中十分不满。为了能让收猪人高兴,快点把猪卖掉,只好强装笑脸,怕画家触怒了收猪人,所以劝画家不要生气。画线句反映了卖猪人十分不满却又无可奈何的心情。解析本题考查对人物心理的揣摩能力。“我一个小老百姓,人家叫咋着就咋着。”这是埋怨之词,但又无可奈何,写出卖猪人表面不生气,实际心中十分不满。卖猪人劝画家不要生气,是因为他想早点把猪卖出去,不想得罪收猪人。4.答案有正义感,好打抱不平。解析本题考查对人物形象的分析能力。从“精神文明你也不懂。可你百般戏弄一个老实巴交的乡下人,你还有没有良知?”可见他肯为弱者出头,热心正直,仗义执言,有同情心,正义感强。 非连续性文本阅读(北大附中中考模拟)【材料一】“青蛙效应”源自十九世纪末美国康奈尔大学曾进行过的一次著名的“青蛙试验”:实验人员将一只青蛙突然放进煮沸的大锅里,青蛙触电般地立即窜了出去。后来,人们又把它放在一个装满凉水的大锅里,任其自由游动。然后用小火慢慢加热。青蛙虽然可以感觉到外界温度的变化,却因惰性而没有立即往外跳,直到后来热度难忍失去逃生能力而被煮熟。科学家经过分析认为,这只青蛙第一次之所以能“逃离险境”,是因为它受到了沸水的剧烈刺激,于是便使出全部的力量跳了出来,第二次由于没有明显感觉到刺激,因此,这只青蛙便失去了警惕,没有了危机意识,它觉得这一温度正适合,然而当它感觉到危机时,已经没有能力从水里逃出来了。【材料二】比尔·盖茨有一句名言:“微软离破产永远只有18个月。”企业要生存要发展,只有具备危机意识,强化危机管理,企业才不致在战略上迷失方向,滑入危机的泥潭之中。值得重视的是, 危机管理并非是企业最高管理层或某些职能部门的事情,而应成为每个职能部门和每位员工共同面临的课题。在最高管理层具备危机意识的基础上,企业要善于将这种危机意识向所有的员工灌输,使每位员工都具备居安思危的思想,提高员工对危机发生的警惕性,使危机管理能够落实到每位员工的实际行动中,做到防微杜渐、临危不乱。企业竞争环境的改变大多是渐热式的,如果管理者与员工对环境之变化没有疼痛的感觉,最后就会像这只青蛙一样,被煮熟、淘汰了仍不知道。一个企业不要满足于眼前的既得利益,不要沉湎于过去的胜利和美好愿望之中,而忘掉危机的逐渐形成和看不到失败一步步地逼近,最后像青蛙一般在安逸中死去。而一个人或企业应居安思危,适时宣扬危机,适度加压,使处危境而不知危境的人猛醒,使放慢脚步的人加快脚步,不断超越自己,超越过去。事实上,造成危机的许多诱因早已潜伏在企业日常的经营管理之中,只是由于管理者麻痹大意,缺乏危机意识,对此没有足够的重视。有时,看起来很不起眼的小事,经过连锁反应、滚雪球效应、恶性循环,都有可能演变成摧毁企业的危机。 【材料三】人天生就是有惰性的,总愿意安于现状,不到迫不得已多半不愿意去改变已有的生活。若一个人久久沉迷于这种无变化、安逸的生活时,就往往忽略了周遭环境的变化,当危机到来时就只能坐以待毙。未雨绸缪、居安思危、有危机意识是我们应该从中领悟的。在生活和职业上都是如此,逆水行舟,不进则退。回顾一下过去,当我们遇上猛烈的挫折和困难时,常常激发了自己的潜能;可一旦趋向平静,便耽于安逸、享乐、奢靡、挥霍的生活,而不断遭遇失败。青蛙效应带给我们一些深刻的启示。启示之一:我们的组织和社会生存的主要威胁,并非来自突如其来的事件,而是由缓慢渐进而无法察觉的过程形成。人们目光短浅,只看到局部,而无法纵观全局,对于突如其来的变化,可以从容面对,对于悄悄发生的变化,而无法察觉,最终会带给我们更加严重的危害!启示之二:青蛙,就好像是我们生活中的芸芸众生,我们要着眼未来,勤于思考新的问题,勤于学习新的知识,不能过“今日有酒今日醉”和“当一天和尚撞一天钟”的醉生梦死的生活,到头来将是非常可悲的! 1.孟子在《生于忧患,死于安乐》一文中提出了“生于忧患,死于安乐”的观点。阅读以上材料,两相比较,青蛙效应该怎样解释?青蛙效应强调的正是孟子的“生于忧患,死于安乐”的主张。人天生就是有惰性的,总是愿意或者习惯于安于现状,不到迫不得已多半不愿意去改变已有的生活。若一个人长久沉迷于这种无变化、安逸的生活时,往往容易忽略周遭环境的变化,当危机到来时就像那只青蛙一样只能坐以待毙。 2.阅读材料二,结合语境解释下面词语的含义。(1)居安思危:(2)防微杜渐:人在安逸和顺利的情境下也要保持一颗戒惧之心。预防和杜绝工作和生活中点点滴滴的小问题小失误,不让小问题成为大问题。 3.阅读材料三,说说“青蛙效应”在我们的日常生活中就个人而言有什么借鉴意义?示例:居安思危、未雨绸缪、有危机意识是我们应该从中领悟借鉴的。在生活上和工作上大多如此,逆水行舟,不进则退。现实生活中可以常常看到,当人们遇上猛烈的挫折和困难时,常常激发了自己的潜能;可一旦趋向平静,便不知不觉地会沉迷于安逸、享乐、奢靡、挥霍的生活,而不断遭遇失败。 什么是权利?当一个人犯了罪,法官依法判他死刑。这不叫权利,这叫正义。而当一个人同样犯了罪,皇帝可以判他死刑,也可以不判他死刑,于是赦免了他,这就叫权利!——电影《辛德勒名单》美文欣赏·台词 部编版九年级语文上册教学课件第五单元19.枣儿 日积月累 课文助读走近作者孙鸿,又名孙宏,化名华生、石坚,洋县华阳镇吊坝河人。1931年10月加入中国共产党,是靖江戏剧小品作家。 写作背景在剧烈深刻的社会变革中,人们的思想观念、生活方式面临着严重的考验。日益强劲的现代化浪潮却无可阻挡地席卷着一切与之不相适应的思想和观念,迫使许多人不得不放弃他们熟悉的生活。 主要内容1999年《枣儿》荣获曹禺优秀话剧小品奖。全剧运用象征手法,围绕着枣儿展开情节,描写了老人和男孩之间的一段亲切交往,表现了老人对儿子、男孩对父亲的深切亲情,反映了我国在现代化进程中的社会变迁。 基础过关 4.(西大附中中考模拟)《儒林外史》以“功名富贵”为镜,照出儒林各种人物的灵魂。阅读书评,回答问题。其书以功名富贵为一篇之骨:有心艳功名富贵而媚人下人者;有倚仗功名富贵而骄人傲人者;有假托无意功名富贵,自以为高,被人看破耻笑者;终乃以辞却功名富贵,品地最上一层,为中流砥柱。(《儒林外史》卧闲草堂本闲斋老人序)从下列选项中任选一项,指出他属于书评所列的哪种人,并结合小说情节加以阐述。A.王冕B.牛浦郎 示例一:我选A。王冕是辞却功名富贵而成中流砥柱者。他蔑视权贵,以卖画为生,为了不与危素等权贵结交,多次推辞见面,甚至远走他乡;他有远见,向朱元璋献策,让其以仁义服人,平定天下;他不慕名利,朱元璋征他做官,他为躲避入仕而归隐会稽山。可见他是一个恪守道德、张扬个性的贤者。示例二:我选B。牛浦郎是心艳功名富贵而媚人下人者。他偷了牛布衣的诗稿,盗用了牛布衣的名号,一心想通过结交权贵改变自己的命运。为结交董瑛,责怪卜家弟兄;为巴结牛现圃,甘愿做牛玉圃的孙子,媚态毕现。可见牛浦郎是一个追名逐利、自甘下流的小人。 1.下列加点字注音有误的一项是(  )A.竹匾(biǎn)   钢盔(kuī)B.囫囵(hú)愣住(lèng)C.蓦然(mò)翘首(qiào)D.中意(zhòng)掸去(dǎn)答案C“翘”应读qiáo。········拓展训练 2.下列加点词语书写有误的一项是(  )A.(老人)从身旁晒满红枣的竹匾子里抓起一把枣儿,喃喃自语。B.男孩温顺地谗老人坐下。C.一颗枣儿从树上落下,男孩蹑手蹑脚地走过去捡枣儿。D.(男孩)踌躇欲下,又垂头丧气站住。答案B 谗→搀。··········· 3.选词填空。(1)我一急,把个枣儿囫囵个儿(吞 咽)下肚了。(2)一颗枣儿从树上落下,男孩(毛手毛脚 蹑手蹑脚)地走过去捡枣儿。(3)正巧,树上一颗枣儿落到鬼子的钢盔上,“咚”的一声,吓得那狗娘养的(抱头就逃 狼狈不堪)。答案(1)吞 (2)蹑手蹑脚 (3)抱头就逃 解析本题考查对近义词的辨析能力。句(1),“吞”指不嚼或不细嚼,整个儿地或成块地咽下去,与“咽”相比,更能突出“我”的心急,与“我一急”相照应。句(2),“毛手毛脚”指“做事粗心大意,不沉着”,“蹑手蹑脚”形容“走路时把脚步放得很轻”,此处,小男孩是怕捡枣儿被发现,故意把脚步放轻些,所以要选“蹑手蹑脚”。句(3),“抱头就逃”暗含“狼狈不堪”的意思,且有逃跑之意,更符合语境。 1.(独家原创试题)下面的台词意蕴丰富,请你揣摩其言外之意。(1)老人甜是甜,不中看,谁要啊。答:(2)老人听好,我讲完一个故事,你才能吃一颗枣儿。噢,慢慢吃才能吃出个甜味。答:(3)男孩我娘说,出远门的人有时候不认识回家的路了。答:能力提升全练 答案(1)委婉地批评了只看外表不注重内在的世俗风气。(2)隐含着对既往岁月、传统生活、精神家园的感情。(3)比喻情感的迷失、人生方向的迷失、精神家园的迷失。解析(1)“甜是甜”是指内在的品质。“不中看”是指外表的普通平凡。所以这句话委婉地批评了只看外表不注重内在的世俗风气。(2)不仅是对吃枣而言,也是对一般生活哲理的泛指。其中隐含的是对既往岁月、对传统生活、对精神家园的感情。(3)注意理解“不认识回家的路”的比喻义。 2.《枣儿》一文发出了对亲情的深切呼唤,歌曲《常回家看看》也表达了父母对子女亲情回归的渴望,中央电视台公益广告中那位做好了饭菜等儿女回家的老妈妈孤独的身影更让我们感受到了人们对亲情的企盼。那么,请你也围绕“亲情”这一主题拟一句公益广告词。答:答案(示例)儿女是父母永远的牵挂——常回家看看。解析解答本题要紧扣主题“亲情”,结合课文中老人对儿子的思念、男孩对爸爸的呼唤,联系题干中提到的公益广告镜头,呼唤儿女常回家看看。语言要简洁、明了,能以情动人。 1.“二人翘首远望,状如雕塑。响起无数个童声呼唤声。”联系全剧,说说这样结尾的表达效果。答:答案这个结尾,既与开头呼应,又强化了剧中的情境和内容,深化了全剧的思想感情。解析本题考查对结尾作用的理解能力。注意此处和开头及主旨的关系。 2.综合实践活动。(1)早前,国外网络上一幅父亲湿身为儿子撑伞的图片(见图1)刷爆了媒体头条和朋友圈。与此同时,一幅南昌的孩子熟练地拖着沉重的行李箱和父亲并肩进入小区的图片(见图2)也瞬间刷爆朋友圈。这两幅图片为什么能刷爆朋友圈?请分别谈谈你的看法。图1          图2 图1:图2:(2)请你设计一条富有感染力的宣传广告语,提醒同学们感恩父母。答: 答案(1)图1:父爱如山,父亲尽其所能为孩子遮风挡雨,做了孩子最好的屏障。图2:父亲培养了孩子独立自主的能力,教会了孩子基本的生存技能,令孩子做好了独立的准备。(2)(示例1)感恩父母亲情,温暖传遍家庭。(示例2)亲情手牵手,感恩心连心。解析(1)本题考查读图能力,要仔细观察画面,注意题干中的提示语,要能揭示出图画的本质意义。(2)注意广告语特点:精练,朗朗上口,言简而意无穷。 阅读提高(重庆八中中考模拟)山脊谈歌褐色的大山裸露着粗壮的骨骼,暴虐的山风狂笑着在绵延几十里的山脊上疯跑。年轻的副部长沿着崎岖的山路攀上山来。十几个随从紧紧跟在他的身后。两年以后,这里若再找不到新的矿点,十里之外,那座赫赫有名的、喧腾了半个世纪的矿山,就要弹尽粮绝,永远地归于沉寂了。年轻的副部长仰头望去,目光焦灼。山脊上,3座几十米高的钻塔,逼入云端。两个月前,这3台钻机被47条汉子推上山脊,在海拔数千米的高山上,引吭高歌。过了一个月,钻机的歌声低哑了。这一个月见不到蔬菜,喝山涧里的泥汤汤过日子,山脊上的汉子们无法忍耐了。请假走了一个,又走了一个,又走了……还有人要走。年轻的分队长急哭了。 这天,年近60岁的前任分队长从医院里跑出来,上山了。走了的7个人,也被这凶神似的老汉像逃兵一样带了回来,满脸惭愧。大队派人追到山上,央求老汉回去。“软骨头的都走,我不走!”老汉吼。47条汉子齐声吼:“老汉不走我们也不走!”老汉笑了。钻井的歌声又重新雄壮起来……年轻的副部长终于攀上了山脊,挥一把汗,检阅着他的士兵。最后,目光盯住了满身泥浆的老汉,皱一下眉,叹口气:“您的身体?”老汉笑了:“不碍。”“您还是下山的好。”副部长用商量的口气说。老汉收住笑,摇摇头:“找不到矿我不走!”副部长从秘书手里接过挎包,取出两瓶“五粮液”塞到老汉手里,说:“少喝点。” 老汉笑弯了眉:“找到矿再喝。”副部长告辞下山,走下去十几步,又停住脚,回头望,老汉正含笑望着他下山。一年零9个月过去了,山脊上钻出了大矿。最后3个月,老汉瘦成了一根柴。他一手按着腹部,一手扶着机台,豹子一样的眼睛盯着旋转的钻杆。47条汉子流着泪,在老汉越来越弱的吼声里干活。终于,在新大矿发现的第二天,老汉说了句“大家喝五粮液”,便含笑倒下了。喜报和丧讯同时发出。大队领导来了,年轻的副部长也来了。还未起灵。47条汉子让老汉安睡在山脊上,那两瓶酒,被47条汉子轮换拿着,洒在老汉周围。一下子温顺了的山风,默默地从老汉身上掠过,带走了浓郁的酒香。年轻的副部长,缓缓地走到老汉身边,猛地跪下,伏在老汉身上,悲痛欲绝地哭喊着:“爹——!”凄绝的喊声,在山脊上传得老远,老远…… 1.简析文章开头第一自然段的作用。2.请简要概括小说中老汉的形象。环境描写,突出了开矿钻探条件的恶劣与艰苦(或“拟人,赋予山风人的特性,生动形象地写出了开矿钻探条件的恶劣与艰苦”),衬托老汉带病坚持开矿,不怕苦、不认输的精神,也为全文奠定了悲凉的感情基调。老汉是一位坚强乐观、不怕苦难、乐于奉献、具有高度责任心、为事业鞠躬尽瘁、死而后已的老钻探工人。 3.结合文本,谈谈你对标题“山脊”含义的理解。4.小说结尾揭示了老汉与副部长之间的父子关系,这样安排有何妙处?一语双关。一方面指矿山的山脊,是人物活动的场所,另一方面指老汉坚持不懈、顽强奋斗、不屈不挠、矢志不渝的伟大品格,这位老汉如同山脊一般,撑起了整个矿队,是矿队的灵魂人物(核心人物)。这个结尾,既在意料之外,又在情理之中。年轻副部长的一声“爹”,出乎读者意料,但细读前文,会发现有多处伏笔。如副部长看到老汉时的“皱眉”“叹气”,询问老汉“身体如何”,这些都是一个儿子面对倔强的父亲时,既充满关爱又无可奈何的表现,而副部长告辞时,老汉“含笑望着他下山”的神情,正是父亲对儿子的深情表露。 非连续性文本阅读(南开中学中考模拟)【材料一】苏州博物馆采用的玻璃、钢铁结构,让现代人可以在室内借到大片天光。开放式钢结构替代传统建筑的木构材料,屋面形态的设计突破了中国传统建筑“大反顶”在采光方面的束缚,由几何形态构成的坡顶和玻璃天窗设计独特,天窗开在了屋顶的中间部位,这样,屋顶的立体几何形天窗和其下的斜坡屋面形成一个折角,呈现出三维造型效果,不仅解决了传统建筑在采光方面的实用性难题,更丰富和发展了中国建筑的屋面造型样式。材料方面,屋面如果用传统的瓦片,易碎易漏,需要经常维修,其坚固性、工艺性以及平整度都难以达到要求。所以,贝聿铭采用素有“中国黑”之称的深灰色花岗石取代了传统的灰瓦,这种黑中带灰的花岗石淋了雨是黑的,太阳一照变成深灰色。石片被加工成菱形,平整地铺设于屋面之上。 【材料三】大家好!欢迎来到苏州博物馆的“片石假山”,说到作为建筑大师的贝聿铭先生,他一直对中国清代书画家——扬州八怪中的石涛的“片石山房”情有独钟。在设计建造苏州博物馆的时候,他有意再现“片石山房”之景。“片石”,就是将浑厚的大石头切片,石片颜色由深入浅,再高低错落排砌,用来摆放成假山,营造出一片逼真的山景。大家往博物馆的围墙之外看,我们面前的这一大片“片石”营造的假山是有“外援”的,建筑师采用了苏州园林常用的“借景”手法,借用和依托了东侧相部的拙政园中树木之景,于是新旧园景笔断意连,巧妙地融为一体。。整体景致非常清雅。【材料四】近年来,人们会在春节假期安排中,给“博物馆”留有一席之地。“博物馆里过大年”已从一个响亮的口号,变为博物馆界的一种常态。苏州博物馆以馆藏文物“秘色瓷莲花碗”为原型,在过年期间开展了制作创意曲奇饼干的活动,吸引了大批学生和家长。 春节前后,故宫每个角落都成为春节文化的展场。位于午门正殿和东西雁翅楼展厅的文物展览以“祈福迎祥、祭祖行孝、敦亲睦族、勤政亲贤、游艺行乐、欢天喜地”为主题,全面展现清代宫廷过年习俗。紫禁城宫殿门口悬挂着寓意吉祥的春联、门神,廊庑下装饰着华美的宫灯,在乾清宫和皇极殿的丹陛上下竖立着高大的天灯、万寿灯。在乾清宫东庑的“宫里过大年”数字沉浸体验展,运用了数字技术、虚拟影像、动作捕捉等科技手段,分为冰嬉乐园、门神佑福、花开岁朝、戏幕画阁、赏灯观焰、纳福迎祥六个部分。尤其值得一提的是,故宫博物院成功修复了天灯、万寿灯等节庆文物,重现康乾盛世的过年景象。 1.根据材料一及材料二中的图1,说说苏州博物馆的屋面有何特色。2.作为(志愿者)解说员的你,发现材料三所提供的解说词在横线的位置漏掉了重要内容,你现在需要阅读观察材料二的图2,为横线处补充出相关内容。(提示:用“我们再看围墙之内,……”开头。要求:至少运用一种修辞手法,并引用一句古诗词,70字左右。)自然光线引入巧妙(或设计上采光效果好),结构造型新颖独特,建筑材料考究,色彩搭配巧妙。我们再看围墙之内,粉墙似乎是白纸,假山和水如同画在纸上,呈现出清晰的轮廓和剪影效果,这就是水墨山水的意境!假山又借了真水,于是层峦叠嶂、粉墙黛瓦在水面映照出清晰的轮廓,古树伸出它苍劲的手臂,碧波荡漾,好一幅空灵迷人的立体山水图。 3.假如你是报社编辑,请从下边三个选项中为材料四选一个你觉得最合适的新闻标题,并说明理由。A.《“博物馆里过大年”已成为博物馆界新常态》B.《走,到博物馆里过大年去!》C.《春节就来博物馆:文化体验,不容错过!》示例一:我选A:这个标题平实、准确,比较全面地概括了文章的主要内容,体现了新闻的特征,能让读者一眼了解主要事件。示例二:我选B:这个标题将新闻事件说得生动有趣,“走”表现了立即可成行,与读者形成了互动。且该题目极具号召力,很吸引人。 幸福,不是长生不老,不是大鱼大肉,不是权倾朝野。幸福是每一个微小的生活愿望达成。当你想吃的时候有得吃,想被爱的时候有人来爱你。——《飞屋环游记》美文欣赏·台词 谢谢观看 部编版九年级语文上册教学课件第六单元20.曹刿论战 日积月累 课文助读走近作者左丘明,中国春秋时期史学家。相传曾著《左氏春秋》(又称《左传》)和《国语》。 作品介绍《左传》是先秦时代内容最丰富、规模最宏大的历史著作,也是我国古代第一部编年体史书。所记载历史年代大致与《春秋》相当,同起于鲁隐公元年(前722年),终于鲁哀公二十七年(前468年),全书共18万字。 主要内容课文通过曹刿与鲁庄公有关齐鲁战争问题的对话和指挥长勺之战的史实,阐明了作战必须取信于民,并运用正确的战略战术,掌握战机才能取胜的道理,表现了曹刿的政治远见和卓越的军事才能,赞美了曹刿的爱国之情。 基础过关 4.翻译下列文言句子。(1)肉食者鄙,未能远谋。(2)忠之属也。可以一战。战则请从。(3)一鼓作气,再而衰,三而竭。有权势的人目光短浅,缺少见识,不能深谋远虑。这是忠心尽力为人民办事。可以凭借这点一战。作战时请让我跟您同往。第一次击鼓可以振奋士气,第二次击鼓士气有所衰减了,第三次击鼓时士气全耗尽了。 6.(西大附中中考模拟)下列句子没有语病的一项是()A.随着私家车的暴增,使车位引发的邻里纠纷频频上演,停车问题已成为烦扰邻里关系的恶疾。B.相比攻坚克难的施工建设,科学运营和长期的管理维护更是考验北调的南水能否永远清澈的关键。C.一些招工的企业负责人表示:虽然用人需求增加,但只有待遇上去了,善待外来工,招工就比较容易。D.谁也不能否认,每个人的心灵深处都有光辉的一面,哪怕是十恶不赦的罪犯。D 7.(巴蜀中学中考模拟)下列句子排序正确的一项是()①雪峰的红光映射到这辽阔的牧场,形成一个金碧辉煌的世界。②你就可以看见无数点点的红火光,那是牧民们烧起铜壶在准备晚餐。③蒙古包、牧群和牧女们,都镀上了一色的玫瑰红。④特别诱人的是牧场的黄昏,周围的雪峰被落日映红,像云霞那么灿烂。⑤当落日沉没,周围雪峰的红光逐渐消退,银灰色的暮霭笼罩草原的时候。A.①④②⑤③B.④③①⑤②C.④①③⑤②D.①③②④⑤C 8.(育才中学中考模拟)阅读下面名著选段,结合相关情节回答问题。“难道就因为我一贫如洗、默默无闻、长相平庸、个子瘦小,就没有灵魂,没有心肠了?——你不是想错了吗?——我的心灵跟你一样丰富,我的心胸跟你一样充实!要是上帝赐予我一点姿色和充足的财富,我也会使你同我现在一样难分难舍,我不是根据习俗、常规,甚至也不是血肉之躯同你说话,而是我的灵魂同你的灵魂在对话,就仿佛我们两个人穿过坟墓,站在上帝脚下,彼此平等——本来就如此!”——节选自《简·爱》(1)选段中的“我”指的是,“你”指的是。“要是上帝赐予我一点姿色和充足的财富”,其实简·爱后来获得了一笔财富,请结合名著内容说说这笔财富的来源。简·爱罗切斯特简·爱得到了约翰叔叔的遗产。 简·爱知道罗切斯特有一个活着的妻子。(2)简·爱深爱着男主人公,但是一开始却拒绝了和他结婚,原因是什么? 1.解释下面各句中加点的词语。(1)牺牲玉帛  (      )(2)望其旗靡  (      )(3)必以信   (      )(4)一鼓作气  (      )(5)既克,公问其故(      )(6)彼竭我盈  (      )答案(1)指祭祀用的纯色全体牲畜 (2)倒下 (3)实情 (4)鼓起 (5)原因,缘故 (6)充满,这里指士气正旺盛解析本题考查对文言实词的解释能力。一是根据课内积累直接作答,二是根据句意推测词义。注意“牺牲”“信”“故”的意思。······· 2.下面加点的“之”与“小大之狱”中的“之”字意思相同的一项是(  )A.属予作文以记之   B.辍耕之垄上C.故时有物外之趣  D.何陋之有答案C 题干中的“之”意为“的”。A.“之”是“代词,代这件事”。B.“之”是“去、往”的意思。C.“之”意为“的”。D.“之”无实义,是宾语前置的标志。···· 3.用现代汉语翻译下列句子。(1)肉食者谋之,又何间焉?译文:(2)小信未孚,神弗福也。译文:(3)忠之属也。可以一战。战则请从。译文: 答案(1)当权者自会谋划这件事,你又何必参与呢?(2)(这只是)小信用,未能让神灵信服,神是不会保佑你的。(3)(这)是尽职分之类的事情。可凭借(这个条件)打一仗。(如果)作战,就请允许(我)跟随着去。解析注意“间”“孚”“福”“忠”“可以”“从”的翻译。 1.选出加点词意义相同的一项(  )A.衣食所安  肉食者鄙B.小信未孚  必以信C.肉食者谋之 未能远谋D.肉食者鄙  二者不可得兼答案C A.名词,食物/动词,吃。B.信用,诚信/实情。C.均指谋划、打算。D.……的人/……的东西、情况。········能力提升全练 2.下列句子与例句句式相同的一项是(  )例句:可以一战。A.战于长勺。   B.夫战,勇气也。C.何以战?  D.再而衰,三而竭。答案D D项句子和例句都是省略句,A、C两项是倒装句,B项是判断句。 3.理解填空。(1)课文中表明鲁庄公在政治上取信于民的句子是:。(2)曹刿认为进攻的有利时机是:。追击的有利时机是:。答案(1)小大之狱,虽不能察,必以情(2)敌军三鼓后 敌军辙乱、旗靡时解析(1)本题考查理解与默写能力。要注意审题,另外,要避免写错别字,如“虽不能察”的“察”。(2)仔细阅读原文最后一段即可找到答案,也可用原句来回答。 1.对下列句子的停顿划分,正确的一项是(  )A.一鼓作/气,再/而衰,三/而竭。B.一鼓/作气,再/而衰,三/而竭。C.一/鼓作气,再而/衰,三而/竭。D.一鼓/作气,再而/衰,三而/竭。答案B 句子停顿划分的依据是划分后的各部分有相对独立的意义或者存在着一定的逻辑关系(如转折等)。根据这个划分的原则看选项的句子:“一鼓”与“作气”存在两个相对独立的意义;“再”和“三”表明次数,“而”表示顺承关系,根据逻辑关系,表示顺承关系的虚词与后面的词联系紧密,故选B。 2.请你阅读课文第三段,将下面内容补充完整。(第②空用原文回答)在本段中,曹刿论述了①;抓住了②的有利战机,后发制人,进攻齐军;看到齐军③后,果断做出追击齐军的决定。从这两处,我们可以看出曹刿是一个④、⑤的人。答案①鲁国取胜的原因 ②彼竭我盈 ③辙乱旗靡 ④有谋略 ⑤善观察解析本题考查对文章内容的理解及对人物形象的分析能力。要仔细审题,根据题目要求细读课文第三段。①要求填写对全段内容的概括语句。②要用原文作答。③可概括原文中的词句作答。④⑤两处应根据曹刿在全文中表现出的性格核心——远谋,以及“下视其辙,登轼而望之”来归纳,填写。 3.长勺之战,齐败鲁胜,曹刿功不可没,且一战成名;鲁庄公虽鄙,但不乏可取之处。鲁庄公身上有哪些值得肯定的方面?答:答案(1)“公将战”表现出鲁庄公面对强敌毫不畏惧的精神。(2)“小大之狱,虽不能察,必以情”表现出鲁庄公勤政爱民的品质。(3)“公与之乘”表现出鲁庄公礼贤下士的品质。(4)“公问其故”表现出鲁庄公不耻下问的品质。(5)鲁庄公发现曹刿才华出众,加以重用,表现出鲁庄公知人善任的品质。(6)在作战时机的选择上,两次遭到曹刿否定,鲁庄公毫不介意,表现出鲁庄公胸怀宽广、从善如流的品质。解析本题考查对人物形象的分析能力。紧扣题干要求,找出鲁庄公身上值得肯定的方面即可。 文言文阅读(重庆八中中考模拟)曹刿论战《左传》十年春,齐师伐我。公将战,曹刿请见。其乡人曰:“肉食者谋之,又何间焉?”刿曰:“肉食者鄙,未能远谋。”乃入见。问:“何以战?”公曰:“衣食所安,弗敢专也,必以分人。”对曰:“小惠未遍,民弗从也。”公曰:“牺牲玉帛,弗敢加也,必以信。”对曰:“小信未孚,神弗福也。”公曰:“小大之狱,虽不能察,必以情。”对曰:“忠之属也。可以一战。战则请从。”公与之乘,战于长勺。公将鼓之。刿曰:“未可。”齐人三鼓。刿曰:“可矣。”齐师败绩。公将驰之。刿曰:“未可。”下视其辙,登轼而望之,曰:“可矣。”遂逐齐师。既克,公问其故。对曰:“夫战,勇气也。一鼓作气,再而衰,三而竭。彼竭我盈,故克之。夫大国,难测也,惧有伏焉。吾视其辙乱,望其旗靡,故逐之。” 3.下列对文章理解有误的一项是()A.本文通过写曹刿对战争的论述以及弱鲁战胜强齐的战例,表现了曹刿在政治上的远见卓识和军事上的卓越才能。B.“小大之狱,虽不能察,必以情。”“忠之属也。”这两句说明了政治上取信于民获得民心,是战争取得胜利的重要保证。C.本文刻画人物精炼传神,如“公将鼓之”“公将驰之”仅八个字就把鲁庄公急于求成、轻率寡谋的特点刻画出来。D.本文紧扣“论战”,详写曹刿言论,而一概不提战争的经过,这样的剪裁,详略得当,叙事清楚,有力地突出了中心。D 4.把文言文中的画线句翻译成现代汉语。(1)肉食者鄙,未能远谋。(2)夫大国,难测也,惧有伏焉。位高禄厚的人目光短浅,不能深谋远虑。像齐国这样的大国,他们的情况是难以推测的,怕他们设下埋伏。 (南开中学中考模拟)曹刿论战《左传》十年春,齐师伐我。公将战,曹刿请见。其乡人曰:“肉食者谋之,又何间焉?”刿曰:“肉食者鄙,未能远谋。”乃入见。问:“何以战?”公曰:“衣食所安,弗敢专也,必以分人。”对曰:“小惠未遍,民弗从也。”公曰:“牺牲玉帛,弗敢加也,必以信。”对曰:“小信未孚,神弗福也。”公曰:“小大之狱,虽不能察,必以情。”对曰:“忠之属也。可以一战。战则请从。”公与之乘,战于长勺。公将鼓之。刿曰:“未可。”齐人三鼓。刿曰:“可矣。”齐师败绩。公将驰之。刿曰:“未可。”下视其辙,登轼而望之,曰:“可矣。”遂逐齐师。既克,公问其故。对曰:“夫战,勇气也。一鼓作气,再而衰,三而竭。彼竭我盈,故克之。夫大国,难测也,惧有伏焉。吾视其辙乱,望其旗靡,故逐之。” 7.对“齐师伐我”一事,曹刿与“乡人”的态度有何不同?8.阅读第二段,从曹刿简短的话语中,你能感受到当时怎样的战斗形势?9.本文借曹刿之口,揭示了战争中的哪些制胜之道?“乡人”漠不关心,抱着事不关己高高挂起的态度;而曹刿则是主动参与,关心国事。两军对垒,紧张的战斗形势。①“取信于民”的战略思想;②善于抓住战机;③谨慎而又果断的战术思想。 非连续性文本阅读(重庆八中中考模拟)这不是魔法,而是垃圾分类材料一:垃圾分类作为垃圾处理的前端环节,其作用早已得到世界的公认,分类收集不仅能大幅度减少垃圾给环境带来的污染、节约垃圾无害化处理费用,更能使宝贵的自然资源得到重复利用。有人将垃圾称为“放错了地方的资源”,据保守估计,我国城市每年丢弃的可回收垃圾价值在300亿元左右。但我国的垃圾分类工作一直难以有效推进。相关调查显示,94.6%的受访民众知道垃圾分类的概念,50.9%的民众“仅了解常见的可回收或者不可回收垃圾”。我们在调查中发现,民众对垃圾分类的认知程度与实践情况大致吻合,基本不了解和从未进行分类的都是少数。约四分之一的受访者认为垃圾分类的效果不明显或完全没效果。(摘编自新华网《垃圾变资源!这不是魔法,而是垃圾分类》)材料二:垃圾分类处理系统是个复杂的系统工程,包括分类投放、分类收集、分类运输、分类处理,各个环节必须配套衔接,形成协同高效的全过程运行系统。分类投放是居民的责任,分类收集、分类运输、分类处理是政府的责任。长期以来,居民分类 投放的参与率较低、准确率较低,成为制约垃圾分类制度实施效果的突出问题,是我国垃圾分类的明显短板;与此同时,垃圾分类的目标和途径并不十分清晰,后端分类处理设施不到位、不完善,又影响了前端居民垃圾分类的积极性。国务院办公厅转发国家发改委、住建部《生活垃圾分类制度实施方案》,给出了我国推进垃圾分类的总体路线图。明确的目标,清晰的路径,让人对推进垃圾分类信心倍增。但是,这一方案再科学合理,缺乏全民动员、全民参与,也难以由蓝图变为现实。我们应当认识到,生活垃圾虽然有资源属性,但它的污染属性更不可忽视。不按要求扔垃圾,并不只是“把资源放错了地方”而已,实际上,那就是随意丢弃污染物、危害环境的行为。垃圾分类是我们作为“污染者”应尽的基本责任,责无旁贷。当然,各地政府同样不能以“居民没有分类”“缺乏资金”等理由拖延、观望,应当落实政府主体责任,坚持政府主导,根据本地特点,做好分类收集、分类运输、分类处理设施体系建设,同时制定完善惩罚和奖励的政策措施。(摘编自刘毅《垃圾分类应自扫门前雪》,《人民日报》) 1.下列有关民众对垃圾分类认知与实践相关情况的表述不正确的一项是()A.大多数民众都知道垃圾分类的概念,而50.9%的民众“仅了解常见的可回收/不可回收垃圾”。B.民众对垃圾分类的认知程度与实践情况大致吻合,基本不了解和从未进行分类的都是少数。C.有些居民不是缺乏垃圾分类意识,而是后端分类处理不到位、不完善,挫伤了他们分类的积极性。D.居民分类投放的参与率和准确率较低,是我国以往垃圾分类工作难以有效推进的主要原因。D 2.下列各项对材料的相关内容进行分析和评价,理解不正确的一项是()A.材料一阐述了垃圾分类的必要性和紧迫性,材料二分析了垃圾分类难以有效推进的原因并提出破解之道。B.居民对垃圾分类的认知与实践有助于垃圾分类的实施效果,我们要增强居民对垃圾进行分类的意识。C.生活垃圾既有资源属性,可以回收再利用,但也有污染属性,会对环境造成污染。D.针对一些地方政府常以各种理由对垃圾分类工作拖延、观望的情况,《生活垃圾分类制度实施方案》制定了完善的惩罚和奖励的政策措施。D 3.怎样才能有效改进我国的生活垃圾分类?请结合两则材料作简要概括。①从居民来说,要提高认识,掌握分类方法,养成良好习惯;②从政府来说,要完善处理设施,建立配套系统,制定奖惩措施。 一年有四个季节,每个季节都有不同的景色,而我最喜欢冬天下雪时的壮丽景色。冬天,大雪纷飞,人们好像来到了一个幽雅恬静的境界,来到了一个晶莹剔透的童话般的世界。松柏的那清香,白雪的那冰香,给人一种凉莹莹的抚慰。一切都在过滤,一切都在升华,连我的心灵也在净化,变得纯洁而又美好。美文欣赏·冬天 部编版九年级语文上册教学课件第六单元21.邹忌讽齐王纳谏 日积月累 课文助读走近作者刘向(约公元前77—公元前6),原名刘更生,字子政,西汉末年经学家、目录学家、文学家,沛县(今属江苏)人。著有《谏营昌陵疏》和《战国策叙录》,叙事简约,理论畅达、舒缓平易是其主要特色。 作品介绍《邹忌讽齐王纳谏》出自《战国策·齐策一》。春秋战国之际,七雄并立,各国间的兼并战争,各统治集团内部新旧势力的斗争,以及民众风起云涌的反抗斗争,都异常尖锐激烈。在这激烈动荡的时代,“士”作为一种最活跃的阶层出现在政治舞台上,他们施展着自己治国安邦的才干。各国统治者也认识到,人心向背,是国家政权能否巩固的决定性因素。所以,他们争相招揽人才,虚心纳谏,争取“士”的支持。 主要内容《邹忌讽齐王纳谏》写的是战国初期齐威王接受其相邹忌的劝谏而采纳群言,终于使齐国大治的故事。文章的主题思想是要求统治者能听取不同意见,而一个人之所以能听取不同意见,又在于他有自知之明。 基础过关 5.(川外附中中考模拟)下列选项中的句子填入横线处与前后衔接最恰当的一项是()中国自古以来就有重视家风家训的传统。通过家风家训建设改善村风民风,我市在这方面做了很多探索。我们深有感触的是,家风家训的确立要切合实际,;,要做好打持久战的思想准备。A.要大力宣传本地优秀的家风家训;也要借鉴外地经验B.要大力宣传本地优秀的家风家训;也要做好经验总结C.避免千篇一律,流于形式;还要认识到社会风气的改变不可能一蹴而就D.避免说一套,做一套;还要认识到社会风气的改变是一个润物细无声的过程C 6.(西大附中中考模拟)下列句子中没有语病的一项是()A.江津四面山、武隆仙女山、万盛黑山谷、北碚缙云山是重庆周边避暑最受欢迎的四大热门景区之一。B.能否贯彻落实科学发展观,是能否构建和谐社会、促进经济可持续发展的重要保证。C.体验生活、留意生活、提炼生活,抓住有意义的素材,捕捉真切的情感,澄清模糊的认识,一定会写出好的作文。D.当望着冉冉升起的五星红旗时,就会情不自禁地想起祖国五千年悠久的历史文化,想起祖国创造的辉煌成就。B 7.(北大附中中考模拟)综合性学习。在“青春随想”综合性学习活动中,同学们展开热烈讨论,现在请你完成以下任务。(1)请为本次活动拟写一条宣传标语。(2)假如你是本次活动的主持人,请写一段70字以内的开场白。宣传标语示例:①用缤纷的画笔来挥洒青春的色彩;②莫让年华付东流;③我的美丽我打造,我的青春我做主;④展望青春,放飞梦想。示例:亲爱的同学们,大家好!青春,一个充满了诗意和活力的词语,一个洋溢着阳光与欢笑的年龄。我们正值青春年少,怀揣着远大的理想。我们经历着青春的迷茫与困惑,今天,就让我们放飞青春吧! (3)请你结合下面两则材料,说说要想成为“青春形象大使”必须具备哪些条件。材料一原国家女篮队长隋菲菲在北京交通大学正式签约《中国青年报》,成为该报青春形象大使。隋菲菲靓丽、青春、健康的外形,刻苦、坚韧的训练精神,比赛时聪明灵活的个性与《中国青年报》理性、敏锐、先锋的办报要求不谋而合。在未来5年内,隋菲菲将主要负责《中国青年报》针对青年市场的品牌代言活动。材料二在某大学青春形象大使评选活动初赛现场,选手们表演了昆曲、武术、快板、书画等在其他选秀活动中难得一见的才艺,博得了评委们的阵阵掌声。有过其他选秀活动参赛经验的选手朱潇轩告诉记者:“通常的选秀活动,更多是关注选手们的外表。我认为,‘校园青春形象大使’良好的文化底蕴和综合素质也是非常重要的。”一是要形象靓丽、青春、健康;二是要有良好的文化底蕴和综合素质。 1.对下面文段中加点词的解释有错误的一项是(  )群臣吏民能面刺寡人之过者,受上赏;上书谏寡人者,受中赏;能谤讥于市朝,闻寡人之耳者,受下赏。A.面刺:当面指责   B.过:过错C.市朝:公共场合  D.闻:听到答案D“闻”在句中是“使……听到”的意思。······拓展训练 2.下列句子朗读停顿有误的一项是(  )A.臣/诚知/不如徐公美。B.群臣吏民/能面刺寡人/之过者,受/上赏。C.四境之内/莫不/有求于王。D.此/所谓/战胜于朝廷。答案B 应为:群臣吏民/能面刺/寡人之过者,受/上赏。 3.将下面的句子翻译成现代汉语。(1)皆以美于徐公。(省略句,“以”后省略了“我”)译文:(2)我孰与城北徐公美?(倒装句,应为“我与城北徐公孰美?”)译文:(3)忌不自信。(倒装句,应为“忌不信自”)译文: 答案(1)都认为(我)比徐公美。(2)我与城北徐公相比,哪一个美?(3)邹忌不相信自己(会比徐公漂亮)。解析解答本题,要根据括号内的提示,或补充省略成分,或调整语序。另外要注意落实关键词的意思,如“孰”“自信”等。 1.对下列句中“于”字的用法分类正确的一项是(  )①欲有求于我也 ②于是入朝见威王 ③以其无礼于晋 ④皆以美于徐公 ⑤且贰于楚也 ⑥能谤讥于市朝,闻寡人之耳者 ⑦燕、赵、韩、魏闻之,皆朝于齐 ⑧此所谓战胜于朝廷A.①③⑤/⑥⑦⑧/②/④  B.①③⑤⑦/②④/⑥⑧C.①③⑤⑦/②⑥⑧/④  D.①③⑤/②⑥⑧/④/⑦答案D 本题考查对文言词一词多义的掌握能力。①③⑤句中的“于”意思为“对,对于”;②⑥⑧句中的“于”的意思为“在”;④句中的“于”的意思为“比”;⑦句中的“于”的意思为“到”。所以选D。能力提升全练 2.填空。(1)《邹忌讽齐王纳谏》选自历史著作《》。“讽”的意思是。(2)文章中写邹忌从妻、妾、客的回答中受到启发的句子是:。表示威王采取纳谏措施后在国内收到效果的句子是:。(3)请列举两个出自《战国策》的成语。答: 答案(1)战国策·齐策一 讽谏,用含蓄的话委婉地规劝(2)吾妻之美我者,私我也;妾之美我者,畏我也;客之美我者,欲有求于我也令初下,群臣进谏,门庭若市;数月之后,时时而间进;期年之后,虽欲言,无可进者(3)(示例)门庭若市、狐假虎威、画蛇添足、鹬蚌相争等。(答出两个即可)解析本题考查对文学常识以及课文内容的理解能力。需在深入理解课文的基础上作答。 3.这篇文章通过叙述邹忌劝齐威王纳谏的故事,强调善于倾听不同意见的好处。这个故事告诉我们:统治者要治理好国家,必须;一个人要不断进步,必须有。(用四字词语填空)答案广开言路(善于纳谏) 自知之明(自省之心)解析“治理好国家”必须怎样,可从齐威王的成功做法(善于纳谏)方面思考;“一个人要不断进步”必须具备什么,可从邹忌没有陶醉于别人的溢美之词,而是“窥镜而自视”,后又“暮寝而思之”的自省精神方面作答。 1.根据成语分析,在横线处选填汉字不正确的一项是(  )A.竽充数分析:这个成语说的是不会吹竽的南郭先生混在吹竽的队伍里充数。后借指没有真正的才干,而混在行家里面充数,或拿不好的东西混在好的里面充数。横线处应填“滥”。B.破沉舟分析:这个成语说的是项羽与秦军打仗,过河后砸锅沉船,表示与秦军决一死战的故事。后用来比喻下决心,不顾一切干到底。横线处应填“斧”。C.闻鸡起分析:这个成语讲的是晋人祖逖立志为国效力,与刘琨互相勉励,半夜听到鸡叫就起床舞剑。后用来指志士及时奋发。横线处应填“舞”。 D.门若市分析:这个成语说的是邹忌讽谏齐王后,门前和院子里向齐王进谏的人很多,像市场一样。现在形容交际来往的人很多。横线处应填“庭”。答案B 横线处应填“釜”。“釜”是一种器物,圆底而无足,必须安置在炉灶之上或是以其他物体支撑煮物,釜口也是圆形,可以直接用来煮、炖、煎、炒等,可视为现代所使用的“锅”的前身。 2.进谏难,需要勇气和智慧;纳谏也难,需要气度和胸怀。文中的邹忌和齐威王,你更欣赏谁?请简要说明理由。答:答案(示例1)欣赏邹忌。因为他善于讽谏,说话讲究技巧。同时,他能正视自己,有自知之明,有反思的习惯和自省能力。(示例2)欣赏齐威王。因为他虚心纳谏,知错能改,有气度,有胸怀,也不失为一个明君。解析本题考查人物评价能力。首先要根据自己对人物形象的把握选择其一,表明态度;然后列举人物的优秀品质,如有智慧、有气度等进行阐述。用词要准确,理由要充分。 文言文阅读(川外附中中考模拟)于是入朝见威王,曰:“臣诚知不如徐公美。臣之妻私臣,臣之妾畏臣,臣之客欲有求于臣,皆以美于徐公。今齐地方千里,百二十城,宫妇左右莫不私王,朝廷之臣莫不畏王,四境之内莫不有求于王:由此观之,王之蔽甚矣。”王曰:“善。”乃下令:“群臣吏民能面刺寡人之过者,受上赏;上书谏寡人者,受中赏;能谤讥于市朝,闻寡人之耳者,受下赏。”令初下,群臣进谏,门庭若市;数月之后,时时而间进;期年之后,虽欲言,无可进者。燕、赵、韩、魏闻之,皆朝于齐。此所谓战胜于朝廷。 2.翻译下列语句。能谤讥于市朝,闻寡人之耳者,受下赏。3.文段写齐威王的态度只用一个“善”字,而写发布命令的内容、群臣进谏的情况、修明政治的结果却较详细,这样写有何好处?能在公共场合指责讥刺我的过失,传到我耳朵里的,可得下等奖赏。因为写了后者,齐威王的一个“善”字就抵得了一大段描写。这一略一详,略得余味无穷,详得具体生动。 (重庆一中中考模拟)邹忌讽齐王纳谏《战国策》邹忌修八尺有余,而形貌昳丽。朝服衣冠,窥镜,谓其妻曰:“我孰与城北徐公美?”其妻曰:“君美甚,徐公何能及君也?”城北徐公,齐国之美丽者也。忌不自信,而复问其妾曰:“吾孰与徐公美?”妾曰:“徐公何能及君也?”旦日,客从外来,与坐谈,问之客曰:“吾与徐公孰美?”客曰:“徐公不若君之美也。”明日徐公来,孰视之,自以为不如;窥镜而自视,又弗如远甚。暮寝而思之,曰:“吾妻之美我者,私我也;妾之美我者,畏我也;客之美我者,欲有求于我也。”于是入朝见威王,曰:“臣诚知不如徐公美。臣之妻私臣,臣之妾畏臣,臣之客欲有求于臣,皆以美于徐公。今齐地方千里,百二十城,宫妇左右莫不私王,朝廷之臣莫不畏王,四境之内莫不有求于王:由此观之,王之蔽甚矣。”王曰:“善。”乃下令:“群臣吏民能面刺寡人之过者,受上赏;上书谏寡人者,受中赏;能谤讥于市朝,闻寡人之耳者,受下赏。”令初下,群臣进谏,门庭若市;数月之后,时时而间进;期年之后,虽欲言,无可进者。燕、赵、韩、魏闻之,皆朝于齐。此所谓战胜于朝廷。 6.下列对文章的分析理解,不正确的一项是()A.本文第一段写出了邹忌头脑冷静,不为奉承所迷惑。B.本文第二段运用两组排比句式增强了语势,给人以无可辩驳之感。C.本文第三段从侧面表现邹忌的精明能干,具有治国之才。D.本文的主旨是通过邹忌“暮寝而思之”,悟出了人们由于种种原因,不会说出事情的真相的道理。D 7.用现代汉语翻译下面句子。(1)期年之后,虽欲言,无可进者。(2)群臣吏民能面刺寡人之过者,受上赏。一年以后,即使想进言,也没有什么可说的了。所有的大臣、官吏、百姓,能够当面批评我的过错的,可得上等奖赏。 (巴蜀中学中考模拟)邹忌讽齐王纳谏《战国策》邹忌修八尺有余,而形貌昳丽。朝服衣冠,窥镜,谓其妻曰:“我孰与城北徐公美?”其妻曰:“君美甚,徐公何能及君也?”城北徐公,齐国之美丽者也。忌不自信,而复问其妾曰:“吾孰与徐公美?”妾曰:“徐公何能及君也?”旦日,客从外来,与坐谈,问之客曰:“吾与徐公孰美?”客曰:“徐公不若君之美也。”明日徐公来,孰视之,自以为不如;窥镜而自视,又弗如远甚。暮寝而思之,曰:“吾妻之美我者,私我也;妾之美我者,畏我也;客之美我者,欲有求于我也。”于是入朝见威王,曰:“臣诚知不如徐公美。臣之妻私臣,臣之妾畏臣,臣之客欲有求于臣,皆以美于徐公。今齐地方千里,百二十城,宫妇左右莫不私王,朝廷之臣莫不畏王,四境之内莫不有求于王:由此观之,王之蔽甚矣。”王曰:“善。”乃下令:“群臣吏民能面刺寡人之过者,受上赏;上书谏寡人者,受中赏;能谤讥于市朝,闻寡人之耳者,受下赏。”令初下,群臣进谏,门庭若市;数月之后,时时而间进;期年之后,虽欲言,无可进者。燕、赵、韩、魏闻之,皆朝于齐。此所谓战胜于朝廷。 10.下列对文章理解和分析不正确的一项是()A.作者对邹忌由“受蔽”到“清醒”过程的叙写,实际上就是对“受蔽”与“除蔽”道理的说明,是紧扣文章的中心来写的。B.文章末尾写齐王纳谏,进谏者由多而少,一年后,由少而无,用的是夸张写法。C.文章对邹忌的描写,不仅注重形体的细致刻画,而且注重细节和对话描写,表现人物丰富的内心世界。D.文章虽短,却有完整的故事情节,写了进谏的缘起,进谏的内容、进谏的结果,环环相扣,结构完整。A 11.你从邹忌进谏、齐威王纳谏中分别得到什么启示?①邹忌善于进谏,给我的启示是:在生活中,在与人的交往中,在提意见时,我们要讲究说话的技巧,要看对象,注意场合,要得体,动之以情晓之以理,让对方心悦诚服。②齐威王纳谏给我的启示:要虚心听取别人的意见;要胸怀宽广;善于听取别人的意见有利于自己的成长。 非连续性文本阅读材料一:载人飞船虽然可以将航天员送入太空,但无法携带较大的载荷进行空间实验,飞船电力供应也无法满足大型载荷或大规模空间实验的需求。因此,建设能部署大量实验载荷并提供充足能源开展实验的空间站,是航天发展的必然趋势。“国际空间站”实践表明,它是人类最好的太空技术实验和应用平台。我国空间站建成后,将具备完全自主的载人航天能力。(摘编自欧阳自远主编《十万个为什么》)材料二:天舟一号作为我国首个空间货物运输飞船,飞行任务取得圆满成功。它突破了货物运输、推进剂补加等关键技术,使我国跻身于少数几个具备大吨位空间货运能力的国家。天舟一号还是全球首个掌握全自主快速交会对接技术的飞船,并且在航天领域首次实现了100V高压锂离子蓄电池组在轨应用。天舟一号是我国载人航天工程“三步走”发展战略第二步的收官之作,标志着我国即将开启空间站时代。 注:“t”是计量单位,吨。上行货重比=上行能力/发射质量,上行货重比越高,货物上行效率越高。(摘编自白明生等《天舟一号货运飞船研制》)材料三:中国空间站具备后发优势。建成后,会单独发射一个光学舱,光学舱里架设一套口径两米的巡天望远镜,可以对40%以上的天区进行观测,可以源源不断地为科学家们研究宇宙学和天文学提供海量的科学数据。同时,空间站还将拥有完整的可再生生命保障系统。(摘编自《中国的空间站什么样》) 1.下列对材料有关内容的理解,不正确的一项是()A.载人飞船能将航天员送入太空,但其电力供应无法满足大型载荷的需求。B.空间站是航天发展的必然选择,是人类最好的太空技术实验和应用平台。C.目前,我国已跻身于世界少数几个具备大吨位空间货运能力的国家之列。D.天舟一号飞行任务取得圆满成功,标志着我国已经正式进入空间站时代。D 2.中国天舟一号与国外货运飞船相比,具有哪些优势?请根据材料二简要分析。3.我国建成后的空间站具有什么特点?请根据材料三简要概括。①全球率先掌握全自主快速交会对接技术;②在航天领域首次实现了100V高压锂离子蓄电池组在轨应用;③货物运送上行能力位居第二;④货物运送上行货重比最高。①配备一套能对40%以上的天区进行观测的巡天望远镜;②拥有完整的可再生生命保障系统。 冬天是一位风烛残年的老人,他步伐蹒跚,历尽沧桑,饱受风雪的磨难,但是他依然不改对大自然的爱恋。你看,越冬的麦苗身上,是他盖上的棉被;繁叶落尽的树下,是他执意培上的冬肥;害虫肆虐的田间,是他施上的杀虫剂。他使青松更苍翠,使冬梅更芬芳,使天更高,地更远,人们更坚强。他把自己最好的东西都奉献给了这个世界,对于自己,没有丝毫保留,当春天的脚步逐渐走近的时候,他又是那样断然离去,把希望与祝愿给了新的一年。美文欣赏·冬天 日积月累 课文助读走近作者司马迁(约前145—?),西汉著名史学家、文学家。字子长,夏阳人。 作品介绍《史记》记载了上自传说中的黄帝时代,下至汉武帝时代,长达三千年的历史。全书包括12本纪、10表、8书、30世家、70列传,共130篇。本纪记述了历代帝王的事迹;世家主要记述诸侯王国的事迹;列传则是人臣之传。《史记》是我国第一部纪传体史书。鲁迅称它是“史家之绝唱,无韵之离骚。” 主要内容课文以陈胜、吴广的活动为线索,生动地描述了这一场起义的发生、发展过程,鲜明地揭示了它的实质:一场得到广大人民支持的,反对暴力统治的农民运动。作者通过对典型历史环境的描写和人物语言、行动的刻画,生动地再现了农民起义领导者和组织者的形象。 基础过关 5.(巴蜀中学中考模拟)下列句子没有语病的一项是()A.中美两国经济总量约占全球40%左右,贸易战不能解决问题,只能导致两败俱伤。B.国务院教育督导委员会办公室发布预警提醒各地教育行政部门和学校要时刻绷紧安全弦,坚决防止溺水事故不再发生。C.只有从不同文明中寻求智慧汲取营养我们才能携手解决人类共同面临的各种问题。D.通过开展经典诗文诵读活动,使我们对中华民族优秀传统文化有了进一步的了解。C 6.(重庆一中中考模拟)把下列句子组成语段,顺序排列最合理的一项是()①这就需要具有辩证眼光和战略定力,发现那些不受短期波动影响,能为中国经济长远发展保驾护航的“深层优势”。②这样的“深层优势”,体现为善于求变应变的“改革优势”。③这样的“深层优势”,首先表现为巨大的“市场优势”。④正确研判中国经济,不仅要看短期波动,也要从时间轴上看整体、看大势、看实质。⑤这样的“深层优势”,更是中国特色社会主义所独有的“制度优势”。A.①③②⑤④B.④①③②⑤C.④①③⑤②D.①③④⑤②B 文言文阅读(川外附中中考模拟)陈胜者,阳城人也,字涉。吴广者,阳夏人也,字叔。陈涉少时,尝与人佣耕,辍耕之垄上,怅恨久之,曰:“苟富贵,无相忘。”佣者笑而应曰:“若为佣耕,何富贵也?”陈涉太息曰:“嗟乎!燕雀安知鸿鹄之志哉!”二世元年七月,发闾左適戍渔阳,九百人屯大泽乡。陈胜、吴广皆次当行,为屯长。会天大雨,道不通,度已失期。失期,法皆斩。陈胜、吴广乃谋曰:“今亡亦死,举大计亦死;等死,死国可乎?”陈胜曰:“天下苦秦久矣。吾闻二世少子也,不当立,当立者乃公子扶苏。扶苏以数谏故,上使外将兵。今或闻无罪,二世杀之。百姓多闻其贤,未知其死也。项燕为楚将,数有功,爱士卒,楚人怜之。或以为死,或以为亡。今诚以吾众诈自称公子扶苏、项燕,为天下唱,宜多应者。”吴广以为然。乃行卜。卜者知其指意,曰:“足下事皆成,有功。然足下卜之鬼乎?”陈胜、吴广喜,念鬼,曰:“此教我先威众耳。”乃丹书帛曰“陈胜王”,置人所罾鱼腹中。卒买鱼烹食,得鱼腹中书,固以怪之矣。又间令吴广之次所旁丛祠中,夜篝火,狐鸣呼曰:“大楚兴,陈胜王。”卒皆夜惊恐。旦日,卒中往往语,皆指目陈胜。 3.结合全文,说说下列对文章理解分析不正确的一项是()A.本文节选自《史记》,“世家”是《史记》的一种体例,是记述世代相承的诸侯王国兴衰事迹的。司马迁把雇农出身的陈涉列入“世家”,主要是肯定他发动起义的历史功绩。B.“陈胜者,阳城人也,字涉。吴广者,阳夏人也,字叔。”句中“……者,……也”是古汉语判断句式的一种标志。C.“嗟乎!燕雀安知鸿鹄之志哉!”充分表达了陈涉非凡的个性和远大抱负。成语“鸿鹄之志”即出自此处,用以指人有远大抱负。D.“天下苦秦久矣”和“会天大雨,道不通,度已失期。失期,法皆斩。”分别说明了本次起义的直接原因和根本原因。4.陈胜能够从一个佣耕之人成为封建社会第一次农民起义的领导人,你认为是什么样的才能成就了他?请根据选段内容作答。D①对当前的形势具有敏锐的洞察力;②做事周密细致;③有很强的组织领导才能。 阅读提高牛犊面①牛犊面是内蒙古东部地区的特色面食。虽叫“牛犊面”,但无论它的原料还是形状,都与牛犊没有任何关系,只是据说一碗牛犊面的奶香能引来周围的牛犊,故称“牛犊面”。②牛犊面的主料是稀奶油和面片。③如何制作牛犊面呢?首先,制作稀奶油。稀奶油的做法很讲究:把鲜牛奶盛进盆里,将其自然放酸之后,上面会结一层稀奶油。所结稀奶油的厚度与盆里的鲜牛奶量成正比,鲜牛奶越多,所结的稀奶油也就越多。稀奶油通常都被放进用纱布做成的袋子里,吊在空中存放。这样,能滤净残留在稀奶油中的奶精,稀奶油的浓度就会增加。稀奶油的浓度越高,做出来的牛犊面就越香。其次,制作面片。在内蒙古东部,80%的家庭喜欢用荞面做牛犊面,若没有荞面,也可以用白面代替。将荞面或白面和好擀开之后,切成边长2厘米左右的四方形面片,等待下锅。接着,文火煎炸。锅热之后,先将事先准备好的稀奶油放入热锅里用文火煮,当锅内的稀奶油微微沸腾时,放入切好的面片,加入少许盐,煎炸。等锅内的稀奶油变成黄油,面片像西下的夕阳般白里泛着金黄时,即可出锅。出锅 时面片与奶油融为一体,入口时浓浓的奶香和面片丝滑的口感,让人久久难忘。④尽管牛犊面是蒙古族的特色面食,却不经常吃。早些年,由于技术条件有限,牛犊面只能在夏天和初秋时节吃,稀奶油的炼制过程需要一个适合的温度,夏天和初秋的温度适宜炼制稀奶油。不过,随着炼乳技术的提高,现在,冬季也能吃到这道可口的面食了。机器提炼的稀奶油与人工提炼的相差无几,味道同样鲜美。⑤小时候,只有逢重大节日母亲才会做牛犊面。为了做一顿牛犊面,母亲要做近半个月的准备。每天挤完牛奶之后盛在盆里,等发酵后提炼稀奶油。母亲对每个步骤都很小心,生怕整个过程中洒了一滴牛奶。蒙古族的习俗中,牛奶是高贵、纯美和善良的象征。牛奶做成的奶豆腐,奶酪、奶油、奶皮等奶食品也具有很高的地位。记得我每次出远门母亲都会弹洒奶汁,保佑我平安。起初我只觉得母亲过于迷信,后来才感知到这一习俗是希望孩子在大自然的怀抱中平平安安。已故的内蒙古大学教授、著名的蒙古族诗人巴·布林贝先生,曾写过一首名为《心与乳》的诗,用诗意语言描绘蒙古与乳汁的关系。稀奶油作为牛奶直接提炼的奶 食品,在蒙古族的生活习俗中被看作是上等美食。吃到牛犊面的人一定是远到而来的贵客或受家族爱戴尊敬的长辈。若有朋自远方来,勤劳的蒙古族妇女也会做一顿可口的牛犊面给宾朋享用。⑥小小一碗牛犊面里酸、香、咸三味融为一体,让你有一种妙不可言的感觉。看似简简单单的牛犊面,其背后包含着近半个月不为人知的等待与准备,这也正印证了蒙古族的生活哲学:将复杂的生活简单化,将复杂藏在简单背后。(摘自《读者〈饮食科学〉》,有删改) 1.本文的说明对象是什么?围绕说明对象主要介绍了哪些内容?2.文段①中加点词能否删去?为什么?虽叫“牛犊面”,但无论它的原料还是形状,都与牛犊没有任何关系,只是据说一碗牛犊面的奶香能引来周围的牛犊,故称“牛犊面”。牛犊面;牛犊面的主料、牛犊面的制作方法、“我”与牛犊面相关的记忆、牛犊面背后的生活哲学。不能去掉。因为“据说”表示这只是作者道听途说而来,如果去掉,就成了肯定的语气,与事实不符,“据说”体现了说明文语言的准确性,所以不能去掉。 3.文段③主要运用了哪些说明方法?请选择其中一种说明方法分析其作用。4.文段⑤插入小时候母亲做牛犊面、弹洒奶汁及引用蒙古族诗人巴·布林贝的诗,有何作用?5.看似简简单单的牛犊面却印证了蒙古族的什么生活哲学?结合生活实际,谈谈你的理解。列数字和打比方。打比方:面片像西下的夕阳般白里泛着金黄时,即可出锅。作用:生动形象地介绍了面片的颜色。介绍牛犊面背后蒙古人的生活哲学。生活哲学:将复杂的生活简单化,将复杂藏在简单背后。理解:生活其实是很复杂,有很多的人或事让我们的心变得纷乱,此时,就需要我们进行取舍,关注自己的本心,把复杂的问题与生活简单化。 冬,她是圣洁的象征,当冰雪跨进冬的门槛,整个世界便被凝结。湖面、河面上都结上了一层厚厚的冰,房屋楼阁在雪中静默,土墩、田坎在银光中陶醉,“山舞银蛇,原驰蜡象”。雪花如明月轻洒,树枝如梨花绽放,绵绵的“柳絮”在空中荡游,梦幻般的“天使的羽毛”从天而降,整个世界都踏进了雪国,都为之沉醉了。美文欣赏·冬天 谢谢观看 部编版九年级语文上册教学课件第六单元23.出师表 日积月累 课文助读走近作者诸葛亮(181—234),字孔明,琅琊阳都(今山东沂南)人,三国时著名的政治家、军事家。 主要内容《出师表》是诸葛亮出师伐魏临行前写给后主刘禅的奏章。文中以恳切的言辞,劝说后主要继承先帝遗志,广开言路,严明赏罚,亲贤臣,远小人,完成兴复汉室的大业。也表达了诸葛亮报答先帝的知遇之恩的真挚感情和北定中原的决心。 基础过关 7.(西大附中中考模拟)下列句子中没有语病的一项是()A.中国企业、基金会、公益组织等发起相关倡议,进一步展现推动《巴黎协定》的决心与努力,引发关注。B.气象专家提醒,华南地区将面临严峻的风雨,广东、香港、澳门、海南、广西、云南等地须严加防范。C.文章的主题确定以后,还要根据主题的需要收集、组织、选择材料,以确保材料为主题服务。D.《故宫日历》系列至今已出版10年,作为故宫特色出版物,至今已形成了品牌效应,受到社会公众所喜爱和追捧。A 8.(2019重庆B)依次填入下面语段横线处的句子,排序正确的一项是()哲学,一门深奥晦涩的学科;儿童,一个直接简单的群体。这两者似乎风马牛不相及。。。。。。也因此,不少思想家认为:儿童天生就是哲学家。①不少大人都对哲学退避三舍,小孩子就更没办法学②而对世界充满好奇心、爱提问的儿童无时无刻不表现出一种“爱智慧”的天性③因此,不少人初听到“儿童哲学”这个名词时,都觉得不可思议④从这个意义上来说,和大多数对世界司空见惯、冷漠麻木的成人相比,儿童离哲学要近得多⑤实际上,哲学的本义是“爱智慧”,就是对智慧的热爱与追求A.③①⑤②④B.⑤②③①④C.⑤④②①③D.③①②⑤④A 9.(育才中学中考模拟)综合性学习。为了让学生走进端午、传承文化,重庆育才中学初三年级某班准备举行“话端午·思传统”的主题活动。请你也积极参与并按要求作答。【诗话端午】(1)根据竺思嘉搜集的端午材料,不是描写端午活动的一项是()A.杨柳枝,芳菲节,所恨年年赠离别。B.彩线轻缠红玉臂,小符斜挂绿云鬟。C.樱桃桑葚与菖蒲,更买雄黄酒一壶。D.共骇群龙水上游,不知原是木兰舟。端午节起源说法不一,最早可追溯到春秋,距今已有两千五百多年的历史。端午节在农历五月初五,吃粽子和赛龙舟是端午的两大主题。但由于地域广大,民族众多,加上许多故事传说,各地也有着不尽相同的习俗,主要有:女儿回娘家、挂钟馗像、贴午叶符,割艾草、挂菖蒲、佩香囊,赛龙舟、放风筝、荡秋千,饮雄黄酒,吃五毒饼、咸蛋、粽子等。A 【寄语端午】(2)请仿照例句,自己创作一句端午寄语,表现端午节的特点,句式不限。[示例]清明节:清明雨洗烈士墓,挥笔祭出英雄魂。中秋节:月到中秋圆,亲人心相连。[仿句]端午节:,。五月初五端午节家家户户粽飘香 1.(2020独家原创)下列加点字的注音完全正确的一项是(  )A.崩殂(cú) 忠谏(jiàn) 疲弊(bì) 陛下(bì)B.费祎(yī)行阵(xíng)郭攸之(yōu) 性行(háng)C.驽钝(nú)攘除(rǎng)  夙夜(sù)  遗诏(zhāo)D.庶竭(shù)  斟酌(zhuó)菲薄(fēi)  和睦(mù)答案A B.行háng阵,性行xíng。C.诏zhào。D.菲fěi。················拓展训练 2.解释下面句子中加点的词语。(1)益州疲弊        (    )(2)此诚危急存亡之秋也  (    )(3)以光先帝遗德  (    )(4)是以先帝简拔以遗陛下  (    )(5)恐托付不效  (    )答案(1)困苦穷乏 (2)时候 (3)发扬光大 (4)给予 (5)成功解析可根据积累直接作答,也可根据句意推测词义。注意古今异义词“秋”字在句中的意思。······ 3.下列加点词的解释完全正确的一项是(  )答案C A.痛恨:痛心、遗憾。B.光:发扬光大。D.第二个“以”应为连词“来”。 1.下列加点词语的古今意义没有发生变化的一项是(  )A.先帝不以臣卑鄙       B.诚宜开张圣听C.未尝不叹息痛恨于桓、灵也  D.苟全性命于乱世答案D A.“卑鄙”古义为“社会地位低微,见识短浅”,今义为“(语言、行为)恶劣;不道德”。B.“开张”古义为“扩大”,今义为“商店等设立后开始营业;经商的人指一天中第一次成交;泛指某种事物开始”。C.“痛恨”古义为“痛心、遗憾”,今义为“极端憎恨或悔恨”。D.“苟全”古今义均为“苟且保全(生命)”。········能力提升全练 2.下列各项中的“以”与例句中的“以”意思相同的一项是(  )例句:以塞忠谏之路也A.以告先帝之灵B.以伤先帝之明C.愚以为宫中之事D.故临崩寄臣以大事也答案B B项和例句中的“以”都是“以致”的意思。A.用来。C.与“为”连用,认为。D.把。···· 1.下列加点词语意思和用法相同的一项是(  )答案A A.在。B.因为;把。C.成功;功效。D.用来……的;……的原因。 2.下列句中的“亡”与“此诚危急存亡之秋也”中的“亡”意思相同的一项是(  )A.亡羊补牢     B.河曲智叟亡以应C.亡命天涯  D.出则无敌国外患者,国恒亡答案DA.“亡”意为“丢失”。B.“亡”同“无”,没有。C.“亡”是“逃亡”之意。D项和例句中的“亡”都是“灭亡”。···· 3.本文虽题为《出师表》,但很少谈出师,而是用大量篇幅规劝后主修明政治,可谓晓之以理、动之以情。请以“报答”“启发”为首词写一组不少于七言的对偶句,表达出诸葛亮的情辞恳切、忠心耿耿。报答;启发。答案(报答)先帝知遇之恩,句句忠言 (启发)后主奋发图强,声声顺耳解析对偶句要涵盖课文内容,注意符合对偶的特点。 文言文阅读(重庆八中中考模拟)【甲】水调歌头丙辰中秋,欢饮达旦,大醉,作此篇,兼怀子由。明月几时有?把酒问青天。不知天上宫阙,今夕是何年。我欲乘风归去,又恐琼楼玉宇,高处不胜寒。起舞弄清影,何似在人间。转朱阁,低绮户,照无眠。不应有恨,何事长向别时圆?人有悲欢离合,月有阴晴圆缺,此事古难全。但愿人长久,千里共婵娟。【乙】北冥有鱼北冥有鱼,其名为鲲。鲲之大,不知其几千里也;化而为鸟,其名为鹏。鹏之背,不知其几千里也;怒而飞,其翼若垂天之云。是鸟也,海运则将徙于南冥。南冥者,天池也。《齐谐》者,志怪者也。《谐》之言曰:“鹏之徙于南冥也,水击三千里,抟扶摇而上者九万里,去以六月息者也。”野马也,尘埃也,生物之以息相吹也。天之苍苍,其正色邪?其远而无所至极邪?其视下也,亦若是则已矣。 【丙】出师表(节选)先帝创业未半而中道崩殂,今天下三分,益州疲弊,此诚危急存亡之秋也。然侍卫之臣不懈于内,忠志之士忘身于外者,盖追先帝之殊遇,欲报之于陛下也。诚宜开张圣听,以光先帝遗德,恢弘志士之气,不宜妄自菲薄,引喻失义,以塞忠谏之路也。宫中府中,俱为一体,陟罚臧否,不宜异同。若有作奸犯科及为忠善者,宜付有司论其刑赏,以昭陛下平明之理,不宜偏私,使内外异法也。侍中、侍郎郭攸之、费祎、董允等,此皆良实,志虑忠纯,是以先帝简拔以遗陛下。愚以为宫中之事,事无大小,悉以咨之,然后施行,必能裨补阙漏,有所广益。将军向宠,性行淑均,晓畅军事,试用于昔日,先帝称之曰能,是以众议举宠为督。愚以为营中之事,悉以咨之,必能使行阵和睦,优劣得所。亲贤臣,远小人,此先汉所以兴隆也;亲小人,远贤臣,此后汉所以倾颓也。先帝在时,每与臣论此事,未尝不叹息痛恨于桓、灵也。侍中、尚书、长史、参军,此悉贞良死节之臣,愿陛下亲之信之,则汉室之隆,可计日而待也。 1.下列对《水调歌头·明月几时有》赏析正确的一项是()A.“水调歌头”是词牌名。词前小序点明作词的时间、地点和缘由。B.“起舞弄清影,何似在人间”,望月亮,无限惆怅,流露出作者抑郁伤感之情。C.“转朱阁,低绮户”,写词人在朱阁转悠,倚户沉思,间接写出词人赏月时间之长。D.全词运用形象的描绘和浪漫主义的想象,紧紧围绕中秋之月展开描写、抒情和议论。D 4.用现代汉语翻译下面的句子。(1)怒而飞,其翼若垂天之云。(2)亲贤臣,远小人,此先汉所以兴隆也。当它奋起而飞的时候,那展开的双翅就像天边的云。亲近贤臣,疏远小人,这是先汉兴旺发达的原因。 5.【乙】文如何表现大鹏“志存高远”的形象特点?6.诸葛亮对后主有哪几方面的担忧?表现了诸葛亮怎样的性格特点?这只神奇的大鸟岂止是大,还要腾空而起,还要乘海风作万里之游,由北海直飞南海天池。①不宜妄自菲薄,引喻失义,以塞忠谏之路;②刑赏偏私,使内外异法;③亲小人,远贤臣。表现了诸葛亮忠心耿耿、任人唯贤的性格特点。 非连续性文本阅读(巴蜀中学中考模拟)【材料一】非物质文化遗产(简称“非遗”),是指各族人民世代相传并视为其文化遗产组成部分的各种传统文化表现形式,以及与传统文化表现形式相关的实物和场所。它是人类世代相传、口传心授的活态文化遗产,凝聚着民族的集体智慧和情感精神。有别于物质文化遗产,其最大的特点就是“活态流变”,强调的是以人为核心的技艺、经验、精神,并多以身口相传而延续。因此对于非遗的保护和传承来说,传承人显得尤为重要。近几年公布的第四批、第五批国家级非遗代表性传承人年龄构成出现了“①”的发展趋势,但整体结构依然呈现“老年型”的格局。随着老艺人的辞世,新的传承人难以为继,一些民间传统文化濒临灭绝或正在消失。因此,想要中国非遗多彩绽放,年轻力量不可或缺。 【材料二】非遗保护的根本目标是“保护非遗的生命力”,因此,保护非遗更要传承非遗,要使其与时代同行。作为华阴老腔第十代传人,张喜民老人一直为后继无人而忧虑。孙子张猛一直对老腔不太上心。直到歌手谭维维搭档张喜民为华阴老腔赋予了新的内涵与活力,并将新编歌曲《华阴老腔一声喊》带上猴年春晚舞台,张猛看后才终于开了窍。京剧是国粹艺术,但在传承上也依然面对着社会需求与传统艺术隔膜的困境。京剧裘派嫡系第四代继承人裘继戎,没有按部就班的延续传统,他另辟蹊径,将西方Popping的舞蹈元素、太极的身段与京剧的唱念做打多元融合,用新的形式实现了传统艺术的跨界。他说:“艺术要摆脱匠气,没有火气,流淌人气。”但是,像华阴老腔、京剧流派面对的传承困境是当今普遍现象,我们需要有更多思考和更多探索,不断赋予其②。【材料三】非遗以其独特的文化色彩,谱写了灿烂的中华文明。然而,当今非遗的保护和传承却遭遇了前所未有的危机:工业文明对植根于农耕文化的非遗形成致命冲击、城镇化出现和传统村落的消 失使一些地方民俗和技艺濒临灭绝、新一代非遗传承人的断层状况导致技艺失传、多种文化观的现代人对非遗产生文化需求的距离感。2012年底被撤销非遗资格的黎族文身,虽被称为海南黎族的“敦煌壁画”,但因不适应现代社会的发展需求,中断了活态传承,其消失成为不可避免的趋势。另一方面,人们也通过各种方式保护和传播非遗。抖音近期公布相关数据:截至2019年4月,1372项国家级非遗代表项目中,有1214项在抖音平台上有相关内容的传播,产生了超过2400万条视频和超过1065亿次播放量。2018年,抖音联合中国民协皮影艺术委员会、北京龙在天皮影文化博物馆共同发起“皮一下很开心”话题,累计播放超28亿次。龙在天戏团的王熙团长表示,按照他们演出的平均观众数,要达到这个播放量,需要每天演出,且持续800年。过去的经典,重新被现代大众所欣赏。 1.阅读材料一、材料二,在横线处依次填入词语,全都正确的一项是()A.①年轻化②新的时代内涵和现代表达形式B.①年轻型②现代表达形式和新的时代内涵C.①老年化②现代表达形式和新的时代内涵D.①老年型②新的时代内涵和现代表达形式2.依据上面三则材料。当今非遗的传承面临的共同困境可以概括为。(限8个字以内)A后继无人 3.针对非遗的保护和传承,人们也有自己的思考。阅读下面网友的评论,结合文本内容,说说你是如何看待非遗传承的。潇潇雨:如果根据现代人的需要对这些非遗进行改造,不就很好地传承下来了吗?远方的时光:对于老祖宗留下的非遗文化,咱现代人可不能用自己的爱好去随便篡改,否则不就缺失了传统文化的本真吗?麦芽糖不甜:有些非遗早就不适应现代人的需求,消失也是必然的,有什么可惜呢?示例:对于适应时代发展需求的非遗,我们应积极传承。因为非遗凝聚着民族智慧和情感精神,是我国的文化瑰宝。对于非遗传承的困境,我们可以尝试一些积极的探索。 黄昏的雪,深切切的,好像有千丝万缕的情绪似的,又像海水一般汹涌,能够淹没一切,还有一丝揭开藏头露尾般的裸露感。雪花形态万千、晶莹透亮,好像出征的战士,披着银色的盔甲,又像是一片片白色的战帆在远航……美文欣赏·冬天 部编版九年级语文上册教学课件第六单元24.诗词曲五首 日积月累 课文助读走近作者郭茂倩(1041—1099),字德粲,宋代郓州须城(今山东东平)人。编有《乐府诗集》百卷传世,以解题考据精博,为学术界所重视。岑参(715—770),唐代边塞诗人,荆州江陵(现湖北江陵)人,代表作是《白雪歌送武判官归京》。辛弃疾(1140—1207),原字坦夫,后改字幼安,号稼轩,山东东路济南府历城县人。南宋豪放派词人、将领,有词集《稼轩长短句》传世。文天祥(1236—1283),初名云孙,字宋瑞,一字履善,道号浮休道人、文山。江西吉州庐陵人,宋末政治家、文学家,爱国诗人。著有《文山诗集》《指南录》《指南后录》《正气歌》等。张养浩(1270—1329),汉族,字希孟,号云庄,山东济南人,元代著名散曲家。作品有《山坡羊·潼关怀古》等。 写作背景《十五从军征》出自《乐府诗集》。描绘了一个在外征战的老兵返乡途中与到家之后的种种场景。这首乐府诗不仅抒发了这一老兵“少小离家老大回”的情感,也反映了当时社会现实的黑暗,具有一定的史诗意义。《白雪歌送武判官归京》作于公元754年(唐玄宗天宝十三载)。当时西北边疆一带,战事频繁,岑参怀着到塞外建功立业的志向,两度出塞,因而对鞍马风尘的征战生活与冰天雪地的塞外风光有长期的观察与体会。 辛弃疾在公元1203年6月末被起用为绍兴知府兼浙东安抚使后不久,第二年阳春三月,改派到镇江去做知府。镇江,在历史上曾是英雄用武和建功立业之地,每当他登临京口(即镇江)北固亭时,触景生情,不胜感慨。《南乡子·登京口北固亭有怀》这首词就是在这一背景下写成的。《过零丁洋》这首诗作于公元1279年。公元1278年,文天祥在广东海丰北五坡岭兵败被俘,押到船上,次年过零丁洋时作此诗。《山坡羊·潼关怀古》是元曲作家张养浩的散曲作品。这是他赴陕西救灾途经潼关所作。 主要内容《十五从军征》通过主人公的遭遇,揭示了封建兵役制度给劳动人民造成的苦难,表达了诗人对封建兵役制度给劳动人民造成的苦难的怨恨和对劳动人民的同情。《白雪歌送武判官归京》描写西域八月飞雪的壮丽景色,抒写塞外送别、雪中送客之情,表现离愁和乡思,却充满奇思异想,并不令人感到伤感。 《南乡子·登京口北固亭有怀》此词通过对古代英雄人物的歌颂,表达了作者渴望像古代英雄人物那样金戈铁马,收拾旧山河,为国效力的壮烈情怀;饱含着浓浓的爱国思想,但也流露出作者报国无门的无限感慨,蕴含着对苟且偷安、毫无振作的南宋朝廷的愤懑之情。《过零丁洋》全诗表现了慷慨激昂的爱国热情和视死如归的高风亮节,以及舍生取义的人生观,是中华民族传统美德的崇高表现。《山坡羊·潼关怀古》此曲抚今追昔,从历代王朝的兴衰更替,想到人民的苦难,一针见血地点出了封建统治与人民的对立,表现了作者对历史的思索和对人民的同情。 基础过关 4.(重庆一中中考模拟)将下列句子组成语意连贯的一段话,语序排列正确的一项是()①少年的一个特点是对声音非常敏感,这使他们能够通过“听”而轻松地记住那些语言优美的古诗文。②语言学家赵元任回忆儿时的学习情景时也说:“晚上念诗我们都觉得比白天念书轻松一点儿,我觉着也好玩一点儿。我念的是《唐诗三百首》。我哥哥跟姊妹们另外还念《千家诗》跟别的诗集。他们念的诗,我就是没念也渐渐地背得出来了。”③美学家朱光潜回忆自己的学习经历时说:“五经之中,我幼时全读的是《书经》《左传》。《诗经》我没正式地读,家塾里有人常在读,我听了多遍,就能成诵大半。于今我记得最熟的经书,除《论语》外,就是听会的一套《诗经》。”④他们都是通过“听”而记住了这些作品。⑤背诵诗文,需要“从娃娃抓起”,要最大限度发挥儿童时期听觉记忆的作用。A.①③②⑤④B.⑤①③②④C.⑤①②③④D.①②③⑤④B 1.下列词语中加点字的注音无误的一项是(  )A.舂谷(chōng) 狗窦(dǒu)阑干(lán)辕门(yuán)B.瀚海(hàn)  锦衾(jīn)羹饭(gēng)  兜鍪(móu)C.野雉(zhì)  干戈(gē)  宫阙(quē)寥落(liáo)D.踌躇(chú)羌笛(qiāng)潼关(tóng)  狐裘(qiú)答案D A.“窦”应读dòu。B.“衾”应读qīn。C.“阙”应读què。················拓展训练 2.(独家原创试题)解释下列加点词在文中的意思。(1)松柏冢累累(      )(2)瀚海阑干百丈冰(      )(3)风掣红旗冻不翻(      )(4)留取丹心照汗青(      )答案(1)众多的样子 (2)纵横交错的样子 (3)拉,扯 (4)这里指史册解析要结合诗句的意思推测词义,如“瀚海阑干百丈冰”意思是“无边沙漠结成百丈坚冰”。阑干:纵横交错的样子。······· 3.下列与诗歌有关的常识搭配有误的一项是(  )A.《十五从军征》—《乐府诗集》—郭茂倩选编—乐府诗B.《过零丁洋》—《过零丁洋》—文天祥—绝句C.《南乡子·登京口北固亭有怀》—《稼轩词编年笺注》—辛弃疾—词D.《山坡羊·潼关怀古》—《全元散曲》—张养浩—曲答案B 《过零丁洋》选自《文天祥诗集校笺》,是一首律诗。律诗和绝句的区别是:律诗是每首有八句的格律诗,绝句是每首有四句的格律诗。 1.下列句子节奏划分有误的一项是(  )A.遥看/是/君家,松柏/冢/累累。B.将军/角弓/不得/控,都护/铁衣/冷/难着。C.山河/破碎/风飘/絮,身世/浮沉/雨打/萍。D.伤心/秦汉/经行处,宫阙/万间/都做了土。答案C 正确的划分:山河/破碎/风/飘絮,身世/浮沉/雨/打萍。能力提升全练 2.下列诗句没有使用对偶修辞的一项是(  )A.山河破碎风飘絮,身世浮沉雨打萍。B.惶恐滩头说惶恐,零丁洋里叹零丁。C.散入珠帘湿罗幕,狐裘不暖锦衾薄。D.瀚海阑干百丈冰,愁云惨淡万里凝。答案C 对偶是用字数相等、结构相同、意义相关的一对短语或句子来表达相同或相对意思的修辞方法。C项显然不符合这一特点。 3.下列对《过零丁洋》理解不正确的一项是(  )A.《过零丁洋》是一首七言律诗,四联八句,全诗押的是ing韵。B.《过零丁洋》多用对偶句,如“山河破碎风飘絮,身世浮沉雨打萍”和“惶恐滩头说惶恐,零丁洋里叹零丁”,对仗极为工整。C.“山河破碎风飘絮,身世浮沉雨打萍”承接上联,从国家和个人两个方面继续抒写事态的发展和深沉的忧愤。这一联对仗工整,比喻贴切,真实反映了当时的社会现实和诗人的遭遇。国家、民族的灾难,个人坎坷的经历,万般痛苦煎熬着诗人的情怀,使其言辞充满凄楚。D.“人生自古谁无死?留取丹心照汗青”表明诗人以死明志的决心,全诗透露出诗人内心的恐惧和对前途的迷茫。答案D“全诗透露出诗人内心的恐惧和对前途的迷茫”的说法不当。在这首诗中,诗人通过追忆自己抗元的艰辛经历,表现了自己的忧国之痛和愿意以死明志、为国捐躯的豪情壮志。 1.下列关于诗词的文学常识表述不正确的一项是(  )A.近体诗的格律有严格的规定,一般分为两大类:律诗、绝句。B.在诗歌发展史上,把唐代以前的诗歌和唐代讲究格律的诗称为“古体诗”。C.律诗由四联组成,每联两句,分别为首联、颔联、颈联、尾联。D.近体诗用韵很严,除首句可以入韵也可不入韵外,各联的偶数句必须押韵。答案B 唐代讲究格律的诗一般被称为“近体诗”。 2.《过零丁洋》《南乡子·登京口北固亭有怀》和《山坡羊·潼关怀古》的体裁分别是(  )A.诗 词 词  B.词 词 曲C.诗 词 曲  D.诗 曲 词答案C“南乡子”是词牌名,“山坡羊”是曲牌名。 3.阅读《山坡羊·潼关怀古》,完成(1)—(3)题。山坡羊·潼关怀古峰峦如聚,波涛如怒,山河表里潼关路。望西都,意踌躇。伤心秦汉经行处,宫阙万间都做了土。兴,百姓苦;亡,百姓苦。(1)本曲作者是,朝家。“山坡羊”是。(2)点明主旨的句子是:。(3)仔细品味画线的句子,说说加点的字好在什么地方,会使人产生怎样的联想。答: 答案(1)张养浩 元 文学 曲牌名(2)兴,百姓苦;亡,百姓苦(3)聚:使静态物具有动态美,使人仿佛看到群山争着向潼关奔来的情景。怒:使人觉得黄河如兽般在咆哮,并联想到河水奔腾澎湃的情景。解析(1)牢记课下注释中的文学常识。注意作者的朝代和作品的体裁。(2)诗人感慨历代王朝兴亡,表达了对在动乱中遭受苦难的人民的同情。“兴,百姓苦;亡,百姓苦”两句议论,道出了全文的主旨,揭示了时局动荡给人民带来的苦难,十分精辟。(3)“聚”和“怒”写出了山河的动态与灵性,将山的雄伟与水的奔腾之势勾勒出来,有力地烘托了作者吊古伤今的悲愤、伤感之情。 阅读提高(2019重庆A)一把老钥匙王举芳①回到家时,母亲正翻箱倒柜。杂乱的地面让我无处落脚。我说:“妈呀,您这是在翻传家宝吗?”②母亲停住手看着我说:“见到我的钥匙没?”③“喏,在这儿。”我从玄关柜上拿起属于母亲的那串钥匙。④“我说的不是这个,是老宅的,老宅的那个。”母亲的语气和神情有些焦急。我和母亲几乎把家里翻了个底朝天,也没找到母亲要找的钥匙。母亲坐下来,情绪有些低落。我说:“妈,您就别总想着老宅了,咱又不回去住了,有没有钥匙都一样。”母亲叹了一口气,开怕收拾地上的凌乱。⑤其实,我没有告诉母亲,弟弟正四处托人,要把老宅卖掉。弟弟说老宅总空着,时间久了,房屋会倒塌的,到时候想出手都不好意思谈价钱。⑥一周后,弟弟告诉我,老宅卖掉了,卖了两万元。看着那些钱,不知怎么,我的心里像坠了一块石头。 ⑦那天下班,在小区外碰到三婶。三婶是我家邻居,和我们家没有亲属关系,按村里辈分我这么喊她。我让三婶到家里坐坐,她直摇手,说没啥大事,就不去家里了,说着掏出一把钥匙交给我,说这钥匙是你们家老宅的,啥时候想回家就回。原来是她家买了我们家的老宅。⑧母亲没再提钥匙的事儿。我想着老宅现在已经是别人家的了,也就再不能回去,就没跟母亲提钥匙的事儿,把它包裹好,放在了柜子顶上的一个盒子里。⑨农历六月六,我们老家有传统庙会。母亲执意要回去看看。无奈,我和弟弟只好依着母亲。一路上母亲说着故乡风俗和旧年往事,精神从未有过地爽朗,她没有看到我和弟弟偶尔交会的眼神里都藏着忐忑。⑩三婶听说我们回来了,招呼我们去她家里。做邻居的那些年,三婶和母亲一直处得很好,像亲姐妹一样。吃过晚饭,三婶拿了几床铺盖说:“你们别嫌,都是干净的。走,到你们家去,你们还睡你们各自的屋。”三婶掏出钥匙打开老宅的锁,我们怔怔地望着那干净整洁的院子,有些恍惚,仿佛我们从未离开过。 ⑪我送三婶到大门口,对她说谢谢。三婶说,咱不说远亲近邻,我懂你妈的心思。我知道她舍不下老家。庄稼人走到哪里,其实根都牢牢扎在老家的土里。另外,我给你们钥匙,还有一个原因。还记得你在家的时候,经常问我为什么总带着一把老钥匙吗?我的老家在遥远的山里,是土房子,因为一场突来的泥石流,房子没有了,但母亲一直让我们自个儿保存着属于自己的那把老钥匙。想家的时候,我就看看老钥匙,摸摸老钥匙,想象着转动钥匙打开门锁,爹娘兄妹啊,那些熟悉的物件啊,一下子呼啦啦在眼前演电影,心里就热乎乎的,就连当初的一些懊悔、吵闹都成了好。你们想回来看看的时候就回来,这里啥时候都是你们的家。⑫三婶眼里有亮光闪烁,我也感觉似乎有水滴落进了眼里。⑬回城后,我把三婶送来的钥匙给了母亲。母亲摩挲着钥匙说:“家门的钥匙在手里,不论何种身份何种境遇,你还是个有家可归的人。”⑭从那以后,不知为什么,有时候我也会摩挲那把老宅的钥匙。那一个个齿痕,似一个个密码,打开岁月的珍藏。 ⑮三婶来电话说村里要建社区了,老房子要拆迁了,用不了多久,老家的人也都要住进楼房里了。停了停,她说:“也好,咱们有钥匙。”⑯几年过去了,母亲一直保存着那把老钥匙,再也未丢过。⑰老钥匙陪着母亲风来雨往,不经意间常生斑斑锈迹,但都会被母亲那厚重、灵巧的双手反反复复摩挲着擦亮。(选自《时代文学》2018年第7期,有删改) 3.理解文中第⑭段画线语句的含义。那一个个齿痕,似一个个密码,打开岁月的珍藏。4.结合文本,简要分析三婶这一人物形象。5.文中的“老钥匙”承载着哪些情感?请简要分析。这里的“齿痕”指的是老宅钥匙的齿痕,“打开岁月的珍藏”指的是对老屋的回忆。老屋钥匙上的齿痕,就像一把记忆的开关钥匙,拿到手里,就会唤起对老屋的记忆和家的归属感。善良、淳朴、真诚。三婶买了老宅并把钥匙交给了我们,体现了她的善良;三婶一直保存着自己老家的钥匙,写出了她的淳朴。三婶对母亲一直真诚相待,体现了她的真诚。①对老屋的怀念和归属感;②对母亲的愧疚、对三婶的感激;③对根的寻找。 那雪花洁白如玉,它是天宫派下的小天将,还是月宫桂树上落下的玉叶呢?雪花像美丽的玉色蝴蝶,似舞如醉;像吹落的蒲公英;似飘如飞;像天使赏赠的小白花儿;忽散忽聚。飘飘悠悠,轻轻盈盈,无愧是大地的杰作!只见眼前的雪花像蝴蝶一样调皮,一会儿落在屋檐下,一会落在树枝上,还不时飘在行人的脸上。美文欣赏·冬天

版权提示

  • 温馨提示:
  • 1. 部分包含数学公式或PPT动画的文件,查看预览时可能会显示错乱或异常,文件下载后无此问题,请放心下载。
  • 2. 本文档由用户上传,版权归属用户,莲山负责整理代发布。如果您对本文档版权有争议请及时联系客服。
  • 3. 下载前请仔细阅读文档内容,确认文档内容符合您的需求后进行下载,若出现内容与标题不符可向本站投诉处理。
  • 4. 下载文档时可能由于网络波动等原因无法下载或下载错误,付费完成后未能成功下载的用户请联系客服vx:lianshan857处理。客服热线:13123380146(工作日9:00-18:00)

文档下载

所属: 初中 - 语文
发布时间:2022-02-19 22:07:40 页数:1330
价格:¥8 大小:22.28 MB
文章作者:追求真实

推荐特供

MORE